Você está na página 1de 425

.

Aulas Tericas e de Problemas


de
lgebra Linear

Nuno Martins

Departamento de Matemtica
Instituto Superior Tcnico
Maio de 2011

ndice
Parte I (Aulas tericas e chas de exerccios)

Matrizes e sistemas de equaes lineares.................................................................3


1a cha de exerccios para as aulas de problemas............................................................23
1a cha de exerccios facultativos....................................................................................29
Determinantes.............................................................................................................33
2a cha de exerccios para as aulas de problemas...........................................................39
Espaos lineares (ou Espaos vectoriais)...............................................................44
3a cha de exerccios para as aulas de problemas...........................................................63
2a cha de exerccios facultativos....................................................................................71
Transformaes lineares............................................................................................73
4a cha de exerccios para as aulas de problemas...........................................................88
Valores prprios e vectores prprios. Diagonalizao..........................................98
5a cha de exerccios para as aulas de problemas..........................................................111
3a cha de exerccios facultativos..................................................................................115
Produtos internos e ortogonalizao.....................................................................116
6a cha de exerccios para as aulas de problemas...........................................................130
4a cha de exerccios facultativos..................................................................................134
Produto externo e produto misto..........................................................................135
Diagonalizao unitria e diagonalizao ortogonal...........................................137
Formas quadrticas..................................................................................................141
Mnimos quadrados..................................................................................................147
7a cha de exerccios para as aulas de problemas............................................................151
Bibliograa...................................................................................................................152

Parte II (Resolues das chas de exerccios)

Resoluo da 1a cha de exerccios para as aulas de problemas.......................................153


Resoluo da 1a cha de exerccios facultativos..............................................................186
Resoluo da 2a cha de exerccios para as aulas de problemas.......................................200
Resoluo da 3a cha de exerccios para as aulas de problemas.......................................215
Resoluo da 2a cha de exerccios facultativos..............................................................281
Resoluo da 4a cha de exerccios para as aulas de problemas.......................................292
Resoluo da 5a cha de exerccios para as aulas de problemas.......................................348
Resoluo da 3a cha de exerccios facultativos..............................................................378
Resoluo da 6a cha de exerccios para as aulas de problemas.......................................383
Resoluo da 4a cha de exerccios facultativos..............................................................416
Resoluo da 7a cha de exerccios para as aulas de problemas.......................................419

Matrizes e sistemas de equaes lineares


Denio 1. (i) Sejam m; n 2 N. Uma matriz A, do tipo m
uma tabela de mn nmeros dispostos em m linhas e n colunas:
2
3
a11 a12
a1n
6 a21 a22
a2n 7
6
7
A = 6 ..
..
.. 7 :
4 .
.
. 5
am1 am2
amn

n (l-se m por n),

Usa-se tambm a notao A = (aij )m n ou simplesmente A = (aij ), na qual aij a entrada


(i; j) da matriz A. Se m = n, diz-se que A uma matriz quadrada do tipo n n (ou de
ordem n) e as entradas a11 ; a22 ; :::; ann formam a chamada diagonal principal de A. Se
m 6= n, diz-se que A uma matriz rectangular.
(ii) A matriz linha i de A :
j de A :

ai1 ai2

ain , para i = 1; :::; m. A matriz coluna

a1j
a2j
..
.

6
6
6
4

para j = 1; :::; n.

amj

3
7
7
7
5

(iii) matriz do tipo m n cujas entradas so todas iguais a zero, chama-se matriz
nula e representa-se por 0m n ou simplesmente por 0. Por exemplo
02

(iv) matriz do tipo n

0 0
0 0

02

0 0 0
0 0 0

n
2
6
6
6
4

a11 0
0 a22
..
.
0

...

0
0
..
.
ann

7
7
7
5

tal que aij = 0 se i 6= j para todos os i; j, isto , matriz cujas entradas fora da diagonal
principal so todas nulas, chama-se matriz diagonal.
(v) matriz do tipo n

2
6
6
6
4

3
1 0
0
0 1
0 7
7
,
..
. . .. 7
. . 5
.
0 0
1

chama-se matriz identidade e representa-se por In


3

ou simplesmente por I.

(vi) matriz do tipo n

n
2
6
6
6
4

a11 a12
0 a22
.. . .
.
.
0

..
0

a1n
a2n
..
.
ann

3
7
7
7
5

cujas entradas por baixo da diagonal principal so todas nulas, isto , tais que aij = 0 se
i > j, chama-se matriz triangular superior. matriz do tipo n n
2
3
a11 0
0
.. 7
.
6
. 7
6 a21 a22 . .
6 .
7
.
.
..
.. 0 5
4 ..
an1 an2
ann

cujas entradas por cima da diagonal principal so todas nulas, isto , tais que aij = 0 se
i < j, chama-se matriz triangular inferior.
Uma matriz diz-se triangular se fr triangular superior ou triangular inferior.
Exemplo 1. As matrizes

A=

1
2

1
2

, B=

1 2
2 0

3 4
2 0

so dos seguintes tipos: A 2 2, B 2


a21 = 2, b13 = 3, c12 = 0 e d41 = 1.

, C=

4, C 1

Observao 1. Uma matriz (real) A do tipo m

0 0 7

3, D 4

3
4
6 3 7
7
e D=6
4 2 5
1

1. Tem-se, por exemplo,

n uma aplicao:

A : f1; :::; mg f1; :::; ng ! R


(i; j)
! aij
Notao 1. O conjunto de todas as matrizes reais (complexas) do tipo m n denotado
por Mm n (R) (Mm n (C)). Tem-se Mm n (R) Mm n (C).
Denio 2. Duas matrizes so iguais se forem do mesmo tipo e se as entradas correspondentes forem iguais, isto , A = (aij )m n e B = (bij )p q so iguais se m = p, n = q e
aij = bij , para i = 1; :::; m e j = 1; :::; n.
Denio 3. A soma de duas matrizes do mesmo tipo
A = (aij )m

e B = (bij )m

a matriz
A + B = (aij + bij )m
4

n.

Exemplo 2. Sejam
A=

A+B =

1 4
3 2

1
3

0
7

, B=

3
1
5
, C = 4 1=2
p
2

2 4
3 9

3
1
1=2 5 :
e D=4 p
2

3
0
1 2 3
, C + D = 4 0 5 e no possvel, por exemplo, somar B com C.
4 5 6
0

Denio 4. O produto de um escalar (nmero real ou complexo)


matriz A = (aij )m n a matriz:
A = ( aij )m

n.

Notao 2. A matriz ( 1)A ser denotada por

Exemplo 3. Seja A =

1 4
3 2

por uma

A.

1
. Tem-se, por exemplo,
6

2
6

2A =

Observao 2. 1A = A, 0A = 0 (matriz nula), A + A = 2A,

8
4

2
.
12

A
::: + A} = nA.
| + {z
n vezes

Denio 5. A diferena entre duas matrizes A e B do mesmo tipo denida por


A

B = A + ( B),

ou seja, a soma de A com o simtrico de B.


Denio 6. (i) O produto AB de duas matrizes A e B s pode ser efectuado se o
nmero de colunas da 1a matriz, A, fr igual ao nmero de linhas da 2a matriz, B. Nesse
caso, o produto AB de A = (aij )m p por B = (bij )p n denido por:
!
p
X
AB = (ai1 b1j + ::: + aip bpj )m n =
aik bkj
,
k=1

m n

isto ,
2

a11
6 ..
6 .
6
6 ai1
6 .
4 ..
am1

3
a1p
.. 7 2
. 7
76
aip 7 4
.. 7
. 5
amp

b11
..
.

b1j
..
.

bp1

bpj

p
P

a1k bk1
3 6
6 k=1
b1n
p
P
.. 7 = 6
aik bkj
. 5 6
6
k=1
6
p
bpn
4 P
amk bk1
k=1

p
P

a1k bkn 7
7
7
7
7
7
p
P
5
amk bkn
k=1

k=1

Note que sendo b1 ; :::; bp as colunas da matriz B, ento


AB = A

b1

bp

Ab1

Abp

e sendo a1 ; :::; ap as linhas da matriz A, ento


2
3
2
3
a1
a1 B
6
7
6
7
AB = 4 ... 5 B = 4 ... 5
am
am B
(ii) Sejam A uma matriz do tipo n

n e p 2 N. A potncia p de A denida por

0
Ap = A:::A
| {z } e para p = 0 dene-se (se A fr no nula) A = I.
p vezes

(iii) Diz-se que duas matrizes A e B comutam se AB = BA.


0
2

Exemplo 4. (i)
=

2
3

1 1
3 2

1
2

1 + ( 2) ( 3) 0 1 + ( 2) 2 0 ( 1) + ( 2) ( 2)
2 1 + 3 ( 3)
2 1+3 2
2 ( 1) + 3 ( 2)

(ii)

1 1
2

3
1
5= 1
1 4 1=2
p
2
3
2

1
5 1 1
(iii) 4 1=2
p
2
2
a11 0
6 0 a22
6
(iv) n 2 N, 6 ..
4 .
0

=4

..

0
0
..
.
ann

1
2

( 1) + 1

( 1)

+ ( 1)

4
8

4
8

1
2

3 2
( 1)
1
1
5=4
1
( 1)
2
2
p
p
2
1
2
( 1)
3
0
0
(a22 )n
0 7
7
7.
..
..
5
.
.
n
0
(ann )

( 1)

1
1
2
p
2
1
3n 2
(a11 )n
6 0
7
6
7
7 =6
..
4
5
.

6
7

( 1)

Observao 3. (i) O produto de matrizes no comutativo. Por exemplo, para


A=

0 1
1 0

eB=

0
1

1
0

tem-se AB =

1
0

0
1

e BA =

1 0
0 1

Logo AB 6= BA.
(ii) CD = 0 ; (C = 0 ou D = 0), pois, por exemplo, para
C=

1 1
1 1

D=

1
1

1
1

CD = 0:

(iii) Se A (B) tem uma linha (coluna) nula ento AB tem uma linha (coluna) nula.
6

1
1

2
p

1
1

2
p

1
1

p2
2
2

3
5

Denio 7. (i) A transposta de uma matriz A = (aij )m


isto
2
3T 2
a11 a12
a1n
a11 a21
6 a21 a22
7
6
a2n 7
6
6 a12 a22
6 ..
..
.. 7 = 6 ..
..
4 .
4 .
.
. 5
.
am1 am2
amn
a1n a2n
(ii) Sendo A = (aij )m
gada de A.

(iii) Sendo A = (aij )m


conjugada de A.
2

1
Exemplo 5. 4 4
1

a matriz AT = (aji )n
am1
am2
..
.
amn

m,

7
7
7:
5

chama-se matriz conju-

2 Mm

(C), matriz A = (aij )m

2 Mm

(C), matriz AH = A chama-se matriz transposta

3T
3
2 5 =
6

1 4
3 2

3H
3
i 5 =
6

1 + 2i
4 4
1

1
.
6

Teorema 1. Sejam A, B, C e D matrizes de tipos apropriados,


vlidas as seguintes propriedades para as operaes matriciais.

2i 4
3
i

1
6

escalares. So

(a) (Comutatividade da soma) A + B = B + A.


(b) (Associatividade da soma) A + (B + C) = (A + B) + C.
(c) (Elemento neutro da soma) Existe uma nica matriz 0 do tipo m
0 + A = A, para toda a matriz A do tipo m n.

n tal que A + 0 =

(d) (Simtrico) Para cada matriz A existe uma nica matriz B tal que A+B = B+A = 0.
Esta matriz B denota-se por A.
(e) (Associatividade do produto por escalares)

( A) = (

) A.

(f) (Distributividade) ( + ) A = A + A.
(g) (Distributividade)

(A + B) = A + B.

(h) (Associatividade do produto de matrizes) A (BC) = (AB) C.


(i) (Distributividade) A (B + C) = AB + AC
(j)

e (B + C) D = BD + CD.

(AB) = ( A) B = A ( B).

(k) AI = A e IB = B, para todas as matrizes A = (aij )m


matriz identidade do tipo n n.
(l) A0 = 0 e 0B = 0, para todas as matrizes A = (aij )m
matriz nula do tipo n n.
(m) AT

= A:

AH

= A:
7

e B = (bij )n

m,

onde I a

e B = (bij )n

m,

onde 0 a

(n) (A + B)T = AT + B T .
(o) ( A)T = AT .

(A + B)H = AH + B H .

( A)H = AH .

(p) (AB)T = B T AT .

(AB)H = B H AH .

(q) (A1 A2 :::An )T = ATn :::AT2 AT1 , com A1 , A2 , :::, An matrizes de tipos apropriados.
H H
(A1 A2 :::An )H = AH
n :::A2 A1
Denio 8. Uma matriz A do (tipo n n) diz-se invertvel se existir uma matriz B
(do tipo n n) tal que
AB = BA = I.
matriz B chama-se matriz inversa de A e denota-se por A 1 .

Exemplo 6.

0 1
1 0

invertvel e

0 1
1 0

0 1
.
1 0

Observao 4. (i) Sendo A 1 a matriz inversa de A, ento A


1
inversa a prpria matriz A, isto , (A 1 ) = A.
I

invertvel e a sua

(ii) A matriz nula no invertvel. No entanto, a matriz identidade I invertvel tendo-se


= I.
(iii) Se uma matriz quadrada tiver uma linha ou uma coluna nula ento no invertvel.
Teorema 2. A inversa de uma matriz invertvel nica.
Dem. Sejam B e C as inversas de A. Ento, B = BI = B (AC) = (BA) C = IC = C.

Denio 9. (i) Uma matriz A diz-se simtrica se A = AT , isto , se aij = aji , para
i; j = 1; :::; n. Diz-se que A anti-simtrica se A = AT , isto , se aij = aji , para
i; j = 1; :::; n.
(ii) Uma matriz A 2 Mm n (C) diz-se hermitiana (ou hermtica) se AH = A. Diz-se
que A anti-hermitiana se AH = A.
(iii) Uma matriz A 2 Mn

(R) diz-se ortogonal se fr invertvel e se A

(iv) Uma matriz A 2 Mn

(C) diz-se unitria se fr invertvel e se A

(v) Uma matriz A diz-se normal se AH A = AAH .

Exemplo 7.

0 1
1 0

uma matriz simtrica.

0 1
1 0

0 1
.
1 0

= AT .

= AH .

1
1

1+i
i
1

cos
sen
1
2

2
i
3

2
i
3

uma matriz hermitiana.

sen
cos
1
3

2
i
3

1
1

1+i
i
1

1+i
.
i
1

uma matriz ortogonal ( 2 R):


uma matriz unitria.

2
i
3

Teorema 3. (i) Se A = (aij )n


invertvel e (AB) 1 = B 1 A 1 .

e B = (bij )n

3i
1
i
1 2i

uma matriz normal.

so duas matrizes invertveis, ento AB

(ii) Sendo
um escalar no nulo e A uma matriz invertvel ento
( A) 1 = 1 A 1 .

A invertvel e

(iii) Seja m 2 N. Se A = (aij )n n uma matriz invertvel, ento Am invertvel e


m
(Am ) 1 = (A 1 ) e escreve-se A m = (Am ) 1 .
(iv) Seja A = (aij )n
invertvel.

uma matriz. Se existir l 2 N tal que Al = 0 ento A no

(v) Sejam A e B matrizes com A invertvel tais que AB = 0. Ento B = 0.


(vi) Sejam A e B matrizes com B invertvel tais que AB = 0. Ento A = 0.
(vii) Sejam A, B e C matrizes com A invertvel tais que AB = AC. Ento B = C.
(viii) Sejam A, B e C matrizes com B invertvel tais que AB = CB. Ento A = C.
(ix) A = (aij )n
(x) A = (aij )n

uma matriz invertvel se e s se AT invertvel e AT

invertvel se e s se AH invertvel e AH

(xi) Se A = (aij )n

uma matriz simtrica invertvel, ento A

(xii) Se A = (aij )n

n
n

uma matriz ortogonal, ento AT e A

(xiv) Se A = (aij )n

uma matriz unitria, ento AH e A

1
1

= (A 1 ) :

= (A 1 ) :
1

uma matriz hermitiana invertvel, ento A

(xiii) Se A = (aij )n

simtrica.
1

hermitiana.

so matrizes ortogonais.

so matrizes unitrias.

(xv) Se A e B so duas matrizes ortogonais ento AB uma matriz ortogonal.


(xvi) Se A e B so duas matrizes unitrias ento AB uma matriz unitria.
(xvii) Se A e B so duas matrizes simtricas ento AB uma matriz simtrica se e s
se A e B comutarem.
(xviii) Se A e B so duas matrizes hermitianas ento AB uma matriz hermitiana se e
s se A e B comutarem.
9

Denio 10. Uma equao linear com n incgnitas x1 ; x2 ; :::; xn uma equao da
forma
a1 x1 + a2 x2 + ::: + an xn = b;
em que a1 ; a2 ; :::; an e b so constantes (reais ou complexas). A b chama-se termo independente.
Denio 11. Um sistema de m equaes lineares com n incgnitas um conjunto
de equaes da forma
8
a11 x1 + a12 x2 + ::: + a1n xn = b1
>
>
<
a21 x1 + a22 x2 + ::: + a2n xn = b2
( )
:::
>
>
:
am1 x1 + am2 x2 + ::: + amn xn = bm
em que aij e bk so constantes (reais ou complexas), para i; k = 1; :::; m e j = 1; :::; n.
Usando o produto de matrizes denido na seco anterior, o sistema de equaes lineares
acima pode ser escrito como uma equao matricial
AX = B
em que
2

6
6
A=6
4

a11
a21
..
.

a12
a22
..
.

a1n
a2n
..
.

am1 am2

amn

7
7
7,
5

6
6
X=6
4

x1
x2
..
.
xn

3
7
7
7
5

6
6
B=6
4

b1
b2
..
.
bm

7
7
7.
5

A matriz A a matriz dos coecientes do sistema, X a matriz coluna das incgnitas


e B a matriz coluna dos termos independentes. A matriz
2
3
a11 a12
a1n j b1
6 a21 a22
a2n j b2 7
6
7
[A j B] = 6 ..
..
..
.. .. 7
4 .
.
.
. . 5
am1 am2
amn j bm
associada ao sistema ( ) chama-se matriz aumentada do sistema.
MUITO IMPORTANTE: Note que
2

6
6
AX = 6
4

a11
a21
..
.
am1

6
7
7
6
7 x1 + 6
5
4

a12
a22
..
.
am2
10

6
7
7
6
7 x2 + ::: + 6
5
4

a1n
a2n
..
.
amn

7
7
7 xn .
5

Denio 12. Uma soluo do sistema de equaes lineares ( ) de variveis reais,


o elemento (s1 ; s2 ; :::; sn ) 2 Rn := f(a1 ; a2 ; :::; an ) : a1 ; a2 ; :::; an 2 Rg tal que as equaes do
sistema so satisfeitas quando substitumos
x 1 = s1 ;

x2 = s 2 ;

x n = sn .

:::;

(No caso das variveis serem complexas ter-se-ia solues em Cn .)


Note que isso equivale a dizer que
2
3
s1
6 s2 7
6
7
S = 6 .. 7
4 . 5
sn

satisfaz a equao matricial AX = B, isto , fazendo X = S tem-se a condio verdadeira


AS = B. Ao conjunto de todas as solues do sistema chama-se conjunto soluo ou
soluo geral do sistema.

Exemplo 8. O sistema linear de duas equaes e duas incgnitas

x + 2y = 1
pode
2x + y = 0

ser escrito do seguinte modo:


1 2
2 1

x
y

1
0

A soluo geral do sistema acima dada por f(x; y) : x + 2y = 1 e 2x + y = 0g = f( 1=3; 2=3)g,


1=3
1 2
1
isto , X =
a nica matriz que satisfaz AX = B, com A =
eB =
.
2=3
2 1
0
Teorema 4. Sejam A uma matriz do tipo m n e B uma matriz do tipo m 1. Se o
sistema de equaes lineares AX = B tem duas solues distintas X0 e X1 (X0 6= X1 ), ento
ter innitas solues.
Dem. Basta vericar que X = (1
qualquer 2 R.

) X0 + X1 soluo do sistema AX = B, para

Denio 13. A um sistema de equaes lineares da forma


8
a11 x1 + a12 x2 + ::: + a1n xn = 0
>
>
<
a21 x1 + a22 x2 + ::: + a2n xn = 0
:::
>
>
:
am1 x1 + am2 x2 + ::: + amn xn = 0
chama-se sistema linear homogneo. Este sistema pode ser escrito na forma AX = 0.

11

Observao 5.
soluo trivial:

(i) Todo o sistema linear homogneo AX = 0 admite pelo menos a


2

6
6
X=6
4

x1
x2
..
.
xn

7 6
7 6
7=6
5 4

0
0
..
.
0

7
7
7.
5

Assim, todo o sistema linear homogneo tem soluo. Alm disso, ou tem apenas a soluo
trivial ou tem innitas solues.
(ii) Como iremos ver num prximo captulo, soluo geral do sistema linear homogneo
AX = 0 d-se o nome de ncleo de A e escreve-se N (A).
Teorema 5. Se A = (aij )m
AX = 0 tem innitas solues.

tal que m < n, ento o sistema linear homogneo

Dem. Como o sistema tem menos equaes do que incgnitas (m < n), o no de linhas
no nulas r da matriz em escada de linhas obtida da matriz aumentada do sistema tambm
tal que r < n. Assim, h r pivots e n r incgnitas livres as quais podem assumir qualquer
valor. Logo, o sistema linear homogneo AX = 0 tem innitas solues.
Teorema 6. Sejam A = (aij )m

e ;

escalares.

(i) Se Y e W so solues do sistema AX = 0, ento Y + W tambm o .


(ii) Se Y soluo do sistema AX = 0, ento Y tambm o .
(iii) Se Y e W so solues do sistema AX = 0, ento Y + W tambm o .
(iv) Sejam Y e W solues do sistema AX = B. Se Y + W (para quaisquer escalares
; ) tambm soluo de AX = B, ento B = 0. (Sugesto: basta fazer = = 0.)
Teorema 7. Seja A uma matriz do tipo m n e B 6= 0 uma matriz do tipo m 1.
Qualquer soluo X do sistema AX = B escreve-se na forma X = X0 + Y onde X0 uma
soluo particular do sistema AX = B e Y uma soluo do sistema linear homogneo
AX = 0. Assim:
soluo geral de
soluo particular de
soluo geral de
=
+
.
AX = B
AX = B
AX = 0
Dem. Sendo X0 uma soluo particular do sistema AX = B e Y1 uma soluo qualquer
de AY = 0 ento A (X0 + Y1 ) = AX0 = B pelo que X0 + Y1 tambm uma soluo de
AX = B e no h soluo de AX = B que no seja deste tipo uma vez que, se X1 fr uma
soluo qualquer de AX = B tem-se
AX1 = B = AX0 , A (X1
e assim X1

X0 ) = 0

X0 = Y2 soluo de AY = 0 tendo-se X1 = X0 + Y2 .
12

Teorema 8. Seja A uma matriz do tipo n

n.

(i) O sistema AX = B tem soluo nica se e s se A fr invertvel. Neste caso a soluo


geral
X = A 1 B:
(ii) O sistema homogneo AX = 0 tem soluo no trivial se e s se A fr no invertvel.
Teorema 9. (i) Sejam A e B duas matrizes do tipo n
e B so invertveis.

n. Se AB invertvel, ento A

n tal que AB = I ento BA = I e B = A 1 :

(ii) Se A uma matriz do tipo n

Dem. (i) Considere o sistema (AB) X = 0. Se B no fosse invertvel, ento pelo teorema
anterior existiria X 6= 0 tal que BX = 0. Logo, X 6= 0 seria soluo no trivial de ABX = 0,
o que contraria o teorema anterior uma vez que por hiptese AB invertvel. Assim, B
invertvel. Finalmente, A invertvel por ser o produto de duas matrizes invertveis:
A = (AB) B 1 .
(ii) Atendendo alnea anterior, B invertvel. Logo B
A = AI = A BB
isto , A invertvel e A

= (B 1 )

= (AB) B

= IB

1
1

tambm invertvel e
= B 1,

= B.

Denio 14. (i) s seguintes operaes que se podem aplicar s equaes de um


sistema de equaes lineares, chamam-se operaes elementares.
(a) Trocar a posio de duas equaes do sistema;
(b) Multiplicar uma equao por um escalar diferente de zero;
(c) Substituio de uma equao pela sua soma com um mltiplo escalar de outra
equao.
(ii) Dois sistemas de equaes lineares que se obtm um do outro atravs de um nmero
nito de operaes elementares, dizem-se equivalentes.
Observao 6. (i) Dois sistemas de equaes lineares que se obtm um do outro atravs
de um nmero nito de operaes elementares, tm o mesmo conjunto soluo.
(ii) Quando aplicamos operaes elementares s equaes de um sistema de equaes
lineares, s os coecientes e os termos independentes do sistema so alterados. Logo, aplicar
as operaes elementares anteriores s equaes de um sistema linear ( ) equivale a aplicar
s linhas da matriz aumentada
2
3
a11 a12
a1n j b1
6 a21 a22
a2n j b2 7
6
7
[A j B] = 6 ..
..
..
.. .. 7
4 .
.
.
. . 5
am1 am2
amn j bm

as seguintes operaes.

13

Denio 15. As operaes elementares que podem ser aplicadas s linhas (i e j)


de uma matriz so:
(i) Trocar a posio de duas linhas (i e j) da matriz:

Li $ Lj

(ii) Multiplicar uma linha (i) da matriz por um escalar ( ) diferente de zero:

L i ! Li

(iii) Substituio de uma linha (j) pela sua soma com um mltiplo escalar ( ) de outra
linha (i):
L i + Lj ! L j
Teorema 10. Se dois sistemas lineares AX = B e CX = D so tais que a matriz
aumentada [C j D] obtida de [A j B] atravs de uma ou mais operaes elementares, ento
os dois sistemas so equivalentes.
Denio 16. Uma matriz A = (aij )m

diz-se em escada de linhas se:

(i) Todas as linhas nulas (formadas inteiramente por zeros) esto por baixo das linhas
no nulas;
(ii) Por baixo (e na mesma coluna) do primeiro elemento no nulo de cada linha e por
baixo dos elementos nulos anteriores da mesma linha, todas as entradas so nulas. Esse
primeiro elemento no nulo de cada linha tem o nome de pivot.
Exemplo 9. As seguintes matrizes esto em escada de linhas:
2
3
2
1 1=2 0 p0
6 0 0
3 0
2 7
6
7
4
1
0 1 3 0
6
A1 =
; A2 =
; A3 = 6 0 0
0 0
5 7
7.
0 0
0 0
5 1
4 0 0
0 0 0 5
0 0
0 0 0
Denio 17. O mtodo de resolver sistemas lineares que consiste em aplicar operaes
elementares s linhas da matriz aumentada do respectivo sistema de modo a que essa matriz
que em escada de linhas, chama-se mtodo de eliminao de Gauss.
Exemplo 10. O sistema de equaes lineares de variveis reais
8
2
32
1 0 1
< x+z =3
4 1 2 2 54
x + 2y + 2z = 6
equivalente a
:
3y + 3z = 6
0 3 3

Considere-se ento
2
1 0 1 j
4 1 2 2 j
0 3 3 j

a matriz aumentada
3
2
3
1
4 0
6 5
!
L1 +L2 !L2
6
0

x; y e z
3 2 3
x
3
y 5 = 4 6 5.
z
6

e o consequente mtodo de eliminao de Gauss:


2
3
3
0 1 j 3
1 0 1 j 3
4 0 2 1 j 3 5.
2 1 j 3 5 3 !
L
+L
!L
2
3
3
2
3 3 j 6
0 0 32 j 32
14

Logo,

8
8
< x+z =3
< x=2
2y + z = 3 ,
y=1
: 3
:
3
z=2
z = 1.
2

Neste exemplo o sistema tem a soluo nica f(2; 1; 1)g e diz-se possvel e determinado.
Exemplo 11. O sistema de equaes lineares de variveis reais x; y; z e
2
8
2
3
0 0
3
9 6
< 3z 9w = 6
4
5x + 15y 10z + 40w = 45 equivalente a
5 15
10 40 5 6
4
:
x + 3y z + 5w = 7
1 3
1 5

w
3
2
x
7
y 7 4
=
z 5
w

3
6
45 5 .
7

Considere-se ento a matriz aumentada e o consequente mtodo de eliminao de Gauss:


2
3
2
3
1 3
1 5 j
7
0 0
3
9 j 6
4 5 15
2 8 j
9 5
10 40 j
45 5
!
! 4 1 3
L1 +L2 !L2
L1 $L3
0 0 3
9 j 6
1 3
1 5 j
7
1
L !L2
5 2
2
3
2
3
1 3
1 5 j
7
1 3
1 5 j
7
4 0 0
4 0 0
1 3 j
2 5
1 3 j
2 5.
!
!
L1 +L2 !L2
3L2 +L3 !L3
0 0 3
9 j 6
0 0 0 0 j 0
Logo,

x + 3y z + 5w =
z + 3w = 2

x = 3y 2w
z = 3w + 2.

As incgnitas y e w so livres e as incgnitas x e z so no livres. A soluo geral do sistema


:
82
9
3
3s 2t 5
>
>
>
>
<6
=
7
s
6
7 : s; t 2 R
4
5
3t + 2
>
>
>
>
:
;
t
isto , o conjunto soluo dado por: f( 3s 2t 5; s; 3t + 2; t) : s; t 2 Rg. Neste exemplo
o sistema tem innitas solues e diz-se possvel e indeterminado.
Exemplo 12. Seja a 2 R. O sistema de equaes lineares de variveis reais x; y e z
8
2
32 3 2 3
1 2
1
x
3
< x + 2y + z = 3
x+y z =2
1 54 y 5 = 4 2 5.
equivalente a 4 1 1
:
x + y + (a2 5) z = a
1 1 a2 5
z
a

Considere-se
2
1
4 1
1

ento a matriz aumentada e o consequente mtodo de eliminao de Gauss:


3
2
3
2
1
j 3
1 2
1
j
3
4 0
1
1
j 2 5
1
2
j
1 5
!
!
L1 +L2 !L2
L2 +L3 !L3
2
2
1 a
5 j a
0
1 a
6 j a 3
L1 +L3 !L3
15

L2 +L3 !L3

1
4 0
0

3
1
j
3
2
j
1 5.
2) (a + 2) j a 2

2
1
0 (a

Se a = 2, ento o sistema possvel e indeterminado:


x + 2y + z = 3
,
y 2z = 1

x = 3z + 1
y = 2z + 1,

a incgnita z livre, as incgnitas x e y so no livres e a soluo geral do sistema


82
9
3
< 3t + 1
=
4 2t + 1 5 : t 2 R
:
;
t

isto , o conjunto soluo dado por: f(3t + 1; 2t + 1; t) : t 2 Rg.


Assim, se a = 2, o sistema tem innitas solues e diz-se possvel e indeterminado.
Se a = 2, o sistema no tem soluo e diz-se impossvel.
a+5
Se a 6= 2 e a 6= 2, o sistema tem a soluo nica
; a ; 1
e diz-se possvel
a+2 a+2 a+2
e determinado.
Observao 7. (Como inverter matrizes invertveis do tipo n n). Seja A uma
matriz do tipo n n e consideremos a equao AX = B. Se A fr invertvel temos
AX = B , X = A 1 B,

AX = IB , IX = A 1 B.

isto ,

Assim, para determinar a inversa de A, iremos transformar a matriz aumentada [A j I] na


matriz [I j A 1 ], por meio de operaes elementares aplicadas s linhas de [A j I]:
[A j I]

:::

IjA

Este mtodo tem o nome de mtodo de eliminao de Gauss-Jordan e consistir na


continuao do mtodo de eliminao de Gauss agora aplicado a [matriz triangular superior j ],
efectuando-se as eliminaes de baixo para cima de modo a obter-se [I j A 1 ].
Exemplo 13. Vejamos que

1
L +L2 !L2
2 1

2 1 j
0 32 j

1
1
2

0
1

2 1
1 2

2 1
1 2

2
3
1
3

1
3
2
3

2
3
1
3

=
2
3
1
3

16

1
3
2
3

1
3
2
3

2 1 j 1 0
1 2 j 0 1

: Tem-se
4
3

2 0 j
0 23 j

2
L +L1 !L1
3 2

2 1
1 2

1
3
2
3

2
3
1
3

Isto

De facto

1
2

2
3

2
L !L2
3 2
1
L !L1
2 1

.
2 1
1 2

=I

1 0 j
0 1 j

1
L +L2 !L2
2 1

2
3
1
3

1
3
2
3

0
Exemplo 14. (i) Seja A = 4 1
1
2

Logo,

0
4
1
[A j I] =
1

Verique(!) que: AA

1
5
4

1
1
2

1
2

0
4 1
1

1
5
4

5. Tem-se

1
2

1
2

3
2
3
j 1 0 0
1 0 0 j 1 2 3
j 0 1 0 5 ! 4 0 1 0 j 2 4 4 5.
:::
j 0 0 1
0 0 1 j 3 4 4

1
5
4

1
1
2

1
2

= I.

3
1 2 3
= 4 2 4 4 5:
3 4 4

3
9 8 7
(ii) Seja A = 4 6 5 4 5. Tem-se
3 2 1
2
3
2
9 8 7 j 1 0 0
3 2 1 j 0
4
5
4
[A j I] = 6 5 4 j 0 1 0
! 0 1 2 j 0
:::
3 2 1 j 0 0 1
0 0 0 j 1

0
1
2

Logo, A no invertvel.
1 2
3 4

(iii) Sejam A =

4 0
0 8

B=

C=
1

2X T

A I

1
8

0
1
4

3
1
2 5
1

. Determine-se X tal que

= C:

Tem-se
A I

2X T

, XT =
1
,X=
2

1
I
2

1 0
0 1

B 1C

1 3
2 4

2X T

=C, I
1

A ,X=
0
1
8

2X T = A 1 CB

= A 1 CB , I

1
4

1
I
2
1

AT C T

4 0
0 8

BT

,X=

,
1
2

1
2
1
2

Denio 18. (Ver-se- mais adiante a consistncia desta denio.) Seja A uma matriz
em escada de linhas. Ao no de colunas de A que no contm pivots chama-se nulidade de A
e escreve-se nul A. Ao no de pivots de A, isto , ao no de linhas no nulas de A, d-se o nome
de caracterstica de A e escreve-se car A. Se A fr a matriz em escada de linhas obtida de
C atravs de operaes elementares ento diz-se que a caracterstica de C car A, tendo-se
car C = car A e diz-se que a nulidade de C nul A, tendo-se nul C = nul A.
17

Exemplo 15. Considere-se as matrizes do exemplo 9. Pivot de A1 : 4. Pivots de A2 :


1; 5. Pivots de A3 : 2; 3; 5. Tem-se: car A1 = 1, car A2 = 2 e car A3 = 3. Alm disso:
nul A1 = 1, nul A2 = 2 e nul A3 = 2.
Denio 19. Uma matriz A = (aij )n n diz-se no singular se aps o mtodo de
eliminao de Gauss esta fr transformada numa matriz triangular superior cujas entradas
da diagonal principal sejam todas no nulas. Uma matriz A = (aij )n n diz-se singular se
aps o mtodo de eliminao de Gauss existir (pelo menos) uma linha nula na matriz obtida
de A.
Teorema 11. Seja A = (aij )n

n.

Tem-se

A invertvel , A no singular , car A = n ,

, para todo o B o sistema AX = B tem uma nica soluo (X = A 1 B),


isto ,
A no invertvel , A singular , car A < n ,
, existe pelo menos um B para o qual o sistema AX = B no tem soluo.
Observao 8. Seja [A j B] a matriz aumentada associada a um sistema de equaes
lineares com n incgnitas.
(i) Se car A = car [A j B] = n ento o sistema possvel e determinado (tem uma
nica soluo).
(ii) Se car A = car [A j B] < n ento o sistema possvel e indeterminado (tem um
no innito de solues).
(iii) Se car A < car [A j B] ento o sistema impossvel (no tem soluo).
(iv) As incgnitas livres (podem tomar valores arbitrrios) do sistema so aquelas que
correspondem s colunas, que no contenham pivots, da matriz em escada de linhas obtida
de A atravs de operaes elementares.
(v) As incgnitas no livres do sistema so aquelas que correspondem s colunas,
que contenham pivots, da matriz em escada de linhas obtida de A atravs de operaes
elementares.
(vi) car A = no de linhas no nulas da matriz em escada de linhas obtida de A =
= no de pivots = no de incgnitas no livres:
nul A = no de incgnitas livres:
(vii) Seja A uma matriz do tipo m

n. Ento 0

car A + nul A = n:

18

car A

min fm; ng e

Denio 20. Uma matriz elementar uma matriz do tipo n n obtida da matriz
identidade I (do tipo n n) atravs de uma nica operao elementar.
(i) A matriz Pij , chamada matriz de permutao, a matriz elementar obtida por
troca da linha i com a linha j da matriz I. Tem-se:
2

0
6 0 ...
6
6 . .
6 .. . .
6
6
6
6
6
6
Pij = 6
6
6
6
6
6
6
6
6 .
4 ..

..

1
0

1
1
...
1

0
1
..
.

3
0
.. 7
. 7
7
7
7
7
7
7
7
7
7
7
7
7
7
7
.
... . 7
. 7
7
..
. 0 5
0 1

i
.
j

(ii) A matriz Ei ( ) a matriz elementar obtida da matriz I atravs do produto do escalar


6= 0 pela linha i da matriz I. Tem-se:
3
2
1 0
0
6
.. 7
6 0 ... ...
. 7
7
6 .
.
7
6 . ..
1
7
6 .
7
6
i .
Ei ( ) = 6
7
7
6
.
.
6
. . .. 7
1
7
6
6 ..
.
.
..
.. 0 7
5
4 .
0
0 1
(iii) A matriz Eij ( ) a matriz elementar obtida da matriz I por soma da linha j com
um mltiplo escalar da linha i. Por exemplo para i < j tem-se:
2
3
1 0
0
.. 7
6 0 ... ...
. 7
6
6 . .
7
i
6 .. . . 1
7
6
7
6
7
..
Eij ( ) = 6
.
7
.
6
7
. . .. 7
6
. . 7
j
1
6
6 .
7
.
.
4 ..
..
.. 0 5
0

19

Observao 9. (i) As matrizes elementares Eij ( ), com i < j, so matrizes triangulares


inferiores.
(ii) As matrizes elementares Eij ( ) e Eik ( ) comutam, isto , Eij ( )Eik ( ) = Eik ( )Eij ( ).
Exemplo 16. Sejam ;
so:

P12 = P21 =

0 1
1 0

escalares com

, E1 ( ) =

0
0 1

6= 0. As matrizes elementares do tipo 2

, E2 ( ) =

1 0
0

, E12 ( ) =

1 0
1

e E21 ( ) =

Teorema 12. Sejam E uma matriz elementar do tipo m m e A uma matriz qualquer
do tipo m n. Ento, EA a matriz obtida de A atravs da mesma operao elementar que
originou E. Isto , aplicar uma operao elementar a uma matriz corresponde a multiplicar
essa matriz esquerda por uma matriz elementar.
2
3
0 0
3
9 j 6
10 40 j
45 5. A opExemplo 17. Considere-se a matriz aumentada 4 5 15
1 3
1 5 j
7
erao elementar:
2
3
2
3
0 0
3
9 j 6
1 3
1 5 j
7
4 5 15
10 40 j
45 5 ! 4 5 15
10 40 j
45 5 ,
L1 $L3
1 3
1 5 j
7
0 0
3
9 j 6
corresponde
2
0
4 0
1

seguinte multiplicao (
32
0 1
0 0
3
9
5
4
1 0
5 15
10 40
0 0
1 3
1 5

A operao elementar:
2
1 3
1 5 j
4 5 15
10 40 j
0 0
3
9 j

esquerda):
3 2
1 3
j 6
4
5
j
45 = 5 15
0 0
j
7

3
7
45 5
6

1 3
4
1 3
!
1
L !L2
5 2
0 0

1
2
3

corresponde seguinte multiplicao ( esquerda):


2
32
3 2
1 0 0
1 3
1 5 j
7
1 3
4 0 1=5 0 5 4 5 15
10 40 j
45 5 = 4 1 3
0 0 1
0 0
3
9 j 6
0 0
A operao elementar:
2
1 3
1
4 1 3
2
0 0 3

5 j
8 j
9 j

3
7
9 5
6

L1 +L2 !L2

20

1 3
4 0 0
0 0

1
1
3

1 5 j
10 40 j
3
9 j

5 j
8 j
9 j

1
2
3

5 j
3 j
9 j

3
7
45 5 .
6

3
7
9 5,
6
5 j
8 j
9 j

3
7
9 5.
6

3
7
2 5,
6

1
0 1

corresponde seguinte multiplicao ( esquerda):


2
32
3 2
1 0 0
1 3
1 5 j
7
1 3
4 1 1 0 54 1 3
2 8 j
9 5=4 0 0
0 0 1
0 0 3
9 j 6
0 0

Finalmente, a operao elementar:


2
2
3
1 3
1 5 j
7
1 3
4 0 0
4 0 0
1 3 j
2 5
!
3L2 +L3 !L3
0 0 3
9 j 6
0 0

corresponde seguinte multiplicao ( esquerda):


2
32
3 2
1 0 0
1 3
1 5 j
7
1 3
4 0 1 0 54 0 0
5
4
1 3 j
2 = 0 0
0 3 1
0 0 3
9 j 6
0 0

1
1
3

5 j
3 j
9 j

1 5 j
1 3 j
0 0 j

1 5 j
1 3 j
0 0 j

Tem-se ento:

E23 (3) E12 ( 1) E2

1
5

0 0
4
P13 5 15
1 3

3
9 j
10 40 j
1 5 j

3 2
1 3
6
4
5
45 = 0 0
0 0
7

3
7
2 5.
6

3
7
2 5,
0
3
7
2 5.
0
1 5 j
1 3 j
0 0 j

3
7
2 5.
0

Teorema 13. Toda a matriz elementar invertvel e a respectiva inversa tambm uma
matriz elementar. Tem-se:
(i) (Pij )

= Pij .

(ii) (Ei ( ))

= Ei (1= ), para

6= 0.

(iii) (Eij ( ))

= Eij (

).

Teorema 14. Uma matriz A invertvel se e s se fr igual ao produto de matrizes


elementares.
Observao 10. O teorema anterior indica um modo alternativo para calcular a matriz
inversa de uma matriz invertvel.
Teorema 15. (Factorizao triangular). Duas consequncias do mtodo de
eliminao de Gauss:
(i) Seja A uma matriz do tipo m n. Ento ou A admite a factorizao A = LU ou
existe uma matriz de permutao P tal que P A admite a factorizao P A = LU , onde L
uma matriz triangular inferior com as entradas da diagonal principal todas iguais a 1 e U
uma matriz em escada.
(ii) Seja A uma matriz no singular do tipo n n. Ento ou A admite a factorizao
nica A = LU ou existe uma matriz de permutao P tal que P A admite a factorizao
nica P A = LU , onde L uma matriz triangular inferior com as entradas da diagonal
principal todas iguais a 1 e U uma matriz triangular superior cujas entradas da diagonal
21

principal so os pivots que resultam de aplicar o mtodo de eliminao de Gauss matriz


A.
2

3
1 1 1
Exemplo 18. Seja A = 4 2 1 4 5. Tem-se:
2 3 5

3
1 1
1 2 5.
0 5

Logo,

1
4
E23 (1)E13 ( 2)E12 ( 2)A = 0
0

Isto ,

1
A = (E12 ( 2)) 1 (E13 ( 2)) 1 (E23 (1)) 1 4 0
0

com

1
4
A = E12 (2)E13 (2)E23 ( 1) 0
0
2
1
4
L = E12 (2)E13 (2)E23 ( 1) = 2
2
2

3
1 1
1 2 5,
0 5
3
0 0
1 0 5
1 1

3
1 1
1 2 5.
0 5

ou ainda,

1
4
U= 0
0

3
2
2 3 4
6
0 5 6 7
7. Tem-se P24 A = 6
4
0 10 6 5
1 7 8
2
3
1 2 3 4
6 0 1 7 8 7
7
E34 ( 1=2) P24 A = 6
4 0 0 10 6 5
0 0 0 3

1
6 0
Exemplo 19. Seja A = 6
4 0
0

Logo

com

1
6 0
P24 A = E34 (1=2) 6
4 0
0
P = P24 ,

3
2 3 4
1 7 8 7
7,
0 10 6 5
0 0 3

1
6 0
L = E34 (1=2) = 6
4 0
0

0 0
1 0
0 1
0 1=2
22

1
0
0
0

3
1 1
1 2 5.
0 5

3
2 3 4
1 7 8 7
7e
0 10 6 5
0 5 6

3
2 3 4
1 7 8 7
7
0 10 6 5
0 0 3

1
6
0
P24 A = (E34 ( 1=2)) 1 6
4 0
0

Isto ,

A = LU ,

ou ainda,

3
0
0 7
7
0 5
1

P A = LU ,

1
6 0
U =6
4 0
0

3
2 3 4
1 7 8 7
7:
0 10 6 5
0 0 3

1a Ficha de exerccios para as aulas de problemas


1. Verique que:
6
4

(i)

9
6

0 0
0 0

(ii)

222

0 1
1 0

220

0 1
(iv)
+
1 0
2
32 2
1
1
1
0
4
5
4
1
1
1
(vi)
= 0
2
2
2
0
2 3
2
1
1
4
5
4
1 0 1 = 2
(viii) 2
3
3
T

4
2

(x)

1
2
1

a b
c d

1
0

1
1

cos
sen

(xiii)

2
2

1
1

cos
sen
sen
cos
2 p
p

6
(xiv) 4
1
2

3
p3
3
p3
3
3

2
i
3

2
i
3

(xvi)
2

2
2p

2
i
3

cos
sen
3

6
p6
6
6p

2
i
3

1
2

3i
1
i
1 2i

3i
1
i
1 2i

2
3
2

1
1

5 4
5
=
6 11
6
32 2
4
2
5
4
4
1
=
3
1

4
11

1
1

1
4

p2
2
3

1
2

bc 6= 0)
3 2
9
1
5
4
6
1

7
1

2
3
2

0
1

2
1

7
9

p
2p 2 11
2 2 + 10

1 2
3 4
3

4
4 5
3

7
2

5
2

(se ad
2
1
1

32 3 2
3
1
2
2
1 54 1 5 = 4 2 5
1
1
0

1
1
1

ortogonal ( 2 R). Isto ,

6
3

0
1
3

2
2

2
1
1

0 1
1 0

(iii)

1 2
3 4

(v)

2
0 0
0 0 5 (vii) 4
0 0
3
0 1
0 2 5 (ix) 3
0 3

sen
cos

=I

0 0
0 0
3

d
b
c
a
ad bc
2
32
1 1
1
6
a
4
5
4
1
1 1
7
A 2 coluna de
1
1
1
9
(xi)

1000

sen
cos

cos
sen

1
2

7
5 ortogonal. (xv)
2
i
3

2
i
3

1
3

2
i
3

1
2

2
i
3

sen
cos
2
i
3

2
i
3

2
i
3

2
i
3

1
3

1
3

2
i
3

2
i
3

cos
sen

sen
cos

=I

unitria. Isto ,

2
i
3
H
2
i
3

1
2

2
i
3

2
i
3

1
3

2
i
3

2
i
3

uma matriz normal. Isto ,


2

3i
1
i
1 2i

3i
1
i
1 2i

3i
1
i
1 2i

(xvii) As constantes a; b e c que denem a funo y = ax2 + bx + c cujo grco passa


pelos pontos
constituem a 3
2 3(x1 ; y1 ) ; (x2 ; y2 ) e (x3 ; y3 ) (de abcissas distintas entre
2 si),
2
a
x1 x1 1 j y1
soluo 4 b 5 do sistema linear cuja matriz aumentada dada por: 4 x22 x2 1 j y2 5.
c
x23 x3 1 j y3
23

=I

2. Efectue, sempre que possvel, as seguintes operaes.


1
3

(i)
(iv) 2

1
3
p

(vii)
0

2
3

1
(x) 4 0
6

1
3

0
1
3

1
2

1
2

0
2
1
2

1
2

0
1
4

0 1

(ii)

1
3

1
2

2
B
B
4
(ix) B
B2
@
2

6
3

(viii) @4

3T 2

5 4

1
2

1
3

1
2

3
0
4 5
3

3T 2
2
1
0 5 4 2
1
1
3

3
2

(iii)

1
3
02

(v)

1
2

1
0

2
1
4

0
1
3

52

6
6
6
6
4

2
1

1
3

p2 41
3
5 2
p
2
3 4

2
1

(vi)
p

8
9

1
3

1
3

1
2

5
3

1T

2
1

3 A

31T

7C
7C
C
1 7
7C
5A
5
2

3T 2
0
1
4 5 4 0
3
6

1
(xi) 4 2
1
3

0
1

1
2

1
2

1
4

3
2
0 5
1

3. Pretende-se arrumar livros em caixas. Ao colocar 7 livros em cada caixa, ca um livro


de fora. Ao colocar 8 livros por caixa, h uma caixa que s tem 1 livro. Quantos livros
se pretende arrumar? Quantas caixas existem?

4. C = Celsius, F = Fahrenheit. A partir do ponto de congelao (C; F ) = (0; 32) e do


ponto de ebulio (C; F ) = (100; 212), deduza a equao linear
9
F = C + 32:
5
Verique que o nico valor comum a ambas as escalas
5. Escreva a matriz A = (aij )4
a)

c)

aij = j 2 ( 1)i+j

2 M4

i+j 1

d)

6. Verique se a matriz (aij ) 2 M2


simtrica.
7. Determine as caractersticas
os respectivos pivots.
2
3
2
0 0
1 2
(i) 4 0 0 5
(ii) 4 0 1
0 0
1 2

(R) em cada um dos seguintes casos:


8
< 0 se i > j
aij =
: 1
caso contrrio,

b)

8
i se i = j
>
>
>
>
<
j se j = i + 1
>
>
>
>
:
i j caso contrrio,

40 .

aij =

8
<
:

aji

para todo i; j

se j > i

(R) denida por aij = 3i + 2j, para todo i; j = 1; 2,

e as nulidades das seguintes matrizes reais, identicando


3
3
1 5
3

(iii) 4
24

2
2
1

3
1
4 5
2

3
1 2 3 4
(iv) 4 5 6 7 8 5
9 10 11 12

0
(v) 4 1
1
(viii)

1
1
1
5
0

1
1
2
1 2
2 0

1
0 5
1

6
(vi) 6
4

1
1
2
3

2
1
7
3

1
3
1
2
3

3
2
9
4

3 6 9
(ix) 4 2 4 6 5
1 2 3

3
2
1 7
7
8 5
6
2

(x) 4

1
6 0
(vii) 6
4 2
4
2
1
2

10
5
10

8. Quais das seguintes equaes so equaes lineares em x; y e z ?


p
1
(b) x + z = 0
(c) x 1 + 3y z = 2
(a) 3 x + 3y + z = 1
2

3
11
5
1
6
3
6

(d) x

1
5
3
1
8
4
8

3
2
3 7
7
1 5
5
3
4
2 5
4

yz = 1

9. Diga qual dos seguintes pontos: (0; 0) ; (1; 1) ; (1; 1) ; ( 1; 1) a soluo do seguinte
sistema de equaes lineares nas variveis x; y.
8
< x+y =0
x 2y = 3
:
x y = 2.

10. Diga quais dos seguintes pontos: (0; 0; 0; 0) ; (1; 1; 1; 0) ; (1; 1; 1; 2) ; 3; 9; 7;

p
3

so solues do sistema de equaes lineares nas variveis x; y; z e w.


x 2y 3z = 0
x + y + z = 1.
11. (i) Determine os coecientes a; b; c e d da funo polinomial p(x) = ax3 + bx2 + cx + d,
cujo grco passa pelos pontos P1 = (0; 10); P2 = (1; 7); P3 = (3; 11) e P4 = (4; 14).
(ii) Determine os coecientes a; b e c da equao da circunferncia
x2 + y 2 + ax + by + c = 0;
que passa pelos pontos P1 = ( 2; 7); P2 = ( 4; 5) e P3 = (4; 3).
12. Seja
2 R. Em funo do parmetro , calcule a caracterstica e a nulidade das
seguintes matrizes. Em cada alnea, indique ainda (se existirem), justicando, os valores de para os quais essas matrizes so invertveis:
2
3
2
3
2
3
2
1 0 1
1
2
5
1
2 5
1
1 5
(i) 4 1
(ii) 4 2
(iii) 4 2
2
0
1
3
2
1
0
1
+1
3
2
3
2
2
3
1
1
0
1
0 1
1 0 1
6
7
6 0
6
1 0 7
1
0
1 0 0 7
7
7 (v) 6 0 1
7 (vi) 6 1
(iv) 6
2
3
5
4 1 0
4 1
4 3
1
0
1 5
0
0 5
2
1 1
1
1
1 1 2
2 0
2
2
13. Determine valores para x; y; z e w de modo a que nas reaces qumicas seguintes
os elementos qumicos envolventes ocorram em iguais quantidades em cada lado da
respectiva equao.
(a) xC3 H8 + yO2 ! zCO2 + wH2 O

(b) xCO2 +yH2 O ! zC6 H12 O6 + wO2


25

14. Resolva os seguintes sistemas de equaes lineares.


2x + 4y = 10
4x 2y = 5
(c)
3x + 6y = 15
6x + 3y = 1
8
8
< 2x + y 3z = 5
< 2x + 3y 2z = 5
3x 2y + 2z = 5
x 2y + 3z = 2
(d)
(e)
:
:
5x 3y z = 16
4x y + 4z = 1
8
8
< x + 2y + 3z = 3
< 2x + 3y = 3
2x + 3y + 8z = 4
x 2y = 5
(f)
(g)
:
:
3x + 2y + 17z = 1
3x + 2y = 7
8
8
z + 3w = 3
< x + 2y
< x + 5y + 4z 13w = 3
2x + 4y + 4z + 3w = 9
3x y + 2z + 5w = 2
(h)
(i)
:
:
3x + 6y z + 8w = 10
2x + 2y + 3z 4w = 1
8
8
2x3 + 3x4 = 4
>
>
<
< x 2y + 3z w = 1
2x1 6x3 + 9x4 = 7
3x y + 2z + 5w = 2
(j)
(k)
2x1 + 2x2 5x3 + 2x4 = 4
>
:
>
3x + 6y 9z + 3w = 6
:
100x2 + 150x3 200x4 = 50

(a)

2x + 3y = 1
5x + 7y = 3

(b)

15. Discuta em funo do parmetro real os seguintes sistemas de equaes lineares (nas
variveis x; y e z). Nos casos em que existirem solues, determine-as.
8
8
< x+y+z =1
< x+y+ z =2
x + 2y + z = 1
x+ y +z =1
3x + 4y + 2z =
(a)
(b)
(c)
2x + y + 8z = 3
:
:
x+y+ z =1
2x + 3y z = 1
8
8
< x +y+z =1
< x+y + z =1
x+ y+z =
2x + y 2 z =
(d)
(e)
:
:
2
x+y + z =
x+ y +z = 1+2

16. Discuta os seguintes sistemas de equaes lineares em termos dos parmetros reais
e . Nos casos em que existirem solues, determine-as.
8
8
8
2z + w =
>
>
< x+y z+ w =0
< x + 4y + 3z = 10
<
x + y + z + 3w = 1
2x + 7y 2z = 10 (b)
x 2y + 2z + w = 1
(a)
(c)
2x + 2y + z + w = 2
:
>
:
>
x + 5y + z =
x y + z + ( + 1) w =
:
x + y + 3z + 14w = 4
17. Diga para que valores de a; b e c tm solues os sistemas.
8
8
< x + 2y 3z = a
< x 2y + 4z = a
3x y + 2z = b
2x + 3y z = b
(a)
(b)
:
:
x 5y + 8z = c
3x + y + 2z = c

18. Determine um sistema de equaes lineares cujo conjunto de solues seja:


(a) S = f(1 + t; 1

t) : t 2 Rg

(c) S = f(3t; 2t; t) : t 2 Rg


(d) S = f(3t
(e) S = f(2t

(f) S = f(1

(b) S = f(t; 1
2t + 1) : s; t 2 Rg

s; t + 2s

1; s

3s; t + s

1; 2s + 1; t

s; s

t; 2s; t

2t; 1) : t 2 Rg

1) : s; t 2 Rg

1) : s; t 2 Rg
26

(g) S = ?

19. Determine todas as matrizes reais 2


20. Existem 16 matrizes 2
vertveis?
21. Seja A 2 Mn
a sua inversa.

1 2
.
3 4

2 que comutam com a matriz

2 s com 0 e 1 nas respectivas entradas. Quantas so in-

(R) tal que A2 + 2A + 2I = 0:Verique que A invertvel e determine

3 4
. Em cada
7 9
um dos seguintes casos, determine a matriz X que satisfaz a equao

22. Sejam A; B; X 2 Mn

(R) matrizes invertveis tais que (AB)2 =

(i) AXB + AB = 0
23. Determine A 2 M2

(ii) BXA

(R) tal que 2I

(3A 1 )

4 3
7 5

A 1B

=0

24. Determine (se existirem) as inversas das seguintes matrizes.


0
1
2
1
4
(v) 4
7
2

1 2
(iii) [1] (iv)
3 4
2
3
3
2
2 1
2 3
1 2 3
1
2 5
5 6 5
(vi) 4 0 1 2 5
(vii) 4 1 2
3
8 9
0 0 1
2 1
2
3
2
3
1 0 2
1 2 1
cos
sen
4
5
4
(ix) 4 0 6 5
(x)
(viii) 0 3 0
sen
cos
4 0 5
1 8 1
2
3
2
3
k 0 0 0
0 0 0 k1
6 1 k 0 0 7
6
7
7, com k 6= 0 (xii) 6 0 0 k2 0 7, com k1 ; k2 ; k3 ; k4 6= 0
(xi) 6
4 0 1 k 0 5
4 0 k3 0 0 5
0 0 1 k
k4 0 0 0
2 5
3
2
3
2
2
8
1
1
1
1
13
13
13
13
2
2
2
6
7
6
7
6 2
7
6 1
7
7
6
2
1
6
7
6
7
1
0
13
13
13 7
2 7
6 13
6 2
6
7
6
7
(xiii) 6
(xiv) 6
7
7
6
7
2
1 7
6 2
7
6 1 0
1
13
13
13 7
2 7
6 13
6 2
4
5
4
5
1
8
2
2
5
1
1
1
13
13
13
13
2
2
2
(i)

1
0

(ii)
3

25. (i) Seja A 2 Mn

1 0
0 1
2

(R) tal que

Ak = 0

para algum k 2 Nn f1g. Verique que


(I
2

3
1 1 0
(ii) Calcule 4 0 1 1 5
0 0 1

A)

= I + A + ::: + Ak

27

2 2
26. Seja A = 4 5 1
1 5

3
2
3 5.
3
2

3
0 0 0
(i) Verique que A3 = 4 0 0 0 5.
0 0 0

(ii) Calcule (I

A) (I + A + A2 ) :

27. Para cada parmetro real , considere o sistema de equaes lineares de variveis reais
cuja matriz aumentada dada por:
2
3
1 4 2 j 10
4 2 7 2 j 20 5 .
1 5
j 10
a) Discuta em termos de
lineares anterior.

a existncia ou no de soluo do sistema de equaes

b) Para = 4, determine o conjunto soluo do sistema de equaes lineares correspondente.


2
3
1 0
1
0
2
6 1
7
+
7, com ; 2 R:
28. Seja A ; = 6
4 0 1
5
2
1
+
+
(a) Determine a caracterstica e a nulidade de A

em funo de

(b) Determine os valores dos parmetros e para os quais A


2
3
1 0
2
2
6 2
4 7
7, com 2 R.
29. Seja A = 6
3
4 4 0
8 5
2
2
0

e .

invertvel.

(a) Determine a caracterstica e a nulidade de A em funo do parmetro


justicando, quais so os valores de para os quais A invertvel.
(b) Para

30. Seja Ba;b

= 1;
2
0
6 2
=6
4 0
3

e diga,

determine a inversa da matriz A1 .


3
0 a 1
2 0 a 7
7, com a; b 2 R:
0 a b 5
0 6 0

(a) Determine a caracterstica e a nulidade de Ba;b em funo de a e b.


(b) Para a = 1 e b = 0 calcule a matriz inversa da matriz B1;0 , isto , (B1;0 ) 1 .
(c) Determine a soluo geral do sistema linear B1;0 X = C, C =

2 3

(d) Para b = 1, determine a soluo geral do sistema linear Ba;1 X = D, em que D o


simtrico da 3a coluna de Ba;1 .

28

1a Ficha de exerccios facultativos


1. Sendo A; B; C matrizes de tipos apropriados, mostre que:
(i) (AB) C = A (BC)

n, mostre que se AT A = 0 ento A = 0.

2. Sendo A uma matriz do tipo m


3. Sendo A =

(iii) (AB)T = B T AT

(ii) A (B + C) = AB + AC

1 2
, determine todos os u 6= 0 tais que Au = 5u.
2 4

4. Obtenha, por induo, uma frmula para An onde A dada por:


2
3
1 1 0
0
1
1 0
(i)
(ii)
(iii) 4 0 1 1 5
1 0
2 1
0 0 1
2
3
0 1 0 0
6 0 0 1 0 7
cos
sen
7
(iv) 6
(v)
( 2 R)
4 0 0 0 1 5
sen
cos
0 0 0 0

5. Mostre que se AB = A e BA = B ento A2 = A e B 2 = B.


6. Sendo A uma matriz 2
A=

cos
sen

2 ortogonal, isto , tal que AAT = AT A = I, mostre que


sen
cos

cos
sen

ou A =

sen
cos

( 2 R):

7. Diga de que tipos devero ser as matrizes A e B de modo a poderem ser efectuados os
seguintes produtos e desenvolva esses mesmos produtos.
(i) (A + B)(A

B)

(ii) (AB)2

(iii) (A + B)2

1 0
0 0
eB=
no satisfazem a relao:
2 0
3 4
AB = 0 ) A = 0 ou B = 0. O que pode concluir? E no caso de A ser invertvel, o
que concluiria acerca da veracidade da relao anterior?

8. (i) Verique que as matrizes A =

1 2
0 0
2 2
, B =
e C =
no
1 2
1 1
0 0
satisfazem a relao: AB = AC ) B = C. O que pode concluir? E no caso de A ser
invertvel, o que concluiria acerca da veracidade da relao anterior?

(ii) Verique que as matrizes A =

9. Sejam A uma matriz do tipo n

n e B uma matriz do tipo m


T

n quaisquer.

(i) Prove que se A simtrica (isto A = A ) ento BAB tambem simtrica.


(ii) Prove que se A normal (isto AH A = AAH ) e B unitria ento BAB H
normal.
(iii) Prove que B T B e BB T so matrizes simtricas e que B H B e BB H so matrizes
hermitianas.

29

10. Uma matriz A do tipo n

n diz-se anti-simtrica se AT =

A. Mostre que:

(i) Os elementos da diagonal principal de uma qualquer matriz anti-simtrica so todos


nulos.
(ii) Para qualquer matriz A do tipo n

n, a matriz A

AT anti-simtrica.

(iii) Escrevendo A = 12 (A + AT ) + 21 (A AT ), toda a matriz quadrada pode ser


decomposta de modo nico pela soma de uma matriz simtrica com uma anti-simtrica.
11. Verique que todas as matrizes X =
1
c

I;

0
1

a b
c d
1
0

que satisfazem a equao X 2 = I so:


b
1

1 a2
b

Observe assim que a equao matricial X 2 = I tem um nmero innito de solues em


contraste com a equao escalar x2 = 1 que tem apenas duas solues (1 e
1).
12. Mostre que:
0

fX 2 M2 2 (R) : XA = AX; para todo o A 2 M2 2 (R)g =


Isto , as matrizes 2
escalares da matriz I.

2 que comutam com todas as matrizes 2

13. Sendo A uma matriz do tipo m

2R .

2 so mltiplos

n, seja N (A) = fX : AX = 0g. Mostre que:

(i) Sendo A e B matrizes de tipos apropriados, ento N (B)


(ii) Sendo A 2 Mm

N (AB).

(R), tem-se N A A = N (A) :

(iii) Sendo A e B matrizes do tipo m n com m < n tais que AB T invertvel, ento
B T A no invertvel. Alm disso, nenhuma linha de B pertence a N (A).
(iv) Sendo A 2 Mm n (R) tal que para todo o B 2 Rm ; o sistema AX = B possvel,
ento N AT = f0g.
14. Sejam A; B 2 Mn
que A = B.

n (R)

tais que que Au = Bu para qualquer u 2 Mn 1 (R). Prove

15. Sejam A; B matrizes no nulas do tipo n


Justique.
16. Sendo A uma matriz do tipo m
n m tal que AB = I.
17. Duas matrizes A e B do tipo n
A = SBS 1 . Mostre que:

1. Determine a caracterstica de AB T .

n tal que car A = m, mostre que existe B do tipo


n dizem-se semelhantes se existir S invertvel tal que

(i) Sendo A ou B invertveis ento AB e BA so semelhantes.


(ii) Sendo A e B semelhantes ento X 2 N (A) se e s se S

30

X 2 N (B).

18. Seja A uma matriz quadrada (do tipo n


(i) Se A fr invertvel ento A

n). Mostre que:

tambem invertvel e (A 1 )

(ii) Se A fr invertvel ento AT tambem invertvel e (AT )


(iii) Se A fr invertvel e simtrica ento A
19. Sejam A e B matrizes do tipo n

1
1

= A.

= (A 1 )T .

tambem simtrica.

n. Mostre que:

(i) Se A; B forem invertveis ento A + B no necessariamente invertvel.


(ii) Se A; B e A + B forem invertveis ento A
(A

+ B 1)

+B

invertvel e

= A(A + B) 1 B = B(A + B) 1 A.

Sugesto: comece por vericar que


I + B 1 A = B 1 (A + B) e I + A 1 B = A 1 (A + B):

20. Seja A do tipo n


(i) I

n tal que A2 = A (A diz-se idempotente). Mostre que:

A idempotente.

(ii) 2A I invertvel e (2A I)


2A I uma matriz ortogonal.

= 2A

I. Alm disso, se A fr simtrica ento

(iii) Se car A = n, ento A = I.


21. Uma matriz B (do tipo n

n) diz-se idempotente se B 2 = B. Mostre que

A2 = I

1
(I + A) idempotente
2

22. Sendo A = (aij ) uma matriz invertvel e B = (bij ) a inversa da A, mostre, para k 6= 0,
a matriz (k i j aij ) invertvel e a sua inversa (k i j bij ).
a b
invertvel se e s se
c d
ad bc 6= 0. No caso de A ser invertvel, utilize o mtodo de eliminao de Gauss-Jordan
para encontrar a matriz inversa de A.

23. Seja A uma matriz do tipo 2

2. Mostre que A =

24. Que condies devem ser vericadas para que a seguinte matriz diagonal do tipo n
2
3
k1 0
0
.
..
6
. .. 7
6 0 k2
7
D=6 . .
7
.
..
.. 0 5
4 ..
0
0 kn
seja invertvel? Qual a sua inversa?

31

25. Para matrizes quadradas A = (aij )n n dene-se o trao de A, tr(A), como sendo a
soma de todas as entradas da diagonal principal de A, isto ,
tr(A) =

n
X

aii :

i=1

Sejam A = (aij )n
que

e B = (bij )n

duas matrizes do tipo n

ne

um escalar. Mostre

(i)
tr(A + B) = tr(A) + tr(B);
(ii)
tr( A) =

tr(A);

(iii)
tr(AT ) = tr(A);
(iv)
tr(AB) = tr(BA):
Esta ltima igualdade continua a ser verdadeira se A = (aij )m
26. Para cada matriz A do tipo n

n, verique que no existe X do tipo n


AX

27. Sejam A e B matrizes do tipo n


que

(R) tais que uT v 6=

1. Seja

A = I + uv T .
Verique que A invertvel e que
A

n tal que

n tais que A simtrica e B anti-simtrica. Mostre

(R). Mostre que tr(AT A) = 0 se e s se A = 0.

29. Sejam u; v 2 Mn

m.

XA = I.

tr(AB) = 0:
28. Seja A 2 Mm

e B = (bij )n

=I

1
uv T .
1 + uT v

Alm disso verique que


uT v = tr uv T

32

Determinantes
Denio 21. Dados os nmeros naturais 1; 2; :::; n chama-se permutao desses n
nmeros a qualquer lista em que os mesmos sejam apresentados por ordem arbitrria.
Denio 22. Seja (i1 i2 :::in ) uma permutao dos nmeros naturais 1; 2; :::; n. Diz-se que um par (ij ik ) uma inverso quando (j k) (ij ik ) < 0 (isto , quando ij e ik
aparecerem na permutao por ordem decrescente).
Denio 23. Uma permutao (i1 i2 :::in ) diz-se par (mpar) quando o no mximo de
inverses includas fr par (mpar).

Exemplo 20. A permutao (21453) mpar pois o no mximo de inverses nela includas mpar: (21); (43) e (53).
Denio 24. Seja A uma matriz do tipo n n. Chama-se determinante de A, e
escreve-se jAj ou det A, o nmero que se obtm do seguinte modo:
(i) Formam-se todos os produtos possveis de n factores em que intervenha um elemento
de cada linha e, simultaneamente, um elemento de cada coluna de A.
(ii) Afecta-se cada produto do sinal + ou do sinal
conforme as permutaes (dos
nmeros naturais 1; 2; :::; n) que guram nos ndices de linha e de coluna tenham a mesma
paridade ou no.
(iii) Somam-se as parcelas obtidas.
Em resumo: xando, por exemplo, a permutao (i1 i2 :::in ) de 1; 2; :::; n
X
jAj =
( 1) ai1 j1 ai2 j2 :::ain jn ,
(j1 j2 :::jn )
permutao de 1;2;:::;n

em que
=

8
< 0 se (i1 i2 :::in ) e (j1 j2 :::jn ) tm a mesma paridade
:

1 se (i1 i2 :::in ) e (j1 j2 :::jn ) tm paridade diferente.

Observao 11. Pode ainda escrever-se

jAj =

( 1) a1j1 a2j2 :::anjn , em que

(j1 j2 :::jn )
permutao de 1;2;:::;n

33

8
< 0 se (j1 j2 :::jn ) par
:

1 se (j1 j2 :::jn ) mpar.

ou
jAj =

( 1) ai1 1 ai2 2 :::ain n , em que

(i1 i2 :::in )
permutao de 1;2;:::;n

Denio 25. Seja A = (aij )n


da diagonal principal de A, isto ,

n.

8
< 0 se (i1 i2 :::in ) par
:

1 se (i1 i2 :::in ) mpar.

O trao de A, tr(A), a soma de todas as entradas

tr(A) =

n
X

aii :

i=1

Teorema 16. Sejam A = (aij )n


escalar. Tem-se

e B = (bij )n

duas matrizes do tipo n

ne

um

(i)
tr(A + B) = tr(A) + tr(B);
(ii)
tr( A) =

tr(A);

(iii)
tr(AT ) = tr(A);
(iv)
tr(AB) = tr(BA):
Teorema 17. (i) Se A do tipo 2
jAj =

a11 a12
a21 a22

(ii) Se A do tipo 3

jAj =

a11 a12 a13


a21 a22 a23
a31 a32 a33

2, ento

= a11 a22

a12 a21

S no caso
2 2

1
(tr A)2
2

tr A2

3, ento

= a11 a22 a33 + a12 a23 a31 + a13 a21 a32

Observao 12. (i) Se A uma matriz do tipo n

a13 a22 a31

a12 a21 a33

a11 a23 a32 .

n ento jAj tem n! parcelas.

(ii) O determinante de cada um dos trs tipos de matrizes elementares dado por
det Pij =

1;

det Ei ( ) = ;
det Eij ( ) = 1:
34

Exemplo 21. (i)


1
2

1
2

= 1( 2)

( 1)2 = 0:

(ii)
1
3
2

2
1
1

1
2
3

= 1( 1)( 3) + 3 + 8

1( 1)2

6( 3)

2 = 32:

Denio 26. Seja A = (aij ) uma matriz do tipo n n, com n > 1. Seja Aij a matriz
do tipo (n 1) (n 1) que se obtem de A suprimindo a linha i e a coluna j de A. Chama-se
a Aij o menor-ij da matriz A.
Teorema 18. (Frmula de Laplace.) Seja A uma matriz do tipo n
Tem-se
n
X
det A =
aij ( 1)i+j det Aij ,
com i 2 f1; :::; ng xo:

n, com n > 1.

j=1

Observao 13. Seja A uma matriz do tipo n


det A =

n
X

aij ( 1)i+j det Aij ,

n, com n > 1. Tem-se


com j 2 f1; :::; ng xo:

i=1

Exemplo 22.
1
2
0
1

0
1
1
0

2
1
0
2

3
4
2
3

3+2

= ( 1)( 1)

= ( 1)( 3) + ( 2)4 + 2( 2)3

( 1)3

1
2
1

2
1
2

3
1 0
3+4
4 + ( 2)( 1)
2 1
3
1 0

( 2)2( 3)

Teorema 19. Sejam A e B matrizes do tipo n

4( 2) + 2 [( 2)

n. Seja

2
1
2

( 2)] =

18

um escalar.

(i) det AT = det A.


(ii) Se A fr uma matriz diagonal, triangular superior ou triangular inferior ento o
determinante de A igual ao produto dos elementos da diagonal principal de A.
(iii) Se A tiver uma linha (ou coluna) nula ento det A = 0.
(iv) Se B fr obtida de A trocando duas linhas (ou colunas) de A ento det B =
35

det A.

(v) Se B fr obtida de A multiplicando uma linha (ou coluna) de A por um escalar


ento det B = det A.
(vi) Se duas linhas (ou colunas) de A forem iguais ento det A = 0.
(vii) Se B fr obtida de A somando a uma linha (ou coluna) de A um mltiplo escalar
de uma outra linha (ou coluna) de A ento det B = det A.
(viii) det A 6= 0 se e s se A invertvel.
n

(ix) det ( A) =

det A.

(x) det (EA) = det E det A, em que E uma matriz elementar (Pij ; Ei ( ) ou Eij ( )).
(xi) det (AB) = det A det B.
(xii) det (A1 A2 :::Al ) = det A1 det A2 ::: det Al , em que A1 ; A2 ; :::; Al so l (l 2 N) matrizes
do tipo n n.
1
.
det A

(xiii) Se A fr invertvel det (A 1 ) =

(xiv) det (AB) = 0 ) det A = 0 ou det B = 0.


(xv) det (AB) = det (BA).
Exemplo 23.
9
7
5
3
1

7
7
5
3
1

5
5
5
3
1
1
0
0
0
0

3
3
3
3
1
1
2
0
0
0

1
1
1
1
1
1
2
2
2
2

1
2
2
4
4

1
3
5
7
9
1
2
2
4
6

1
3
5
7
7

1
3
5
5
5
1
0
0
0
0

1
3
3
3
3

1
1
1
1
1

1
2
0
0
0

1
1
1
1
1

1
2
2
0
0

1
2
2
2
2

1
2
2
2
4

1
3
3
3
3

Observao 14. Sendo A e B matrizes do tipo n


jA + Bj =
6 jAj + jBj

Por exemplo, se n par, A = I e B =


jA + Bj = 0 6= 2

jA

1
3
5
5
5
1
0
0
0
0

1
3
5
7
7
1
2
0
0
0

1
3
5
7
9
1
2
2
0
0

1
2
2
2
0

1
0
0
0
0
1
2
2
2
2

Bj =
6 jAj

jBj .

1 + ( 1)n = jAj + jBj :

36

1
2
4
4
4

1
2
4
6
6

1
2
4
6
8

= 24 = 16.

n, em geral:

I, tem-se

n par

1
2
2
2
2

Denio 27. Seja A = (aij ) uma matriz do tipo n n, com n > 1. Seja a0ij =
( 1)i+j det Aij onde Aij o menor-ij da matriz A. Chama-se a a0ij o cofactor-ij da matriz
A e matriz cof A = (a0ij ) do tipo n n, com n > 1, a matriz dos cofactores de A.
Teorema 20. Para qualquer matriz A do tipo n

n, com n > 1, tem-se

A (cof A)T = (cof A)T A = (det A) I.


Se det A 6= 0 ento A invertvel e
A

B 1
C
1
(cof A)T = B
( 1)j+i det Aji C
@
A
det A
|det A
{z
}
entrada (i;j) de A

Exemplo 24. (i) Seja A =

A
Note que ad

.
n n

a b
c d

tal que det A 6= 0. Ento A invertvel e

ad

bc

d
c

b
a

bc = det A.

(ii) Podemos usar o teorema anterior para calcular no s a inversa de uma matriz (no
singular) mas tambm (e sobretudo) entradas concretas dessa inversa. Seja
2
3
1 0 3
A = 4 4 5 6 5.
7 8 9
A entrada (1; 2) da matriz A
(A 1 )12 =

1
(cof A)T
det A

dada por

=
12

1
( 1)2+1 det A21 =
det A

1
12

0 3
8 9

det

Note que apesar da entrada (1; 2) de A ser nula, a entrada (1; 2) de A

2.

no nula.

(iii) Para calcular A 1 a partir do teorema anterior, preciso calcular (cof A)T . Assim,
usando por exemplo A da alnea anterior, tem-se
2
3
4 5
5 6
4 6
6
7 2
3
8 9
7 9
7 8
6
7
3
6
3
6
0 3
1 3
1 0 7
7 = 4 24
12
8 5
cof A = 6
6
8 9
7 9
7 8 7
6
7
15 6
5
4
5
0 3
1 3
1 0
5 6
4 6
4 5
37

pelo que

3
(cof A)T = 4 6
3

e assim
A

1
(cof A)T =
det A

De facto

2
4

1
4
1
4

1
2

3
1 4
6
12
3
5
4

2
1

1
2
5
12

2
3

3
15
6 5
5

24
12
8

24
12
8

32

3 2
15
6 5=4
5

1
4
1
4

1
2

5
4

2
1

1
2
5
12

2
3

3 2
3
1 0 3
1 0 0
54 4 5 6 5 = 4 0 1 0 5.
7 8 9
0 0 1

5.

Teorema 21. (Regra de Cramer.) Seja A uma matriz do tipo n n tal que A no
singular. Ento a nica soluo do sistema de equaes lineares AX = B dada por
X = A 1B =
Isto , sendo X =

x1 ::: xn

eB=

1
(cof A)T B.
det A
T

b1 ::: bn

n
det (Bj )T
1 X 0
xj =
a bi =
det A i=1 ji
det A

tem-se, para j = 1; :::; n,

det Bj
,
det A

onde Bj a matriz obtida de A substituindo a coluna j de A pela matriz coluna B dos


termos independentes.
Exemplo 25. O sistema de equaes lineares
8
< y + 2z = 8
4x + 2y z = 7
:
x z=1

pode ser resolvido usando a regra de Cramer:

x=

8
7
1
0
4
1

1
2
0
1
2
0

2
1
1
2
1
1

= 14,

y=

0
4
1
0
4
1

8
7
1
1
2
0

2
1
1
2
1
1

38

18

z=

0
4
1
0
4
1

1
2
0
1
2
0

8
7
1
2
1
1

= 13.

2a Ficha de exerccios para as aulas de problemas


1. Classique quanto paridade as seguintes permutaes de nmeros de 1 a 6:
(i) (312645)

(ii) (234516)

(iii) (654321)

(iv) (123456)

(v) (546321)

(vi) (453261)

(vii) (634125)

(viii) (123465)

2. Na expresso do determinante de uma matriz do tipo 6


cada uma das seguintes parcelas:

6 diga qual o sinal que afecta

(i) a23 a31 a42 a56 a14 a65

(ii) a16 a25 a34 a43 a52 a61

(iii) a54 a45 a63 a32 a26 a11

(iv) a16 a23 a34 a41 a62 a55

3. Verique que
(i)

0
0 a13
0 a22 a23
a31 a32 a33

(iii)

6
det 6
4

a13 a22 a31

0 an
an1

(ii)

a1n
an 1 n
ann

12

0
0
0
0
0 a23
0 a32 a33
a41 a42 a43

a14
a24
a34
a44

= a14 a23 a32 a41

7
n 1
7 = ( 1)n 2 a1n :::an
5

1 2 an1

4. Calcule os seguintes determinantes e diga quais so as matrizes singulares:


p
1 2
18563 18573
1 + p2 2
(i)
(ii)
(iii)
3 4
21472 21482
2+ 3 1
(iv)

(vii)

(xi)

(xiv)

cos
sen
2
5
2

sen
cos
1
1
3

2
0
0
0

2
1
0
0

12
22
32
42

22
32
42
52

(v)

1
3
2

(viii)

8
2
3
0
32
42
52
62

6
0
23
5
42
52
62
72

2 0 1
5 3 0
5 1 2

8 12
5
1
2 1

8
3
1

1
1
2
2

3
2
1
0

(xii)

(xv)

0
0
0
0
5

4
0
0
0
0

39

0
0
0
3
0

(vi)

(ix)

1
1
1
2
0
2
0
0
0

0
0
1
0
0

1
1
1
0

2
5
2

1 2 3
4 5 6
7 8 9

0
0
0
2
a
b
0
0
0

0
a
b
0
0

2
3
1
0
1
0
0

(x)

(xiii)

(xvi)

3
1
1

1
0
1
0

0
0
1
2
0
0
a
b
0

p
p3
2

0
1
0
1

0
3
2
8
0
0
0
a
b

b
0
0
0
a

0
0
1
0
5
23
0
6

7
1
(xvii)
2
2

(xx)

4
2
1
2

2 1 0
7 3 0
3 1
3 9 0

0
0
3
0

5
2
8
4

9
5
8
6
7

(xviii)

0
3
0
0
0

6
0
4
3
5

0
2
1
0

4
1
1
1
2

1 n

(xxi)

5. (i) Verique que a matriz

2
6
6
6
6
4

0
e
0
0
0

..
.
..
.
..

2
1
a
0
f
0
0

0
b
0
g
0

2
1
0
0
c
0
h

0
0
0
d
0

7
0
(xix) 0
0
5

2
2
1
3
3

4
0
0
0
3

1
0
5
3
7

3
2
8
9

2 1
2
.
. ..
2
1

1
..
.
1
1

3
7
7
7
7
5

no invertvel para quaisquer a; b; c; d; e; f; g; h 2 R.

(ii) Diga, para que valores de a; b; c; d; e; f; g; h; i; j 2 R, invertvel a seguinte matriz


2
3
0 a 0 0 0 0
6 f 0 b 0 0 0 7
6
7
6 0 g 0 c 0 0 7
6
7
6 0 0 h 0 d 0 7
6
7
4 0 0 0 i 0 e 5
0 0 0 0 j 0

6. Determine todos os valores do escalar para os quais a matriz A


I no invertvel,
onde A dada por:
2
3
2
3
2
3
1
1
1 0
2
1 1 1
0 3
6
7
1
2 5 (iii) 4 1 1 1 5 (iv) 4 ... . . . ... 5
(i)
(ii) 4 0
2
1
2
2 0
1 1 1
1
1 n n

7. Indique trs matrizes A do tipo 2

2 tais que tr A =

= det A.

8. Seja
2

6
A =6
4

1
0
1

0
1
0

1
0

a) Diga, justicando, quais so os valores de


b) Seja n 2 N. Calcule det (A0 )n + (A0 )n+2 .
c) Considerando os valores de
da matriz inversa de A .

3
1
0 7
7 ; com
1 5

2R

para os quais A invertvel.

para os quais A invertvel, calcule a entrada (3; 1)

40

6
0
1
p0
3

9. Use a frmula de inverso de matrizes para inverter:


2
3
1 1 1
1 2
(i)
(ii) 4 0 1 1 5
(iii)
3 4
0 0 1

10. Sejam

3
6 1
A=6
4 0
3

(i) Sem calcular A

2
0
9
1

0
0
2
0

3
2
3 7
7
0 5
2

1
4 1
0

1
6 4
B=6
4 0
0

0
0
1
1

e B 1 , determine as entradas (2; 2) de A

(ii) Verique que det (A + B) 6= det A + det B e det (A

3
4
3 5
0

0
1
6
0
1
0
2
1

3
2
0 7
7.
3 5
2

e (2; 3) de B 1 .

B) 6= det A

det B.

11. Use a regra de Cramer para calcular as solues dos sistemas:


8
< x+y =1
2x + 3y = 1
2x + z = 1
(i)
(ii)
5x + 7y = 3
:
x + 2y + 2z = 1
2
3
2
3
1 0 1
9
8
1
3 0 5.
12. Sejam C = 4 2 3 2 5 e D = 4 7
0 1
2
2 0
0
Verique que C e D so invertveis e calcule:
(i) det (2C

(iv) det C T 12 C
2C T

(vi) det

(ii) det C 3 (2C)

)
2

(iii) det

C T 2C

(v) det (C 2 + 2D)


1

2 3
D
3

Sugesto: Sejam m 2 N,

DT

escalar, A; B e S matrizes n

n com S invertvel, tem-se

(a) det (AB) = (det A) (det B)


(b) det ( B) =
det B
(c) det AT = det A
1
m
(e) ( B)T = B T
(f) S m = (S 1 )
(d) det (A 1 ) =
det A
p
13. Sejam A e B matrizes 3 3 tais que det A = 3 e det B = 12 . Calcule det(2AT B 3 ).

14. Sejam a; b; c; d; e; f 2 R. Sabendo que

(i)

d e f
g h i
a b c

(ii)

a
2d
g

b
c
2e 2f
h
i

b
e
h
y

0
0
0
1

(iii)

2i

(iv)

2h
2g
f 3c e 3b d 3a
c
b
a

a
d
g
x

(v)

41

c
f
i
z

= 5; calcule:

a+d b+e c+f


d
e
f
g
h
i

a g d
b h e
c i f

a b c
2 1 0
1 2 1

15. Sejam a; b; c 2 R. Sabendo que


(i)

a
6

b
3
1

1
2

a
(iii)

c
0

(ii)
2 c

3
2

1
1
1

1
1
1
2
1
0
3a + 1 3b + 2 3c + 1

(iv)
1

16. Sejam ;

a
b
c
2a + 2 2b + 1 2c
a+1 b+2 c+1

1
2

1 b
3
1

2 R. Sabendo que

= 1; calcule:

2
1
+

1
+1
+1
+1
+1
+1

1
2

= 1; calcule

1
2
+2
+2
+2
+2

1
2
3
+3
+3
+3

1
2
3
4
+4
+4

1
2
3
4
5
+5

17. Seja

2 R. Verique que

18. Seja

2 R. Calcule o determinante da seguinte matriz do tipo n


2
3
6
6
6
6
6
6
6
6
4

19. Sejam

1
..
.

6= 0 e A = (aij )n

+1

1
..
.

+1

1
n.

...
..

1
..
.

...

..
.

+1
1

1
+1

Mostre que det A = det

i j

2
+

n.

7
7
7
7
7
7
7
7
5

aij .

3
1 a a2
20. Que condies devem os parmetros reais a; b e c vericar para que a matriz 4 1 b b2 5
1 c c2
seja invertvel?
21. Verique que
2
1 1
(i) det 4 x1 y1
x2 x2
2
(x1 )3
6 (x2 )3
(ii) det 6
4 (x3 )3
(x4 )3

3
1
y1 5 = (y1
y2

(x1 )2
(x2 )2
(x3 )2
(x4 )2

x1
x2
x3
x4

x1 ) (y2

3
1
1 7
7 = (x1
1 5
1

x2 )

x2 ) (x1

42

x3 ) (x1

x4 ) (x2

x3 ) (x2

x4 ) (x3

x4 )

22. Mostre que:


(i)

b+c c+a b+a


a
b
c
1
1
1

(iii)

a1 + b 1 a1
a2 + b 2 a2
a3 + b 3 a3

(iv)

a1 b 1 a1 + b 1 + c 1
a2 b 2 a2 + b 2 + c 2
a3 b 3 a3 + b 3 + c 3

23. Verique que

b1 c 1
b2 c 2
b3 c 3

=0

b1 + c 1 b2 + c 2 b3 + c 3
c 1 + a1 c 2 + a2 c 3 + a3
a1 + b 1 a2 + b 2 a3 + b 3

(ii)

a1 b1 c1
2 a2 b 2 c 2
a3 b 3 c 3

a1 b 1 c 1
a2 b 2 c 2
a3 b 3 c 3

a1 + b1 c1 + d1
a2 + b2 c2 + d2

a1 c 1
a2 c 2

a1 c 1
b2 d2

a1 b 1 c 1
a
= 2 2 b2 c 2
a3 b 3 c 3

b1 d1
a2 c 2

b1 d1
:
b2 d2

24. Sem calcular explicitamente o determinante, mostre que para x = 0 e x = 2 se tem


x2 x
2 1
0 0

2
1
5

= 0.

25. Sem calcular o determinante, diga qual o coeciente de x3 na expresso


2x
1
3
9

x
x
2
8

1
1
x
7

x
4
x
x

x
x
4
x

x
x
x
4

2
1
.
1
x

26. Resolva as seguintes equaes.


(i)

1
0
1

x 1
1 1
0 2

=0

(ii)

x
x
x
x

=0

(iii)

x
1
1
1

27. Sabendo que 533; 715 e 871 so mltiplos de 13, justique que

1
x
1
1

1
1
x
1

1
1
1
x

5 3 3
7 1 5
8 7 1

=0

tambm

mltiplo de 13, sem calcular o determinante.


28. Sem calcular o determinante, verique que

2
1
3

1 8
0 10
7 4

mltiplo de 5.

29. Seja A = (aij )n n com n mpar e tal que aij + aji = 0, para todos os i; j = 1; :::; n:
Mostre que A no invertvel. Isto , toda a matriz anti-simtrica de ordem mpar
no invertvel.
30. Mostre que se uma matriz fr ortogonal ento o seu determinante ou 1 ou
se a matriz fr unitria?

43

1. E

Espaos lineares (ou Espaos vectoriais)


Denio 28. Um conjunto no vazio V um espao linear (real) se existirem duas
operaes associadas a V , uma soma de elementos de V e um produto de escalares (nmeros
reais) por elementos de V , com as seguintes propriedades:
(a) (Fecho da soma). Para quaisquer u; v 2 V
u + v 2 V:
(b) (Fecho do produto por escalares). Para quaisquer

2Reu2V

u 2 V:
(c) (Comutatividade da soma). Para quaisquer u; v 2 V ,
u + v = v + u:
(d) (Associatividade da soma). Para quaisquer u; v; w 2 V ,
u + (v + w) = (u + v) + w:
(e) (Elemento neutro da soma). Existe um elemento de V designado por 0 tal que, para
qualquer u 2 V ,
u + 0 = u:
(f) (Simtrico). Para cada (qualquer) u 2 V existe v 2 V tal que
u + v = 0:
A v chama-se o simtrico de u e denota-se por

u.

(g) (Associatividade do produto por escalares). Para quaisquer ;


( u) = (

2R e u2V,

) u:

(h) (Distributividade em relao soma de vectores). Para quaisquer

2 R e u; v 2 V ,

(u + v) = u + v:
(i) (Distributividade em relao soma de escalares). Para quaisquer ;
( + ) u = u + u:
(j) Para qualquer u 2 V ,

1u = u:
44

2R e u2V,

Observao 15. Aos elementos de V chamaremos vectores.


Exemplo 26. Exemplos de espaos lineares. Seja

2 R.

(i) Rn = f(x1 ; :::; xn ) : x1 ; :::; xn 2 Rg, com as operaes usuais:


(u1 ; :::; un ) + (v1 ; :::; vn ) = (u1 + v1 ; :::; un + vn ),
(u1 ; :::; un ) = ( u1 ; :::; un ).
(ii) Mm n (R) (conjunto de todas as matrizes reais do tipo m
(usuais): A + B e A.

n), com as operaes

(iii) O conjunto de todas as funes reais de varivel real denidas num conjunto S
com as operaes usuais:
(f + g)(x) = f (x) + g(x),

R,

( f )(x) = f (x).
(iv) O conjunto P = fa0 + a1 t + ::: + as ts : a0 ; a1 ; :::; as 2 R e s 2 N0 g de todos os polinmios
reais de varivel real, com as operaes usuais.
(v) Seja n 2 N xo. O conjunto Pn = fa0 + a1 t + ::: + an tn : a0 ; a1 ; :::; an 2 Rg de todos
os polinmios reais de varivel real e de grau menor ou igual a n, com as operaes usuais.
(a0 + a1 t + ::: + an tn ) + (b0 + b1 t + ::: + bn tn ) = a0 + b0 + (a1 + b1 ) t + ::: + (an + bn ) tn
(a0 + a1 t + ::: + an tn ) = a0 + ( a1 ) t + ::: + ( an ) tn .

Observao 16. Existem espaos lineares com operaes no usuais:


(i) O conjunto dos nmeros reais R, com a soma denida por
u

v = u + v + 1,

e o produto por escalares denido por


u= u+
um espao linear. (Neste caso o elemento neutro

1,
1.)

(ii) O conjunto dos nmeros reais maiores do que zero, com a soma denida por
u

v = uv,

e o produto por escalares denido por


u=u ,
um espao linear. (Neste caso o elemento neutro 1.)
45

Observao 17. Alteraes nos conjuntos considerados anteriormente podem resultar


em conjuntos que no so espaos lineares.
(i) O conjunto f(x; y) 2 R2 : x 0 e y 0g, com as operaes usuais, no um espao
linear. Por exemplo, os simtricos no esto no conjunto.
(ii) O conjunto V = fa0 + a1 t + ::: + an tn : a0 ; a1 ; :::; an 2 R e an 6= 0g de todos os polinmios
reais de grau igual a n, com as operaes usuais, no um espao linear. Por exemplo, para
n > 1:
tn ; tn + t 2 V , mas tn + ( tn + t) = t 2
= V.
(iii) O conjunto U = ff : R ! R tais que f (1) = 2g, com as operaes usuais, no
um espao linear. Por exemplo, se f1 ; f2 2 U ,
(f1 + f2 ) (1) = f1 (1) + f2 (1) = 2 + 2 = 4 6= 2.
Logo, f1 + f2 2
= U.
Denio 29. Seja V um espao linear. Diz-se que S um subespao de V se S um
subconjunto de V e se S, com as operaes de V , fr um espao linear.
Observao 18. No entanto, para mostrar que um certo conjunto S V um subespao
do espao linear V , no ser necessrio vericar as 10 propriedades da denio de espao
linear, como se pode ver no seguinte teorema.
Teorema 22. Um subconjunto no vazio S de um espao linear V um subespao de
V se e s se:
(i) Para quaisquer u; v 2 S tem-se u + v 2 S.
(ii) Para quaisquer

2 R e u 2 S tem-se u 2 S.

Exemplo 27. Exemplos de subespaos:


(i) Os nicos subespaos do espao linear R, com as operaes usuais, so f0g e R.
(ii) Os subespaos do espao linear R3 , com as operaes usuais, so: f(0; 0; 0)g, R3 ,
todas as rectas que passam pela origem e todos os planos que passam pela origem.
(iii) O conjunto de todas as matrizes (reais) triangulares superiores (do tipo n
subespao do espao linear Mn n (R), com as operaes usuais.

n) um

(iv) O conjunto de todas as funes reais denidas e contnuas em I


R (I um
intervalo) um subespao do espao linear de todas as funes f : I ! R, com as operaes
usuais.

46

(v) Seja A uma matriz (real) do tipo m

n. O conjunto

C(A) = fb 2 Rm : Au = b tem pelo menos uma soluo ug


um subespao do espao linear Rm , com as operaes usuais, ao qual se d o nome de
espao das colunas de A.
(vi) Seja A uma matriz (real) do tipo m

n. O conjunto

N (A) = fu 2 Rn : Au = 0g
um subespao do espao linear Rn , com as operaes usuais, ao qual se d o nome de
espao nulo ou ncleo de A.
Observao 19. (i) Se A invertvel ento N (A) = f0g.
(ii) Se N (A) = f0g ento A invertvel.
Denio 30. Seja S um subconjunto no vazio de um espao linear V . Diz-se que um
vector u combinao linear nita dos elementos de S, se existir um no nito de elementos
de S, u1 ; :::; uk , e de escalares 1 ; :::; k tais que
u=

1 u1

+ ::: +

k uk

k
X

i ui .

i=1

Ao conjunto de todas as combinaes lineares nitas de elementos de S chama-se expanso


linear de S e designa-se por L(S). Isto ,
L(S) = f 1 u1 + ::: +

k uk

1 ; :::;

2 Rg ,

no caso do corpo dos escalares ser R. Se S o conjunto vazio ?, escreve-se L(?) = f0g.
Teorema 23. Seja S um subconjunto no vazio de um espao linear V . A expanso
linear L(S) de S o menor subespao de V que contm S. Deste modo, a L(S) tambm se
chama o subespao gerado por S, e diz-se que S gera L(S) ou ainda que S um conjunto
gerador de L(S).
Observao 20. (i) Seja S e T dois subconjuntos no vazios de um espao linear V ,
com S T . Se L(S) = V ento L(T ) = V .
(ii) Todo o subespao do espao linear Rn pode ser escrito como o ncleo de uma matriz.
Exemplo 28. (i) O espao linear R2 gerado por qualquer dos seguintes conjuntos de
vectores:
f(1; 0); (0; 1)g, f(1; 2); ( 1; 11)g e f(23; 8); (6; 14)g.

47

(ii) O subespao f(x; y) 2 R2 : y = 2xg do espao linear R2 gerado por qualquer dos
seguintes conjuntos de vectores:
f(1; 2)g, f( 2; 4)g e f(77; 154)g.
(iii) O espao linear Pn de todos os polinmios reais de varivel real e de grau menor ou
igual a n, gerado por qualquer dos seguintes conjuntos de vectores:
f1; t; t2 ; :::; tn g, f1; 1 + t; (1 + t)2 ; :::; (1 + t)n g e f1;

t t2
tn
; ; :::; g.
1! 2!
n!

(iv) O espao linear P de todos os polinmios reais de varivel real, gerado pelo conjunto
innito de vectores:
f1; t; t2 ; :::g.
(v) Seja U o espao linear de todas as funes reais com primeira derivada contnua em
R (isto , pertencentes a C 1 (R)) e tais que f 0 (x) = af (x) (em R) com a 2 R. Ento U
gerado pela funo g (x) = eax , tendo-se U = L (fgg).
(vi) Seja A uma matriz (real) do tipo m

n. O espao das colunas de A,

C(A) = fb 2 Rm : Au = b tem pelo menos uma soluo ug ,


o subespao (do espao linear Rm ) gerado pelas colunas de A, uma vez que:
2
3 2
32
2
2
3
3
3
b1
a11
a1n
u1
a11
a1n
6 .. 7 6 ..
.. 7 6 .. 7 = u 6 .. 7 + ::: + u 6 .. 7 .
4 . 5=4 .
14
n4
. 54 . 5
. 5
. 5
bm
am1
amn
un
am1
amn
(vii) Seja A uma matriz (real) do tipo m n. Ao subespao linear de Rn gerado pelas
linhas de A d-se o nome de espao das linhas de A e designa-se por L(A).
2
3
2
3
1
3 1
1 2
0 0 0
2 0
4 5, D =
(viii) A =
, B = 4 0 0 7 5, C = 4 2
.
0 0 0
0
1
0 0 0
2 4
C(A) = f(0; 0)g,

N (A) = R3 ,

L(A) = f(0; 0; 0)g.

C(B) = L (f(1; 0; 0) ; (1; 7; 0)g) , N (B) = L (f(3; 1; 0)g) ;


C(C) = L (f( 1; 2; 2)g) ;

N (C) = L (f(2; 1)g) ;

C(D) = L (f(2; 0) ; (0; 1)g) , N (D) = f(0; 0)g;

(ix) Seja U = fA 2 M3 2 (R) : a12 = a21 = a32 = 0


A 2 U,
2
3 2
3
2
a11 a12
2a31 0
a22 5 = a31 4
A = 4 a21 a22 5 = 4 0
a31 a32
a31
0
48

L(B) = L (f(1; 3; 1) ; (0; 0; 7)g) .


L(C) = L (f( 1; 2)g) :

L(D) = L (f(2; 0) ; (0; 1)g) .


e a11 + 2a31 = 0g. Tem-se, para

3
2
3
2 0
0 0
0 0 5 + a22 4 0 1 5 ,
1 0
0 0

com a31 ; a22 2 R. Logo,

082
3 2
391
2 0
0 0 =
<
U = L@ 4 0 0 5;4 0 1 5 A.
:
;
1 0
0 0

(x) Seja U = fp(t) = a0 + a1 t + a2 t2 2 P2 : p(1) = p(0)g. Tem-se, para p(t) 2 U ,


p(1) = p(0) , a0 + a1 + a2 = a0 , a1 + a2 = 0 , a1 =
Logo, p(t) = a0

a2 .

a2 t + a2 t2 = a0 1 + a2 ( t + t2 ), com a0 ; a2 2 R. Assim,
1; t + t2

U =L

Teorema 24. Se U e V so subespaos do espao linear W , ento:


(i) O conjunto U \ V um subespao linear de W .
(ii) O conjunto U + V = fu + v : u 2 U e v 2 V g um subespao de W . o
menor subespao de W que contm U [ V . O conjunto U [ V em geral no um subespao.
Escreve-se U + V = L(U [ V ).
Exemplo 29. (i) Em R3 , considere os subespaos:
U = f(x; y; z) 2 R3 : x + y

2z = 0g e V = L (f(1; 1; 1); (1; 2; 1)g) .

Seja v 2 V , ento
v = (1; 1; 1) + (1; 2; 1) = ( + ;
com ;

+2 ;

+ ),

2 R. Para que v esteja tambm em U preciso que:


( + )+( +2 )

A ltima equao equivalente a 4 +


U \ V = f( 3 ; 7 ; 5 ) :

=0,

2(
=

+ ) = 0.
4 . Logo,

2 Rg = f ( 3; 7; 5) :

2 Rg = L (f(3; 7; 5)g) .

(ii) Em R3 , considere os subespaos:


U = L (f(1; 1; 1); (1; 2; 2)g)

e V = L (f(2; 1; 1); ( 1; 1; 3)g) .

Seja v 2 U , ento
v = (1; 1; 1) + (1; 2; 2) = ( + ;
com ;

+2 ;

+ 2 ),

2 R. Para que v esteja tambm em V preciso que:


( + ;

+2 ;

+ 2 ) = (2; 1; 1) + ( 1; 1; 3) = (2
49

; + ; + 3 ),

com ;

2 R. Deste modo,

8
<
:

=2
+2 = +
+2 = +3 .

Considerando a matriz aumentada tem-se


2
2
3
1 1 j 2
1 1 j
4 1 2 j
4
5
+
0 3 j
!
L1 +L2 !L2
1 2 j
+3
0 1 j
L1 +L3 !L3
Logo,

2
3
+4

8
<

+ =2
=
:
0= 2 +4 .

Assim,

3
5

8
<

1
L +L3 !L3
3 2

1 1 j
4 0 3 j
0 0 j

2
3
2 +4

5.

=
=2
=2 .

(1; 1; 1) + (1; 2; 2) = (1; 1; 1) + 2 (1; 2; 2) = (3 ; 3 ; 5 ) = (3; 3; 5).


Logo,
U \ V = f(3 ; 3 ; 5 ) :

2 Rg =f (3; 3; 5) :

2 Rg = L (f(3; 3; 5)g) .

Observao 21. Neste exemplo (ii), os subespaos U e V poderiam ter sido apresentados
inicialmente na forma:
U = f(x; y; z) 2 R3 : 4x + y
uma vez que
2
3
1 1 x
4 1 2 y 5
1 2 z

4
x
3

1
y
3

L1 +L2 !L2
L1 +L3 !L3

e logo (x; y; z) 2 U , z

U = f(x; y; z) 2 R3 : 4x + y
pois sendo y =

3z = 0g e V = f(x; y; z) 2 R3 : 2x
3
1 1
x
4 0 3 x+y 5
0 1 z x
= 0 , 4x + y

1
L +L3 !L3
3 2

7y + 3z = 0g,

3
1 1
x
4 0 3
5
x+y
4
1
0 0 z 3x 3y

3z = 0. Por outro lado,

3z = 0g = L (f(1; 4; 0); (0; 3; 1)g) = L (f(1; 1; 1); (1; 2; 2)g)

4x + 3z,

U = f(x; 4x+3z; z) : x; z 2 Rg = fx(1; 4; 0)+z(0; 3; 1) : x; z 2 Rg = L (f(1; 4; 0); (0; 3; 1)g) .


De facto, como
(1; 4; 0)) = 2(1; 1; 1)

(1; 2; 2) e (0; 3; 1) =

(1; 1; 1) + (1; 2; 2)

(1; 1; 1) = (1; 4; 0) + (0; 3; 1) e (1; 2; 2) = (1; 4; 0) + 2(0; 3; 1)


ou seja
2

3 2
3
1 1
1 0
4 1 2 5=4 4 3 5
1 2
0 1

1 1
1 2

3
1 1
,4 1 2 5
1 2
50

2
1

1
1

3
1 0
=4 4 3 5
0 1

2
1
1 1
=
, tem-se U = L (f(1; 4; 0); (0; 3; 1)g) = L (f(1; 1; 1); (1; 2; 2)g).
1 1
1 2
Analogamente se mostra que

em que

V = f(x; y; z) 2 R3 : 2x 7y+3z = 0g = L (f(7; 2; 0); ( 3; 0; 2)g) = L (f(2; 1; 1); ( 1; 1; 3)g) .


(iii) Seja U o subespao de Mn n (R) das matrizes triangulares superiores e seja V o
subespao de Mn n (R) das matrizes triangulares inferiores. Ento
U + V = Mn

n (R)

U \ V = subespao das matrizes diagonais.

(iv) Sejam U = L(f(1; 0)g) e V = L(f(0; 1)g) subespaos de R2 . O conjunto


U [ V = f(x; y) 2 R2 : x = 0 _ y = 0g
= U [ V . No entanto, tem-se U + V = R2 .
no um espao linear pois (1; 0) + (0; 1) = (1; 1) 2
| {z } | {z }
2U

2V

Teorema 25. Se U e V subespaos do espao linear W , ento U [ V subespao de W


se e s se U V ou V
U.
Teorema 26. Sejam W1 e W2 subespaos de um espao linear V tais que W1 \W2 = f0g:
Se V = W1 + W2 ento todo o vector v 2 V pode ser escrito de modo nico na forma
v = w1 + w2
com w1 2 W1 e w2 2 W2 . Neste caso escreve-se V = W1
directa dos espaos W1 e W2 .
Teorema 27. Seja A 2 Mm

n (R).

W2 e diz-se que V a soma

Tem-se C(A) = L(AT ) e L(A) \ N (A) = f0g.

Observao 22. Seja A 2 Mm n (R). Se A0 fr a matriz em escada que se obtem de A


por aplicao do mtodo de eliminao de Gauss, tem-se
C(A) 6= C(A0 ).
Teorema 28. Seja A 2 Mm n (R). O espao das linhas L(A) e o ncleo N (A) mantmse invariantes por aplicao do mtodo de eliminao de Gauss. Isto , sendo A0 a matriz
em escada que se obtem de A por aplicao desse mtodo, tem-se
L(A) = L(A0 )

51

N (A) = N (A0 ).

Independncia linear
Denio 31. Seja V um espao linear. Seja
S = fv1 ; :::; vk g

V:

Diz-se que o conjunto S linearmente dependente se e s se algum dos vectores de S se


escrever como combinao linear dos restantes, isto , se e s se existir algum i 2 f1; :::; kg
e escalares 1 ; :::; i 1 ; i+1 ; :::; k 2 R tais que
vi =

1 v1

+ ::: +

i 1 vi 1

i+1 vi+1

+ ::: +

k vk .

Denio 32. Seja V um espao linear. Seja


S = fv1 ; :::; vk g

V:

Diz-se que o conjunto S linearmente independente se e s se nenhum dos vectores de


S se puder escrever como combinao linear dos restantes, isto , se e s a nica soluo do
sistema homogneo
1 v1 + ::: + k vk = 0
fr a soluo trivial, ou seja, 1 = ::: = k = 0. No caso em que V = Rn , sendo A a
matriz cujas colunas so os vectores de S V , diz-se que S linearmente independente
se e s se N (A) = f0g.
Teorema 29. Seja A0 uma matriz em escada de linhas.
(i) As colunas de A0 que contm pivots so linearmente independentes.
(ii) As linhas no nulas de A0 so linearmente independentes.
(iii) O no de linhas independentes e o no de colunas independentes (de A0 ) so ambos
iguais caracterstica de A0 .
Observao 23. (i) Assim, atendendo ao teorema anterior, a independncia linear de
S = fv1 ; v2 ; :::; vk g V (espao linear) pode ser decidida aplicando o mtodo de eliminao
matriz A cujas colunas so os vectores de S, de modo a coloc-la em escada de linhas.
Sendo A0 essa matriz em escada, tem-se
N (A) = N (A0 ) (*).
Uma vez que as colunas de A0 que contm pivots so linearmente independentes ento, devido
a (*), as colunas de A nas posies correspondentes tambm sero linearmente independentes.
(ii) Em R, quaisquer dois vectores so linearmente dependentes.
(iii) Em R2 , dois vectores so linearmente independentes se no forem colineares.
52

(iv) Em R3 , trs vectores so linearmente independentes se no forem coplanares.


(v) Qualquer conjunto que contenha o vector nulo (elemento neutro) linearmente dependente. Em particular, o conjunto f0g, formado apenas pelo vector nulo, linearmente
dependente.
(vi) O conjunto vazio ? linearmente independente.

Teorema 30. Sejam S1 e S2 dois subconjuntos nitos de um espao linear, tais que
S1 S2 .
(i) Se S1 linearmente dependente ento S2 tambm linearmente dependente.
(ii) Se S2 linearmente independente ento S1 tambm linearmente independente.

S1

Observao 24. Sejam S1 e S2 dois subconjuntos nitos de um espao linear, tais que
S2 .

(i) Se S2 fr linearmente dependente ento S1 tanto pode ser linearmente dependente


como linearmente independente.
(ii) Se S1 fr linearmente independente ento S2 tanto pode ser linearmente dependente
como linearmente independente.

Exemplo 30.
2
1
A=4 0
2

Seja S = f(1; 0; 2); (2; 0; 4); (0; 1; 2)g. Tem-se


2
3
2
3
3
2 0
1 2 0
1 2 0
4 0 0 1 5
4 0 0 1 5 = A0 :
0 1 5
!
!
2L1 +L3 !L3
2L2 +L3 !L3
4 2
0 0 2
0 0 0

Logo, como apenas existem dois pivots e portanto uma varivel livre, as trs colunas de A
so linearmente dependentes, isto , o conjunto S linearmente dependente. O subconjunto
de S:
f(1; 0; 2); (2; 0; 4)g

tambm linearmente dependente. No entanto, uma vez que a 1a e 3a colunas de A so


independentes pois correspondem s colunas da matriz em escada A0 que contm os pivots,
o subconjunto de S:
f(1; 0; 2); (0; 1; 2)g
linearmente independente.

53

Bases e dimenso de um espao linear


Denio 33. Chama-se base de um espao linear V a qualquer subconjunto B de V
que verique as duas condies:
(i) B gera V , isto , L(B) = V .
(ii) B linearmente independente.
Teorema 31. Qualquer espao linear V 6= f0g tem pelo menos uma base.
Observao 25. Qualquer espao linear V 6= f0g tem um no innito de bases. Por exemplo, se B = fu1 ; :::; uk g fr uma base de V ento para cada 6= 0 o conjunto f u1 ; :::; uk g
tambm uma base de V .
Teorema 32. (i) Seja V 6= f0g um espao linear. Sejam p; q 2 N tais que fu1 ; :::; up g
um conjunto gerador de V e fv1 ; :::; vq g um subconjunto de V linearmente independente.
Ento p q.
(ii) Todas as bases de um espao linear V 6= f0g tm o mesmo no de vectores.
Denio 34. Chama-se dimenso de um espao linear V 6= f0g ao no de vectores de
uma base qualquer de V , e escreve-se dim V . Se V = f0g ento dim V = 0 uma vez que o
conjunto vazio ? base de f0g. Um espao linear ter dimenso nita se uma sua base tiver
um no nito de vectores.
Observao 26. A dimenso de um espao linear, isto , o no de elementos de uma sua
base igual ao no mnimo de vectores possam constituir um conjunto gerador desse espao
e tambm igual ao no mximo de vectores que possam constituir um conjunto linearmente
independente nesse espao.
Exemplo 31. (i) O conjunto f1g uma base de R, chamada base cannica ou natural
de R. Logo,
dim R = 1.
(ii) O conjunto f(1; 0); (0; 1)g uma base de R2 , chamada base cannica ou natural de
R2 . Logo,
dim R2 = 2.
(iii) O conjunto f(1; 0; 0); (0; 1; 0); (0; 0; 1)g uma base de R3 , chamada base cannica
ou natural de R3 . Logo,
dim R3 = 3.
(iv) O conjunto
1 0 0
0 0 0

0 1 0
0 0 0

0 0 1
0 0 0

;
54

0 0 0
1 0 0

0 0 0
0 1 0

0 0 0
0 0 1

uma base de M2 3 (R), chamada base cannica ou natural de M2 3 (R). Logo,


dim M2 3 (R) = 6.
(v) Tem-se
dim Rn = n e dim Mm

n (R)

= mn.

(vi) O conjunto f1; t; t2 ; :::; tn g uma base de Pn (espao linear de todos os polinmios
reais de varivel real e de grau menor ou igual a n), chamada base cannica ou natural de
Pn . Logo,
dim Pn = n + 1.

(vii) O conjunto f1; t; t2 ; :::g uma base de P (espao linear de todos os polinmios reais
de varivel real), chamada base cannica ou natural de P. Logo,
dim P = 1.
Observao 27. Seja A uma matriz do tipo m
nul A = dim N (A)

Teorema 33. Seja A uma matriz do tipo m

n. Tem-se
car A = dim L(A).

n. Tem-se

dim C(A) = dim L(A) = car A.


Dem. Suponhamos que car A = k. Sendo A0 a matriz m n em escada (reduzida)
de linhas, ento A0 tem exactamente k linhas no nulas. Sejam R1 ; :::; Rk essas linhas.
Como L(A) = L(A0 ), ento as linhas L1 ; :::; Lm de A podem ser expressas como combinaes
lineares das linhas R1 ; :::; Rk , ou seja, existem escalares cij ; com i = 1; :::; m e j = 1; :::; k tais
que
L1 = c11 R1 + ::: + c1k Rk
:::
Lm = cm1 R1 + ::: + cmk Rk
Para i = 1; :::; m, sejam aij e rij as componentes j das linhas Li e Ri respectivamente.
Assim, tem-se
a1j = c11 r1j + ::: + c1k rkj
:::
amj = cm1 r1j + ::: + cmk rkj
ou seja, matricialmente,

3
2
3
2
3
a1j
c11
c1k
6 .. 7
6 . 7
6 . 7
4 . 5 = r1j 4 .. 5 + ::: + rkj 4 .. 5 .
amj
cm1
cmk

3
a1j
6
7
Como 4 ... 5 a coluna j de A, a ltima igualdade mostra que os vectores
amj
3
2
3
2
c11
c1k
6 .. 7
6 . 7
4 . 5 ; :::; 4 .. 5
cmk

cm1

55

geram C (A). Logo, tem-se

dim C (A)

k = dim L (A) .

Deste modo, substituindo A por AT tem-se tambm


dim C AT
| {z }

dim L AT :
|
{z
}

=dim L(A)

Ou seja, tem-se
dim C (A)

dim L (A)

=dim C(A)

dim L (A)

dim C (A) .

Isto ,
dim C (A) = dim L (A) .
Teorema 34. Sejam W1 e W2 dois subespaos de dimenso nita de um espao linear
V . Ento,
dim (W1 + W2 ) = dim W1 + dim W2 dim (W1 \ W2 ) .
Teorema 35. Sejam V um espao linear de dimenso nita e W um subespao de V .
(i) Seja S = fu1 ; :::; uk g V . Se S linearmente independente ento S ser um subconjunto de uma base de V e ter-se- dim V
k.
(ii) Se dim V = n, ento quaisquer m vectores de V , com m > n, so linearmente
dependentes.
(iii) Se dim V = n, ento nenhum conjunto com m vectores de V , em que m < n, pode
gerar V .
(iv) O subespao W tem dimenso nita e dim W

dim V .

(v) Se dim W = dim V , ento W = V .


(vi) Se dim V = n, ento quaisquer n vectores de V linearmente independentes constituem uma base de V .
(vii) Se dim V = n, ento quaisquer n vectores geradores de V constituem uma base de
V.
Exemplo 32. Seja A 2 Mm

n (R).

Recorde que:

car A + nul A = n.
Como L(A) e N (A) so subespaos de Rn ento
L(A) + N (A) = L (L(A) [ N (A))
56

tambm um subepao de Rn . Por outro lado, atendendo a que


L(A) \ N (A) = f0g
tem-se
dim (L(A) \ N (A)) = 0.
Assim,
dim (L(A) + N (A)) = dim L(A) + dim N (A)
= car A + nul A 0 =
= n.

dim (L(A) \ N (A)) =

Logo
Rn = L(A)

N (A).

Exemplo 33. (i) Os seguintes conjuntos so todos os subespaos de R:


f0g e R.
(ii) Os seguintes conjuntos so todos os subespaos de R2 :
f(0; 0)g , todas as rectas que contm a origem e R2 .
(iii) Os seguintes conjuntos so todos os subespaos de R3 :
f(0; 0; 0)g , todas as rectas que contm a origem,
todos os planos que contm a origem e R3 .
Observao 28. O mtodo de eliminao de Gauss permite determinar a dimenso
e uma base quer para o espao das linhas L(A) quer para o espao das colunas C(A) de
uma matriz A. Seja A0 a matriz em escada que se obtem de A por aplicao do mtodo de
eliminao de Gauss. Ento,
(i) Uma base para L(A) ser formada pelas linhas no nulas de A0 .
(ii) Uma base para C(A) ser formada pelas colunas de A que correspondem s posies
das colunas de A0 que contm os pivots.
Exemplo 34. Seja

2
4
4
A=
6

Tem-se
2

2
4
4
A=
6

3
1 1 1
2 3 3 5
3 1 1

2L1 +L2 !L2


3L1 +L3 !L3

3
1 1 1
2 3 3 5.
3 1 1

3
2 1 1 1
4 0 0 1 1 5
0 0 4 4
57

4L2 +L3 !L3

3
2 1 1 1
4 0 0 1 1 5 = A0 .
0 0 0 0

Logo, f(2; 1; 1; 1); (0; 0; 1; 1)g uma base de L(A) e f(2; 4; 6); (1; 3; 1)g uma base de C(A).
Assim,
dim L(A) = 2 = dim C(A)
e
L(A) = L (f(2; 1; 1; 1); (0; 0; 1; 1)g) , C(A) = L (f(2; 4; 6); (1; 3; 1)g) .
Por outro lado,
8
2
3 2 39
x
0 >
>
>
>
<
6
7
6
7=
0
0
4
06 y 7
N (A ) =
(x; y; z; w) 2 R : A 4
=6 7 =
z 5 4 0 5>
>
>
>
:
;
w
0
= f(x; 2x; w; w) : x; w 2 Rg = L (f(1; 2; 0; 0); (0; 0; 1; 1)g) .
Como o conjunto f(1; 2; 0; 0); (0; 0; 1; 1)g linearmente independente e gera N (A0 ) ento
uma base de N (A0 ). Finalmente, uma vez que N (A) = N (A0 ), o conjunto
f(1; 2; 0; 0); (0; 0; 1; 1)g
uma base de N (A) e portanto dim N (A) = 2, com
N (A) = L (f(1; 2; 0; 0); (0; 0; 1; 1)g) .
R3 . Determinemos

Exemplo 35. Seja S = f1; 2; 1); (2; 1; 1); ( 1; 2; 1); (0; 1; 0)g
uma base para L(S).
Considerando a
2
1 2
1
4 2 1
2
1 1 1

matriz cujas colunas so os


3
2
0
1 2
5
4
1
0
3
!
2L1 +L2 !L2
0
0
3
L1 +L3 !L3

vectores de S, tem-se
3
2
1 0
1
5
4
0 1
0
!
L2 +L3 !L3
0 0
0

2
3
0

3
1 0
0 1 5.
0 1

Logo, S 0 = f1; 2; 1); (2; 1; 1); (0; 1; 0)g uma base de L(S). Como dim R3 = 3, ento tem-se
mesmo: L(S) = R3 e S 0 uma base de R3 .
2
6
6
4

Resoluo alternativa: Considerando a matriz cujas linhas so os vectores de S, tem-se


2
3
3
2
3
2
1 2
1
1
2
1
1 2
1
1 2
1
7
6
7
6
7
6
3 3
0
3 3 7
2
1
1 7
3 3 7
6 0
6 0
!
!
! 6
1
0 5 13 L2 +L3 !L3 4 0 0
1
2 1 5 2L1 +L2 !L2 4 0 0
0 5 L3 $L4 4 0 1
L1 +L3 !L3
0
1
0
0 1
0
0 0
0
0 0
0

Logo, S 0 = f1; 2; 1); (0; 3; 3); (0; 0; 1)g uma base de L(S). Como dim R3 = 3, ento
tem-se mesmo: L(S) = R3 e S 0 uma base de R3 .
Exemplo 36. Seja Sa;b = f1; 0; 1); (0; 1; a); (1; 1; b); (1; 1; 1)g
valores dos parmetros a e b para os quais Sa;b no gere R3 .

58

R3 . Determinemos os

7
7.
5

Considerando a matriz cujas


2
2
3
1 0 1 1
4 0 1 1 1 5
4
!
L1 +L3 !L3
1 a b 1

colunas so os vectores de S, tem-se


2
3
1 0
1
1
1 0
4 0 1
0 1
1
1 5
!
aL2 +L3 !L3
0 a b 1 0
0 0 b

Logo, Sa;b no gera R3 se e s se b

Teorema 36. (i) Seja A 2 Mm


(ii) Seja A 2 Mm
car A = n.

n (R).

1=0e

n (R).

1
1
a

3
1
1 5.
a

a = 0, isto , se e s se a = 0 e b = 1.

As colunas de A geram Rm se e s se car A = m.

As colunas de A so linearmente independentes se e s se

(iii) Seja A 2 Mn n (R). A matriz A invertvel se e s se as colunas de A (ou as linhas


de A) formarem uma base de Rn . No caso de A ser invertvel tem-se
C(A) = L(A) = Rn .
Observao 29. Seja A 2 Mm

n (R)

e considere o sistema de equaes lineares Au = b.

(i) O sistema Au = b impossvel (no tem soluo) se e s se b 2


= C(A), isto , se e s
se car A < car [A j b].
(ii) O sistema Au = b possvel e indeterminado (tem um no innito de solues) se
e s se b 2 C(A) e as colunas de A forem linearmente dependentes, isto , se e s se car A =
car [A j b] < n, isto , se e s se car A = car [A j b] e nul A 6= 0.
(iii) O sistema Au = b possvel e determinado (tem uma nica soluo) se e s
se b 2 C(A) e as colunas de A forem linearmente independentes, isto , se e s se car A =
car [A j b] = n, isto , se e s se car A = car [A j b] e nul A = 0.
Observao 30. Seja A 2 Mm

n (R)

(i) Existncia de soluo: Se m


soluo u para cada b 2 Rm se e s se

e considere o sistema de equaes lineares Au = b.


n ento o sistema Au = b tem pelo menos uma

car A = m:
(ii) Unicidade de soluo: Se m
n ento o sistema Au = b tem no mximo uma
soluo u para cada b 2 Rm se e s se car A = n, isto , se e s se
nul A = 0:
(iii) Existncia e unicidade de soluo: Se m = n ento o sistema Au = b tem
soluo nica u para cada b 2 Rm se e s se A fr invertvel.
59

Teorema 37. Seja A 2 Mn

n (R).

As seguintes armaes so equivalentes.

(i) A no singular.
(ii) A igual ao produto de matrizes elementares.
(iii) A invertvel.
(iv) AT A invertvel.
(v) nul A = 0.
(vi) car A = n.
(vii) Au = 0 tem apenas a soluo trivial u = 0.
(viii) Au = b tem soluo nica u para cada b 2 Rn .
(ix) det A 6= 0.
(x) N (A) = f0g.
(xi) As colunas de A geram Rn .
(xii) As colunas de A so independentes.
(xiii) As colunas de A formam uma base de Rn .
(xiv) As linhas de A geram Rn .
(xv) As linhas de A so independentes.
(xvi) As linhas de A formam uma base de Rn .
(xvii) A transformao linear T : Rn ! Rn denida por T (u) = Au, para u 2 Rn ,
sobrejectiva. (Num prximo captulo.)
(xviii) A transformao linear T : Rn ! Rn denida por T (u) = Au, para u 2 Rn ,
injectiva. (Num prximo captulo.)
(xix) A transformao linear T : Rn ! Rn denida por T (u) = Au, para u 2 Rn ,
bijectiva. (Num prximo captulo.)
(xx) A transformao linear T : Rn ! Rn denida por T (u) = Au, para u 2 Rn ,
invertvel. (Num prximo captulo.)
(xxi) 0 no valor prprio de A. (Num prximo captulo.)
(xxii) (N (A))? = Rn . (Num prximo captulo.)
(xxiii) (L (A))? = f0g. (Num prximo captulo.)
60

Coordenadas de um vector numa base e matriz de mudana de base


Denio 35. Seja B = fv1 ; :::; vk g uma base ordenada de um espao linear V e seja u
um vector de V . Chamam-se coordenadas do vector u na base ordenada B aos escalares
1 ; :::; k da combinao linear:
u=

1 v1

k vk .

+ ::: +

Teorema 38. Seja V um espao linear.


(i) Um conjunto B de vectores no nulos de V uma base de V se e s se todo o vector
de V puder ser escrito de modo nico como combinao linear dos vectores de B.
(ii) Se dim V = n, ento dados u; w 2 V e B = fv1 ; :::; vn g uma base ordenada de V ,
tem-se u = w se e s se as coordenadas de u e de w na base B forem iguais.
Teorema 39. Seja V um espao linear de dimenso n. Sejam B1 = fv1 ; :::; vn g e
B2 = fw1 ; :::; wn g duas bases ordenadas de V . Seja SB1 !B2 a matriz cujas colunas so as
coordenadas dos vectores de B1 em relao base B2 . Isto ,
SB1 !B2 = (sij )n

com vj =

n
X

sij wi

para todo o j = 1; :::; n.

i=1

A matriz SB1 !B2 no singular e chama-se matriz de mudana de base (da base B1 para
B2 ). Assim, se tivermos
n
X
u=
i vi ,
i=1

isto , se ( 1 ; :::; n ) forem as coordenadas do vector u na base B1 ento as coordenadas


( 1 ; :::; n ) de u na base B2 so dadas por
2
3
2
3
1

6 .. 7
6 . 7
4 . 5 = SB1 !B2 4 .. 5 .

Dem. Tem-se
u=

n
X

i wi

i=1

n
X
j=1

j vj

n
X
j=1

n
X

sij wi =

i=1

n
n
X
X
i=1

como as coordenadas de um vector u numa base so nicas, tem-se


2
3
2
!
n
1
X
6 .. 7
6
=
s
.
Isto
,
4 . 5 = SB1 !B2 4
ij j
i
j=1

61

sij

j=1

wi .

para todo o i = 1; :::; n,


3
1

.. 7 .
. 5
n

Observao 31. Tem-se


SB2 !B1 = (SB1 !B2 )

Exemplo 37. Consideremos


Bc = f(1; 0); (0; 1)g
a base cannica de R2 . Seja
B = f(1; 2); (3; 4)g

uma outra base ordenada de R2 . Sejam (5; 6) as coordenadas de um vector u na base


cannica Bc e determinemos as coordenadas de u na base B usando a matriz de mudana de
base SBc !B . Tem-se
2 32
SBc !B =
,
1
1
2
uma vez que
3
2(1; 2) + 1(3; 4) e (0; 1) = (1; 2)
2

(1; 0) =

1
(3; 4). (*)
2

Logo, as coordenadas de u na base B so dadas por


SBc !B
Logo,

5
6

3
2

2
1

5
6

1
2

1
2

1 e 2 so as coordenadas de (5; 6) na base ordenada B, isto


(5; 6) =

1(1; 2) + 2(3; 4).

Observao 32. Colocando os vectores em coluna, note que as duas igualdades em (*)
podem ser escritas na forma:
1 0
0 1

1 3
2 4

2
1

3
2

1
2

sendo esta ltima igualdade equivalente a

1 0
0 1
2

=4

1 3
2 4
| {z }
3
2 32 5
1
1
2

1 3
2 4

8
< (1; 2) = 1(1; 0) + 2(0; 1)
:

(3; 4) = 3(1; 0) + 4(0; 1)

querendo isto dizer que as coordenadas dos vectores (1; 2) e (3; 4) relativamente base
cannica (ordenada) f(1; 0); (0; 1)g so respectivamente (1; 2) e (3; 4).

62

3a Ficha de exerccios para as aulas de problemas


1. Verique que os seguintes subconjuntos de R2 , com as operaes usuais, no so subespaos de R2 .
(i) f(x; y) 2 R2 : x

(ii) f(x; y) 2 R2 : xy = 0g

0g

(iii) f(x; y) 2 R2 : y = x2 g

2. Verique que os seguintes conjuntos, com as operaes usuais, so (todos os) subespaos
de R2 .
(i) f(0; 0)g

(ii) Vk = f(x; kx) : x 2 Rg com k 2 R

(iii) U = f(0; a) : a 2 Rg

(iv) R2

3. No espao linear R3 , considere o subconjunto Uk = f(x; y; k) : x; y 2 Rg onde k uma


constante real. Determine os valores de k para os quais Uk subespao de R3 .
4. Considere o espao linear V = R3 . Diga quais dos seguintes subconjuntos de V , com
as operaes usuais, so subespaos de V e indique os respectivos conjuntos geradores.
(i) f(x; y; z) 2 R3 : z = 2g

(iii) f(x; y; z) 2 R3 : x > 0g


3

(ii) f(x; y; z) 2 R3 : x + y

z = 0g

(iv) f(0; 0; z) : z 2 Rg

(v) f(x; y; z) 2 R : y = 2x e z = 3xg

(vii) f(x; y; z) 2 R3 : x + y + z = 0 e x

(vi) f(x; y; z) 2 R3 : x + y = 1g
y

z = 0g

(viii) f(x; y; z) 2 R3 : x = y ou y = zg
(ix) f(x; y; z) 2 R3 : x

y = 0 e 2y + z = 0g (x) f(x; y; z) 2 R3 : xy = 0g

5. Seja Pn o espao linear de todos os polinmios reais de varivel real e de grau menor ou
igual a n, com as operaes usuais: Diga quais dos seguintes subconjuntos de P2 , com
as operaes usuais, so subespaos de P2 e indique os respectivos conjuntos geradores.
(i) fa0 + a1 t + a2 t2 2 P2 : a0 = 0g (ii) fa0 + a1 t + a2 t2 2 P2 : a2 = 2a0 e a1 = 0g

(iii) fa0 + a1 t + a2 t2 2 P2 : a1 = 1g (iv) fa0 + a1 t + a2 t2 2 P2 : a2


2

(v) fa0 + a1 t + a2 t 2 P2 : a2

a1 = 2g

a1 + 2a0 = 0g

6. Seja Mm n (R) o espao linear de todas as matrizes do tipo m n com entradas


reais. Diga quais dos seguintes subconjuntos de M2 3 (R), com as operaes usuais,
so subespaos de M2 3 (R) e indique os respectivos conjuntos geradores.
(i)
(iii)

a b c
d 0 0

a b c
d e f

2 M2 3 (R) : b = a + c
2 M2 3 (R) : a =

(ii)

a b c
d 0 f

2 M2 3 (R) : b < 0

2c e f = 2e + d :

7. Determine o espao das colunas, o espao das linhas e o ncleo das seguintes matrizes.
2
3
2 1 1
1
1
1 2 3
0 0 0
(i)
(ii)
(iii)
(iv) 4 0 0 1 5
0
0
0 0 0
0 0 0
0 0 0
2
3
2
3
2
3
2
3
1 0
1 2
0 0
1 0 1
(v) 4 2 3 5 (vi) 4 2 4 5
(vii) 4 0 0 5
(viii) 4 2 3 0 5
2 1
2 4
0 0
2 1 0
63

8. Verique que, com as operaes usuais, o seguinte conjunto de matrizes


82
3 2
3 2
3 2
39
0 0
0 0
0 0 =
< 1 0
4 0 0 5 ;4 1 0 5;4 0 1 5;4 0 0 5
:
;
0 0
0 0
0 0
0 1
82
9
3
< a 0
=
gera o subespao 4 b c 5 2 M3 2 (R) : a; b; c; d 2 R do espao linear M3 2 (R).
:
;
0 d

9. Considere, no espao linear R3 , os vectores v1 = (1; 2; 1), v2 = (1; 0; 2) e v3 = (1; 1; 0).


Mostre que os seguintes vectores so combinaes lineares de v1 ; v2 e v3 .
(i) (3; 3; 0)

(ii) (2; 1; 5)

(iii) ( 1; 2; 0)

(iv) (1; 1; 1)

10. Considere, no espao linear R4 , os vectores v1 = (1; 0; 0; 1), v2 = (1; 1; 0; 0) e v3 =


(0; 1; 2; 1). Diga quais dos seguintes vectores pertencem ao subespao L (fv1 ; v2 ; v3 g).
(i) ( 1; 4; 2; 2)

(ii) (2; 0; 2; 2)

(iii) (1; 1; 2; 2)

(iv) (0; 1; 1; 0)

11. Determine o valor de k para o qual o vector u = (1; 2; k) 2 R3 combinao linear


dos vectores
v = (3; 0; 2) e w = (2; 1; 5):
12. Considere, no espao linear P2 , os vectores p1 (t) = 2 + t + 2t2 , p2 (t) =
p3 (t) = 2 5t + 5t2 e p4 (t) = 2 3t t2 . O vector

2t + t2 ,

q(t) = 2 + t + t2
pertence expanso linear L (fp1 (t); p2 (t); p3 (t); p4 (t)g)? Podem os vectores p1 (t),
p2 (t), p3 (t) e p4 (t) gerar P2 ?
13. Verique que os seguintes conjuntos de vectores geram R3 .
(i) f(1; 0; 0); (0; 1; 0); (0; 0; 1)g (ii) f(1; 1; 1); (0; 1; 1); (0; 0; 1)g
(iii) f(1; 1; 1) ; ( 1; 1; 1); (1; 1; 1); ( 1; 1; 1)g

14. Escreva a matriz

3
1

1
1

como combinao linear das matrizes

1 1
1 0

A=
Encontre uma matriz 2
L

0 0
1 1

B=

, C=

0
0

2
1

2 que no pertena a
1 1
1 0

0 0
1 1

0
0

2
1

Antes de a determinar, explique porque que essa matriz existe.


15. Determine os vectores (a; b; c) de R3 que pertencem a L (fu; v; wg) onde
u = (2; 1; 0);

v = (1; 1; 2) e w = (0; 3; 4):


64

16. Sejam
A=

1 1 5
2 3 13

1
e B=4 4
3

1
3
1

3
1
1 5:
3

Verique que o espao das linhas de A igual ao espao das linhas de B: Conclua ento
que os espaos das colunas de AT e de B T so iguais.
17. Encontre um conjunto de geradores para cada um dos seguintes subespaos do espao
linear R4 .
(i) f(x; y; z; w) 2 R4 : x = 0 e y + z = 0g
(ii) f(x; y; z; w) 2 R4 : x + y + z + w = 0g
(iii) f(x; y; z; w) 2 R4 : x + 2y

z = 0 e x + y + 2w = 0 e y

z + w = 0g

18. Dena por meio de sistemas de equaes homogneas os seguintes subespaos.


(i) Em P2 : L (f1

t2 ; 1 + tg)

(ii) L (f(1; 0; 1); (0; 1; 0); ( 2; 1; 2)g)

(iii) L (f(0; 1; 0); ( 2; 1; 2)g) (iv) L (f(1; 1; 2); (2; 1; 1)g)

(v) L (f(1; 0; 1; 1)g)

(vi) L (f(1; 2; 5; 3); (2; 4; 6; 2); (3; 6; 11; 1); (0; 0; 1; 2)g)
19. Determine as condies que os parametros i ; i (i = 1; 2) devem vericar para que os
vectores ( 1 ; 1 ; 3) e ( 2 ; 2 ; 9), no espao linear R3 , sejam linearmente independentes.
20. Diga se os seguintes conjuntos de vectores em R3 so linearmente dependentes ou
linearmente independentes? Nos casos em que sejam linearmente dependentes, indique
(para cada um) um subconjunto linearmente independente com o maior no possvel de
elementos e escreva os restantes como combinao linear desses vectores.
(i) f(4; 2; 1); (2; 6; 5); (1; 2; 3)g

(ii) f(1; 2; 1); (3; 2; 5)g

(iii) f(1; 2; 3); (1; 1; 1); (1; 0; 1)g

(iv) f(1; 0; 1); (0; 0; 0); (0; 1; 1)g

(v) f(1; 1; 0); (0; 2; 3); (1; 2; 3); (x; y; z)g (com x; y; z 2 R).

21. Determine todos os valores de a para os quais f(a2 ; 0; 1); (0; a; 2); (1; 0; 1)g uma base
de R3 :
22. Sejam U = L (f(1; 1; 0; 0); (0; 1; 1; 0)g) e Vk = L (f(2; k; 1; 0); (0; 0; 0; 1)g) subespaos de
R4 : Determine os valores de k para os quais dim (U \ Vk ) = 1.
23. No espao linear R3 , construa uma base que inclua os vectores:
(i) (1; 0; 2) e (0; 1; 2).

(ii) (2; 1; 1) e ( 4; 2; 1).

(iii) ( 1; 2; 1) e (1; 0; 1).

24. Verique que os seguintes subconjuntos do espao linear de todas as funes reais de
varivel real so linearmente dependentes. Indique (para cada um) um subconjunto
linearmente independente com o maior no possvel de elementos e escreva os restantes
como combinao linear desses vectores.
(i) S = fcos2 t; sen2 t; cos 2tg

(ii) S = f2; sen2 t; cos2 tg

(iv) S = 1; t; t2 ; (t + 1)2

(iii) S = fet ; e t ; cosh tg

Determine uma base para cada subespao L(S) e calcule a respectiva dimenso.
65

25. Seja V o espao linear de todas as funes reais de varivel real. Sejam f; g; h 2 V , com
f (t) = sen t, g (t) = cos t e h (t) = t. Mostre que o conjunto ff; g; hg linearmente
independente.
26. Diga quais dos seguintes conjuntos de vectores so bases de R2 . Caso no sejam
bases, determine subconjuntos desses conjuntos que sejam bases e as dimenses dos
espaos gerados por cada um desses subconjuntos. Em cada base de R2 encontrada,
exprima o vector (0; 1) como combinao linear dos vectores dessa base ordenada.
Isto , determine as coordenadas do vector (0; 1) em cada base ordenada encontrada.
Relativamente a cada base ordenada de R2 , determine ainda o vector cujas coordenadas
so (0; 1).
(i) f(1; 3); (1; 1)g

(ii) f(0; 0); (1; 2)g

(iv) f( 5; 0); (0; 2)g (v) f(1; 2); (2; 3); (3; 2)g

(iii) f(2; 4)g

(vi) f(1; 0); (0; 1)g

27. Diga quais dos seguintes conjuntos de vectores so bases de R3 . Caso no sejam bases,
determine subconjuntos desses conjuntos que sejam bases e as dimenses dos espaos
gerados por cada um desses subconjuntos. Em cada base de R3 encontrada, exprima
o vector ( 1; 1; 2) como combinao linear dos vectores dessa base ordenada. Isto
, determine as coordenadas do vector ( 1; 1; 2) em cada base ordenada encontrada.
Relativamente a cada base ordenada de R3 , determine ainda o vector cujas coordenadas
so ( 1; 1; 2).
(i) f(1; 2; 3); (0; 0; 0); (0; 1; 2)g

(ii) f(1; 2; 0); (0; 1; 1)g

(v) f(1; 1; 1); (2; 3; 4); (4; 1; 1); (0; 1; 1)g

(vi) f(1; 0; 0); (0; 1; 0); (0; 0; 1)g

(iii) f(3; 2; 2); ( 1; 2; 1); (0; 1; 0)g

(iv) f(1; 1; 1); (0; 1; 1); (0; 0; 1)g

28. Diga quais dos seguintes conjuntos de vectores so bases de R4 . Caso no sejam bases,
determine subconjuntos desses conjuntos que sejam bases e as dimenses dos espaos
gerados por cada um desses subconjuntos. Em cada alnea indique uma base de R4
que inclua pelo menos dois vectores do conjunto apresentado.
(i) f(1; 0; 0; 1); (0; 1; 0; 0); (1; 1; 1; 1); (0; 1; 1; 1)g

(ii) f(1; 1; 0; 2); (3; 1; 2; 1); (1; 0; 0; 1)g

(iii) S = f(1; 0; 0; 1); (0; 1; 1; 0); (0; 1; 0; 1); (1; 0; 1; 0); (0; 0; 1; 1)g
(iv) f(1; 0; 0; 2); (1; 0; 2; 0); (1; 2; 0; 0); (3; 0; 0; 0)g

(v) f(1; 2; 5; 3); (2; 4; 6; 2); (3; 6; 11; 1); (0; 0; 5; 5)g

(vi) S = f(2; 1; 1; 2); ( 1; 1; 1; 2); (4; 2; 2; 2); (5; 2; 2; 2)g : Nesta alnea, verique que (8; 3; 3; 5) 2 L (S) e determine uma base de L (S) que inclua o vector
(8; 3; 3; 5).
29. Diga quais dos seguintes conjuntos de vectores so bases de P2 (espao linear dos
polinmios reais de grau menor ou igual a 2). Caso no sejam bases, determine subconjuntos desses conjuntos que sejam bases e as dimenses dos espaos gerados por
cada um desses subconjuntos. Determine as coordenadas do vector 1 t em cada base
ordenada de P2 encontrada. Relativamente a cada base ordenada de P2 , determine
ainda o vector cujas coordenadas so ( 1; 3; 2).
(i) f2 + t

t2 ; 2t + 2t2 ; t2 g

(ii) f2t
66

t2 ; 1

2t2 ; 2 + t; 1

4tg

(iii) f1 + t2 ; t

(v) f1 + 2t

t2 ; 1

t + 2t2 ; 1 + tg

t2 ; 3 + t2 ; 5 + 4t

1 1
0 0
para o espao linear M2 2 (R):

30. Mostre que as matrizes

t2 ; 2 + 2t
;

0 0
1 1

t2 g
;

(iv) f 1 + 2t + t2 ; 2

tg

(vi) f1; t; t2 g

1 0
0 1

0 1
1 1

formam uma base

1 3
0 11
2
5
4 1
3
2
;
;
,
;
. Seja W um
1 2
5 3
3 1
1 5
2 3
subespao de M2 2 (R) gerado por S. Determine uma base para W que inclua vectores
de S.

31. Seja S =

32. Determine uma base para M3 2 (R). Qual a dimenso do espao linear M3 2 (R)?
33. Determine uma base para cada um dos seguintes subespaos de M3 3 (R) e calcule a
respectiva dimenso:
(i) O conjunto de todas as matrizes (reais) diagonais do tipo 3
(ii) O conjunto de todas as matrizes (reais) simtricas do tipo 3

3:
3:

34. Determine as dimenses e indique bases para: o ncleo, o espao das linhas
das colunas das seguintes matrizes.
2
3
2
0 1 0 0
1
3
1
3 0
6 0
4
5
4
1
(i)
(ii)
(iii) 0 0 1 0
(iv)
6
2
1 0
2 0
0 0 0 1
0
3
2
3
2
2
3
1 0 0
1 2 3
1
1 3 0 2
6 0 1 0 7
6
0 7
6
7
7:
4
5 (vii) 6 2 3 2
0
2
2
0
(v) 4
(vi)
4 3 4 1
0 0 1 5
1 5
1 3 0 2
0 0 0
1 1
1 1

e o espao
1
2
1

3
2
1 5
1

Determine tambem a caracterstica e a nulidade de cada uma delas.

35. Sejam U e V subespaos de W tais que dim U = 4; dim V = 5 e dim W = 7. Diga


quais as dimenses possveis para U \ V .
36. Determine bases e calcule as dimenses de U + V e U \ V , dizendo em que casos U + V
a soma directa U V (determine-a) dos subespaos U e V .
(i) U = L (f(1; 1; 1); (0; 1; 1)g) ; V = L (f(1; 1; 2); ( 1; 1; 1)g) em R3 :
(ii) U = f(x; y; z) 2 R3 : x + y

z = 0 e x + y = 0g ; V = L (f(1; 1; 1)g) em R3 :

(iii) U = L (f(1; 0; 1); ( 1; 1; 2)g) ; V = f(x; y; z) 2 R3 : x + y + 3z = 0g em R3 :


(iv) U = f(x; y; z) 2 R3 : x = y = zg ; V = f(x; y; z) 2 R3 : x = 0g em R3 :
(v) U = L (f1 + t; 1

(vi) U = L (f1 + t; 1

t2 g), V = fa0 + a1 t + a2 t2 2 P2 : a2
3

t g), V = L (f1 + t + t ; t

a1 + a0 = 0g em P2 .

t ; 1 + t + t3 g) em P3 .

(vii) U = L (f(2; 2; 1; 2); ( 1; 1; 1; 3); (0; 0; 6; 8); ( 1; 1; 5; 5)g) ;


V = L (f(0; 0; 0; 1); (0; 1; 2; 3); (0; 2; 4; 8)g) em R4 :
(viii) U = f(x; y; z; w) 2 R4 : x + 2y + 3z = 0 e y + 2z + 3w = 0g,
67

V = L (f(2; 5; 4; 1); (0; 9; 6; 1); ( 4; 1; 2; 1)g) em R4 :


Neste alnea (viii) mostre que U = V .
(ix) Seja U o subespao de R5 gerado por
f(1; 1; 1; 2; 0); (1; 2; 2; 0; 3); (1; 1; 2; 2; 1)g .
Seja V o subespao de R5 gerado por
f(1; 2; 3; 0; 2); (1; 1; 3; 2; 4); (1; 1; 2; 2; 5)g .
Comece por escrever U e V como solues de sistemas de equaes lineares homogneas.
(x) Sejam U e V subespaos de R4 gerados respectivamente por F e por G, com

6
6
37. Seja A = 6
6
4

F = f(1; 0; 1; 0); (0; 1; 1; 1); (1; 0; 0; 2) ; (0; 0; 1; 2)g ;


G = f(1; 1; 1; 1); (1; 2; 0; 1); (0; 0; 1; 1)g .
1
0
2
1
0

1
0
2
1
0

0
2
1
2
0

2
4
2
2
0

1
0
1
1
0

7
7
7:
7
5

(i) Calcule a nulidade e a caracterstica de A:


(ii) Determine bases para o espao das colunas de A e para o ncleo de A:
(iii) Usando a alnea anterior, determine a soluo geral do sistema de equaes lineares
homogneo Au = 0.

(iv) Resolva o sistema de equaes Au = b, com b = (1; 0; 2; 1; 0): Note que b igual
1a coluna de A e use esse facto de modo a encontrar uma soluo particular de Au = b.
38. Utilize a informao da seguinte tabela para, em cada caso, determinar a dimenso do
espao gerado pelas linhas de A, do espao gerado pelas colunas de A, do ncleo de
A e do ncleo de AT . Diga tambem se o correspondente sistema de equaes lineares
no homogneo AX = B possvel, determinando para esses casos, o nmero de
parmetros que entram na soluo geral de AX = B.
A
3 3
car A
3
car [A j B]
3

3
2
3

3
1
1

9
2
2

5
2
3

0
0

2
2
2

39. Construa uma matriz cujo ncleo seja gerado pelo vector (2; 0; 1).
40. Existe alguma matriz cujo espao das linhas contm o vector (1; 1; 1) e cujo ncleo
contm (1; 0; 0)?
41. Quais so as matrizes do tipo 3

3 cujo ncleo tem dimenso 3?

42. Seja A 2 Mm n (R) tal que C(A) = N (A). Prove que A 2 Mn


um exemplo para n = 4.
68

n (R)

com n par. D

43. Seja A 2 Mn

n (R)

tal que car A = n e A2 = A. Prove que A = I.

44. Sejam B1 = f(1; 2); (0; 1)g e B2 = f(1; 1); (2; 3)g duas bases ordenadas de R2 . Seja
v = (1; 5).
(i) Determine as coordenadas de v em relao base B1 .
(ii) Determine a matriz SB1 !B2 de mudana da base B1 para a base B2 .
(iii) Determine as coordenadas de v em relao base B2 , usando as alneas anteriores.
(iv) Determine, directamente, as coordenadas de v em relao base B2 .
(v) Determine a matriz SB2 !B1 de mudana da base B2 para a base B1 .
(vi) Determine as coordenadas de v em relao base B1 , usando a alnea anterior, e
compare com o resultado obtido em (i).
45. Sejam B1 = fv1 ; v2 g e B2 = fw1 ; w2 g duas bases ordenadas de R2 , onde
v1 = (1; 2), v2 = (0; 1).
Suponha que a matriz SB2 !B1 de mudana da base B2 para a base B1 , dada por:
SB2 !B1 =

2 1
1 1

Determine B2 .
46. Sejam B1 = fv1 ; v2 g e B2 = fw1 ; w2 g duas bases ordenadas de P1 , onde
w1 =

1 + t, w2 = 1 + t.

Suponha que a matriz SB1 !B2 de mudana da base B1 para a base B2 , dada por:
SB1 !B2 =

2 3
1 2

Determine B1 .
47. Sejam B1 = f1; 1

t; t2 g e B2 = f1; 1 + t; 1 + t + t2 g duas bases ordenadas de P2 .

(i) Suponha que as coordenadas de um vector p(t) 2 P2 em relao base B2 so


dadas por (1; 2; 3). Determine as coordenadas do mesmo vector p(t) em relao base
B1 .
(ii) Determine a matriz SB1 !B2 de mudana da base B1 para a base B2 e utilize-a para
determinar as coordenadas do vector 2 t + t2 na base B2 .

48. Sejam B1 = fv1 ; v2 g e B2 = fw1 ; w2 g duas bases ordenadas de P1 , onde


w1 = t, w2 = 1

t.

Suponha que a matriz SB2 !B1 de mudana da base B2 para a base B1 , dada por:
SB2 !B1 =
Determine B1 .
69

2 3
1 2

49. Sejam B1 = fv1 ; v2 ; v3 g e B2 = fw1 ; w2 ; w3 g duas bases ordenadas de R3 , onde


v1 = (1; 0; 1), v2 = (1; 1; 0), v3 = (0; 0; 1).
Suponha que a matriz SB1 !B2 de mudana da base B1 para a base B2 , dada por:
2
3
1
1 2
1 1 5.
SB1 !B2 = 4 2
1
1 1
Determine B2 .

50. Sejam B1 =

1 0
0 0
B2 =

0 1
0 0

0 0
1 0

1 1
1 1

1
1

1
1

0 0
0 1

1 1
1 1

1
1

1
1

duas bases ordenadas de M2 2 (R). Determine a matriz SB1 !B2 de mudana da base
1 2
B1 para a base B2 e utilize-a para determinar as coordenadas do vector
em
3 4
relao base B2 .
51. Seja B = fv1 ; v2 g uma base ordenada de P1 . Sejam (1; 1) e (2; 2) respectivamente as
coordenadas de dois polinmios 1 + t e 1 t em relao base B: Determine B.
52. Sejam B1 = fv1 ; v2 g e B2 = fw1 ; w2 g duas bases ordenadas de P1 . Suponha que (1; 1)
e (2; 2) so respectivamente as coordenadas de um polinmio p (t) em relao s bases
B1 e B2 : Suponha ainda que (1; 1) e (2; 2) so respectivamente as coordenadas de um
polinmio q (t) em relao s bases B1 e B2 : Determine a matriz SB1 !B2 de mudana
da base B1 para a base B2 .

70

2a Ficha de exerccios facultativos


1. Seja V um espao linear real e 0 o seu vector nulo. Mostre que:
(i) Se u + v = u + w, ento v = w:

(ii) 0 = 0 para todo o escalar

(iii) 0u = 0 para todo o vector u 2 V:

(iv)

(v) Mostre que o vector nulo 0 2 V nico.

(vi) Mostre que o simtrico


(vii) (

1)u =

2 R:

( u) = u para todo o u 2 V:

u de um qualquer vector u de V nico.

u para todo o u 2 V:

(ix) Se u 6= 0 e u = u, ento

(viii) Se u = 0, ento

= 0 ou u = 0:

= :

2. Verique que o conjunto de todos os polinmios reais de grau igual a n:


fa0 + a1 t +

+ an tn 2 Pn : an 6= 0g ,

munido das operaes usuais, no um espao linear.


3. (i) Mostre que P2 um subespao de P3 : (ii) Mostre que Pn um subespao de Pn+1 :
(iii) Seja P o espao linear de todos os polinmios reais (de qualquer grau). Mostre
que Pn um subespao de P:
4. Quais dos seguintes subconjuntos de Mn
espaos?

n (R),

com as operaes usuais, so sub-

(i) O conjunto de todas as matrizes simtricas do tipo n

n:

(ii) O conjunto de todas as matrizes invertveis do tipo n

n:

(iii) O conjunto de todas as matrizes diagonais do tipo n

n:

(iv) O conjunto de todas as matrizes singulares do tipo n

n:

(v) O conjunto de todas as matrizes triangulares superiores do tipo n

n:

5. Seja V o espao linear de todas as funes reais de varivel real. Quais dos seguintes
subconjuntos de V , com as operaes usuais, so subespaos?
(i) O conjunto de todas as funes limitadas.
(ii) O conjunto de todas as funes pares, isto , tais que f (x) = f ( x):
(iii) O conjunto de todas as funes racionais, isto , as que so quocientes de funes
polinomiais.
(iv) O conjunto de todas as funes crescentes.
(v) O conjunto de todas as funes f tais que f (0) = f (1):
(vi) O conjunto de todas as funes f tais que f (0) = 1 + f (1):
6. Seja fv1 ; v2 ; v3 g uma base de um espao linear V . Prove que fv1 + v2 ; v2 + v3 ; v1 + v3 g
tambm uma base de V .
7. Seja A uma matriz (real) invertvel do tipo n n. Prove que, se fv1 ; v2 ; : : : ; vn g uma
base de Rn , ento fAv1 ; Av2 ; : : : ; Avn g tambm uma base de Rn .
71

8. Sejam V um espao linear e S = fv1 ; v2 ; : : : ; vn g. Prove que o conjunto S uma base


de V se e s se todo o vector de V se escrever de maneira nica como combinao
linear dos elementos de S.
9. Seja fv1 ; v2 g uma base de um espao linear U . Considere os vectores w1 = av1 + bv2 e
w2 = cv1 + dv2 , com a; b; c; d 2 R. Prove que fw1 ; w2 g tambm uma base de U se e
s se ad 6= bc.
10. Sejam A uma matriz m

n e B uma matriz n

dim C (AB) = dim C (B)

p. Mostre que

dim (N (A) \ C (B)) .

Sugesto: Considere (no caso em que N (A) \ C (B) 6= f0g) uma base fx1 ; : : : ; xs g
para N (A) \ C (B) e suponha (no caso em que AB 6= 0) que fx1 ; : : : ; xs ; y1 ; : : : ; yt g
uma base para C (B). Mostre que fAy1 ; : : : ; Ayt g uma base para C (AB).
11. Considere os seguintes r vectores de Rn :
x1 = (x11 ; x12 ; : : : ; x1n ); x2 = (x21 ; x22 ; : : : ; x2n ); : : : ; xr = (xr1 ; xr2 ; : : : ; xrn ):
Mostre que se jxjj j >

r
P

i=1(i6=j)

jxij j para todo o j = 1; : : : ; r ento o conjunto


x1 ; x2 ; : : : ; xr

linearmente independente.
Sugesto: Considere
v = (v1 ; : : : ; vn ) =
com

1;

2; : : : ;

1
1x

2 R e mostre que se existir

2
2x
j

r
rx ;

6= 0 (com j 2 f1; : : : ; rg) tal que

j j j > j i j;
para todo o i = 1; : : : ; r; ento vj 6= 0.
12. Sejam A; B 2 Mm
13. Seja A 2 Mm

n (R).

n (R).

Mostre que C (A + B)

Mostre que N (A) \ L (A) = f0g.

14. Sejam B e C matrizes m

n. Mostre que
jcar B

15. Seja A 2 Mn

n (R)

C (A) + C (B) :

car Cj

car (B

C) .

tal que A3 6= 0 e A4 = 0. Seja v 2


= N (A3 ). Prove que o conjunto
v; Av; A2 v; A3 v

linearmente independente.

72

Transformaes lineares
Denio 36. Sejam U e V espaos lineares. Diz-se que
T :U !V
uma transformao linear se e s se vericar as duas condies:
(i) T (u + v) = T (u) + T (v), para todos os u; v 2 U .
(ii) T ( u) = T (u), para todos os u 2 U e escalares .
Observao 33. Sejam U e V espaos lineares. Sejam 0 o vector nulo de U e 00 o vector
nulo de V .
(i) Se T : U ! V fr uma transformao linear ento T (U ) um subespao de V e alm
disso tem-se T (0) = 00 (T (0) = T (0 + 0) = T (0) + T (0) , T (0) = 00 ). Logo, se T no
vericar T (0) = 00 ento T no ser uma transformao linear.
(ii) T : U ! V uma transformao linear se e s se
T ( u + v) = T (u) + T (v),
para todos os u; v 2 U e escalares ; .
(iii) Seja T : U ! V uma transformao linear, com U = L (fv1 ; :::; vn g). Seja u 2 U .
Logo, existem escalares 1 ; :::; n tais que
u=

1 v1

+ ::: +

n vn .

Tem-se ento
T (u) =

1 T (v1 )

+ ::: +

n T (vn ).

Exemplo 38. Consideremos a base cannica f(1; 0) ; (0; 1)g de R2 . Seja


T : R2 ! R
uma transformao linear tal que T (1; 0) = 1 e T (0; 1) = 1.
Para qualquer (x; y) 2 R2 tem-se
(x; y) = x(1; 0) + y(0; 1).
Ento,
T (x; y) = T (x(1; 0) + y(0; 1)) = xT (1; 0) + yT (0; 1) = x + y.
Logo, T : R2 ! R a transformao linear denida explicitamente por
T (x; y) = x + y.
73

Teorema 40. Sejam U e V espaos lineares e seja fv1 ; :::; vn g uma base de U . Sejam
T1 ; T2 : U ! V duas transformaes lineares.
Se T1 (vi ) = T2 (vi ) para todo o i = 1; :::; n, ento T1 (u) = T2 (u),
para todo o u 2 U , isto , T1 = T2 .
Exemplo 39. (i) Sejam U e V espaos lineares e seja 0 o vector nulo de V . Seja
O : U ! V denida por
O(u) = 0,
para todo o u 2 U . O uma transformao linear e chama-se transformao nula.
(ii) Seja A 2 Mm

n (R).

Seja
T : Rn ! R m

denida por
T (u) = Au,
para todo o u 2 Rn . T uma transformao linear.
(iii) Sejam V um espao linear e k um escalar (xo). Seja Tk : V ! V denida por
Tk (v) = kv;
para todo o v 2 V .
Tk uma transformao linear. Diz-se que Tk uma homotetia.
Se 0 < k < 1 diz-se que Tk uma contraco.
Se k > 1 diz-se que Tk uma dilatao.
Se k = 1 ento chama-se a T1 a transformao identidade e denota-se por I. Tem-se
I(u) = u;
para todo o u 2 U .
(iv) T : R2 ! R2 denida por T (x; y) = (1

y; 2x) no uma transformao linear.

(v) T : R2 ! R denida por T (x; y) = xy no uma transformao linear.


(vi) Seja T : P2 ! P3 denida por
T (p (t)) = tp (t) :
T uma transformao linear.
(vii) Seja T : P3 ! P1 denida por
T (p) = p00 :
T uma transformao linear.

74

(viii) Seja T : C 1 (R) ! C (R) denida por


T (f ) = f 0 ;
onde C 1 (R) o espao linear de todas as funes reais com primeira derivada contnua em R
e C (R) o espao linear de todas as funes reais contnuas em R. T uma transformao
linear.
(ix) Seja a 2 R (xo). Seja T : C 1 (R) ! R denida por
T (f ) = f 0 (a) .
T uma transformao linear.
(x) Seja n 2 N. Seja T : C n (R) ! C (R) denida por
T (f ) = f (n) ;
onde f (n) a derivada de ordem n de f , C n (R) o espao linear de todas as funes reais
com derivada de ordem n contnua em R e C (R) o espao linear de todas as funes reais
contnuas em R. T uma transformao linear.
(xi) Seja T : C (R) ! C 1 (R) denida por
Z
T (f ) =

f (t) dt:

T uma transformao linear.


(xii) Seja T : C ([a; b]) ! R denida por
T (f ) =

f (t) dt:

T uma transformao linear.


(xiii) Seja T : Mn

n (R)

!Mn

n (R)

denida por

T (X) = X T :
T uma transformao linear.
(xiv) Seja T : Mn

n (R)

!Mn

n (R)

denida por

T (X) = AX;
com A 2 Mn

n (R)

xa. T uma transformao linear.

(xv) Seja
tr : Mn

n (R)

!R

denida por
tr(A) = a11 + ::: + ann =

n
X
i=1

75

aii ,

para todo o A = (aij )n

2 Mn

n (R).

tr (trao) uma transformao linear.

Denio 37. Sejam U e V espaos lineares e T : U ! V uma transformao linear.


Seja 0 o vector nulo de V .
(i) Chama-se contradomnio ou imagem de T ao conjunto
T (U ) = fT (u) : u 2 U g ,
que tambm se denota por I(T ).
Note-se que se existir fu1 ; :::; uk g

U tal que U = L (fu1 ; :::; uk g) ento

I(T ) = L (fT (u1 ) ; :::; T (uk )g) :


(ii) Chama-se ncleo ou espao nulo de T ao conjunto
N (T ) = fu 2 U : T (u) = 0g .
Teorema 41. Sejam U e V espaos lineares e T : U ! V uma transformao linear.
Ento, os conjuntos N (T ) e I(T ) so subespaos de U e V respectivamente.
Exemplo 40. (i) Sejam U e V espaos lineares. Sejam 0 e 00 os vectores nulos de U e
V respectivamente.
Considere a transformao nula O : U ! V denida por
O(u) = 00 ,
para todo o u 2 U . Tem-se

N (O) = U e I(O) = f00 g .

(ii) Considere a transformao identidade I : U ! U denida por


I(u) = u,
para todo o u 2 U . Tem-se
(iii) Seja A 2 Mm

N (I) = f0g

n (R).

e I(I) = U .

Seja
T : Rn ! R m

denida por
T (u) = Au,
para todo o u 2 Rn . Tem-se
N (T ) = N (A)

e I(T ) = C(A).
76

(iv) Seja T : C 1 (R) ! C (R) denida por


T (f ) = f 0 :
Tem-se
N (T ) = ff : R ! R tal que f constante em Rg

I(T ) = C (R) :

(v) Seja T : C 2 (R) ! C (R) denida por


T (f (t)) = f 00 (t) + ! 2 f (t) ;
com ! 2 Rn f0g. Tem-se (pg. 72 de [1])
N (T ) = L (fcos (!t) ; sen (!t)g) ;
onde fcos (!t) ; sen (!t)g uma base de N (T ). Observe-se que N (T ) precisamente a soluo
geral da equao diferencial linear homognea
f 00 (t) + ! 2 f (t) = 0:
(vi) Seja T : C 2 (R) ! C (R) denida por
T (f (t)) = f 00 (t)

! 2 f (t) ;

com ! 2 Rn f0g. Tem-se (pg. 74 de [1])


N (T ) = L

!t

; e!t

onde fe !t ; e!t g uma base de N (T ). Note-se que N (T ) precisamente a soluo geral da


equao diferencial linear homognea
f 00 (t)

! 2 f (t) = 0:

Teorema 42. Sejam U um espao linear de dimenso nita e T uma transformao


linear denida em U . Ento, o subespao I(T ) tem dimenso nita e
dim N (T ) + dim I(T ) = dim U .

Denio 38. Sejam U e V espaos lineares e S; T : U ! V transformaes lineares.


Seja um escalar. Sejam S + T; T : U ! V denidas por
(S + T ) (u) = S(u) + T (u)
para todo o u 2 U .
77

( T )(u) = T (u),

Denio 39. Sejam U e V espaos lineares. Chama-se a L(U; V ) o conjunto de todas


as transformaes lineares de U em V .
Teorema 43. Sejam U e V espaos lineares e S; T : U ! V transformaes lineares.
Seja um escalar. Ento:
(i) S + T e T so transformaes lineares.
(ii) O conjunto L(U; V ), com as operaes da denio 38, um espao linear.
Exemplo 41. Seja B = fT1 ; T2 ; T3 ; T4 g com T1 ; T2 ; T3 ; T4 2 L(R2 ; R2 ) denidas por
T1 (x; y) = (x; 0), T2 (x; y) = (y; 0), T3 (x; y) = (0; x) e T4 (x; y) = (0; y),
para todo o (x; y) 2 R2 . O conjunto B uma base de L(R2 ; R2 ). Logo,
dim L(R2 ; R2 ) = 4:

Denio 40. Sejam U; V e W espaos lineares e, T : U ! V e S : V ! W transformaes lineares. Seja S T : U ! W denida por
(S

T ) (u) = S (T (u)) ,

para todo o u 2 U . S T uma transformao linear. Chama-se a S T a composio de


S com T .
Observao 34. Em geral, tem-se S

T 6= T

S.

Teorema 44. (i) Sejam T : U ! V; S : V ! W e R : W ! X. Ento, tem-se


R (S T ) = (R S) T .
(ii) Sejam R; S : U ! V e T : V ! W . Seja
T

(R + S) = T

R+T

2 R. Ento, tem-se

S e T

( R) =

(T

R) .

Se o contradomnio de Q estiver contido em U ento


(R + S) Q = R Q + S

Q e ( R) Q =

Denio 41. Dene-se T 0 = I e T k = T

(R Q) .

T k 1 , para todo o k = 1; 2; :::.

Observao 35. Tem-se T m+n = T m T n para todos os m; n 2 N.


78

Denio 42. T : U ! V diz-se injectiva se e s se


T (u) = T (w) ) u = w,
para todos os u; w 2 U , isto , se e s se
u 6= w ) T (u) 6= T (w),
para todos os u; w 2 U .
Denio 43. T : U ! V diz-se sobrejectiva se e s se T (U ) = V .
(iii) T : U ! V diz-se bijectiva se e s se fr injectiva e sobrejectiva.
Denio 44. Sejam U e V espaos lineares. Diz-se que U e V so isomorfos se e s
se existir um isomorsmo entre U e V , isto , se e s se existir uma transformao linear
bijectiva T : U ! V . Sendo U e V isomorfos escreve-se
U = V.

Teorema 45. Sejam U e V dois espaos lineares de dimenses nitas. Ento, U e V so


isomorfos se e s se dim U = dim V .
Observao 36. (i) Qualquer espao linear real de dimenso n isomorfo a Rn .
(ii) Sejam U e V dois espaos lineares de dimenses nitas. A transformao linear
T : U ! V sobrejectiva se e s se T transformar um qualquer conjunto gerador de U num
conjunto gerador de V .
(iii) Sejam U e V dois espaos lineares de dimenses nitas. Se a transformao linear
T : U ! V fr sobrejectiva ento dim V
dim U .
(iv) Sejam U e V dois espaos lineares de dimenses nitas. Se a transformao linear
T : U ! V fr injectiva ento dim U dim V .
Exemplo 42. (i) A transformao linear T : Rn ! Mn 1 (R) denida por
2
3
a1
6
7
T (a1 ; :::; an ) = 4 ... 5 ;
an

um isomorsmo. Logo Rn = Mn 1 (R).


(ii) A transformao linear T
02
a11
B6 .. ..
T @4 . .
am1

: Mm n (R) ! Rmn denida por


31
a1n
.. 7C = (a ; :::; a ; :::; a ; :::; a ) ;
11
m1
1n
mn
. 5A
amn
79

um isomorsmo. Logo Mm

n (R)

= Rmn .

(iii) A transformao linear T : Rn+1 ! Pn denida por


T (a0 ; a1 ; :::; an ) = a0 + a1 t + ::: + an tn ;
um isomorsmo. Logo Rn+1 = Pn .
(iv) Seja A uma matriz m
dimenso (car A).

n. Os espaos C (A) e L (A) so isomorfos pois tm a mesma


C (A) = L (A) .

Teorema 46. Sejam U e V espaos lineares de dimenses nitas tais que


dim U = dim V:
Seja T : U ! V uma transformao linear. Ento, T injectiva se e s se T sobrejectiva.
Denio 45. Diz-se que T : U ! V invertvel se existir S : T (U ) ! U tal que
S

T = IU e T

S = IT (U ) ,

onde IU e IT (U ) so as funes identidade em U e T (U ) respectivamente. Chama-se a S a


inversa de T e escreve-se
S = T 1.

Teorema 47. Sejam U e V espaos lineares de dimenses nitas. Seja


T :U !V
uma transformao linear. Seja 0 o vector nulo de U . As seguintes armaes so equivalentes.
(i) T injectiva.
(ii) T invertvel e a inversa T

: T (U ) ! U linear.

(iii) N (T ) = f0g.
(iv) dim U = dim T (U ).
(v) T transforma vectores linearmente independentes de U em vectores linearmente independentes de V .
(vi) T transforma bases de U em bases de T (U ).
Teorema 48. Sejam U e V espaos lineares. Seja T : U ! V uma transformao linear.
Seja b 2 V . Ento:
80

(i) Existncia de soluo: a equao linear T (u) = b tem sempre soluo (para qualquer b) se e s se T fr sobrejectiva (T (U ) = V );
(ii) Unicidade de soluo: a equao linear T (u) = b a ter soluo, ela nica se e s
se T fr injectiva;
(iii) Existncia e unicidade de soluo: a equao linear T (u) = b tem soluo nica
u se e s se T fr bijectiva.
Teorema 49. Sejam U e V espaos lineares. Seja T : U ! V uma transformao linear.
Seja b 2 V . A soluo geral da equao linear T (u) = b obtm-se somando a uma soluo
particular dessa equao, a soluo geral da equao linear homognea T (u) = 0 (N (T )).
Teorema 50. (Representao matricial de uma transformao linear). Sejam
U e V espaos lineares de dimenses nitas tais que dim U = n e dim V = m. Sejam
B1 = fu1 ; :::; un g e B2 = fv1 ; :::; vm g duas bases ordenadas de U e V respectivamente. Seja
T : U ! V uma transformao linear. Considere-se a matriz A = (aij )m n cuja coluna j,
para cada j = 1; :::; n, formada pelas coordenadas de T (uj ) na base B2 . Isto ,
T (uj ) =

m
X

aij vi .

i=1

Chama-se a esta matriz A a representao matricial de T em relao s bases B1 e B2 e


escreve-se
A = M (T ; B1 ; B2 ).
Alm disso, sendo 1 ; :::; n as coordenadas de um vector u 2 U na base ordenada B1 ento
as coordenadas 1 ; :::; m de T (u) 2 V na base ordenada B2 so dadas por
2
2
3
3
1

6 .. 7
6 . 7
4 . 5 = M (T ; B1 ; B2 ) 4 .. 5 .

Observao 37. MUITO IMPORTANTE. Nas condies do teorema anterior, temse


u=

n
X

j uj

j=1

v=

m
X

i vi

i=1

uma vez que


T (u) = T

n
X
j=1

j uj

T linear

2 N (T ) , (
2 I(T ) , (

n
X

jT

(uj ) =

j=1

1 ; :::;

1 ; :::;

n
X
j=1

81

n)

m)

2 N (A)
2 C(A)

m
X
i=1

aij vi =

m
n
X
X
i=1

j=1

aij

vi

e sendo fv1 ; v2 ; :::; vm g uma base de V tem-se


u 2 N (T ) , T (u) = 0 ,

n
X

aij

= 0; para i = 1; :::; m

j=1

,(

1 ; :::;

n)

2 N (A):

Alm disso:
I(T ) = L (fT (u1 ); :::; T (un )g) =
= L (fa11 v1 + ::: + am1 vm ; :::; a1n v1 + ::: + amn vm g) .
Observao 38. Seja V um espao linear de dimenso nita, com dim V = n. Sejam
B1 = fu1 ; :::; un g e B2 = fv1 ; :::; vn g duas bases ordenadas de V . A representao matricial
da transformao identidade I : V ! V em relao s bases B1 e B2 igual matriz de
mudana da base B1 para B2 . Isto ,
M (I; B1 ; B2 ) = SB1 !B2 .

Teorema 51. Sejam U e V espaos lineares tais que dim U = n e dim V = m. Seja
T : U ! V uma transformao linear. Sejam B1 e B2 bases (ordenadas) de U e V respectivamente. Seja
A = M (T ; B1 ; B2 ) 2 Mm n (R)
a matriz que representa T em relao s bases B1 e B2 . Tem-se ento:
(i) dim N (T ) = nul A;
(ii) dim I(T ) = car A;
(iii) T injectiva se e s se nul A = 0, isto , se e s se car A = n;
(iv) T sobrejectiva se e s se car A = m.
Teorema 52. Sejam Bcn = fe1 ; :::; en g e Bcm = fe01 ; :::; e0m g as bases cannicas (ordenadas)
de Rn e Rm respectivamente. Seja T : Rn ! Rm uma transformao linear. Considere-se a
matriz A = (aij )m n = M (T ; Bcn ; Bcm ) 2 Mm n (R) cuja coluna j, para cada j = 1; :::; n,
formada pelas coordenadas de T (ej ) na base Bcm . Isto ,
2 3
2 3
2
3
1
0
a
1j
m
6
7
6
7
.
X
6 0 7
6 . 7 6
7
T (ej ) =
aij e0i = a1j 6 .. 7 + ::: + amj 6 . 7 = 4 ... 5 .
4 . 5
4 0 5
i=1
amj
0
1
Ento, tem-se, para todo o u 2 Rn ,

T (u) = Au.

82

Dem. Seja u 2 Rn . Ento, existem


u=

1 e1

1 ; :::;

+ ::: +

2 R tais que

n en

n
X

j ej .

j=1

P
0
Uma vez que, para todo o j = 1; :::; n, T (ej ) = m
i=1 aij ei , tem-se
!
m
m
n
n
n
n
X
X
X
X
X
X
0
a
e
=
aij
T
(e
)
=
T (u) = T
e
=
ij i
j
j
j
j j
T linear

j=1

n
X
j=1

a1j

j ; :::;

n
X
j=1

j=1

amj

j=1

=4

i=1

a11

a1n

am1

amn

i=1

32

j=1

e0i =

. 7
56
4 .. 5 = Au.
n

Observao 39. No caso em que U = Rn , V = Rm e B1 = Bcn , B2 = Bcm , tem-se:


N (T ) = N (A)

I(T ) = C(A),

uma vez que neste caso as coordenadas de um vector numa base coincidem com o prprio
vector.
Exemplo 43. (i) Seja T : R4 ! R3 denida por
T (x; y; z; w) = (3x + y

2z; 0; x + 4z):

T uma transformao linear e a matriz M (T ; Bc4 ; Bc3 ) que representa T em relao s bases
cannicas (ordenadas) Bc4 e Bc3 de R4 e R3 respectivamente, dada por
2
3
3 1
2 0
A = M (T ; Bc4 ; Bc3 ) = 4 0 0 0 0 5 ,
1 0 4 0

uma vez que T (1; 0; 0; 0) = (3; 0; 1), T (0; 1; 0; 0) = (1; 0; 0), T (0; 0; 1; 0) = ( 2; 0; 4) e
T (0; 0; 0; 1) = (0; 0; 0).
Tem-se ento:
2
3
x
6 y 7
7
T (x; y; z; w) = M (T ; Bc4 ; Bc3 ) 6
4 z 5 = (3x + y 2z; 0; x + 4z).
w
Alm disso, tem-se

N (T ) = N (A) = (x; y; z; w) 2 R4 : y = 14z e x = 4z =


= f( 4z; 14z; z; w) : z; w 2 Rg = L (f( 4; 14; 1; 0); (0; 0; 0; 1)g)
e
I (T ) = C (A) = L (f(3; 0; 1); (1; 0; 0)g) .
83

Uma base de I (T ) : f(3; 0; 1); (1; 0; 0)g. Uma base de N (T ) : f( 4; 14; 1; 0); (0; 0; 0; 1)g.
(ii) Sejam B1 = f1; t; t2 g e B2 = f1; t; t2 ; t3 g as bases cannicas (ordenadas) de P2 e P3
respectivamente. Seja D : P2 ! P3 tal que D(1) = 0, D(t) = 1 e D(t2 ) = 2t. D uma
transformao linear e a matriz M (D; B1 ; B2 ) que representa D em relao s bases cannicas
B1 e B2 , dada por
2
3
0 1 0
6 0 0 2 7
7
M (D; B1 ; B2 ) = 6
4 0 0 0 5.
0 0 0
Alm disso tem-se

0
a0
6 0
M (D; B1 ; B2 ) 4 a1 5 = 6
4 0
a2
0

3
2
3
0 2
a1
a0
6 2a2
2 7
7 4 a1 5 = 6
4 0
0 5
a2
0
0

1
0
0
0

isto , D (a0 + a1 t + a2 t2 ) = a1 + 2a2 t, com a0 ; a1 ; a2 2 R.


Alm disso, como

7
7,
5

N (D) = a0 + a1 t + a2 t2 : D a0 + a1 t + a2 t2 = 0 = a0 + a1 t + a2 t2 : a1 = a2 = 0 e a0 2 R ;
tem-se
N (D) = fa0 : a0 2 Rg = L (f1g)

I (D) = L (f1; 2tg) .

Uma base de I (D) : f1; 2tg. Uma base de N (D) : f1g.


(iii) Seja T : R3 ! R2 a transformao linear cuja matriz que a representa em relao
s bases ordenadas B1 = f(1; 1; 1); (0; 1; 1); (0; 0; 1)g e B2 = f(1; 1); (1; 1)g de R3 e R2
respectivamente, dada por
A = M (T ; B1 ; B2 ) =
Seja u 2 R3 e sejam (

1;

2;

3)

1 2 3
2 4 6

as coordenadas de u em relao base B1 . Tem-se

u 2 N (T ) , (

1;

2;

3)

2 N (A)

e como
N (A) = N

1 2 3
0 0 0

= f( 2y

3z; y; z) : y; z 2 Rg = L (f( 2; 1; 0); ( 3; 0; 1)g) ,

logo f( 2; 1; 0); ( 3; 0; 1)g uma base de N (A) (uma vez que gera N (A) e linearmente
independente).
N (T ) = f( 2) (1; 1; 1) + 1(0; 1; 1) + 0(0; 0; 1); ( 3) (1; 1; 1) + 0(0; 1; 1) + 1(0; 0; 1)g =
= L (f( 2; 1; 1); ( 3; 3; 2)g) .

84

Logo f( 2; 1; 1); ( 3; 3; 2)g uma base para N (T ) (uma vez que gera N (T ) e
linearmente independente).
Quanto ao contradomnio:
C(A) = L (f(1; 2)g) ,
logo f(1; 2)g uma base de C(A) (uma vez que gera C(A) e linearmente independente).
I(T ) = L (f1(1; 1) + 2(1; 1)g) = L (f(3; 1)g) .
Uma base de I (T ) : f(3; 1)g (uma vez que gera I (T ) e linearmente independente).
Note-se que:
dim N (T ) = nul A
dim I(T ) = car A
e
dim N (T ) + dim I(T ) = dim U (espao de partida).
Teorema 53. Sejam U; V e W espaos lineares de dimenses nitas. Sejam B1 ; B2 e B3
bases ordenadas de U; V e W respectivamente. Seja escalar. Sejam T; T1 ; T2 2 L(U; V ) e
S 2 L(V; W ). Ento, tem-se
M (T1 + T2 ; B1 ; B2 ) = M (T1 ; B1 ; B2 ) + M (T2 ; B1 ; B2 )
M (S

T ; B1 ; B3 ) = M (S; B2 ; B3 )M (T ; B1 ; B2 )

Teorema 54. Sejam U e V dois espaos lineares de dimenses nitas. Seja T : U ! V


uma transformao linear. Sejam B1 e B2 duas bases ordenadas de U e V respectivamente.
Seja A = M (T ; B1 ; B2 ) a matriz que representa T em relao s bases B1 e B2 .
Se V = T (U ) ento T invertvel se e s se A fr uma matriz quadrada no singular.
Tem-se ento
A 1 = M (T 1 ; B2 ; B1 ),
isto , A

ser a matriz que representa T

em relao s bases B2 e B1 .

Teorema 55. Sejam U e V espaos lineares de dimenses nitas respectivamente n e


m. Isto ,
dim U = n e dim V = m:
Ento, os espaos lineares L(U; V ) e Mm

n (R)

so isomorfos e escreve-se

L(U; V ) = Mm

n (R).

Dem. Fixando bases ordenadas B1 e B2 para U e V respectivamente,


L(U; V ) ! Mm n (R)
T
! M (T ; B1 ; B2 )
uma transformao linear bijectiva.

85

Observao 40. No teorema anterior tem-se dim L(U; V ) = mn.


Teorema 56. Seja V um espao linear de dimenso nita. Seja T : V ! V uma
transformao linear. Sejam B1 e B2 duas bases ordenadas de V . Seja M (T ; B1 ; B1 ) a matriz
que representa T em relao base B1 .
Ento, a matriz M (T ; B2 ; B2 ) que representa T em relao base B2 , dada por
M (T ; B2 ; B2 ) = SB1 !B2 M (T ; B1 ; B1 ) (SB1 !B2 )

onde SB1 !B2 a matriz de mudana da base B1 para B2 .


Isto , o diagrama seguinte comutativo.
(V; B1 )

SB1 !B2 # I
(V; B2 )

M (T ;B1 ;B1 )

T
T

M (T ;B2 ;B2 )

(V; B1 )

I # SB1 !B2
(V; B2 )

Teorema 57. Caso geral. Sejam U e V dois espaos lineares de dimenses nitas. Seja
T : U ! V uma transformao linear. Sejam B1 e B10 duas bases ordenadas de U . Sejam B2
e B20 duas bases ordenadas de V . Seja M (T ; B1 ; B2 ) a matriz que representa T em relao s
bases B1 e B2 .
Ento, a matriz M (T ; B10 ; B20 ) que representa T em relao s bases B10 e B20 , dada por
M (T ; B10 ; B20 ) = SB2 !B20 M (T ; B1 ; B2 ) SB1 !B10

onde SB2 !B20 e SB1 !B10 so as matrizes de mudana das bases B2 para B20 e de B1 para B10
respectivamente.
Isto , o diagrama seguinte comutativo.
(U; B1 )

SB1 !B10 # I
(U; B10 )

M (T ;B1 ;B2 )

M (T ;B10 ;B20 )

(V; B2 )

I # SB2 !B20
(V; B20 )

Exemplo 44. Seja T : R2 ! R3 denida por T (x; y) = (y; x; y x). T uma transformao linear. A matriz M (T ; Bc2 ; Bc3 ) que representa T em relao base cannica (ordenada)
Bc2 de R2 e base cannica (ordenada) Bc3 de R3 , dada por
2
3
0 1
M (T ; Bc2 ; Bc3 ) = 4 1 0 5 .
1 1
Sejam B1 = f(1; 1); ( 1; 1)g uma base ordenada de R2 e B2 = f(0; 0; 1); (0; 1; 1); (1; 1; 1)g
uma base ordenada de R3
86

A matriz M (T ; B1 ; B2 ) que representa T em relao base ordenada B1 de R2 e base


ordenada B2 de R3 , dada por
2
3
1 3
2 5,
M (T ; B1 ; B2 ) = 4 0
1
1

uma vez que

T (1; 1) = (1; 1; 0) = (0; 0; 1) + 0(0; 1; 1) + 1 (1; 1; 1)


T ( 1; 1) = (1; 1; 2) = 3(0; 0; 1) 2(0; 1; 1) + 1 (1; 1; 1) :
Vamos agora vericar que se tem M (T ; B1 ; B2 ) = SBc3 !B2 M (T ; Bc2 ; Bc3 ) SBc2 !B1
Uma vez que

(1; 0; 0) = 0(0; 0; 1) 1(0; 1; 1) + 1 (1; 1; 1) ; (0; 1; 0) = (0; 0; 1) + 1(0; 1; 1) + 0 (1; 1; 1) ;


(0; 0; 1) = 1(0; 0; 1) + 0(0; 1; 1) + 0 (1; 1; 1)
2
3
0
1 1
tem-se ento SBc3 !B2 = 4 1 1 0 5. Logo,
1
0 0
2
32
3
0
1 1
0 1
1
SBc3 !B2 M (T ; Bc2 ; Bc3 ) SBc2 !B1
= 4 1 1 0 5 4 1 0 5 SB1 !Bc2 =
1
0 0
1 1
2
3
2
3
2 1
1 3
1
1
1 5
2 5 = M (T ; B1 ; B2 ).
=4 1
=4 0
1 1
0
1
1
1
Por exemplo, para (2; 1) 2 R2 , tem-se:
coordenadas de (2; 1)
na base Bc2

coordenadas de T (2; 1)
na base Bc3

M (T ;Bc2 ;Bc3 )

SBc2 !B1 # I

I # SBc3 !B2

coordenadas de (2; 1)
na base B1

ou seja
2
1

M (T ;Bc2 ;Bc3 )

SBc2 !B1 # I
1
3

coordenadas de T (2; 1)
na base B2 .

M (T ;B1 ;B2 )

3
1
4 2 5
1
I # SBc3 !B2

M (T ;B1 ;B2 )

87

3
8
4 6 5.
4

4a Ficha de exerccios para as aulas de problemas


1. Sejam a; b 2 R. Considere a aplicao Ta;b : R ! R denida por Ta;b (x) = ax + b.
Determine os valores de a e de b para os quais Ta;b linear.
2. Diga quais das seguintes transformaes so lineares. Determine para cada transformao linear a correspondente matriz que a representa em relao s respectivas
bases cannicas (ordenadas). Determine tambm, se possvel, para cada uma dessas
transformaes lineares, bases para o ncleo N (T ) e para o contradomnio I(T ), bem
como as respectivas dimenses (de N (T ) e de I(T )). Diga ainda quais so injectivas,
sobrejectivas e bijectivas.
(i) T : R2 ! R2 com T (x; y) = (x + 2y; 3x
2

(ii) T : R ! R

com T (x; y) = (1

y).

y; 2x).

(iii) T : R3 ! R3 com T (x; y; z) = (x; 2x; x).

(iv) T : R3 ! R2 com T (x; y; z) = (0; 0).

(v) T : R2 ! R com T (x; y) =


3

com T (x; y; z) = (0; 1; 2).

com T (x) = (2x; 0; x).

(vi) T : R ! R

(vii) T : R ! R

3x.

(viii) T : R3 ! R2 com T (x; y; z) = (x2

(ix) T : R4 ! R2 com T (x; y; z; w) = (x

(x) T : R3 ! R4 com T (x; y; z) = ( z; y


(xi) T : R ! R

y; 2y).
y; 3w).
2z; 2y; y + z).

com T (x) = (0; 0).

(xii) T : R3 ! R3 com T (x; y; z) = (x + 2y; 3z; x

z).

(xiii) T : R3 ! R3 com T (x; y; z) = (x; y; z).

(xiv) T : R2 ! R2 com T (x; y) = (x cos


y sen ; x sen +y cos ), 2 R. Aplicao
que ao ponto de coordenadas (x; y) faz corresponder o ponto obtido por uma rotao
de amplitude em torno da origem e no sentido contrrio ao dos ponteiros de um
relgio.
(xv) T : P2 ! P2 com T (p (t)) = 2p (1

t)

tp0 (t),

onde P2 = fa0 + a1 t + a2 t2 : a0 ; a1 ; a2 2 Rg e p0 a derivada de 1a ordem de p.


(xvi) T : P2 ! P2 com
T (p (t)) = p (0)
(xvii) T : P2 ! M2

p ( 1) + (p ( 1) + p (1)) t + (p ( 1)
2

(R) com T (p (t)) =

p (1)

2p (0)) t2 :

p (1) p (0)
.
p (0) p ( 1)

3. Considere a transformao linear T : R3 ! R3 que em relao base cannica (ordenada) Bc3 = f(1; 0; 0); (0; 1; 0); (0; 0; 1)g de R3 representada pela matriz
2
3
1
2 1
1 0 5.
M (T ; Bc3 ; Bc3 ) = 4 1
2
1 0
88

Determine a expresso geral de T , isto , determine T (x; y; z) para qualquer (x; y; z) 2


R3 . Determine, se possvel, bases para o ncleo N (T ) e para o contradomnio I(T ),
bem como as respectivas dimenses (de N (T ) e de I(T )).
4. Considere a base ordenada B = fv1 ; v2 g de R2 , em que v1 = (1; 1) e v2 = (1; 0) e seja
T : R2 ! R2 a transformao linear tal que
T (v1 ) = (1; 2), T (v2 ) = ( 3; 1).
(i) Calcule T (2; 1).
(ii) Determine a expresso geral de T , isto , determine T (x; y) para qualquer (x; y) 2
R2 .
(iii) Determine a matriz M (T ; Bc2 ; Bc2 ) que representa T em relao base cannica
(ordenada) Bc2 de R2 .
(iv) Determine as matrizes de mudana de base SBc2 !B e SB!Bc2 . Determine as coordenadas do vector (2; 1) na base B.
(v) Determine a matriz M (T ; B; B) que representa T em relao base ordenada B
de R2 . Determine as coordenadas do vector T (2; 1) na base B.

(vi) Determine a matriz M (T ; Bc2 ; B) que representa T em relao s bases ordenadas


Bc2 e B de R2 .

(vii) Determine a matriz M (T ; B; Bc2 ) que representa T em relao s bases ordenadas


B e Bc2 de R2 .

5. Considere as transformaes lineares T1 e T2 cujas matrizes que as representam em


relao s bases cannicas (ordenadas) de R2 e R3 so dadas respectivamente por
2
3
0 1
2 0 1
M (T1 ; Bc3 ; Bc2 ) =
e M (T2 ; Bc2 ; Bc3 ) = 4 0 1 5 .
1 1 0
1 1
Determine as expresses gerais de (T1
(x; y) 2 R2 ; (x; y; z) 2 R3 .

T2 )(x; y) e (T2

T1 )(x; y; z) para quaisquer

6. Considere a transformao linear T : R3 ! R3 denida por


T (x; y; z) = (2y; y

x; x).

Determine a matriz M (T ; B; B) que representa T em relao base ordenada


B = fv1 ; v2 ; v3 g de R3 com v1 = (1; 0; 1), v2 = (1; 2; 0), v3 = ( 1; 1; 1).
7. Seja
Bc2

1 0
0 0

0 1
0 0

0 0
1 0

0 0
0 1

a base cannica (ordenada) de M2 2 (R). Considere a transformao linear


S : M2 2 (R) ! M2 2 (R) denida por S(A) = AT .
Determine a matriz M (S; Bc2 2 ; Bc2 2 ) que representa S em relao base cannica
(ordenada) Bc2 2 .
89

8. Considere a transformao linear T : R3 ! R3 e a base cannica (ordenada)


Bc3 = fv1 ; v2 ; v3 g de R3 , com v1 = (1; 0; 0), v2 = (0; 1; 0), v3 = (0; 0; 1).
Suponha que se tem
T (v3 ) = 3v1 + v2
(i) Calcule T (2v1

2v3 ,

T (v2 + v3 ) = v1 ,

T (v1 + v2 + v3 ) = v2 + v3 .

v2 + 3v3 ).

(ii) Determine a matriz M (T ; Bc3 ; Bc3 ) que representa T em relao base cannica
(ordenada) Bc3 de R3 .

(iii) Determine duas bases ordenadas B1 = fu1 ; u2 ; u3 g e B2 = fw1 ; w2 ; w3 g de R3 de


modo a que a matriz M (T ; B1 ; B2 ) que represente T em relao a essas bases B1 e B2
seja a matriz identidade:
2
3
1 0 0
4 0 1 0 5.
0 0 1

9. Considere a transformao linear T : R2 ! R3 que em relao s bases ordenadas


B1 = fu1 ; u2 g de R2 e B2 = fv1 ; v2 ; v3 g de R3 com
u1 = (1; 1), u2 = (2; 1), v1 = (1; 0; 1), v2 = (1; 1; 2), v3 = (0; 1; 1),
representada pela matriz
2

M (T ; B1 ; B2 ) = 4
0

3
1 2
1 1 5.
3 0

Considere ainda as bases ordenadas B1 = u1 ; u2 de R2 e B2 = v1 ; v2 ; v3 de R3 com


0

u1 = (1; 0), u2 = (1; 1), v1 = (1; 0; 0), v2 = (1; 1; 0), v3 = (1; 1; 1).
(i) Determine as coordenadas do vector T ( 1; 2) na base B2 .

(ii) Determine as coordenadas do vector ( 1; 2) na base B10 .

(iii) Determine as coordenadas do vector T ( 1; 2) na base B20 .

(iv) Determine, se possvel, uma base para o ncleo N (T ). Determine a dimenso de


N (T ). Diga se T injectiva.
(v) Determine, se possvel, uma base para o contradomnio I(T ). Determine a dimenso de I(T ). Diga se T sobrejectiva.
(vi) Determine a expresso geral de T , isto , determine T (x; y) para qualquer (x; y) 2
R2 .
0

(vii) Determine a matriz M (T ; B1 ; B2 ) que representa T em relao s bases ordenadas


0
0
B1 e B2 .

90

10. Considere a transformao linear T : R3 ! R2 denida por


T (x; y; z) = (x + y; x + y

z).

(i) Determine a matriz M (T ; Bc3 ; Bc2 ) que representa T em relao s bases cannicas
(ordenadas) Bc3 e Bc2 de R3 e R2 respectivamente.

(ii) Determine, se possvel, uma base para o ncleo N (T ). Determine a dimenso de


N (T ). Diga se T injectiva.
(iii) Determine, se possvel, uma base para o contradomnio I(T ). Determine a dimenso de I(T ). Diga se T sobrejectiva.
(iv) Determine a soluo geral da equao linear T (x; y; z) = (1; 1).

(v) Considere a equao linear T (x; y; z) = (a; b). Verique se existe algum vector
(a; b) 2 R2 para o qual essa equao seja impossvel.
(vi) Considere a equao linear T (x; y; z) = (a; b). Verique se existe algum vector
(a; b) 2 R2 para o qual essa equao seja possvel e determinada.

11. Considere a transformao linear T : R3 ! R3 cuja matriz M (T ; Bc3 ; Bc3 ) que a representa em relao base cannica (ordenada) Bc3 de R3 dada por
2
3
1 2 2
M (T ; Bc3 ; Bc3 ) = 4 2 1 4 5 .
0 0 2

(i) Determine a expresso geral de T , isto , determine T (x; y; z) para qualquer


(x; y; z) 2 R3 .
(ii) Determine, se possvel, uma base para o ncleo N (T ). Determine a dimenso de
N (T ). Diga se T injectiva.
(iii) Determine, se possvel, uma base para o contradomnio I(T ). Determine a dimenso de I(T ). Diga se T sobrejectiva.
(iv) Determine a soluo geral da equao linear T (x; y; z) = (3; 3; 0).

(v) Considere a equao linear T (x; y; z) = (a; b; c). Verique se existe algum vector
(a; b; c) 2 R3 para o qual essa equao seja impossvel.
(vi) Considere a equao linear T (x; y; z) = (a; b; c). Verique se existe algum vector
(a; b; c) 2 R3 para o qual essa equao seja possvel e indeterminada.

12. Considere a transformao linear T : R3 ! R3 cuja matriz M (T ; B; B) que a representa


em relao base (ordenada) B = fv1 ; v2 ; v3 g de R3 com
v1 = (1; 1; 1), v2 = (1; 1; 0), v3 = (1; 0; 0),
dada por

3
1 2 2
M (T ; B; B) = 4 2 4 4 5 .
0 0 2

(i) Determine, se possvel, uma base para o ncleo N (T ). Determine a dimenso de


N (T ). Diga, justicando, se T sobrejectiva e se T injectiva.
91

(ii) Determine, se possvel, uma base para o contradomnio I(T ). Determine a dimenso de I(T ).

(iii) Mostre que a equao linear T (x; y; z) = (2; 4; 0) no tem solues.

(iv) Determine T (1; 1; 1) e resolva a equao linear T (x; y; z) = ( 1; 1;

1
).
3

(v) Considere a equao linear T (x; y; z) = (a; b; c). Verique se existe algum vector
(a; b; c) 2 R3 para o qual essa equao seja possvel e indeterminada.

(vi) Determine a expresso geral de T , isto , determine T (x; y; z) para qualquer


(x; y; z) 2 R3 .

13. Considere a transformao linear T : R3 ! R3 denida por


T (x; y; z) = (x + y + z; x + 2y

4z; z).

(i) Determine a matriz M (T ; Bc3 ; Bc3 ) que representa T em relao base cannica
(ordenada) Bc3 de R3 .

(ii) Mostre que T injectiva e determine a expresso geral de T


T 1 (x; y; z) para qualquer (x; y; z) 2 R3 .

, isto , determine

(iii) Justique que T um isomorsmo.

(iv) Determine a soluo geral da equao linear T (x; y; z) = (1; 1; 2).


14. Seja
Bc2

1 0
0 0

0 1
0 0

0 0
1 0

0 0
0 1

a base cannica (ordenada) de M2 2 (R). Considere a transformao


T : M2 2 (R) ! M2 2 (R) denida por T (X) = AX

XA, com A =

0 1
1 0

(i) Verique que T linear.


(ii) Determine a expresso geral de T .
(iii) Determine a matriz M (T ; Bc2 2 ; Bc2 2 ) que representa T em relao base cannica
(ordenada) Bc2 2 de M2 2 (R).
(iv) Determine, se possvel, uma base para o ncleo N (T ). Determine a dimenso de
N (T ). Diga se T injectiva.
(v) Determine, se possvel, uma base para o contradomnio I(T ). Determine a dimenso de I(T ). Diga se T sobrejectiva.

15. Considere as transformaes lineares T1 ; T2 : R2 ! R2 denidas respectivamente por


T1 (x; y) = (x + y; x

y) e T2 (x; y) = (2x + y; x

2y).

(i) Determine as matrizes M (T1 ; Bc2 ; Bc2 ) e M (T2 ; Bc2 ; Bc2 ) que representam respectivamente T1 e T2 em relao base cannica (ordenada) Bc2 de R2 .
(ii) Determine a matriz A = M (T2
base cannica (ordenada) Bc2 de R2 .

T1 ; Bc2 ; Bc2 ) que representa T2


92

T1 em relao

(iii) Determine, usando a alnea anterior, a expresso geral de T2


T1 )(x; y) para qualquer (x; y) 2 R2 .

T1 , isto , (T2

(iv) Determine, directamente a partir das expresses de T1 e de T2 , a expresso geral


de T2 T1 .
(v) Mostre que T1 e T2 so invertveis.
(vi) Determine as expresses gerais de T1 1 (x; y); T2 1 (x; y) e T1
qualquer (x; y) 2 R2 .

T2

(x; y) para

(vii) Determine a matriz M ((T2 T1 ) 1 ; Bc2 ; Bc2 ) que representa (T2 T1 ) 1 em relao
base cannica (ordenada) Bc2 de R2 e verique que igual a A 1 , onde A a matriz
determinada em (ii).
(viii) Verique que (T2 T1 )

= T1

T2 1 .

16. Considere a transformao linear T : R2 ! R2 que em relao base cannica ordenada


(Bc2 = f(1; 0) ; (0; 1)g) de R2 representada pela matriz:
M T ; Bc2 ; Bc2 =

1 0
2 1

Justique que T injectiva e resolva a equao linear T (x; y) = (1; 2).


17. Considere a transformao linear T1 : R2 ! R denida por T1 (x; y) = x. Seja
M T2 ; Bc1 ; Bc2 =

1
0

a matriz que representa a aplicao linear T2 : R ! R2 em relao s bases cannicas


ordenadas Bc1 = f1g e Bc2 de R e R2 respectivamente. Determine uma base para o
ncleo: N (T2 T1 ).
18. Considere a transformao linear T : R3 ! R2 cuja representao matricial em relao
as bases ordenadas B1 = f(1; 0; 1); (0; 1; 1); (0; 0; 1)g de R3 e B2 = f(1; 1); (0; 1)g de R2
dada pela matriz:
1 0
1
M (T ; B1 ; B2 ) =
.
1 0
1
Determine uma base para o contradomnio I (T ) e diga, justicando, se T sobrejectiva.
19. Considere a transformao linear T1 : R3 ! R2 denida por T1 (x; y; z) = (2x+y; y+2z).
Considere ainda a transformao linear T2 : R2 ! R3 cuja representao matricial
em relao base (ordenada) B = f(2; 1); (1; 2)g de R2 e base cannica Bc3 de R3
dada pela matriz:
2
3
2 1
M (T2 ; B; Bc3 ) = 4 1 1 5 .
1 2
(i) Determine uma base para o ncleo N (T1 ) de T1 e diga, justicando, se T1 sobrejectiva.

93

(ii) Determine uma base para o contradomnio I(T2 ) de T2 e diga, justicando, se T2


injectiva.
(iii) Diga, justicando, se se tem N (T1 ) + I(T2 ) = R3 e determine a dimenso de
N (T1 ) \ I(T2 ).
(iv) Determine a matriz M (T2 ; Bc2 ; Bc3 ) que representa T2 em relao s bases cannicas
Bc2 e Bc3 de R2 e R3 respectivamente.

(v) Determine a soluo geral da equao (T1 T2 ) (x; y) =

8 8
;
.
3 3

20. Considere a transformao linear T1 : R2 ! R3 denida por T1 (x; y) = (2x+y; 0; x+2y).


Considere ainda a transformao linear T2 : R3 ! R2 cuja representao matricial
em relao base (ordenada) B = f(1; 1; 1); (1; 1; 0); (1; 0; 0)g de R3 e base cannica
Bc2 de R2 dada pela matriz:
1
1

M (T2 ; B; Bc2 ) =

1
1

1
1

(i) Determine T2 (0; 1; 0) e T2 (0; 0; 1).


(ii) Determine uma base para o contradomnio I(T1 ) de T1 e diga, justicando, se T1
sobrejectiva.
(iii) Determine uma base para o ncleo N (T2 ) de T2 e diga, justicando, se T2
injectiva.
(iv) Determine a soluo geral da equao (T2 T1 ) (x; y) = ( 1; 1).
21. Considere a transformao linear T : R3 ! P3 denida por
T (1; 1; 1) = 2 + 2t2 ;

T (1; 1; 1) =

t3

e T ( 1; 1; 1) = 2 + t + 2t2 + t3 .

(i) Determine a expresso geral de T , isto , determine T (x; y; z) para qualquer


(x; y; z) 2 R3 .
(ii) Determine, se possvel, uma base para o ncleo N (T ). Determine a dimenso de
N (T ). Diga se T injectiva.
(iii) Determine, se possvel, uma base para o contradomnio I(T ). Determine a dimenso de I(T ). Diga se T sobrejectiva.
(iv) Resolva, em R3 ; a equao linear T (x; y; z) = 1 + t + t2 + t3 .

22. Seja

2 R. Considere a transformao linear T : R3 ! P2 denida por


T (x; y; z) = z

y + (y

x) t + xt2 .

(i) Determine, se possvel, uma base para o ncleo N (T ). Determine a dimenso de


N (T ). Diga se T injectiva.
(ii) Determine, se possvel, uma base para o contradomnio I(T ). Determine a dimenso de I(T ). Diga se T sobrejectiva.

(iii) Considere

= 0 e resolva a equao linear T0 (x; y; z) = 1 + t2 .


94

23. Considere o espao linear P2 dos polinmios reais de varivel real e de grau menor ou
igual a 2. Considere a transformao linear T : P2 ! P2 denida por
T (p (t)) = p0 (t)

2p (t) ,

onde p0 (t) a derivada de primeira ordem de p (t).


(i) Determine a expresso geral de T .
(ii) Sendo B = f1; t; t2 g a base cannica (ordenada) de P2 , determine a matriz
M (T ; B; B) que representa T em relao base B.

(iii) Justique que T um isomorsmo e verique que a expresso geral do isomorsmo


T 1 dada por
1
1 0
1 00
T 1 (p (t)) =
p (t)
p (t)
p (t)
2
4
8
para todo o p (t) 2 P2 , onde p00 (t) a derivada de segunda ordem de p (t).

(iv) Resolva, em P2 ; a equao diferencial linear p0 (t)

2p (t) = (2

3t)2 .

24. Considere o espao linear P2 dos polinmios reais de varivel real e de grau menor ou
igual a 2. Considere a transformao linear T : P2 ! P2 denida por
T (p (t)) = t2 p00 (t)

2p (t) ,

onde p00 (t) a derivada de segunda ordem de p (t).


(i) Determine a expresso geral de T .
(ii) Sendo B = f1; t; t2 g a base cannica (ordenada) de P2 , determine a matriz
M (T ; B; B) que representa T em relao base B.
(iii) Determine, se possvel, uma base para N (T ) e uma base para I (T ) e diga,
justicando, se T injectiva e/ou sobrejectiva.
(iv) Resolva, em P2 ; as equaes diferenciais lineares:
a) t2 p00 (t)

2p (t) = 2

b) 2tp0 (t)

t;

25. Seja U o subespao das matrizes simtricas de M2


U = A 2 M2

2p (0) = 2

t.

(R), isto ,

(R) : A = AT .

Considere a transformao linear T : U ! U denida por


T (A) = AB + BA
com B =

0 1
.
1 0

(i) Determine a expresso geral de T .


(ii) Determine uma base para U e calcule a matriz que representa T em relao a essa
base.
(iii) Determine, se possvel, uma base para N (T ) e uma base para I (T ) e diga,
justicando, se T injectiva e/ou sobrejectiva.
(iv) Resolva, em U; a equao linear T (A) = B.
95

26. Considere a transformao linear T : M2


representa em relao s bases ordenadas
B1 =
de M2

1 1
1 0

1 1
0 1

(R) ! P3 cuja matriz M (T ; B1 ; B2 ) que a


;

0 1
1 1

(R) e B2 = f1 + t; t + t2 ; t2 + t3 ; t3 g de P3
2
1 1 1
6 0 1 1
M (T ; B1 ; B2 ) = 6
4 0 0 1
0 0 0

(i) Determine a expresso geral de T .

1 0
1 1

dada por
3
1
1 7
7.
1 5
1

(ii) Justique que T um isomorsmo e determine a expresso geral do isomorsmo


T 1 , isto , determine
T 1 a0 + a1 t + a2 t2 + a3 t3 .
a b
c d

(iii) Resolva a equao linear T

= 1 + 2t + 3t2 + 4t3 .

27. Seja U o espao linear das funes reais de varivel real duas vezes diferenciveis.
Considere a transformao linear T : U ! U denida por
T (f ) = f 00
Considere o subespao S = ff 2 U : f 00

2f 0 + f .

2f 0 + f = 0g de U .

(i) Mostre que o conjunto fet ; tet g uma base de S. Sugesto: Mostre que se f 2 S,
ento f (t) e t um polinmio de grau menor ou igual a 1.
(ii) Mostre que dados a; b 2 R, existe uma nica funo f 2 S tal que f (0) = a e
f 0 (0) = b.
(iii) Determine a nica soluo f da equao diferencial linear T (f ) = 1 que verica
f (0) = 1 e f 0 (0) = 0.
28. Seja V o subespao linear de R4 gerado pelos vectores v1 = (1; 0; 0; 1) e v2 =
(0; 1; 1; 0). Considere ainda a transformao linear T : V ! V tal que
T (v1 ) = v2 ;

T (v2 ) =

v1 :

(i) Determine a matriz M (T ; B; B) que representa T em relao base ordenada


B = fv1 ; v2 g de V .

(ii) Encontre, em V , a soluo geral da equao T (u) = (2; 3; 3; 2).

(iii) Sejam w1 = (0; 0; 1; 1); w2 = (0; 0; 0; 1) e considere a transformao linear R :


R4 ! R4 denida por
R(v1 ) = v2 ;

R(v2 ) =

v1 ;

R(w1 ) = R(w2 ) = (0; 0; 0; 0):

Encontre, em R4 , a soluo geral da equao R(u) = (2; 3; 3; 2).


96

29. Seja Pn , com n 2 N, o espao linear real dos polinmios reais de varivel real e
de grau menor ou igual a n. Considere a transformao linear T1 : P2 ! P1 cuja
representao matricial em relao s bases ordenadas B1 = f1 + t; 1 t; t2 g de P2 e
B2 = f1 + t; 1 + 2tg de P1 , dada pela matriz:
M (T1 ; B1 ; B2 ) =

1
0

2 0
1 1

Considere ainda a transformao linear T2 : P1 ! P2 tal que


T2 (1) = 1

T2 (t) = 2 + 8t

2t2 :

a) Determine a matriz M (T2 ; B; B1 ) que representa T2 em relao s bases ordenadas


B = f1; tg de P1 e B1 = f1 + t; 1 t; t2 g de P2 .
b) Determine uma base para N (T1 ) (ncleo de T1 ) e diga, justicando, se T1 sobrejectiva.
c) Determine T1 (t) e encontre, em P2 , a soluo geral da equao T1 (p (t)) = t.
d) Verique que T1 T2 = I.

97

Valores prprios e vectores prprios. Diagonalizao.


Denio 46. Seja V espao linear. Seja T : V ! V uma transformao linear. Diz-se
que um escalar um valor prprio de T se existir um vector no nulo v 2 V tal que
T (v) = v.
Aos vectores no nulos v que satisfaam a equao anterior chamam-se vectores prprios
associados ao valor prprio . Dado um valor prprio de T , o conjunto
E = fv 2 V : T (v) = vg = N (T

I)

um subespao linear de V . Chama-se a E o subespao prprio associado ao valor


prprio . dimenso de E chama-se multiplicidade geomtrica de e denota-se por
mg ( ), isto ,
dim N (T
I) = mg ( ) .
Exemplo 45. (a) Seja V um espao linear e I : V ! V a transformao identidade. Ento todos os vectores de V , exceptuando o vector nulo, so vectores prprios de T associados
ao valor prprio 1.
(b) Seja V o espao linear das funes reais indenidamente diferenciveis em R e T :
V ! V a (transfomao) funo derivada. Como, por exemplo
T e2x = 2e2x
ento e2x vector prprio de T associado ao valor prprio 2.
Observao 41. (i) Sejam V um espao linear e 0 o vector nulo de V . Seja T : V ! V
uma transformao linear. Um escalar um valor prprio de T se e s se N (T
I) 6= f0g.
(ii) Se o espao linear V tiver dimenso nita n e se A = M (T ; B; B) fr a matriz n n
que representa T em relao a uma base ordenada B de V , ento um escalar um valor
prprio de T se e s se esse escalar fr soluo da equao
det(A

I) = 0,

uma vez que se tem, para v 2 V ,


(T

I) v = 0 , (A

6
7
I) 4 ... 5 = 0.
n

onde

1 ; :::;

so as coordenadas de v na base ordenada B, da que

um valor prprio de T , N (T

I) 6= f0g , N (A
98

I) 6= f0g , det(A

I) = 0

isto
um valor prprio de T , det(A

I) = 0

Alm disso, tem-se


v um vector prprio de T , v 2 N (T

I) n f0g , (

1 ; :::;

n)

2 N (A

I) n f0g

isto
v um vector prprio de T , (

1 ; :::;

n)

2 N (A

I) n f0g

e
mg ( ) = dim N (T

I) = dim N (A

I).

(iii) No caso em que V = Rn e A = M (T ; Bcn ; Bcn ), como (neste caso) v = (


tem-se
N (T
I) = N (A
I) :
Denio 47. Seja A uma matriz n

1 ; :::;

n ),

n. Chama-se ao polinmio

p( ) = det(A

I)

o polinmio caracterstico da matriz A. Este polinmio tem grau n, o coeciente do


termo de grau n ( 1)n , o coeciente do termo de grau n 1 ( 1)n 1 tr A e o termo
constante p(0) = det A.
Denio 48. Seja A uma matriz n n. Chama-se valor prprio de A a qualquer
escalar tal que A
I seja singular, isto , tal que det(A
I) = 0. Ao conjunto de
todos os valores prprios de A chama-se espectro de A. multiplicidade de como raz
do polinmio det(A
I) chama-se multiplicidade algbrica de e denota-se por ma ( ).
Chama-se vector prprio de A, associado ao valor prprio de A, a qualquer vector no
nulo v que verique
(A
I)v = 0,
isto , a qualquer vector
v 2 N (A

I)n f0g .

Observao 42. (i) Seja A uma matriz n n. O escalar 0 valor prprio de A se e s


se A fr singular. Isto , a matriz A invertvel se e s se 0 no fr valor prprio de A.
(ii) Seja A uma matriz n n. Ento o polinmio caracterstico de A pode ser escrito na
forma:
p( ) = det(A
I) = ( 1
)m1 ( 2
)m2
( k
)mk ,
onde 1 ; 2 ; : : : ; k so os valores prprios distintos de A e m1 ; m2 ; : : : ; mk so as multiplicidades algbricas desses valores prprios respectivamente, com m1 + m2 +
+ mk = n.
(iii) Seja A uma matriz n
de A.

n. Tem-se mg ( )

99

ma ( ), para qualquer valor prprio

(iv) Seja A uma matriz n n, com os valores prprios 1 ; 2 ; : : : ; n (repetidos de acordo


com a respectiva multiplicidade algbrica). Ento, atendendo alnea anterior e denio
anterior tem-se
det A =

1 2

Denio 49. Sejam A e B matrizes n


se existir uma matriz S invertvel tal que

tr A =

n.

n. As matrizes A e B dizem-se semelhantes

B = SAS

Teorema 58. Duas matrizes so semelhantes se e s se existirem bases ordenadas em


relao s quais essas matrizes representem a mesma transformao linear.
Teorema 59. Sejam A e B matrizes n
tm o(a) mesmo(a):
(i) determinante;

(ii) caracterstica;

n. Se A e B forem semelhantes ento A e B


(iii) nulidade;

(iv) trao;

(v) polinmio caracterstico, e portanto tm os mesmos valores prprios com as mesmas


multiplicidades algbricas e geomtricas.
Dem. (Matrizes semelhantes tm o mesmo polinmio caracterstico.)
det(B

I) = det(SAS 1
SS 1 ) =
I)S 1 ) = det S det(A
I) det S
1
= det S det(A
I)
= det(A
I).
det S

I) = det(SAS
= det(S(A

Teorema 60. (i) Seja V um espao linear. Seja T : V ! V uma transformao


linear. Se T tiver valores prprios distintos 1 ; :::; k e se v1 ; :::; vk forem os vectores prprios
associados a cada um destes valores prprios, ento os vectores v1 ; :::; vk so linearmente
independentes.
(ii) Seja A uma matriz n n. Se A tiver valores prprios distintos 1 ; :::; k e se v1 ; :::; vk
forem os vectores prprios associados a cada um destes valores prprios, ento os vectores
v1 ; :::; vk so linearmente independentes.
Dem. (ii) Seja r = dim L (fv1 ; :::; vk g). Suponhamos que r < k. Suponhamos ainda, a
menos de uma reordenao, que o conjunto fv1 ; :::; vr g linearmente independente.
Como o conjunto fv1 ; :::; vr ; vr+1 g linearmente dependente, ento existem escalares no
todos nulos c1 ; :::; cr ; cr+1 tais que
c1 v1 +

+ cr vr + cr+1 vr+1 = 0. (*)

100

Note que tem que se ter cr+1 6= 0 caso contrrio o conjunto fv1 ; :::; vr g linearmente dependente. Logo
cr+1 vr+1 6= 0
e assim c1 ; :::; cr no podem ser todos nulos.
Por outro lado, atendendo a (*) tem-se
A (c1 v1 +

+ cr vr + cr+1 vr+1 ) = 0 ,c1 Av1 +


, c1

Logo, multiplicando
r+1

(c1 v1 +

+ cr

r vr

r+1

por (*) e somando a (**) tem-se:

, c1 (

r+1 ) v1

1 v1

r+1 vr+1

= 0 (**).

1 v1

+ cr vr + cr+1 vr+1 ) + c1

+ cr+1

+ cr Avr + cr+1 Avr+1 = 0 ,

+ cr (

+ cr
r

r vr

+ cr+1

r+1 ) vr

r+1 vr+1

=0,

=0

Assim, sendo os escalares c1 ; :::; cr no todos nulos e sendo os escalares 1 ; :::; k todos distintos, ento o conjunto fv1 ; :::; vr g seria linearmente dependente, contrariando a hiptese de
o mesmo ser linearmente independente. Logo, tem que se ter r = k.
Denio 50. (i) Seja A uma matriz n

n. Se existir uma matriz P invertvel tal que

D = P AP

com D matriz diagonal, ento diz-se que A uma matriz diagonalizvel e que P
a matriz diagonalizante. No caso de A ser uma matriz diagonal, a matriz diagonalizante
a matriz identidade.
(ii) Seja V um espao linear tal que dim V = n. Seja T : V ! V uma transformao
linear. Diz-se que T diagonalizvel se existir uma base ordenada B de V em relao
qual a matriz M (T ; B; B) que representa T nessa base seja uma matriz diagonal.
Teorema 61. Seja A 2 Mn n (R). A matriz A diagonalizvel se e s se existir uma
base Bvp de Rn apenas constituda por vectores prprios de A. Neste caso, as entradas da
diagonal principal da matriz diagonal D sero os valores prprios de A apresentados pela
ordem dos vectores prprios correspondentes na base ordenada Bvp . Alm disso, a matriz
P 1 ser a matriz cujas colunas sero os vectores prprios de A, da base Bvp de Rn dispostos
pela mesma ordem, tendo-se
D = P AP 1 .
O mesmo se aplica a Cn .
Teorema 62. Seja A uma matriz n

n. Ento as armaes seguintes so equivalentes:

(i) A diagonalizvel.
(ii) A tem n vectores prprios linearmente independentes.
(iii) A soma das multiplicidades geomtricas dos valores prprios de A n.
101

(iv) A multiplicidade geomtrica de cada valor prprio de A igual multiplicidade


algbrica desse valor prprio.
Observao 43. (i) Seja V um espao linear tal que dim V = n. Seja A = M (T; B; B)
a matriz n n que representa a transformao linear T : V ! V em relao base ordenada
B. No caso de haver uma base Bvp (ordenada) de V apenas constituda por vectores prprios
de T , ento tem-se
M (T; Bvp ; Bvp ) = P AP 1 ,
onde P 1 = SBvp !B , sendo deste modo M (T; Bvp ; Bvp ) a matriz diagonal cujas entradas da
diagonal principal so os valores prprios de A apresentados pela ordem dos vectores prprios
correspondentes na base Bvp . Assim, T representada relativamente a uma base ordenada
por uma matriz diagonal, isto , T diagonalizvel.
No caso de se ter V = Rn e B = B nc (base cannica ordenada) as colunas da matriz
1
P = SBvp !Bcn so os vectores prprios de A da base Bvp dispostos pela mesma ordem.

(ii) No caso de se ter D = P AP 1 , com P invertvel e D matriz diagonal, tem-se, para


k 2 N,
D k = P Ak P 1 ,
ou seja,
Ak = P 1 D k P .
Exemplo 46. Nos exemplos que se seguem as matrizes A consideradas podero ser vistas
como matrizes que representam transformaes lineares T relativamente base cannica (ou
outras) ordenada de R3 , tendo-se no caso da base cannica, para todo o v 2 R3 ,
T (v) = Av.
Deste modo, os valores prprios e vectores prprios de T sero respectivamente os valores
prprios e vectores prprios de A.
(i) Uma matriz com valores prprios distintos.
2
3
1
5
1
2 1 5.
A=4 0
4 0
3

O polinmio caracterstico dado por


1
det(A

0
4

I) =
=
=
=
=
=

(1
(1
(3
(3
(3

1
1

=
3
2
) (3
) 20 + 4 (2 + ) =
2
) (3
)+4
12 =
1) ( + 2) 4] =
2
+
6 =
2) ( + 3) .
0

)(
)(
) [(
)
)(

Os valores prprios de A so os valores de para os quais det(A


prprios de A so
3.
1 = 3,
2 = 2 e
3 =
102

I) = 0. Logo, os valores

so os vectores no nulos v 2 R3

Os vectores prprios de A associados ao valor prprio


para os quais
(A
I) v = 0,
isto , so os vectores no nulos de N (A
Determinemos os vectores prprios de A
02
2
@
4
0
N (A
I)
=
N
1
4

Logo, o subespao prprio E

dado por

= N (A

I).
associados ao valor prprio 1 = 3. Tem-se
31
5
1
5 1 5A = L (f(0; 1; 5)g) .
0
0

1 I)

= L (f(0; 1; 5)g) .

Os vectores prprios de A associados ao valor prprio

= 3 so

v = (0; s; 5s) , com s 2 Rn f0g .


Determinemos os vectores prprios de A associados ao valor prprio 2 = 2. Tem-se
02
31
1 5
1
@4 0
4 1 5A = L (f(1; 1; 4)g) .
N (A
2 I) = N
4 0
1

Logo, o subespao prprio E

dado por

= N (A

2 I)

= L (f(1; 1; 4)g) .

Os vectores prprios de A associados ao valor prprio

= 2 so

v = (s; s; 4s) , com s 2 Rn f0g .


Determinemos os vectores prprios de A
02
4
@
4
0
N (A
3 I) = N
4

Logo, o subespao prprio E


E

dado por

= N (A

associados ao valor prprio 3 =


31
5
1
1 1 5A = L (f(3; 2; 2)g) .
0 6

3 I)

3. Tem-se

= L (f(3; 2; 2)g) .

Os vectores prprios de A associados ao valor prprio

3 so

v = (3s; 2s; 2s) , com s 2 Rn f0g .


Atendendo a que os valores prprios de A so distintos, os vectores prprios de A associados a esses valores prprios so linearmente independentes. Como dim R3 = 3, ento 3
vectores em R3 linearmente independentes formaro desde logo uma base de R3 . Logo, o
conjunto
B = f(0; 1; 5) ; (1; 1; 4) ; (3; 2; 2)g
103

uma base de R3 . Deste modo, temos uma base de R3 formada s por vectores prprios de
A. Logo, a matriz A diagonalizvel, isto , existe uma matriz invertvel P diagonalizante
tal que a matriz P AP 1 diagonal, tendo-se
2
3 2
3
2
3
0 0
3 0 0
0 1 3
1
0 5 = 4 0 2 0 5 , com P 1 = 4 1 1
2 5.
D = P AP 1 = 4 0
2
0 0
0 0
3
5 4 2
3

Note que cada coluna de P 1 formada pelo vector prprio associado ao valor prprio
respectivo e na posio respectiva. Alm disso, tem-se
M (T ;Bc3 ;Bc3 )

(R3 ; Bc3 )

SBc3 !B # I
(R3 ; B)

(R3 ; Bc3 )

I # SBc3 !B
(R3 ; B)

M (T ;B;B)

com
SBc3 !B = P , M (T ; B; B) = D
(ii) Uma matriz com valores prprios
2
2
4
A= 2
3

O polinmio caracterstico dado por


2
det(A

1
2
3

I) =

3
3

= (2
) (3
3
=
+9 2
=
(
1) (

e M (T ; Bc3 ; Bc3 ) = A.

repetidos mas diagonalizvel.


3
1 1
3 2 5.
3 4

1
2

=
4
) (4
)+6+6
15 + 7 =
1) (
7) .

3 (3

Os valores prprios de A so os valores de para os quais det(A


prprios de A so
1 = 1 e
2 = 7.
Os vectores prprios de A associados ao valor prprio
para os quais
(A
I) v = 0,
isto , so os vectores no nulos de N (A
Determinemos os vectores prprios de
02
1 1
@
4
2 2
N (A
1 I) = N
3 3
Logo, o subespao prprio E
E

dado por

= N (A

1 I)

6 (2

2 (4

)=

I) = 0. Logo, os valores

so os vectores no nulos v 2 R3

I).
A associados ao valor prprio 1 = 1. Tem-se
31
1
2 5A = L (f( 1; 1; 0) ; ( 1; 0; 1)g) .
3

= L (f( 1; 1; 0) ; ( 1; 0; 1)g) .
104

Os vectores prprios de A associados ao valor prprio


v=( s

= 1 so

t; s; t) , com s e t no simultneamente nulos.

Determinemos os vectores prprios de A associados ao valor prprio 2 = 7. Tem-se


02
31
5 1
1
@4 2
4 2 5A = L (f(1; 2; 3)g) .
N (A
2 I) = N
3
3
3

Logo, o subespao prprio E

dado por
= N (A

2 I)

= L (f(1; 2; 3)g) .

Os vectores prprios de A associados ao valor prprio

= 7 so

v = (s; 2s; 3s) , com s 2 Rn f0g .


Atendendo a que
dim E

+ dim E

= 3,

podemos ter a seguinte base de R3 formada s por vectores prprios de A


B = f( 1; 1; 0) ; ( 1; 0; 1) ; (1; 2; 3)g .
Logo, a matriz A diagonalizvel, isto , existe
que a matriz P AP 1 diagonal, tendo-se
2
3 2
0 0
1 0
1
5
4
0
0 1
D = P AP 1 = 4 0
=
1
0 0
0 0
2

uma matriz invertvel P diagonalizante tal


3
0
0 5 , com P
7

1
=4 1
0

3
1 1
0 2 5.
1 3

Note que cada coluna de P 1 formada pelo vector prprio associado ao valor prprio
respectivo e na posio respectiva. Alm disso, tem-se
(R3 ; Bc3 )

SBc3 !B # I
(R3 ; B)

M (T ;Bc3 ;Bc3 )

M (T ;B;B)

(R3 ; Bc3 )

I # SBc3 !B
(R3 ; B)

com
SBc3 !B = P , M (T ; B; B) = D

e M (T ; Bc3 ; Bc3 ) = A.

(iii) Uma matriz com valores prprios repetidos e no diagonalizvel.


2
3
7 5
1
2 1 5.
A=4 0
20 0
3

105

O polinmio caracterstico dado por


7
det(A

5
0
20

I) =
=
=
=
=

(7
(3
(3
(3

1
1

=
3
)( 2
) (3
) + 100 20 (2 + ) =
) [(7
)( 2
) + 20] =
2
)
5 +6 =
)(
3) (
2) .
0

Os valores prprios de A so os valores de para os quais det(A


prprios de A so
1 = 3 e
2 = 2.

so os vectores no nulos v 2 R3

Os vectores prprios de A associados ao valor prprio


para os quais
(A
I) v = 0,
isto , so os vectores no nulos de N (A
Determinemos os vectores prprios de A
02
4
@
4
0
N (A
1 I) = N
20
Logo, o subespao prprio E

I).
associados ao valor prprio 1 = 3. Tem-se
31
5
1
5 1 5A = L (f(0; 1; 5)g) .
0
0

dado por
= N (A

I) = 0. Logo, os valores

1 I)

= L (f(0; 1; 5)g) .

Os vectores prprios de A associados ao valor prprio

= 3 so

v = (0; s; 5s) , com s 2 Rn f0g .


Determinemos os vectores prprios de A associados ao valor prprio 2 = 2. Tem-se
02
31
5 5
1
@4 0
4 1 5A = L (f(1; 5; 20)g) .
N (A
2 I) = N
20 0
1

Logo, o subespao prprio E


E

dado por

= N (A

2 I)

= L (f(1; 5; 20)g) .

Os vectores prprios de A associados ao valor prprio

= 2 so

v = (s; 5s; 20s) , com s 2 Rn f0g .


Atendendo a que
dim E

+ dim E

= 2 < 3,

no possvel ter uma base de R3 formada s por vectores prprios de A. Logo, a matriz
A no diagonalizvel, isto , no existe uma matriz invertvel P diagonalizante tal que a
matriz P AP 1 seja diagonal.
106

(iv) Uma matriz com apenas um valor prprio real.


2
3
1 0 0
1 5.
A=4 0 0
0 1 0

O polinmio caracterstico dado por

1
det(A

0
0
0

I) =

0
1

= 2 (1
) + (1
)=
2
= (1
)
+1 .
Os valores prprios de A so os valores de para os quais det(A
prprios de A so
i.
1 = 1,
2 = i e
3 =

I) = 0. Logo, os valores

Logo, a matriz A no diagonalizvel numa matriz de entradas reais, isto , no existe


uma matriz invertvel P diagonalizante tal que a matriz P AP 1 seja diagonal com entradas
reais. No entanto e atendendo a que os trs valores prprios so distintos, a matriz A
diagonalizvel numa matriz de entradas complexas:
2
3
1 0 0
4 0 i 0 5
0 0
i
Exemplo 47. A sucesso de Fibonacci (Leonardo de Pisa, 1202). Seja (vn )n2N tal que
v1 = 1;

v2 = 1

vn+2 = vn + vn+1 , n 2 N.

Considerando a igualdade vn+1 = vn+1 , podemos escrever o sistema


vn+1 = vn+1
vn+2 = vn + vn+1
isto
vn+1
vn+2

0 1
1 1

vn
vn+1

para todo o n 2 N. Aplicando sucessivamente a igualdade anterior tem-se


vn+1
vn+2

=
=

0 1
1 1
=

vn
vn+1
0 1
1 1

Calculemos agora os valores prprios de


det

1
1

=0,(

0 1
1 1

v1
v2

0 1
1 1

) (1
107

0 1
1 1

0 1
1 1
n

1=0,

vn 1
vn
1
1

1=0,

Valores prprios: 1 =
Atendendo a que
1

p
1+ 5
2

,
e

0 1+
1
1

=N

p
1+ 5
=
2

ou

=N

0
1

.
2
1

1
1

0p

=L

p
1+ 5
;1
2

1+
2

!)!

;1

um vector prprio associado ao valor prprio 1+2 5 , sendo todos os vectores


n
o
p
p
1+ 5
prprios associados ao valor prprio 1+2 5 dados por L
;
1
n f(0; 0)g.
2
Atendendo a que
(
!)!
p
2
0
0
1
+
5
1
0 1+ 2
2
2
p
=N
N
=N
=L
;1
1 1
1
1
1 1+2 5
2
2
2
p

p
1+ 5
;1
2

um vector prprio associado ao valor prprio 1 2 5 , sendo todos os vectores


n
o
p
p
1+ 5
prprios associados ao valor prprio 1 2 5 dados por L
;
1
n f(0; 0)g.
2

Como existe uma base de R2 formada s por vectores prprios (os dois valores prprios
so distintos logo os vectores prprios correspondentes so linearmente independentes) ento
0 1
a matriz
diagonalizvel. Assim, fazendo
1 1
P

tem-se
P =
e
D=

"

p
1+ 5
2
1

p
1+ 5
2

1
"

5
5p

5
5

0p

=P

#
1

=P

"

p
1+ 5
2

p
1+ 5
2

0 1
1 1

0p

2
p
1+ 5
2

#n

2
4

1
1

1
1

"

p
1+ 5
2

p
1+ 5
2

=P

0 1
1 1

=P

Logo
vn+1
vn+2

p
5+ 5
10
p
5
5
10

0 1
1 1

p
1+ 5
2

"

2
4

p
2

108

0p

2
p
1+ 5
2

p
1+ 5
2

0
n

P
#

P.

0p

0
3"
5

# !n
0

2
p

5
5p

5
5

1
1

p
5+ 5
10
p
5
5
10

1
1

5P
#

1
1

Isto ,

=4
vn+1 =

10
p
5 3 5
10

2
p

+
n

p
p
n
5+ 5 1+ 5
10
2
p
p
n
3 5+5 1+ 5
10
2

p !n
p
5
5+ 5
+
2
10

10

5.

p !n
1+ 5
2

para todo o n 2 N, com v1 = 1.


Verique que (por exemplo) v2 = 1, v3 = 2; v4 = 3:
Exemplo 48. (Um processo de difuso.) Considere duas clulas adjacentes separadas
por uma membrana permevel e suponha que um udo passa da 1a clula para a 2a a uma
taxa (em mililitros por minuto) numericamente igual a 4 vezes o volume (em mililitros) do
udo da 1a clula. Em seguida, passa da 2a clula para a 1a a uma taxa (em mililitros por
minuto) numericamente igual a 5 vezes o volume (em mililitros) do udo da 2a clula.
Sejam v1 (t) e v2 (t) respectivamente o volume da 1a clula e o volume da 2a clula no
instante t. Suponha que inicialmente a primeira clula tem 10 mililitros de udo e que a
segunda tem 8 mililitros de udo, isto v1 (0) = 10 e v2 (0) = 8.
Determinemos o volume de udo de cada clula no instante t.
Tem-se
8 0
< v1 (t) = 4v1 (t)
:

isto

4e

v10 (t)
v20 (t)

v20 (t) = 4v1 (t)

5v2 (t)

4
4

v1 (t)
v2 (t)

5 so os valores prprios da matriz

0
5
4
4

: (*)

0
, sendo os vectores prprios associados
5

(1; 4) e (0; 1) respectivamente.


Como existe uma base de R2 formada s por vectores prprios (os dois valores prprios
so distintos logo os vectores prprios correspondentes so linearmente independentes) ento
4 0
a matriz
diagonalizvel. Assim, fazendo
4
5
P

1 0
4 1

1 0
4 1

tem-se P =

e
4
0

,D=
,

4
4

0
5
0
5

4
4

=P
=P

4
0

0
5

0
5

P.

o sistema (*) equivalente a


v10 (t)
v20 (t)

4
0

109

0
5

v1 (t)
v2 (t)

v10 (t)
v20 (t)

,P

4
0

0
5

v1 (t)
v2 (t)

Assim, considerando a mudana de varivel


u1 (t)
u2 (t)
v10 (t)
v20 (t)

u01 (t)
u02 (t)

8 0
< u1 (t) =
:

=P

c1 e
c2 e

5u2 (t)

Se u1 6=0
e u2 6=0

u1 (t)
u2 (t)

8 0
u1 (t)
>
>
=
>
>
< u1 (t)
>
>
u0 (t)
>
>
=
: 2
u2 (t)

4t

5t

,
,
4
,
5

4t + k1

1 0
4 1

c1 e
c2 e

4t

5t

c1 e 4t
4c1 e 4t + c2 e

5t

8
< v1 (0) = 10
:

v2 (0) = 8

32 e assim a soluo geral do sistema de equaes diferenciais lineares


8 0
< v1 (t) = 4v1 (t)
:

com os valores iniciais

dada por

0
5

log ju2 (t)j = 5t + k2


8
< u1 (t) = c1 e 4t
,
:
u2 (t) = c2 e 5t

Como

ento c1 = 10 e c2 =

4
0

v1 (t)
v2 (t)

8
< log ju1 (t)j =

com c1 ; c2 2 R. Logo
v1 (t)
v2 (t)

0
5

4u1 (t)

u02 (t) =

4
0

v1 (t)
v2 (t)

=P

2
4

v20 (t) = 4v1 (t)

v1 (t)
v2 (t)

5v2 (t)

8
< v1 (0) = 10
:

v2 (0) = 8
2

5=4

10e
40e

110

4t

4t

32e

5t

5.

5a Ficha de exerccios para as aulas de problemas


2

9
4
1. Seja A = 6
3
vector prprio

3
8 7
5 4 5. Verique se 0 valor prprio de A e caso seja determine um
2 1
associado.

2. Sem calcular o polinmio caracterstico, indique um valor prprio e dois vectores


prprios associados linearmente independentes para a matriz
2
3
5 5 5
4 5 5 5 5:
5 5 5
3. Determine os valores prprios de uma matriz A 2
determinante seja igual a 6.

2 cujo trao seja igual a 5 e cujo

4. Determine uma matriz A real simtrica (AT = A) 2 2 cujos valores prprios sejam
2 e 2 e tal que (2; 1) seja um vector prprio associado ao valor prprio 2.
5. Considere a transformao linear T : R3 ! R3 que admite os vectores prprios
v1 = (1; 2; 1);

v2 = ( 1; 0; 1);

v3 = (0; 1; 0);

associados respectivamente aos valores prprios 1; 2 e 3.


Determine a expresso geral de T .
6. Considere a transformao linear T : R3 ! R3 denida por
T (x; y; z) = (0; y + 3z; 3y + z):
(i) Diga quais dos seguintes vectores:
v1 = (2; 1; 1); v2 = (0; 1; 1); v3 = (1; 0; 0); v4 = ( 1; 1; 3); v5 = (0; 3; 3)
so vectores prprios.
(ii) Determine os valores prprios de T .
(iii) Diga, justicando, se T invertvel e se T diagonalizvel.
(iv) Determine os subespaos prprios de T .
7. Considere a transformao linear T : R2 ! R2 denida por T (1; 2) = (5; 5) = T (2; 1).
(i) Verique que os vectores v1 = (1; 1) e v2 = (1; 1) so vectores prprios de T .

(ii) Diga, justicando, se T invertvel e se T diagonalizvel.


(iii) Indique uma base ordenada de R2 relativamente qual a matriz que representa
T seja uma matriz diagonal.
(iv) Determine os valores prprios e os subespaos prprios de T .
111

8. Considere a transformao linear T : R3 ! R3 que em relao base cannica de R3


representada pela matriz:
2
3
0 1 0
A = 4 0 1 0 5.
0 1 0

(i) Verique que os vectores v1 = (1; 0; 0); v2 = (1; 1; 1) e v3 = (0; 0; 1) so vectores


prprios de T .
(ii) Diga, justicando, se T invertvel e se T diagonalizvel.
(iii) Determine os valores prprios e os subespaos prprios de T .

(iv) Diagonalize T . Isto , determine uma matriz de mudana de base P


matriz diagonal D tais que
D = P AP 1 :

e uma

9. Considere a transformao linear T : R2 ! R2 que em relao base ordenada


f(1; 2) ; (2; 1)g de R2 representada pela matriz:
A=

2 3
3 2

(i) Determine os valores prprios de T e diga, justicando, se T invertvel e se T


diagonalizvel.
(ii) Determine bases para os subespaos prprios de T .
(iii) Diagonalize a transformao linear T , isto , determine uma base ordenada de R2
relativamente qual a matriz que represente T seja uma matriz diagonal.
10. Seja V um espao linear de dimenso nita. Seja T : V ! V uma transformao
linear tal que T 2 = T . Uma tranformao linear nas condies anteriores chama-se
projeco.
(i) Mostre que os valores prprios de T so 0 e 1:
(ii) Justique que T diagonalizvel.
11. Considere a transformao linear T : R3 ! R3 denida por T (x; y; z) = (x; y; x

y).

(i) Determine os valores prprios e os subespaos prprios de T .

(ii) A transformao linear T representa geometricamente uma projeco sobre um


plano, paralelamente a um vector. Determine esse plano e esse vector.
12. Considere a transformao linear T : R3 ! R3 que representa geometricamente a
projeco sobre o plano x + y + z = 0, paralelamente ao vector (0; 0; 1).
(i) Explique o signicado do plano e do vector referidos no enunciado.
(ii) Determine a expresso geral de T .
13. Considere a transformao linear T : R2 ! R2 que em relao base cannica de R2
representada pela matriz:
2 1
A=
.
0 2
112

(i) Determine os valores prprios e os subespaos prprios de T .


(ii) Mostre que no existe nenhuma base de R2 constituda por vectores prprios de
T . T diagonalizvel?
14. Considere a transformao linear T : R3 ! R3 denida por
T (x; y; z) = (3x; 2y + z; 2z):
(i) Determine os valores prprios e bases dos subespaos prprios de T .
(ii) Mostre que no existe nenhuma base de R3 em relao qual T possa ser representada por uma matriz diagonal.
15. Considere a transformao linear T : R3 ! R3 denida por
T (x; y; z) = (y + z; 2y + z; y + 2z).
(i) Determine o polinmio caracterstico de T .
(ii) Determine os valores prprios e bases dos subespaos prprios de T .
(iii) Determine uma base de R3 constituda por vectores prprios de T . Determine a
matriz que representa T nesta base ordenada.
(iv) Seja A a matriz que representa T na base cannica de R3 , isto , A = M (T ; Bc3 ; Bc3 ).
Diagonalize a matriz A. Isto , determine uma matriz de mudana de base P 1 e uma
matriz diagonal D tais que D = P AP 1 .
(v) Determine An e T n (x; y; z).
16. Considere a transformao linear T : R3 ! R3 que em relao base ordenada
f(0; 1; 0) ; (1; 0; 1) ; (1; 0; 1)g de R3 representada pela matriz:
2
3
7 4 2
1 5.
A=4 1 7
1 2 10
(i) Determine o polinmio caracterstico de T .
(ii) Determine os valores prprios e bases dos subespaos prprios de T .
(iii) Diagonalize a transformao linear T , isto , determine uma base ordenada de R3
relativamente qual a matriz que represente T seja uma matriz diagonal.
(iv) Determine An e T n (x; y; z).
17. Sabendo que os vectores (1; 1; 1); (1; 0; 1) e
2
1 1
4 a b
d e
determine a; b; c; d; e; f .

113

(1; 1; 0) so vectores prprios da matriz


3
1
c 5,
f

18. Considere a transformao linear T : M2 2 (R) ! M2 2 (R) denida por


T (A) = A + AT :
(i) Escolha uma base ordenada para M2 2 (R) e determine a matriz que representa T
em relao a essa base ordenada.
(ii) Determine os valores prprios e os vectores prprios de T .
(iii) Diga se T pode ou no ser representada por uma matriz diagonal em relao a
uma base ordenada apropriada de M2 2 (R). Em caso armativo, indique uma tal base
ordenada e a correspondente matriz diagonal que representa T .
2
3
2
3
2 1 1
1 1 0
2 1
19. Considere as matrizes A1 =
; A2 = 4 0 3 1 5 ; A3 = 4 1 1 0 5.
2 5
0 1 3
0 0 0
Verique que A1 ; A2 e A3 so diagonalizveis. Isto , determine matrizes de mudana
de bases P1 1 ; P2 1 e P3 1 e matrizes diagonais D1 ; D2 e D3 tais que
D1 = P1 A1 P1 1 , D2 = P2 A2 P2

e D3 = P3 A3 P3 1 .

Ou seja, verique que existe uma base de R2 formada por vectores prprios
de A1 , uma base de R3 formada por vectores prprios de A2 e outra base de
R3 formada por vectores prprios de A3 .
20. Considere a transformao linear T : R4 ! R4 que em relao base cannica de R4
representada pela matriz
2
3
0 0 0 0
6 a 0 0 0 7
6
7
4 0 b 0 0 5,
0 0 c 0

com a; b; c 2 R. Determine os valores de a; b; c de modo a que exista uma base de R4


constituda s por vectores prprios de T .
2
3
2 3
2
3
1 1
1
1
21. Para cada parmetro 2 R, sejam A = 4 0 3 0 5, u1 = 4 0 5, u2 = 4 0 5 :
1 0
1
1

(i) Prove que u1 e u2 so vectores prprios de A. Determine os valores prprios


associados.
(ii) Determine os valores prprios de A e indique os valores de
3 valores prprios todos distintos.

para os quais A tem

(iii) Determine, em funo de , bases para os espaos prprios associados.


(iv) Identique, justicando, os valores de

114

para os quais a matriz A diagonalizvel.

3a Ficha de exerccios facultativos


1. Seja T : Rn ! Rn uma transformao linear invertvel. Seja u um vector prprio de T
associado a um valor prprio de T . Verique que u tambm um vector prprio de
T 1 e determine o valor prprio de T 1 que lhe est associado.
2. Seja V um espao linear. Seja T : V ! V uma transformao linear. Seja u um vector
prprio de T associado a um valor prprio de T . Verique que u tambm um vector
prprio de T 2 associado ao valor prprio 2 de T 2 .
k

3. Seja A uma matriz do tipo n n. Mostre que se um valor prprio de A ento


um valor prprio de Ak , onde k um inteiro positivo.

4. Uma matriz A do tipo n n diz-se nilpotente se Al = 0 para algum inteiro positivo l.


Mostre que se A nilpotente ento o nico valor prprio de A 0.
5. Seja A uma matriz n

n. Verique que A e AT tm os mesmos valores prprios.

6. Seja A uma matriz n n cuja soma das suas colunas constante e igual a r. Mostre
que r um valor prprio de A:
7. Seja A 2 Mn n (R). Seja P uma matriz diagonalizante para A. Determine uma matriz
diagonalizante para AT em termos de P .
8. Seja Q uma matriz n

n real ortogonal, isto , tal que Q

= QT .

Mostre que se n fr mpar ento Q tem o valor prprio 1 ou tem o valor prprio
9. Determine uma matriz A real 2

1.

2 tal que det A < 0. Mostre que A diagonalizvel.

10. Seja A uma matriz n n e seja um valor prprio de A com multiplicidade algbrica
igual a n. Mostre que se A fr diagonalizvel ento A uma matriz diagonal.
11. Seja V um espao linear e seja T : V ! V uma transformao linear tal que todos
os vectores no nulos de V so vectores prprios. Mostre que T tem um nico valor
prprio.
12. Sejam A e B duas matrizes do tipo n n. Mostre que AB e BA tm os mesmos valores
prprios.
13. Sejam A e B duas matrizes tais que AB = BA. Mostre que A e B tm um vector
prprio em comum.
Sugesto: Sendo um valor prprio de A, considere C a matriz cujas colunas formam
uma base ordenada S de N (A
I) e verique que (A
I) BC = 0. Finalmente
considere a matriz P cujas colunas so respectivamente as coordenadas das colunas de
BC em relao base S e sendo v um vector prprio de P mostre que Cv um vector
prprio comum a A e B.
14. Seja A uma matriz n

n e sejam
(A

1;

escalares, com

1 I) (A

Mostre que A diagonalizvel.

115

2 I)

= 0:

6=

2,

tais que

Produtos internos e ortogonalizao


Denio 51. Sejam V um espao linear real e 0 o vector nulo de V . Chama-se produto
interno em V a uma aplicao
h; i : V

V !R

(u; v) ! hu; vi
que verique as trs condies seguintes.
(i) Simetria: para todos os u; v 2 V
hu; vi = hv; ui .
(ii) Linearidade: para todo o v 2 V (xo) a aplicao
V !R
u ! hu; vi
linear.
(iii) Positividade: para todo o u 2 V tal que u 6= 0,
hu; ui > 0.
Tendo-se hu; ui = 0 se e s se u = 0.
Observao 44. (a) Um produto interno num espao linear real uma forma bilinear,
simtrica e denida positiva.
(b) Num espao linear V sobre C (espao linear complexo), um produto interno uma
aplicao que a cada par de vectores (u; v) 2 V V associa o nmero complexo hu; vi e que
verica as seguintes condies:
(i) Para todos os u; v 2 V

hu; vi = hv; ui.

(ii) Para todo o v 2 V (xo) tem-se


h u + w; vi =
para todos os u; w 2 V e ;
para todo o u 2 V (xo)

hu; vi + hw; vi

2 C, (onde por exemplo


V !C
v ! hu; vi

linear.
116

=a

bi se

= a + bi) e a aplicao,

(iii) Para todo o u 2 V tal que u 6= 0,


hu; ui > 0.
Tendo-se hu; ui = 0 se e s se u = 0.
(c) A um espao linear real de dimenso nita com um produto interno chama-se espao
euclidiano. A um espao linear complexo de dimenso nita com um produto interno
chama-se espao unitrio.
Observao 45. (i) Seja V um espao linear real. Seja B = fw1 ; w2 ; :::; wn g uma base
ordenada de V . Sejam u; v 2 V . Sejam
1;

2 ; :::;

1;

2 ; :::;

as coordenadas de u e de v na base ordenada B respectivamente, isto ,


u=

1 w1 +

2 w2 + ::: +

n
X

n wn =

i wi

i=1

e
v=

1 w1

2 w2

+ ::: +

n wn

n
X

i wi .

i=1

Logo,
hu; vi =

:::

* n
X
i=1

i wi ;

2
6
6
6
4

n
X
i=1

i wi

n X
n
X
i=1 j=1

i j

hwi ; wj i =

hw1 ; w1 i hw1 ; w2 i : : : hw1 ; wn i


hw2 ; w1 i hw2 ; w2 i : : : hw2 ; wn i
..
..
..
.
.
.
hwn ; w1 i hwn ; w2 i : : : hwn ; wn i

32

76
7
76 2 7
7 6 .. 7 .
54 . 5
n

Assim, a aplicao h; i : V V ! R que a cada (u; v) 2 V V faz corresponder hu; vi, um


produto interno em V se e s se a matriz
2
3
hw1 ; w1 i hw1 ; w2 i : : : hw1 ; wn i
6 hw2 ; w1 i hw2 ; w2 i : : : hw2 ; wn i 7
6
7
G=6
7
..
..
..
4
5
.
.
.
hwn ; w1 i hwn ; w2 i : : : hwn ; wn i
fr simtrica (G = GT ) e denida positiva (uT Gu > 0, para todo o u 6= 0). Note-se que a
linearidade consequncia das propriedades referentes s operaes matriciais envolvidas.
(ii) matriz G anterior d-se o nome de matriz da mtrica do produto interno.
(iii) Num prximo captulo, como consequncia da diagonalizao ortogonal, sendo G
simtrica (G = GT ), ser estabelecida a equivalncia:
(uT Gu > 0, para todo o u 6= 0) , (todos os valores prprios de G so positivos).
117

(iv) Observe-se ainda que no caso de se ter um espao unitrio pode-se encontrar uma
T
matriz G cujos valores prprios sejam todos positivos e tal que G = G , (onde G a matriz
que se obtem de G passando todas as entradas desta ao complexo conjugado), tendo-se
2
3
1

hu; vi =

:::

6
7
6 2 7
G 6 .. 7 .
4 . 5

Uma matriz A que satisfaa a condio A = A diz-se hermitiana.


Teorema 63. Seja V um espao linear real com dim V = n. Seja fw1 ; w2 ; :::; wn g uma
base ordenada de V . Ento, uma aplicao
h; i : V

V !R

um produto interno (em V ) se e s se

hu; vi =

:::

7
6
6 2 7
G 6 .. 7 ,
4 . 5

com
u=

1 w1

2 w2

+ ::: +

n wn

v=

1 w1

2 w2

+ ::: +

n wn .

e G uma matriz simtrica cujos valores prprios so todos positivos. Se a aplicao h; i fr


um produto interno tem-se
2
3
hw1 ; w1 i hw1 ; w2 i : : : hw1 ; wn i
6 hw2 ; w1 i hw2 ; w2 i : : : hw2 ; wn i 7
6
7
G=6
7.
..
..
..
4
5
.
.
.
hwn ; w1 i hwn ; w2 i : : : hwn ; wn i
Exemplo 49. (i) Seja h; i : R2
h(

1;

R2 ! R a aplicao denida por:


2) ; ( 1;

2 )i

1 1

2 2,

com ( 1 ; 2 ) ; ( 1 ; 2 ) 2 R2 . Esta aplicao um produto interno em R2 a que se d o nome


de produto interno usual em R2 , uma vez que
h(

1;

2) ; ( 1;

2 )i

1 1

2 2

1 0
0 1
118

1
2

com
G=

A matriz G simtrica e o nico valor prprio de G 1 > 0.


(ii) Seja h; i : R2

R2 ! R a aplicao denida por:


h(

com (

1;

2) ; ( 1;

h(

2)
1;

1;

2) ; ( 1;

2 )i

1 1

2 2,

+3

2 R2 . Esta aplicao no um produto interno em R2 , uma vez que


2) ; ( 1;

2 )i

1 1

+3

2 2

1
2

com
2 0
0 3

G=

A matriz G simtrica, no entanto, os valores prprios de G:

2 e 3 no so ambos positivos.

(iii) O produto interno usual em Rn dado por:


h; i : Rn

onde u =

u1 u2 : : : un

6
6
ev=6
4

v1
v2
..
.
vn

Rn ! R
(u; v) ! hu; vi = uT v,
3
7
7
7.
5

(iv) O produto interno usual em Cn dado por:


h; i : Cn

onde u = u =

u1 u 2 : : : u n

(v) Um produto interno em Mm


h; i : Mm

Cn ! C

(u; v)
2
v1
6 v2
6
e v = 6 ..
4 .
vn
n

(R).

(R)

! hu; vi = uH v,
3
7
7
7.
5

Mm

(A; B) ! hA; Bi =

(R) ! R
m X
n
X

aij bij = tr AT B .

i=1 j=1

(vi) Um produto interno em C ([a; b]).


h; i : C ([a; b])

C ([a; b]) ! R
Z b
(f; g) ! hf; gi =
f (x) g (x) dx.
a

119

Exemplo 50. R2 com um produto interno no usual. Seja h; i : R2


aplicao denida por:
h(

1;

2) ; ( 1;

2 )i

=2

1 1

1 2

2 2,

+3

2 1

com ( 1 ; 2 ) ; ( 1 ; 2 ) 2 R2 .
fcil ver que esta aplicao simtrica e linear em relao a (
Vejamos por exemplo que a condio
h(

1;

> 0, para todo o (

1;

=2

2
1

+(

h(

1;

2) ; ( 1;

2 )i

R2 ! R a

1;

2)

(xando (

1;

2 )).

6= (0; 0),

2)

satisfeita.
Atendendo a que
h(

1;

2) ; ( 1;

2 )i

+2

+3

2
2

2
1

2
2)

+2

2
2,

tem-se
,(
,(

2) ; ( 1;

=0 e

=0 e

1
2

2 )i

=0,

=0 e

= 0) , (

1;

2
2)

= 0)

= (0; 0).

Em alternativa, podemos escrever


h(

1;

2) ; ( 1;

2 )i

=2

1 1

1 2

2 1

+3

2 2

+
2

com
2 1
1 3

G=

A matriz G simtrica e os valores prprios de G:

.
p
5+ 5
2

so ambos positivos.

Denio 52. Sejam V um espao linear com um produto interno e 0 o vector nulo de
V . Sejam u; v 2 V .
(i) Chama-se norma de u a:
kuk =

hu; ui:

(ii) Chama-se projeco ortogonal de v sobre u 6= 0 a:


proju v =

hv; ui
u:
kuk2

(iii) Diz-se que u e v so ortogonais se hu; vi = 0.


(iv) Chama-se ngulo entre dois vectores no nulos u e v tais que hu; vi 2 R a:
= arccos

hu; vi
.
kuk kvk

120

Note que este ngulo est bem denido atendendo ao prximo teorema.
Observao 46. (i) O ngulo
so ortogonais.

entre dois vectores no nulos u e v

se e s se u e v

(ii) Para cada u 2 V (xo) com u 6= 0, a aplicao proju : V ! V que a cada v 2 V faz
corresponder proju v, uma transformao linear.
Teorema 64. Desigualdade de Cauchy-Schwarz. Seja V um espao linear com um
produto interno. Ento, para todos os u; v 2 V ,
jhu; vij

kuk kvk

Observao 47. (i) Teorema de Pitgoras. Sejam u; v 2 R2 . Tem-se u e v ortogonais


se e s se
ku vk2 = kuk2 + kvk2 .
Dem.
vk2 = hu v; u vi =
= hu; ui hv; ui hu; vi + hv; vi =
= kuk2 2 hu; vi + kvk2 = kuk2 + kvk2

ku

se e s se
hu; vi = 0,
isto , se e s se u e v forem ortogonais.
(ii) Em R2 com o produto interno usual, a desigualdade de Cauchy-Schwarz dada por
q
q
2
2
2
2
j 1 1 + 2 2j
1+ 2
1 + 2,

uma vez que


com (

1;

2) ; ( 1;

2)

2 R2 .

h(

1;

2) ; ( 1;

2 )i

1 1

2 2,

(iii) Em Rn com o produto interno usual, a desigualdade de Cauchy-Schwarz dada por


v
v
u n
u n
n
X
uX uX
2
2t
t
i i
i,
i
i=1

i=1

i=1

uma vez que

h(
com (

1 ; :::;

n ) ; ( 1 ; :::;

1 ; :::;
n)

n ) ; ( 1 ; :::;

n )i

2 Rn .
121

1 1

+ ::: +

n n,

Teorema 65. Sejam V um espao linear com um produto interno e 0 o vector nulo
de V . Sejam u; v 2 V e escalar. A norma uma aplicao kk : V ! R que satisfaz as
seguintes propriedades.
(i) Positividade: kuk > 0 se u 6= 0.
(ii) Homogeneidade: k uk = j j kuk
(iii) Desigualdade triangular: ku + vk

kuk + kvk

Observao 48. Pode denir-se norma num espao linear V , sem estar associada a
qualquer produto interno, como sendo uma aplicao de V em R que satisfaz as propriedades
do teorema anterior. A um espao linear com uma norma chama-se espao normado.
Observao 49. Seja V um espao linear real com um produto interno. Sejam u; v 2 V .
Tem-se
hu; vi =

1
ku + vk2
2

kuk2

kvk2 .

Observao 50. Seja V um espao normado. Sejam u; v 2 V . Ento, a norma pode


dar origem a um produto interno se e s se
ku

vk2 + ku + vk2 = 2 kuk2 + 2 kvk2 .

Esta ltima equao conhecida por lei do paralelogramo.


Exemplo 51. Uma norma que no d origem a um produto interno. Seja
kk : R2 ! R a aplicao denida por
k(

1;

2 )k

=j

1j

+j

2j ,

com ( 1 ; 2 ) 2 R2 . fcil vericar que esta aplicao satisfaz as trs condies da norma.
Logo, uma norma. No entanto, tambm fcil vericar que esta norma no satisfaz a lei
do paralelogramo. Logo, esta norma no poder originar um produto interno.
Denio 53. Sejam V um espao euclidiano (ou unitrio) e S
ortogonal se para todos os u; v 2 S com u 6= v, se tiver

V . Diz-se que S

hu; vi = 0.
Diz-se que S ortonormado se fr ortogonal e se, para todo o u 2 S, se tiver
kuk = 1.
Teorema 66. Sejam V um espao linear com um produto interno e S
V . Seja 0
o vector nulo de V . Se S ortogonal e 0 2
= S ento S linearmente independente. Em
122

particular, se n = dim V ento qualquer conjunto S ortogonal de n vectores no nulos uma


base de V .
Teorema 67. Seja V um espao euclidiano (ou unitrio) com dim V = n. Seja B =
fu1 ; :::; un g uma base (ordenada) ortogonal de V . Ento, as coordenadas de um vector v 2 V
em relao base (ordenada) B so dadas por:
j

hv; uj i
,
huj ; uj i

com j = 1; :::; n. Se B fr ortonormada ento as coordenadas de um vector v 2 V em relao


base (ordenada) B so dadas por:
j

= hv; uj i ,

com j = 1; :::; n.
Teorema 68. Seja V um espao euclidiano com dim V = n. Seja B = fw1 ; :::; wn g uma
base (ordenada) ortonormada de V . Ento, para todos os u; v 2 V , tem-se
hu; vi =

n
X
i=1

hu; wi i hv; wi i

(frmula de Parseval)

v
u n
uX
kuk = t
hu; wi i2 :
i=1

Observao 51. Seja V um espao euclidiano com dim V = n. Seja B = fw1 ; :::; wn g
uma base (ordenada) ortonormada de V . Sejam u; v 2 V , com
u=

1 w1

2 w2

+ ::: +

n wn

v=

1 w1

2 w2

+ ::: +

n wn .

Ento a frmula de Parseval dada por:


hu; vi =
e tem-se

n
X

i i

1 1

2 2

+ ::: +

n n

i=1

v
u n
uX
kuk = t

2
i.

i=1

Notao 3. Sejam V um espao euclidiano (ou unitrio) e 0 o vector nulo de V . Para


1
v
qualquer v 2 V , com v 6= 0, o vector
v ser denotado por
.
kvk
kvk
123

Teorema 69. Mtodo de ortogonalizao de Gram-Schmidt. Seja V um espao


euclidiano (ou unitrio) no nulo. Ento V tem bases ortonormadas. Mais concretamente,
considere o conjunto linearmente independente:
fv1 ; v2 ; :::; vk g

e sejam
u1 = v 1 ,
u2 = v2 proju1 v2 ,
:::
uk = vk proju1 vk

:::

projuk

vk

ento
(i) L(fu1 ; u2 ; :::; uk g) = L(fv1 ; v2 ; :::; vk g);
(ii) o conjunto fu1 ; u2 ; :::; uk g uma base ortogonal de L(fv1 ; v2 ; :::; vk g).
(iii) o conjunto

u1
u2
uk
;
; :::;
ku1 k ku2 k
kuk k

uma base ortonormada de L(fv1 ; v2 ; :::; vk g).

Exemplo 52. Considere-se R4 com o produto interno usual. Seja


U = L(f(1; 1; 1; 1); (1; 2; 3; 4); (2; 1; 6; 7); (1; 3; 7; 9)g).
Determinemos a dimenso
2
1 1 2
6 1 2 1
6
4 1 3
6
1 4
7

de U e uma base ortonormada para U . Tem-se


3
2
3
2
1
1 1 2 1
1 1 2 1
7
6
7
6
3 7
0 1
1 2 7
0 1
1 2
!6
!6
5
4
5
4
7
0 4
4 8
0 0 0 0
9
0 5
5 10
0 0 0 0

7
7.
5

Logo, o conjunto fv1 ; v2 g, com v1 = (1; 1; 1; 1) e v2 = (1; 2; 3; 4), uma base de U e como
tal dim U = 2.
Sejam u1 = v1 e u2 = v2 proju1 v2 .
Logo, o conjunto fu1 ; u2 g, com u1 = (1; 1; 1; 1) e
1+2

(1; 1; 1; 1) = (2; 3; 2; 3),


4
uma base ortogonal de U . Uma base ortonormada para U :
(
p
p
p
p !)
u1
u2
1 1 1 1
26 3 26 26 3 26
;
;
=
; ;
;
;
;
;
ku1 k ku2 k
2 2 2 2
13
26
13
26
u2 = (1; 2; 3; 4)

Teorema 70. Seja B = fu1 ; u2 ; :::; un g uma base (ordenada) de um espao euclidiano
(ou unitrio). A base B ortonormada se e s se a matriz da mtrica G em relao a essa
base fr a matriz identidade. Em Rn o produto interno usual aquele em relao ao qual a
base cannica ortonormada.
124

Teorema 71. Seja fv1 ; v2 ; :::; vn g uma base (ordenada) de Rn . Ento, existe um nico
produto interno em Rn para o qual esta base ortonormada.
Exemplo 53. Considere em R2 a base (ordenada) B = fv1 ; v2 g, com v1 = (1; 0) e v2 =
(1; 1). Vejamos que existe um e um s produto interno para o qual a base B ortonormada.
Seja Bc2 = f(1; 0); (0; 1)g a base cannica de R2 . Sejam u; v 2 R2 , com u = ( 1 ; 2 ) e
v = ( 1 ; 2 ), onde 1 ; 2 e 1 ; 2 so as coordenadas na base Bc2 de u e v respectivamente.
Logo, a aplicao h; i : R2 R2 denida por
T

hu; vi = h(
=

1
0

1
1

1;

2) ; ( 1;
T

1
2

2 )i

1 0
0 1
=

SBc2 !B
1
0

1
1

1
2

SBc2 !B

1 2

2 1

+2

p
3+ 5
2

1 1

1
1

1 0
0 1

2 2

1
2

um produto interno e o nico para o qual a base B ortonormada, onde


S

Bc2 !B

NOTE QUE: sendo G =

= S

B!Bc2

1
1

1
2

1 1
0 1

1
0

1
1

( simtrica e os valores prprios

p
2

so

1 0
( simtrica e o nico
0 1
valor prprio 1 positivo) a matriz da mtrica em relao a B, tem-se
ambos positivos) a matriz da mtrica em relao a Bc2 e G0 =
G = SBc2 !B

G0 SBc2 !B

fcil vericar que para este produto interno a base B ortonormada:


h(1; 0) ; (1; 1)i = 0 e h(1; 0) ; (1; 0)i = h(1; 1) ; (1; 1)i = 1.
Denio 54. Sejam V um espao euclidiano (ou unitrio) e U um subespao de V .
Diz-se que um elemento de V ortogonal a U se fr ortogonal a todos os elementos de U .
Ao conjunto de todos os elementos ortogonais a U chama-se complemento ortogonal de
U e designa-se por U ? ,
U ? = fv 2 V : hv; ui = 0 para todo o u 2 U g .
Observao 52. Seja V um espao linear com produto interno.
(i) Qualquer que seja o subespao U de V , tambm U ? um subespao de V .
(ii) Sendo S um subconjunto de V , no necessariamente um subespao de V , (tambm)
pode denir-se S ? :
S ? = fv 2 V : hv; ui = 0 para todo o u 2 Sg .
125

Apesar de S no ser necessariamente um subespao de V , S ? sempre um subespao de V ,


tendo-se
S ? = (L (S))? .
Teorema 72. Seja V um espao linear com produto interno.
(i) Seja U um subespao de V . Tem-se
U \ U ? = f0g :
(ii) Seja S um subconjunto de V . Ento
S

S?

No prximo teorema ver-se- que no caso de se ter dim V < 1, ento


L (S) = S ?

ou ainda, sendo U um subespao de V (de dimenso nita)


U = U?

(iii) Sejam S1 ; S2 subconjuntos de V . Ento


S1

S2 ) (S2 )?

(S1 )?

(iv) Seja U um subespao de V . Se fv1 ; v2 ; :::; vn g uma base de U ento


U ? = fv 2 V : hv; v1 i = hv; v2 i = ::: = hv; vn i = 0g .
(v) Sejam U1 ; U2 subespaos de V . Tem-se
(U1 + U2 )? = (U1 )? \ (U2 )?
e
(U1 \ U2 )?
Se dim V < 1 tem-se

(U1 )? + (U2 )? .

(U1 \ U2 )? = (U1 )? + (U2 )? .

Exemplo 54. (i) Se U R3 um plano que passa pela origem, ento U ? uma recta
que passa pela origem e perpendicular ao plano.
(ii) Se U R3 uma recta que passa pela origem, ento U ? um plano que passa pela
origem e perpendicular recta.
(iii) Seja A 2 Mm

n (R).

Ento,

N (A) = (L(A))? = C(AT )


(iv) Seja A 2 Mn

n (R)

N (AT ) = L(AT )

= (C(A))? :

tal que A invertvel. Ento, (N (A))? = Rn e (L (A))? = f0g :


126

Teorema 73. Se U um subespao de um espao euclidiano (ou unitrio) V , ento V


a soma directa de U e U ? , isto , V = U U ? . Logo, cada elemento v 2 V pode ser escrito
de modo nico como soma de um elemento de U com um elemento de U ? :
v = vU + vU ? , com vU 2 U

e vU ? 2 U ? .

transformao linear PU : V ! V denida por


PU (v) = vU
e tal que PU = PU PU = (PU )2 chama-se projeco ortogonal de V sobre U e
transformao linear PU ? : V ! V denida por
PU ? (v) = vU ?
e tal que PU ? = PU ? PU ? = (PU ? )2 chama-se projeco ortogonal de V sobre U ? .
Tem-se
?
I = PU + PU ? ,
dim U + dim U ? = dim V;
U? = U
De facto, se fw1 ; w2 ; :::; wl g fr uma base ortogonal de U , ento
PU (v) =

l
X
hv; wi i
i=1

kwi k2

wi =

l
X

projwi v = vU

i=1

N (PU ) = U ?

I (PU ) = PU (V ) = U

para todo o v 2 V .
Se fu1 ; u2 ; :::; uk g uma base ortogonal de U ? , ento, para todo o v 2 V
PU ? (v) =

k
X
hv; uj i
j=1

kuj k2

uj =

I (PU ? ) = PU ? (V ) = U ?

k
X

projuj v = vU ?

j=1

N (PU ? ) = U

Neste caso, fw1 ; w2 ; :::; wl ; u1 ; u2 ; :::; uk g uma base ortogonal de V .


Tem-se ainda:
(i) hPU (u) ; vi = hu; PU (v)i,
hPU ? (u) ; vi = hu; PU ? (v)i, para todos os u; v 2 V ;
2
2
(ii) kuk = kPU (u)k + kPU ? (u)k2 , para todo o u 2 V (Teorema de Pitgoras);
Teorema 74. Seja U um subespao de dimenso nita de um espao euclidiano (ou
unitrio) V . Seja v 2 V . Ento, existe um elemento de U mais prximo de v do que
qualquer dos outros pontos de U . Este elemento a projeco ortogonal PU (v) de v
sobre U e tem-se
kv PU (v)k kv uk ,
para todo o u 2 U , e a igualdade verica-se se e s se u = PU (v). Alm disso, tendo-se
0 2 U , a distncia d de um ponto v 2 V a um subespao U dada por:
d (v; U ) = kPU ? (v

0)k = kPU ? (v)k = kv

127

PU (v)k .

Denio 55. Seja V um espao euclidiano (ou unitrio). Seja U um subespao de V


com dim U = k. Seja q 2 V . Chama-se ao conjunto
fqg + U
um k-plano. A distncia d de um ponto p 2 V a um k-plano P = fqg + U dada por:
d (p; P) = kPU ? (p

q)k .

Observao 53. A distncia entre dois k-planos paralelos


P1 = fpg + U

e P2 = fqg + U

dada por:
d (P1 ; P2 ) = kPU ? (p

q)k .

Exemplo 55. Considere-se R3 com o produto interno usual.


(i) Seja P o plano (em R3 ) que passa pelos pontos:
(1; 2; 1), (1; 0; 1) e (1; 1; 1):
Tem-se
P = f(1; 2; 1)g + L (f(0; 2; 2); (0; 1; 0)g)
uma vez que
(0; 2; 2) = (1; 0; 1)

(1; 2; 1)

(0; 1; 0) = (1; 1; 1)

(1; 2; 1).

Equao vectorial de P:
(x; y; z) = (1; 2; 1) + (0; 2; 2) + (0; 1; 0),
com ;

2 R.

Equaes paramtricas de P:
8
< x=1
y=2
:
z=1

com ;

2
2

2 R.

Equao cartesiana de P:

x = 1.

Podemos determinar uma equao cartesiana de P do seguinte modo. Atendendo a


que
P = f(1; 2; 1)g + L (f(0; 2; 2); (0; 1; 0)g)
128

seja
U = L (f(0; 2; 2); (0; 1; 0)g) .
Logo,
U ? = (x; y; z) 2 R3 : h(x; y; z); (0; 2; 2)i = 0 e h(x; y; z); (0; 1; 0)i = 0 =
0
0

=N

2
1

2
0

= L (f(1; 0; 0)g)

e assim, a equao cartesiana do plano P que passa pelo ponto (1; 2; 1) dada por:
(h(x

1; y

, (1 (x

ou seja por

2; z

1) + 0 (y

1); (1; 0; 0)i = 0) ,


2) + 0 (z

1) = 0) ,

x = 1.
NOTE QUE:
U = U?

= (x; y; z) 2 R3 : h(x; y; z); (1; 0; 0)i = 0 =


= (x; y; z) 2 R3 : x = 0 :

(ii) Determinemos a equao cartesiana da recta que passa pelos pontos (1; 1; 0) e
(1; 2; 1). Tem-se
r = f(1; 1; 0)g + L (f(0; 1; 1)g) ,
uma vez que

(0; 1; 1) = (1; 2; 1)

(1; 1; 0):

Seja
U = L (f(0; 1; 1)g) .
Logo,
U ? = (x; y; z) 2 R3 : h(x; y; z); (0; 1; 1)i = 0 =
=N

0 1 1

= L (f(1; 0; 0); (0; 1; 1)g)

e assim, a equao cartesiana da recta r dada por:


(h(x

1; y

1; z); (1; 0; 0)i = 0 e h(x


, (1 (x

ou seja por

1; y

1) = 0 e 1 (y

1)

1; z); (0; 1; 1)i = 0) ,


1z = 0) ,

x=1
y z = 1.
NOTE QUE:
U = U?

= (x; y; z) 2 R3 : h(x; y; z); (1; 0; 0)i = 0 e h(x; y; z); (0; 1; 1)i = 0 =


= (x; y; z) 2 R3 : x = 0 e y
129

z=0 :

6a Ficha de exerccios para as aulas de problemas


1. Diga quais das seguintes aplicaes h; i : R2
interno.
(i) h(x1 ; x2 ); (y1 ; y2 )i = x21 y12 + x22 y22
(ii) h(x1 ; x2 ); (y1 ; y2 )i = x1 y1
(iii) h(x1 ; x2 ); (y1 ; y2 )i =

x2 y1

R2 ! R denem em R2 um produto

x1 y2 + 3x2 y2

2x1 y1 + 3x2 y2

2. Diga quais das seguintes aplicaes h; i : R3


interno.

R3 ! R denem em R3 um produto

(i) h(x1 ; x2 ; x3 ); (y1 ; y2 ; y3 )i = x1 y1 + x2 y2 + x3 y3


(ii) h(x1 ; x2 ; x3 ); (y1 ; y2 ; y3 )i = x1 y2

x2 y1

(iii) h(x1 ; x2 ; x3 ); (y1 ; y2 ; y3 )i = 2x1 y1 + x1 y3 + x3 y1 + 2x2 y2 + x3 y3

3. Determine um produto interno em R2 tal que h(1; 0); (0; 1)i = 2.


4. Considere os vectores u =

p1 ;
5

p1
5

e v =

p2 ; p3
30
30

. Verique que o conjunto

fu; vg ortonormado relativamente ao produto interno denido em R2 por:


hu; vi = 3u1 v1 + 2u2 v2 ,
onde u = (u1 ; u2 ) e v = (v1 ; v2 ). Verique porm que o mesmo conjunto fu; vg no
ortonormado relativamente ao produto interno usual denido em R2 .
5. Considere em R4 o produto interno usual. Determine o subespao de R4 ortogonal aos
vectores (1; 0; 0; 0) e (1; 0; 0; 1).
6. Considere em R3 o produto interno denido por:
h(x1 ; x2 ; x3 ); (y1 ; y2 ; y3 )i = x1 y1 + x1 y2 + x2 y1 + 2x2 y2 + x3 y3 .
(i) Calcule kuk, para qualquer vector u = (x1 ; x2 ; x3 ) 2 R3 .

(ii) Considere os vectores u1 = (1; 0; 0), u2 = ( 1; 1; 0) e u3 = (0; 0; 1). Calcule os


ngulos formados pelos vectores: u1 e u2 ; u1 e u3 ; u2 e u3 .

(iii) Justique que o conjunto fu1 ; u2 ; u3 g uma base ortonormada de R3 . Calcule as


coordenadas de um vector u 2 R3 em relao a esta base.
7. Considere R4 com o produto interno usual. Determine uma base ortonormada para o
subespao de R4 gerado pelos vectores:
(1; 0; 1; 0); ( 1; 2; 0; 1) e (2; 0; 2; 1).
8. Considere R3 com o produto interno usual. Considere tambm os seguintes subespaos
de R3 :
U = L (f(0; 1; 1); (0; 0; 1)g)

e
130

V = (x; y; z) 2 R3 : y

z=0 .

(i) Determine uma base ortogonal para U e uma base ortonormada para V .
(ii) Determine duas bases ortonormadas para R3 : uma que inclua dois vectores de U
e outra que inclua dois vectores de V .
(iii) Determine o elemento de U mais prximo de (1; 1; 1) e a distncia entre (1; 1; 1)
e V ?.
2
3
1 0 2
9. Seja A = 4 0 0 0 5 e considere o produto interno usual. Sejam N (A), C (A) e L (A)
2 0 1
respectivamente o ncleo, espao das colunas e espao das linhas de A.
(i) Determine uma base ortonormada para R3 que inclua dois vectores de C (A).

(ii) Determine o elemento de L (A) mais prximo de (1; 1; 1) e a distncia entre (1; 1; 1)
e N (A).
2
3
1 0 1
10. Seja A = 4 0 2 0 5 e considere o produto interno usual. Sejam N (A), C (A) e L (A)
1 0 1
respectivamente o ncleo, espao das colunas e espao das linhas de A.
(i) Determine uma base ortonormada para (N (A))? (o complemento ortogonal do
ncleo de A).
(ii) Determine uma base ortonormada para R3 que inclua dois vectores de C (A).

(iii) Determine o elemento de L (A) mais prximo de (1; 2; 3) e a distncia entre (1; 2; 3)
e (L (A))? .

11. Considere em R4 o seguinte subespao: U = L (f(1; 1; 1; 0); (0; 1; 1; 1)g). Determine


uma matriz A do tipo 2 4 cujo ncleo seja igual a U , isto , tal que U = N (A).
12. Dena o produto interno em R2 em relao ao qual a base f(1; 0); (1; 1)g ortonormada.
13. Considere a aplicao h; i : R3

R3 ! R denida por

h(x1 ; x2 ; x3 ); (y1 ; y2 ; y3 )i = x1 y1

x1 y2

x2 y1 + 4x2 y2 + x3 y3 .

(i) Verique que h; i dene um produto interno em R3 .


(ii) Seja V = L (f(3; 4; 0)g)

R3 . Diga qual o ponto de V mais prximo de (0; 1; 0).

(iii) Determine uma base ortogonal para o complemento ortogonal de V , em relao


ao produto interno h; i.

(iv) Seja PV : R3 ! R3 a projeco ortogonal de R3 sobre V . Indique, em relao ao


produto interno h; i, uma base ortonormada de R3 para a qual a representao matricial
de PV seja dada por
2
3
1 0 0
4 0 0 0 5.
0 0 0

14. Seja U o subespao de R3 gerado pelos vectores v1 = (0; 1; 0) e v2 =


u = (1; 2; 3) na forma u = u1 + u2 , com u1 2 U e u2 2 U ? .
131

4
; 0;
5

3
5

. Escreva

15. Considere R4 com o produto interno usual. Em cada alnea seguinte, determine uma
base ortogonal para o complemento ortogonal de U , isto , para U ? .
(i) U = L (f(1; 0; 0; 0); (1; 1; 0; 1)g)
(ii) U = L (f(1; 0; 1; 1)g)
(iii) U = f(x; y; z; w) 2 R4 : x + 2y + z + 2w = 0g
(iv) U = f(x; y; z; w) 2 R4 : x

z = 0 e 2x

y + 2z

w = 0g

16. Considere R3 com o produto interno usual. Considere tambm o seguinte subespao
de R3 :
U = L (f(1; 1; 1); (1; 0; 0)g) .
(i) Determine uma base ortogonal para U .
(ii) Determine u 2 U e v 2 U ? tais que
(3; 2; 1) = u + v.
(iii) Determine a distncia entre o ponto (1; 0; 1) e o plano f(1; 1; 0)g + U .
(iv) Determine a distncia entre o ponto (x; y; z) e o plano U .

17. Considere R4 com o produto interno usual. Considere tambm o seguinte subespao
de R4 :
U = (x; y; z; w) 2 R4 : x y + z = 0 e y z + w = 0 .
(i) Determine uma base ortonormada para U .
(ii) Determine uma base ortonormada para U ? .
(iii) Determine as projeces ortogonais de (0; 0; 1; 0) sobre U e U ? respectivamente.
(iv) Determine as representaes matriciais de PU : R4 ! R4 e de PU ? : R4 ! R4 em
relao base cannica de R4 .
(v) Determine a distncia entre o ponto (0; 0; 1; 0) e o subespao U .
(vi) Determine a distncia entre o ponto (x; y; z; w) e o subespao U .
18. Considere P2 = fa0 + a1 t + a2 t2 : a0 ; a1 ; a2 2 Rg a aplicao h; i : P2
por
hp(t); q(t)i = p( 1)q( 1) + p(0)q(0) + p(1)q(1).

P2 ! R denida

Considere tambm o seguinte subespao de P2 : U = fp(t) 2 P2 : p(0) = 0g.

(i) Verique que h; i dene um produto interno em P2 .


(ii) Determine uma base ortonormada para U .

(iii) Determine uma base ortonormada para U ? .


(iv) Determine as projeces ortogonais do polinmio 1 + t sobre U e U ? respectivamente.
(v) Determine as representaes matriciais de PU : P2 ! P2 e de PU ? : P2 ! P2 em
relao base cannica f1; t; t2 g de P2 .
(vi) Determine a distncia entre 1 + t e U .

(vii) Determine a distncia entre o polinmio a0 + a1 t + a2 t2 e o subespao U .


132

19. Considere a aplicao h; i : M2 2 (R)

M2 2 (R) ! R denida por

hA; Bi = tr(AB T ).
Considere tambm o subespao U de M2 2 (R) constitudo por todas as matrizes
simtricas reais do tipo 2 2:
U=

a b
c d

2 M2 2 (R) : b = c .

(i) Verique que h; i dene um produto interno em M2 2 (R).


(ii) Determine uma base ortonormada para U .

(iii) Determine uma base ortonormada para U ? .


(iv) Determine as representaes matriciais de PU : M2 2 (R) ! M2 2 (R) e de PU ? :
M2 2 (R) ! M2 2 (R) em relao base cannica
1 0
0 0

0 1
0 0

0 0
1 0

de M2 2 (R).
(v) Determine as projeces ortogonais da matriz

0 0
0 1

1 1
0 1

sobre U e U ? respectiva-

mente.
(vi) Qual a matriz simtrica mais prxima da matriz
(vii) Determine a distncia entre
(viii) Determine a distncia entre

1 1
0 1
a b
c d

133

e U.
e U.

1 1
?
0 1

4a Ficha de exerccios facultativos


1. Seja V um espao euclidiano real. Verique que para todos os u; v; w 2 V;
tem:
(i) hu; vi = hv; ui

(ii) h u; vi = hu; vi =

(iii) hu; v + wi = hu; vi + hu; wi

2 R se

hu; vi

(iv) hu + v; wi = hu; wi + hv; wi

(v) hu + w; v + wi = hu; vi + hu; wi + hw; vi + kwk2

(vi) hu; 0i = h0; ui = 0

(vii) hu; vi = 0 se e s se ku + vk = ku

vk.

(viii) hu; vi = 0 se e s se ku + vk = kuk + kvk2 :


(ix) hu; vi = 0 se e s se ku + cvk
(x) hu + v; u

kuk para todo o real c.

vi = 0 se e s se kuk = kvk.

(xi) Lei do paralelogramo ku

vk2 + ku + vk2 = 2 kuk2 + 2 kvk2 .

2. Seja V um espao euclidiano real.


(i) Seja u 2 V . Verique que se hu; vi = 0 para qualquer v 2 V ento u = 0.

(ii) Sejam u; v 2 V . Verique que u = v se e s se hu; wi = hv; wi para qualquer


w 2V.

3. Seja V um espao euclidiano com dim V = n. Seja S = fu1 ; :::; un g uma base ortonormada de V . Seja T : V ! V uma transformao linear. Verique que a matriz
A = (aij ) que representa T em relao base S dada por
A = (aij ) = (hT (uj ); ui i) .
4. Seja V um espao euclidiano de dimenso n. Seja fu1 ; :::; uk g um conjunto linearmente
independente de k vectores de V . Considere a transformao linear T : V ! V denida
por
k
X
T (v) =
hv; ui i ui ,
i=1

com v 2 V .

Mostre que T invertvel se e s se k = n.


5. Seja V um espao euclidiano real. Seja T : V ! V uma transformao linear tal que
kT (w)k = kwk para qualquer w 2 V . Mostre que
hT (u); T (v)i = hu; vi ,
para quaisquer u; v 2 V .

134

Produto externo e produto misto


Denio 56. Sejam u = (u1 ; u2 ; u3 ) ; v = (v1 ; v2 ; v3 ) 2 R3 . Ento o produto externo
(vectorial) de u por v, denotado por u v, o vector de R3 denido por
u

v = (u2 v3

u3 v2 ; u3 v1

u1 v3 ; u1 v2

u2 v1 ) ,

isto ,
u

v =

u 2 u3
;
v2 v3

u2 u 3
e
v2 v3 1

u1 u 3
u u
; 1 2
v1 v3
v1 v2

u1 u 3
u1 u 2
e +
e =
v1 v3 2
v1 v2 3

e1 e2 e3
u 1 u2 u 3 ,
v1 v2 v3

onde fe1 ; e2 ; e3 g a base cannica de R3 .


Observao 54. Sejam u; v; w 2 R3 e
(i) e1

e2 = e3

(ii) e2

e3 = e1

(iii) e3

e1 = e2

(iv) u

v=

(v) u

(v

u)

(v + w) = u

(vi) (u + v)
(vii)

2 R. Ento, tem-se:

(u

(viii) u
(ix) u

w=u

v+u
w+v

v) = ( u)
0=0

w
w

v=u

( v)

u=0

u=0

(x) Se u e v forem linearmente dependentes ento u


(xi) u
(xii) (u
(xiii) ku
(xiv) u

w) = hu; wi v

(v
v)

w = hw; ui v

v=0

hu; vi w
hw; vi u

vk2 + hu; vi2 = kuk2 kvk2


(v

w) + v

(w

u) + w

(u

v) = 0

Teorema 75. Considere-se R3 com o produto interno usual. Sejam u = (u1 ; u2 ; u3 ) ; v =


(v1 ; v2 ; v3 ) 2 R3 n f0g e seja 2 [0; ] o ngulo formado por u e v. Ento tem-se:
135

(i) ku

vk = kuk kvk sen .

(ii) A rea do paralelogramo de lados adjacentes u e v dada por:


A = ku
Dem. (i) Como

2 [0; ], tem-se sen =

vk .

cos2

e deste modo,

kuk kvk sen =


= kuk kvk

cos2 = kuk kvk

hu; vi2
=
kuk2 kvk2

kuk2 kvk2

hu; vi2 =

q
= (u21 + u22 + u23 ) (v12 + v22 + v32 ) (u1 v1 + u2 v2 + u3 v3 )2 =
q
= (u2 v3 u3 v2 )2 + (u3 v1 u1 v3 )2 + (u1 v2 u2 v1 )2 =
= k(u2 v3

u3 v2 ; u3 v1

u1 v3 ; u1 v2

u2 v1 )k = ku

vk .

(ii) A = (base) (altura) = kuk kvk sen .


Denio 57. Considere-se R3 com o produto interno usual. Sejam u = (u1 ; u2 ; u3 ) ; v =
(v1 ; v2 ; v3 ) ; w = (w1 ; w2 ; w3 ) 2 R3 . expresso
hu; v

wi

chama-se produto misto de u; v e w.


Observao 55. Considere-se R3 com o produto interno usual. Sejam u = (u1 ; u2 ; u3 ) ; v =
(v1 ; v2 ; v3 ) ; w = (w1 ; w2 ; w3 ) 2 R3 . Ento, tem-se:
(i)
hu; v

wi =

u 1 u2 u3
v1 v2 v3
w1 w2 w3

(ii) Sendo o ngulo formado por u e v w, o volume do paraleleppedo com um vrtice


e arestas u; v; w com origem em (0; 0; 0), dado por
V

kv wk
| {z }

rea da face determinada por v e w

= kv

wk kuk

(iii) hu; u

vi = 0

(iv) hv; u

vi = 0

(v) hu; v

wi = hu

kuk jcos j =
| {z }
altura

jhu; v wij
= jhu; v
kuk kv wk

v; wi.
136

02

31
u 1 u2 u3
wij = det @4 v1 v2 v3 5A
w1 w2 w3

Matrizes hermitianas, matrizes simtricas e matrizes normais. Diagonalizao


unitria e diagonalizao ortogonal
T

Denio 58. Seja A = (aij ) 2 Mn n (C). Denota-se por AH a matriz A , isto , a


transposta da matriz conjugada A = (aij ), onde aij o complexo conjugado de aij . Ou seja,
escreve-se
T
AH = A .
A matriz A diz-se hermitiana se AH = A.
Observao 56. (a) Sejam ; 2 C, A; C 2 Mm n (C) e B 2 Mn r (C). Tem-se:
H
(i) AH
=A
(ii) ( A + B)H = AH + B H
(iii) (AC)H = C H AH
(b) Seja A 2 Mn n (R) tal que A simtrica (AT = A). Ento A hermitiana uma vez
que sendo A real tem-se AH = AT .
Teorema 76. Todos os valores prprios de uma matriz hermitiana so reais. Alm disso,
os vectores prprios associados a valores prprios distintos, de uma matriz hermitiana, so
ortogonais.
Dem. Seja A 2 Mn n (C) tal que A hermitiana. Seja um valor prprio de A e seja
u um vector prprio associado. Seja = uH Au. Ento, tem-se
=
Ou seja,

= uH Au

= u H AH u H

uH Au = .

real. Por outro lado, como


= uH Au = uH u =

tem-se

A hermitiana

kuk2 ,

2 R.

kuk2

Sejam agora u1 e u2 vectores prprios associados respectivamente a valores prprios


distintos 1 e 2 . Ento, tem-se
H
(Au1 )H u2 = uH
1 A u2

A hermitiana

uH
1 Au2 =

H
2 u 1 u2

e
(Au1 )H u2 = ( 1 u1 )H u2 =

H
1 u1 u2

1 2R

H
1 u1 u2 .

Logo, tem-se
H
1 u1 u 2

E assim, como

6=

2,

H
2 u1 u2

,(

H
2 ) u1 u2

= 0.

ento uH
u = 0, ou seja, u1 e u2 so ortogonais.
| 1{z }2
=hu1 ;u2 i

Observao 57. Todos os valores prprios de uma matriz simtrica real so reais. Alm
disso, os vectores prprios associados a valores prprios distintos, de uma matriz simtrica,
so ortogonais.
137

Denio 59. (i) Seja U 2 Mn n (C). A matriz U diz-se unitria se se tiver U H U = I,


isto , se U H = U 1 , ou seja, se as colunas de U constituirem um conjunto ortonormado em
Cn .
(ii) Seja A 2 Mn n (R). A matriz A diz-se ortogonal se se tiver AT A = I, isto , se
A = A 1 , ou seja, se as colunas de A constituirem um conjunto ortonormado em Rn .
T

Observao 58. (i) Seja U 2 Mn n (R) tal que U unitria. Como, sendo U real, se
tem U H = U T , ento U ortogonal. Isto , toda a matriz real unitria ortogonal.
(ii) Seja A uma matriz hermitiana. Se os valores prprios de A forem distintos, ento
existe uma matriz unitria que diagonaliza A, isto , existe U unitria tal que U AU H uma
matriz diagonal, ou seja, A diz-se unitariamente diagonalizvel.
(iii) A armao anterior (ii) continua vlida mesmo se os valores prprios no forem
distintos, como ser provado no Teorema 78.
Teorema 77. Seja A uma matriz n
U AU H triangular superior.

n. Ento, existe uma matriz unitria U tal que

Dem. A demonstrao ser efectuada por induo em n. O resultado bvio para n = 1.


Suponhamos que a hiptese vlida para matrizes k k e seja A uma matriz (k + 1) (k + 1).
Sejam 1 um valor prprio de A e w1 um vector prprio associado de norma 1. Aplicando
o mtodo de ortogonalizao de Gram-Schmidt, seja fw1 ; : : : ; wk+1 g uma base ortonormada
para Ck+1 . Seja W H a matriz cuja coluna i igual ao vector wi , para i = 1; : : : ; k + 1. Ento,
por construo, a matriz W H unitria. Por outro lado, a primeira coluna de W AW H
igual a W Aw1 , tendo-se
2 3 2
3
1
1
6 0 7 6 0 7
6 7 6
7
W Aw1 = W 1 w1 = 1 W w1 = 1 6 .. 7 = 6 .. 7 .
4 . 5 4 . 5
0
0
Logo, tem-se

W AW H

6
6
6
=6
6
4

onde M uma matriz k k.


Pela hiptese de induo, existe uma
onde T1 uma matriz triangular. Seja
2
1
6
6
6
V =6 0
6 ..
4 .
0

0
..
.
0

j
j
j
j
j

matriz k
j 0
j
j
j
j

138

7
7
7
7,
7
5

k unitria V1 tal que V1 M (V1 )H = T1 ,


0

V1

7
7
7
7.
7
5

Ento V unitria e tem-se

(V W ) A (V W )H = V W AW H V H

6
6
6
=6
6
4

0
..
.
0

j
j
j

j
j

V1 M (V1 )

7 6
7 6
7 6
7=6
7 6
5 4

0
..
.
0

j
j
j
j
j

3
T1

onde T uma matriz triangular. Como a matriz V W unitria, pondo U = V W , tem-se

7
7
7
7 = T,
7
5

U AU H = T ,
com T triangular e U unitria.
Teorema 78. Seja A uma matriz hermitiana. Ento existe uma matriz unitria U que
diagonaliza A, isto , A diagonalizvel unitariamente. Ou seja, existe U unitria tal que
tal que a matriz U AU H diagonal.
Dem. Pelo teorema anterior, existe uma matriz unitria U tal que a matriz U AU H
triangular. Seja T = U AU H . Tem-se ento
T H = U AU H

= UH

AH U H

A hermitiana

U AU H = T .

Logo, como T = T H e T triangular ento T diagonal.


Observao 59. Atendendo ao resultado anterior tem-se ento o seguinte. Seja A 2
Mn n (R) tal que A simtrica, isto , tal que A = AT . Ento existe uma matriz ortogonal
P (matriz que verica: P T = P 1 ) tal que P AP T diagonal, isto , A ortogonalmente
diagonalizvel relativamente a uma base ortonormada formada s por vectores prprios de
A. A matriz P T a matriz cujas colunas so os vectores prprios de A que formam uma base
ortonormada de Rn , sendo P AP T a matriz diagonal onde se coloca na entrada i da diagonal
principal o valor prprio correspondente ao vector prprio da coluna i da matriz P T .
Observao 60. (i) Existem matrizes no hermitianas que so diagonalizveis relativamente a bases ortonormadas formadas s por vectores prprios, como por exemplo as
matrizes anti-hermitianas (AH = A) e as matrizes anti-simtricas (AT = A).
(ii) Seja A 2 Mn n (R). Suponhamos que A ortogonalmente diagonalizvel relativamente a uma base ortonormada formada s por vectores prprios de A. Sejam D diagonal e
P ortogonal tais que A = P T DP . Ento
AT = P T DP

= P T DT P T

= P T DP = A.

Logo A simtrica. Tem-se ento, atendendo tambem observao 59, sendo A real:
A ortogonalmente diagonalizvel
139

, A simtrica

(iii) Seja A 2 Mn n (C). Suponhamos que A unitariamente diagonalizvel relativamente a uma base ortonormada formada s por vectores prprios de A. Sejam D diagonal
e U unitria tais que A = U H DU . Como em geral se tem DH 6= D, ento
AH = U H DU

= U H DH U 6= U H DU = A.

Logo A no tem que ser necessariamente hermitiana.


(iv) O prximo teorema diz quais so as matrizes unitariamente diagonalizveis.
Denio 60. Uma matriz A diz-se normal se
AAH = AH A:

Teorema 79. Uma matriz A normal se e s se fr unitariamente diagonalizvel relativamente a uma base ortonormada formada s por vectores prprios de A.
Dem. ()) Suponhamos que A normal. Pelo teorema 77, existe uma matriz unitria U
e uma matriz triangular superior T tais que T = U AU H . Vejamos que T normal. Tem-se
T HT =

U AU H

U AU H = U AH U H U AU H = U AH AU H

A normal

= U AAH U H = U AU H U AH U H = T T H .
Logo T normal. Seja T = (tij ) do tipo n
principais de T T H e T H T tem-se:
jt11 j2 + jt12 j2 + jt13 j2 +
jt22 j2 + jt23 j2 +

n. Comparando as entradas das diagonais

+ jt1n j2 = jt11 j2
+ jt2n j2 = jt12 j2 + jt22 j2
..
.
2
jtnn j = jt1n j2 + jt2n j2 + jt3n j2 +

+ jtnn j2

e assim, tij = 0 sempre que i 6= j. Logo T diagonal e portanto A unitariamente


diagonalizvel.
(() Suponhamos agora que A unitariamente diagonalizvel. Queremos mostrar que A
normal. Sejam D diagonal e U unitria tais que D = U AU H , ou seja, A = U H DU . Tem-se
AAH = U H DU U H DU
e
AH A = U H DU
Como

= U H DU U H DH U = U H DDH U

U H DU = U H DH U U H DU = U H DH D U .
2

6
6
DDH = DH D = 6
4

j 1 j2
0
..
.

ento tem-se AAH = AH A e assim A normal.


140

0
j 2 j2
..
.

...

0
..
.

..

.
j

2
nj

7
7
7,
5

Formas quadrticas
Denio 61. Uma equao quadrtica em duas variveis x1 e x2 uma equao da
forma
ax21 + bx22 + 2cx1 x2 + dx1 + ex2 + f = 0
a qual pode ser escrita na forma
x1 x2

a c
c b

x1
x2

d e

x1
x2

+ f = 0.

Sejam
x=

x1
x2

A=

a c
c b

(A uma matriz real simtrica). funo real a duas variveis reais Q : R2 ! R denida
por Q (x) = xT Ax, com
xT Ax = ax21 + bx22 + 2cx1 x2
chama-se forma quadrtica real a 2 variveis reais associada equao quadrtica anterior.
Podem haver equaes do 2o grau e formas quadrticas com um no de variveis superior
a 2. Uma equao quadrtica em n variveis x1 ; x2 ; : : : ; xn uma equao da forma
xT Ax + Bx +
2

6
6
onde x = 6
4

x1
x2
..
.

= 0,

7
7
7, A = (aij ) uma matriz real simtrica do tipo n
5

n, B 2 M1

(R) e

xn
um escalar. funo real a n variveis reais Q : Rn ! R denida por
!
n
n
X
X
T
Q (x) = x Ax =
aij xj xi
i=1

j=1

chama-se forma quadrtica real a n variveis reais associada equao quadrtica anterior.
Teorema 80. (Teorema dos eixos principais). Seja A 2 Mn n (R) tal que A simtrica.
Ento existe uma mudana de variveis ortogonal que transforma a forma quadrtica xT Ax
na forma quadrtica y T Dy sem termos cruzados. Isto , se P diagonalizar A ortogonalmente
(D = P AP T ), ento a mudana de variveis x = P T y transforma a forma quadrtica xT Ax
na forma quadrtica y T Dy:
xT Ax = y T P AP T y = y T Dy =
2
=

y1 y2

yn

6
6
6
4

0
..
.
0

141

2
1 y1

0
2

..
.
0

...

2
2 y2

+
+
32
0
y1
7
6
0 7 6 y2
.. 7 6 ..
. 54 .
yn
n

2
n yn

7
7
7,
5

onde 1 ; 2 ; : : : ; n so os valores prprios de A associados respectivamente aos vectores


prprios que constituem as colunas de P T e que formam uma base ortonormada de Rn .
Observao 61. (i) Chama-se cnica ou seco cnica curva plana obtida por meio de
um corte efectuado por um plano relativamente a uma superfcie cnica. As seces cnicas
que se obtm quando o plano que efectua o corte no passa pelo vrtice da superfcie cnica,
so elipses (os valores prprios tm o mesmo sinal) (podendo ter-se circunferncias: quando
o corte efectuado perpendicularmente ao eixo de simetria do cone), parbolas (um dos dois
valores prprios zero) e hiprboles (os dois valores prprios tm sinais contrrios).

(ii) Em R3 tem-se
2
3
x1
x = 4 x2 5 ;
x3

3
a d e
A=4 d b f 5
e f c

3
g
B=4 h 5
i

ax21 + bx22 + cx23 + 2dx1 x2 + 2ex1 x3 + 2f x2 x3 + gx1 + hx2 + ix3 +

= 0.

supercie resultante da equao anterior chama-se qudrica. Existem quatro tipos de


qudricas no degeneradas): elipsides, hiperbolides (de uma ou duas folhas), cones e
parabolides (elpticos ou hiperblicos).

Exemplo 56. Considere-se a forma quadrtica Q : R2 ! R denida por


Q(x; y) = 3x2 + 4xy + 3y 2 .
Tem-se
Q(x; y) =

x y

x
y

3 2
. Os valores prprios de A so 1 = 1 e
2 3
forma quadrtica diagonal (isto , sem termos cruzados)
com A =

Q0 (x0 ; y 0 ) =

x0 y 0

com
D = P AP T ,
e

2 p

6
PT = 4

2
2
p
2
2

x0 y 0
1 0
0 5
x0
y0

x0
y0

= 5. Tem-se ento a seguinte

x0
y0
=P

x
y

3 2
3
2
sen
cos
4
4 5,
p2 7
5=4
2
sen
cos
4
4
2
142

em que P T a matriz diagonalizante obtida colocando na 1a coluna um vector prprio


de norma 1 associado ao valor prprio 1 e na 2a coluna um vector prprio de norma 1
associado ao valor prprio 2 , de tal modo que ambos os vectores prprios constituam uma
base ortonormada de R2 . Observe-se que a matriz P ortogonal, isto , tem-se P T = P 1 .
Tem-se ento
x
Q(x; y) = x y A
=
y

=
=

x0 y 0

x
y

P T DP

x y

x
y

DP
x0
y0

=
x
y

= Q0 (x0 ; y 0 ).

Por exemplo, relativamente equao quadrtica 3x2 + 4xy + 3y 2 = 4

-5

-4

-3

-2

-1

-2

-4

tem-se a elipse:
2

(x0 ) + 5 (y 0 ) = 4 ,

x0
2

y0

p
2 5
5

!2

= 1:

Denio 62. Seja A uma matriz real simtrica do tipo n


quadrtica Q : Rn ! R dada por Q (x) = xT Ax so:

n. Diz-se que A e a forma

(i) denidas positivas se xT Ax > 0, para todo o x 6= 0;


(ii) denidas negativas se xT Ax < 0, para todo o x 6= 0;
(iii) semidenidas positivas se xT Ax

0, para todo o x;

(iv) semidenidas negativas se xT Ax

0, para todo o x;

(v) indenidas se existirem pontos onde xT Ax seja positiva e pontos onde xT Ax seja
negativa.
143

Teorema 81. Seja A 2 Mn

n (R)

tal que A simtrica. Ento,

(i) A denida positiva se e s se todos os valores prprios de A forem positivos;


(ii) A denida negativa se e s se todos os valores prprios de A forem negativos;
(iii) A semidenida positiva se e s se todos os valores prprios de A forem no
negativos;
(iv) A semidenida negativa se e s se todos os valores prprios de A forem no
positivos;
(v) A indenida se e s se A tiver pelo menos um valor prprio positivo e outro
negativo.
Dem. (i) ()) Suponhamos que A denida positiva, isto ,
xT Ax > 0;
para todo o x 6= 0. Seja
associado a tem-se

um valor prprio de A. Ento, para qualquer vector prprio x


kxk2 ,

xT Ax = xT x =
com x 6= 0. Logo
=

xT Ax
> 0.
kxk2

(() Suponhamos que todos os valores prprios de A so positivos. Seja fx1 ; : : : ; xn g um


conjunto ortonormado de vectores prprios de A. Logo fx1 ; : : : ; xn g uma base ortonormada
de Rn . Se x 6= 0, ento tem-se
x=
com

1 ; :::;

1 x1

n xn ,

no todos nulos, pelo que


xT Ax = (
=
=

(x1 )T +

(x1 )T +

1 x1

T
n xn )

(xn )T (

(xn )T (

n
X

( i )2

i=1

Logo A denida positiva.

144

A(

1 x1

n xn )

1 Ax1

n Axn )

1 1 x1

n n xn )

> 0.

=
=

Teorema 82. Seja A 2 Mn


so equivalentes.

n (R)

tal que A simtrica. Ento, as seguintes armaes

(i) A denida positiva.


(ii) Existe uma matriz simtrica denida positiva B tal que A = B 2 .
(iii) Existe uma matriz invertvel S tal que A = S T S.
Dem. (i) ) (ii) Supondo que A denida positiva, vejamos que existe uma matriz
simtrica denida positiva B tal que A = B 2 .
Como A simtrica, ento A ortogonalmente diagonalizvel, isto , existe uma matriz
ortogonal P tal que
2
3
0
0
1
.
..
..
6
.
. .. 7
6 0
7
T
P AP = D = 6 . .
7
..
4 ..
0 5
0
0
n

onde 1 ; :::; n so os valores prprios de A, os quais so todos positivos por A ser denida
positiva, tendo-se
2
D = (D0 )
com

6
6
D =6
4
0

Assim

2 p

0
..
.

0
..
.
..
.

..

0
..
.
p0

A = P T DP = P T (D0 ) P = P T D0 P

7
7
7.
5
P T D0 P = B 2

com
B = P T D0 P
simtrica:
B T = P T D0 P

= P T (D0 )

PT

= P T D0 P = B

e denida positiva uma vez que os valores prprios de P T D0 P so os de D0 .


(ii) ) (iii) Supondo que existe uma matriz simtrica denida positiva B tal que A = B 2 ,
vejamos que existe uma matriz invertvel S tal que
A = S T S:
Como B simtrica e denida positiva, basta fazer S = B para ter-se
A = B 2 = BB = S T S
com S simtrica e invertvel uma vez que sendo B denida positiva, 0 no valor prprio de
B.

145

(iii) ) (i) Supondo que existe uma matriz invertvel S tal que A = S T S, vejamos que A
denida positiva, isto , vejamos que
xT Ax > 0;
para todo o x 6= 0. Tem-se
xT Ax = xT S T Sx = (Sx)T Sx = kSxk2 > 0
para todo o x 6= 0, uma vez que S invertvel.
Observao 62. (i) Relativamente matriz da mtrica G que dene um produto
interno, sendo B = fw1 ; :::; wn g uma base ordenada e ortonormada de um espao euclidiano
V de dimenso n tem-se
2
3
2
3
hw
;
w
i
hw
;
w
i
:
:
:
hw
;
w
i
1
1
1
2
1
n
1
6
7
6 hw2 ; w1 i hw2 ; w2 i : : : hw2 ; wn i 7
6 2 7
6
7
hu; vi =
G 6 .. 7 ,
G=6
7
..
..
..
1
2 :::
n
4 . 5
4
5
.
.
.
hwn ; w1 i hwn ; w2 i : : : hwn ; wn i
n
com

u=

1 w1

2 w2

+ ::: +

n wn

v=

1 w1

2 w2

+ ::: +

n wn

e
G = ST S = I
onde a matriz invertvel S a matriz de mundana da base B para uma outra base ordenada
B 0 : SB!B0 . No caso da base ordenada B no ser ortonormada, a base ordenada B 0 ser
ortonormada. Em particular, a matriz G invertvel.
(ii) Sendo A matriz simtrica e denida positiva (ou semidenida positiva), s matrizes
reais B tais que A = B 2 chamam-se "razes quadradas" de A:
(iii) Sendo V o volume do hiperparaleleppedo determinado por fw1 ; :::; wn g, tem-se
V 2 = det w1
= det

w1
02

= det @4

Logo

wn
(w1 )T
(wn )T

wn
T

det

5 w1

w1

w1

= det
wn
1

= det
02

B6
wn A = det @4

wn

det

w1

w1
wn

(w1 )T w1

wn
T

w1

(w1 )T wn
..

(wn )T w1
(wn )T wn
02
31
hw1 ; w1 i
hw1 ; wn i
B6
7C
..
= det @4
5A = det G.
.
hwn ; w1 i
hwn ; wn i
V 2 = det G.
146

=
wn
31

7C
5A =

Mnimos quadrados
Existem aplicaes relativamente s quais os erros cometidos nas medies das entradas
de A ou de b podem levar a que o sistema de equaes lineares Au = b no tenha soluo,
quando teoricamente deveria ter. Em tais casos natural a procura da "melhor soluo
aproximada" para esse problema.
Denio 63. Sejam A 2 Mm n (R) e b 2 Rn . Ento, a u
b 2 Rn chama-se melhor
soluo aproximada ou soluo de mnimos quadrados de Au = b se
kb

Ab
uk

kb

Auk ,

para qualquer u 2 Rn . Ao vector b Ab


u chama-se vector erro de mnimos quadrados e ao
escalar kb Ab
uk chama-se erro de mnimos quadrados.
Observao 63. Sejam A 2 Mm n (R) e b 2 Rn . Procuremos ento um mtodo para
determinar as solues de mnimos quadrados de Au = b. Atendendo a que Au 2 C (A) para
todo o u 2 Rn , ento a distncia kb Auk mnima se
Au = PC(A) (b) ,
onde PC(A) a projeco ortogonal de Rn sobre C (A). Como PC(A) (b) 2 C (A), a equao
Au = PC(A) (b) tem sempre soluo e essas solues so as solues de mnimos quadrados
de Au = b. Deste modo, qualquer sistema de equaes lineares tem sempre pelo menos uma
soluo de mnimos quadrados.
Por outro lado, pode escrever-se a equao Au = PC(A) (b) na forma
b

Au = b

PC(A) (b) = PN (AT ) (b)

tendo-se
AT (b

Au) = AT b

pois (C (A))? = N AT . Logo

PC(A) (b) = AT PN (AT ) (b) = 0;


AT Au = AT b.

A esta equao chama-se equao normal associada a Au = b.


Teorema 83. Sejam A 2 Mm

n (R)

e b 2 Rn .

(i) As solues de mnimos quadrados do sistema de equaes lineares


Au = b
so as solues da equao normal
AT Au = AT b:

147

(ii) Se car A = n ento a equao normal


AT Au = AT b
tem a soluo nica
u = AT A

AT b

e tem-se

PC(A) (b) = Au = A AT A

AT b,

isto ,
A AT A

AT

a matriz que representa a projeco ortogonal PC(A) .


Observao 64. Seja A 2 Mm

n (R).

Vejamos que se tem

car A = car AT A :
Basta para isso, mostrar que
N (A) = N AT A :

Seja u 2 N (A). Como Au = 0 ento AT Au = AT 0 = 0 e assim u 2 N AT A .


Reciprocamente, seja u 2 N AT A e vejamos que u 2 N (A). Tem-se AT Au = 0, logo
Au 2 N AT = L AT

= (C (A))?

(usando o produto interno usual) e como tal


hAu; Aui = 0,
ou seja kAuk2 = 0 e ento Au = 0, isto , u 2 N (A).
Observao 65. Vejamos agora o modo como se pode determinar uma curva (ou recta)
especca que se possa "ajustar" a um conjunto de pontos determinados experimentalmente.
(i) A partir de dois ou mais pontos dados
(x1 ; y1 ) ; (x2 ; y2 ) ; : : : ; (xm ; ym ) ;
pretende-se determinar uma recta y = a0 + a1 x que seja a recta que "melhor aproxime" ou
a recta de mnimos quadrados de melhor ajuste aos pontos dados (recta de regresso). Isto
, pretende-se determinar as solues de mnimos quadrados de
8
>
y1 = a0 + a1 x1
>
>
< y2 = a0 + a1 x2
..
>
.
>
>
: y =a +a x
m
0
1 m
148

ou seja de

Sejam

6
6
A=6
4

6
6
6
4

3
1 x1
1 x2 7
7
.. .. 7
. . 5
1 xm

3
1 x1
1 x2 7
7
.. .. 7 ;
. . 5
1 xm

6
6
=6
4

a0
a1

y1
y2
..
.
ym

a0
a1

u=

7
7
7.
5

6
6
b=6
4

y1
y2
..
.
ym

7
7
7:
5

Atendendo a que car A = car AT A , se houver pelo menos dois pontos distintos, tem-se
car A = 2 e nesse caso, a equao normal
AT Au = AT b
tem como nica soluo de mnimos quadrados
1

u = AT A

AT b:

Assim, a recta de mnimos quadrados y = a + bx a recta que torna mnimos os quadrados


cuja soma
(y1

(a0 + a1 x1 ))2 + (y2

(a0 + a1 x2 ))2 +

+ (ym

(a0 + a1 xm ))2

dada por
kb
onde kb

Auk2 ,

Auk o erro de mnimos quadrados.

(ii) A partir de m pontos dados (x1 ; y1 ) ; (x2 ; y2 ) ; : : : ; (xm ; ym ), pretende-se determinar


um polinmio cujo grco esteja to perto quanto possvel desses m pontos dados. Isto ,
com m 2 N previamente xo, pretende-se determinar as solues de mnimos quadrados do
sistema de m equaes a n + 1 incgnitas (a0 ; a1 ; a2 ; : : : ; an )
8
2
>
+ an xn1
> y 1 = a0 + a1 x 1 + a2 x 1 +
>
< y 2 = a0 + a1 x 2 + a2 x 2 +
+ an xn2
2
..
>
.
>
>
: y = a + a x + a x2 +
+ an xnm
m
0
1 m
2 m
ou seja de

2
6
6
6
4

1 x1 x21
1 x2 x22
.. ..
..
. .
.
1 xm x2m

xn1
xn2
..
.
xnm

32
76
76
76
54

149

a0
a1
..
.
an

7 6
7 6
7=6
5 4

y1
y2
..
.
ym

7
7
7.
5

Sejam

6
6
A=6
4

xn1
xn2
..
.

1 x1 x21
1 x2 x22
.. ..
..
. .
.
1 xm x2m

7
7
7;
5

xnm

6
6
u=6
4

a0
a1
..
.
an

3
7
7
7
5

6
6
b=6
4

y1
y2
..
.
ym

7
7
7:
5

Note-se que se n + 1 = m e se os pontos dados forem distintos, ento existe um nico


polinmio de grau n (o chamado polinmio interpolador) que passa por todos esses m pontos.
Por outro lado, atendendo a que car A = car AT A , se n < m e pelo menos n + 1 pontos
forem distintos, tem-se car A = n + 1 e ento a equao normal
AT Au = AT b
tem como nica soluo de mnimos quadrados
u = AT A

AT b:

Exemplo 57. Determinemos a recta de mnimos quadrados relativa aos pontos


(0; 1) ; (1; 3) ; (2; 4) e (3; 4) :
Sejam

3
2 3
1 0
1
6 1 1 7
6 3 7
7
7
A=6
e
b=6
4 1 2 5
4 4 5.
1 3
4
Tem-se car A = 2 e como tal a soluo de mnimos quadrados nica e dada por:
a0
a1

u=
0

1
6 1
6
4 1
1

B 1 1 1 1
=B
@ 0 1 2 3

tendo-se

= AT A

31
0
C
1 7
7C
2 5A
3

AT b =

1 1 1 1
0 1 2 3

3
1
6 3 7
6 7=
4 4 5
4

3=2
1

3
+ x:
2
O vector b Au o vector erro de mnimos quadrados, sendo o erro de mnimos quadrados
dado por:
kb Auk =
y=

q
=
(y1
=

(a0 + a1 x1 ))2 + (y2


3
+0
2

+ 3

25 1
1
1
+ +
+ =
16 4 16 4

(a0 + a1 x2 ))2 + (y3


3
+1
2

+ 4

34
.
4
150

(a0 + a1 x3 ))2 + (y4


3
+2
2

+ 4

(a0 + a1 x4 ))2 =
3
+3
2

7a Ficha de exerccios para as aulas de problemas


2

3
2 0 1
1. Seja A = 4 0 1 0 5. Calcule as dimenses dos espaos prprios de A sem os deter1 0 2
minar.
2
3
3
1
2
3 2 5. Encontre uma matriz ortogonal P T que diagonalize or2. Seja A = 4 1
2 2
0
togonalmente A e indique a correspondente matriz diagonal semelhante a A.
3. Seja

3
A=4 1
0

1
2
1

3
0
1 5:
3

Determine trs razes quadradas de A, isto , determine trs matrizes B simtricas e


denidas positivas B tais que A = B 2 .
4. Diagonalize a seguinte forma quadrtica Q denida por
Q : R3 ! R
Q (x1 ; x2 ; x3 ) = 3x21

2x1 x2 + 2x22

2x2 x3 + 3x23 .

5. Seja
A=

2
1

1+i
3

(i) Mostre que A normal.


(ii) Mostre que A hermitiana e em particular normal.
(iii) Encontre uma matriz unitria U H que diagonalize unitariamente A e indique a
correspondente matriz diagonal semelhante a A.
6. Determine a soluo de mnimos quadrados de Au = b, com
2
3
2 3
1 2
3
4
5
4
A= 2 4
e b= 2 5
1
2
1

calculando o correspondente vector erro de mnimos quadrados.

7. Resolva o seguinte sistema de equaes diferenciais determinando


geral.
8 0
2
3 2
u1 (0)
< u1 = 2u3
4 u2 (0) 5 = 4
u0 = u1 + 2u2 + u3
com
: 20
u3 = u1 + 3u3
u3 (0)
151

a respectiva soluo
3
2
1 5.
0

Bibliograa
1. Luis T. Magalhes, lgebra Linear como Introduo Matemtica Aplicada, 9a edio,
Texto Editora, 2001.
2. Gilbert Strang, Linear Algebra and its Applications, 3rd edition, Thomson Learning,
1988.
3. Steven J. Leon, Linear Algebra with Applications, 8th edition, Pearson, 2009.
4. Bernard Kolman, Introductory Linear Algebra with Applications, Prentice Hall, 1996.
5. Howard Anton and Robert C. Busby, Contemporary Linear Algebra, John Wiley &
Sons, Inc., 2002.
6. Antnio Monteiro e Gonalo Pinto, lgebra Linear e Geometria Analtica, McGrawHill, 1997.
7. Seymour Lipschutz, Linear Algebra, Schaums Outline Series, 4th edition, McGrawHill, 2009.
8. Lus Barreira e Cludia Valls, Exerccios de lgebra Linear, IST Press, 2011.

152

Resoluo da 1a Ficha de exerccios para as aulas de problemas


1
0

1. (ii)

220

0 1
1 0

(xi) se ad

0 1
1 0

0 0
0 0

1
3

2. (i)

(iii) 4

3
2

(vii)

2
3

viii) @4
2

B
B
4
(ix) B
B2
@
2

1
4
(x) 0
6

220

0
2
1
2

52

1
4

2 4

1
3

=[ 2

1
3

54

1
3

0
1
3

1
2

1
2

1
2

1
4

d
c

bc

1
2

02

ad

32

1
3
p

!500

= I 500 = I

0 1
1 0

b
a

220

0 1
1 0

+I

0 0
0 0

2
2

(iv) 2

1
1

bc 6= 0,
a b
c d

1
0

222

0 1
1 0

(iv)

1000

1
1

1
2

1
ad

bc

=I

p
3
2 5 5
5
p
7
5 2
2

8
3

6
2 2
(v) No possvel. (vi) No possvel.
= 20
2
3
3
p 3
p
2 p
1
2
2
2
4 2
2
5
=4
3
12
6
2

2 p
4 2
4
A
=
3
12

1
2

ad

1T

(ii) No possvel.
11
2

5=4

bc
0

bc

1]

ad

3T 2

5 4

3T 2

2
1
5
4
0
2
1
1
3

1
3

0
1
2

1
2

3
2

1
2

1
2

6
3 6
6
0
6
5
4
=6
6
6
3
6
4
153

p 3
2 2 2
5
6

3
5
1
5
6
7
6
7
3

6
6
6
6
4

8
9

1
3

1
3

1
2

5
3

1
3

18

7
7
10 7
7
7
7
16 7
7
5
3

31T

2
3
7C
0
0
0
7C
C
4
5
1 7
7C = 0 0 0
5A
0 0 0
5
2

3T 2
0
1
5
4
4
0
3
6

1
4
2
(xi)
1
3

1
2

1
2

1
4

3
2
0 5=
1

5
6

1
18

10

x
x

3. Sendo x o no de livros e y o no de caixas, tem-se

7
6

7
3

16

7y = 1
,
8y = 7

x = 57
y = 8.

A soluo geral do sistema f(57; 8)g.


4.

212 32
9
= ,
100 0
5

5. a)

c)

1
6 1
6
4 1
1

1
6 1
6
4 2
3

2
2
1
2

4
4
4
4

9
9
9
9

2
3
3
1

3
3
2 7
7
4 5
4

6. Seja (aij ) 2 M2

32
9
9
= , F = C + 32:
0
5
5

9
F = F + 32 , F =
5

F = C,
2

F
C

3
16
16 7
7
16 5
16

b)

d)

1 21 13
6 0 1 1
3
4
6
4 0 0 1
5
0 0 0

0
6 2
6
4 3
4

2
0
3
4

3
3
0
4

40.

1
4
1
5
1
6
1
7

3
7
7
5

3
4
4 7
7 (aii =
4 5
0

aii , aii = 0)

(R) tal que


aij = 3i + 2j

Como
a12 = 3

1+2

2 = 7 6= 8 = 3

2+2

1 = a21

ento A no simtrica.
2

3
0 0
7. (i) Seja A = 4 0 0 5. car A = 0; nul A = 2. No existem pivots.
0 0
2

3
1 2 3
(ii) 4 0 1 1 5
1 2 3

3
1 2 3
4 0 1 1 5.
!
L1 +L3 !L3
0 0 0
2
3
1 2 3
Assim, sendo A = 4 0 1 1 5, tem-se car A = 2 e nul A = 1. Pivots: 1 e 1.
1 2 3
154

(iii) 4

2
3
1
4 5 ! 4
L1 $L3
2

2
2
1

2
2
1

Assim, sendo A = 4
2

3
1 2 3 4
(iv) 4 5 6 7 8 5
9 10 11 12
2

1
Assim, sendo A = 4 0
0
2

0
(v) 4 1
1

1
1
1

L1 +L3 !L3

1
4 0
0

1
1
2
2

2
4
0

1
Assim, sendo A = 4 0
0
2

1
1
0

1
6 1
(vi) 6
4 2
3

2
1
7
3

1
3
1
2

3
2
9
4

1
6 0
6
4 0
0

2
3
0
0

1
2
1
3
2

3
1
2
4

L2 +L3 !L3
L2 +L4 !L4

1
4 0
0

5L1 +L2 !L2


9L1 +L3 !L3

1
1
1

1
6 1
Assim, sendo A = 6
4 2
3
1; 3; 1 e 2.

2
3
2
0 5 ! 4
L1 $L3
3

1
0
0

3
1
4 5, car A = 2 e nul A = 0. Pivots:
2

3
8
0

3
0
1 5
1

1
1
1

2L2 +L3 !L3

1
1
3

3
2
1 7
7
8 5
6

2
1
7
3

2
4
8

3
4
12 5
24

3
8
16

1e

2L2 +L3 !L3

3
2
3 5.
0

1
0
0

3.

1
4 0
0

2
4
0

3
4
12 5, car A = 2 e nul A = 2. Pivots: 1 e
0

3
2
1
1
0 5 ! 4 0
L1 $L2
1
1

1
1
2

2
3
2
4
4 5
!
2L1 +L2 !L2
1 2L1 +L3 !L3

1
2
2

3
0
1 5
1

1
1
2
2

1
4 0
0

3
8
0

4.

L1 +L3 !L3

1
1
0

3
0
1 5.
3

1
1
3

3
0
1 5, tem-se car A = 3 e nul A = 1. Pivots: 1; 1 e 3.
3

L1 +L2 !L2
2L1 +L3 !L3
3L1 +L4 !L4

1
6 0
6
4 0
0

2
3
3
3

1
2
1
1

3
2
2
1 2
1 3
6 0 3 2 1
1 7
7
6
!
1 2
3 5 3L3 +L4 !L4 4 0 0
11
0 0 0 2
3
1 3
2
3
2
1 7
7, tem-se car A =
1 9
8 5
2 4
6
155

3
1
3
5

3
2
1 7
7
4 5
12

L2 +L3 !L3
L2 +L4 !L4

3
2
1 7
7.
3 5
2

4 e nul A = 1. Pivots:

3
4
12 5.
0

1 3
6 0 11
(vii) 6
4 2
5
4 1

3
2
3 7
7
1 5
5

1
5
3
1
2

1 3
1 2
6 0 11
5 3
6
!
4
0
11 5
3
2L1 +L3 !L3
4L1 +L4 !L4
0
11 5
3
3
3
1 2
11
5 3 7
7, tem-se car A = 2
5 3 1 5
1
1 5

1
6 0
Assim, sendo A = 6
4 2
4
5
0

(viii) Sendo A =

3
3 6 9
(ix) 4 2 4 6 5 !
L1 $L3
1 2 3
2
3
4
Assim, sendo A = 2
1
2
4
1
(x)
2

10
5
10

6
3
6
2

7
7
5 L2 +L!
3 !L3

L2 +L4 !L4

1 3
6 0 11
6
4 0 0
0 0

1
5
0
0

3
2
3 7
7.
0 5
0

e nul A = 2. Pivots: 1 e 11.

1 2
, tem-se car A = 2 e nul A = 1. Pivots: 5 e 2.
2 0

3
2
3
1 2 3
1 2 3
4 2 4 6 5
4 0 0 0 5.
!
2L1 +L2 !L2
3 6 9
0 0 0
3L1 +L3 !L3
3
6 9
4 6 5, tem-se car A = 1 e nul A = 2. Pivot: 1.
2 3
8
4
8

2
4
1
Assim, sendo A =
2

3
4
2 5
4
10
5
10

1
L +L2 !L2
2 1

L1 +L3 !L3

6
3
6

8
4
8

2 10
4 0 0
0 0

6 8
0 0
0 0

3
4
0 5.
0

3
4
2 5, car A = 1 e nul A = 4. Pivot: 2.
4

8. As equaes das alneas (a) e (b) so lineares.


9. O ponto (1; 1) a soluo desse sistema de equaes lineares.
10. Os pontos: (1; 1; 1; 0) ; (1; 1; 1; 2) ; 3; 9; 7;

p
3
2

so solues desse sistema de

equaes lineares.
11. (i) Para que o grco da funo polinomial p(x) = ax3 + bx2 + cx + d passe pelos
pontos P1 = (0; 10); P2 = (1; 7); P3 = (3; 11) e P4 = (4; 14), necessrio que
8
p(0) = 10
>
>
<
p(1) = 7
p(3) = 11
>
>
:
p(4) = 14.
156

O que equivalente a existir soluo para o seguinte sistema de equaes lineares nas variveis
a; b; c e d:
8
d = 10
>
>
<
a+b+c+d=7
27a + 9b + 3c + d = 11
>
>
:
64a + 16b + 4c + d = 14.
Ou seja:

8
d = 10
>
>
<
a+b+c= 3
27a + 9b + 3c = 21
>
>
:
16a + 4b + c = 6.

Atendendo a que:
2
3
2
1 1 1 j
3
1 1
4 27 9 3 j
4 0
21 5
18
!
27L1 +L2 !L2
16 4 1 j
6
0
12
16L1 +L3 !L3
2
3
2
1 1
1 j
3
4
3
4 j 10 5
! 4 0
!
1
4L2 +L3 !L3
L
!L
2
2
6
0
12
15 j 42

1
0
0

3
1 j
3
24 j 60 5
15 j 42

tem-se

1
3
0

1
L !L2
6 2

3
1 j
3
4 j 10 5 ;
1 j 2

8
a=1
>
>
<
b= 6
c=2
>
>
:
d = 10.
(ii) Para que os pontos P1 = ( 2; 7); P2 = ( 4; 5) e P3 = (4; 3) pertenam circunferncia de equao x2 + y 2 + ax + by + c = 0; necessrio que
8
2
< ( 2) + 72 + a ( 2) + 7b + c = 0
( 4)2 + 52 + a ( 4) + 5b + c = 0
: 2
4 + ( 3)2 + 4a + b ( 3) + c = 0.

O que equivalente a existir soluo para o seguinte sistema de equaes lineares nas variveis
a; b e c:
8
< 2a + 7b + c = 53
4a + 5b + c = 41
:
4a 3b + c = 25.
Atendendo a que:
2

2
4 4
4

7 1 j
5 1 j
3 1 j

3
53
41 5
25

2L1 +L2 !L2


2L1 +L3 !L3

2 7
4 0
9
0 11
157

1 j
1 j
3 j

3
53
65 5
131

11
L +L3 !L3
9 2

11
L +L3 !L3
9 2

tem-se

7
1
j
9
1 j
0 16=9 j

2
4 0
0

3
53
5;
65
464=9

8
< a= 2
b= 4
:
c = 29.
2

2
3
1 0 1
1 0
4
4
5
1
0
12. (i)
!
L1 +L2 !L2
0
1
0
2
3
1 0 1
5. Se 6= 0 ento
Seja A = 4 1
0
1

Se

L2 +L3 !L3

1 0
4 0
0 0

3
1
+ 1 5.

car A = 3 e nul A = 0.

= 0 ento car A = 2 e nul A = 1.

Assim, A invertvel se e s se
colunas de A .

1
(ii) 4 2
3

2 5
1

1
2

6= 0, uma vez que s neste caso que car A = no de

2L1 +L2 !L2


3L1 +L3 !L3

3
1
4 0 1 + 2 2 (1 + ) 5.
!
2L2 +L3 !L3
0
0
3+
2
3
1
1
2 5. Se
Seja A = 4 2
3
2
1

Se

3
1
+1 5
1

3 ou

2
4
(iii) 2
0

1
2

1
4 0
0

6=

1+2
2 4

3e

6=

2+2
1 3

3
5

2L2 +L3 !L3

1
ento car A = 3 e nul A = 0.
2

1
ento car A = 2 e nul A = 1.
2

Assim, A invertvel se e s se
car A = no de colunas de A .
2

1 5
+1

6=

L1 +L2 !L2

3e

6=

2
4 0 1
0 2

158

1
, uma vez que s neste caso que
2
3

1+ 5
!
L2 +L3 !L3
1
+1
2

3
2
2
5.
) (1 + )
1+
! 4 0 (1
L2 +L3 !L3
0
0
2 ( + 1)
2
3
2
2
1
1 5. Se = 1 ento car A = 1 e nul A = 2.
Seja A = 4 2
2
0
1
+1

Se

= 1 ento car A = 2 e nul A = 1.

Se

6=

1e

6= 1 ento car A = 3 e nul A = 0.

Assim, A invertvel se e s se
car A = no de colunas de A .
2

3
0 1
1 0 0 7
7
0
0 5
1 1 2

1
6 0
(iv) 6
4 3
1
!

L2 +L4 !L4

1
6 0
6
4 0
0

0
1
0
0

1
6 0
Seja A = 6
4 3
1
Se

6= 2 e

6=

3L1 +L3 !L3


L1 +L4 !L4

1
0
0
+3 3
0
2

6=
2

1
6 0
6
4 0
0

1
6 0
(v) 6
4 1
2

0
1
0
0

3
0 1
1 0 0 7
7. Se
0
0 5
1 1 2

0
1
0
1

1
0
0
+3 3
0
2

= 2 ou

7
7
5 L2 +L!
4 !L4

3 ento car A = 3 e nul A = 1.

0 7
7
!
1 5 L1 +L3 !L3
2 2L1 +L4 !L4
1
1

6= 2 e
2

1
6 0
6
4 0
0

6=

3, uma vez que s neste caso que

0
1
0 (1
0

1
6 0
Seja A = 6
4 1
2

0
1
0
0

Se

Se

6= 1 e

1 ento car A = 4 e nul A = 0.

0 7
7. Se
1 5
2

3
1
7
1
0
7.
) (1 + )
1 5
0
2(
1)

= 1 ento car A = 2 e nul A = 2.

1 ento car A = 3 e nul A = 1.


6=

3 ento car A = 4 e nul A = 0.

1
1

6= 1, uma vez que s neste caso que

7
7.
5

Assim, A invertvel se e s se
car A = no de colunas de A .
2

1 e

159

Assim, A invertvel se e s se
car A = no de colunas de A .
2

1
6 1
(vi) 6
4 1
1
!

L1 +L2 !L2
L1 +L3 !L3
L1 +L4 !L4

1
1
3

1
1
2

1
6 0
6
4 0
0
2

1
+1
0
0

1
6 0
Seja A = 6
4 1
2

0
1
0
0

0
0
0
1

1
1
2

7
7
5

6= 1 e

1
1) ( + 1)
0
(

0 7
7. Se
1 5
2

= 0 ento car A = 3 e nul A = 1.

Se

Se

= 1 ento car A = 2 e nul A = 2.

Se

6= 0 e

1 ento car A = 1 e nul A = 3.

6= 1 e

6=

1 ento car A = 4 e nul A = 0.


6= 1 e

8
< 3x
13. (a) Tem-se 2y
:
8x

8
< 3x
2y
Logo,
: 8
z
3

1
s
4

3
0
7
0
7.
5
0
1) ( + 1)

= 1 ento car A = 2 e nul A = 2.

Se

Assim, A invertvel se e s se
car A = no de colunas de A .

1, uma vez que s neste caso que

L1 +L2 !L2
L1 +L3 !L3
L1 +L4 !L4

6=

6=

1, uma vez que s neste caso que

2
z=0
3 0
2z w = 0 e assim, 4 0 2
2w = 0
8 0
8
x = 14 w
>
>
>
>
z=0
<
2z w = 0 ,
y = 45 w A soluo
>
>
2w = 0.
>
>
:
z = 34 w.

1
2
0

3
0 j 0
1 j 0 5
2 j 0

8
L +L2 !L2
3 1

3 0
4 0 2
0 0

3
x
6 y 7
7
geral do sistema : X = 6
4 z 5=
w

6
7
6 5 7
6 s 7
6 4 7
6
7
6 3 7, para qualquer s 2 R, isto , o conjunto soluo dado por: S =
6 s 7
6 4 7
4
5
s
160

1
s; 54 s; 34 s; s
4

:s2R .

1
2
8
3

0 j
1 j
2 j

Para s = 4, tem-se a seguinte soluo para a equao qumica: x = 1; y = 5; z = 3; w =


4:
8
< x 6z = 0
(b) Tem-se 2x + y 6z 2w = 0 e assim,
:
2y 12z = 0
2
2
2
3
3
3
1 0
6 0 j 0
1 0
6 0 j 0
1 0
6 0 j 0
4 2 1
4 0 1 6
4 0 1 6
6
2 j 0 5
2 j 0 5
2 j 0 5:
!
!
2L1 +L2 !L2
2L2 +L3 !L3
0 2
12 0 j 0
0 2
12 0 j 0
0 0
24 4 j 0
8
8
< x 6z = 0
< x=w
y + 6z 2w = 0 ,
y=w
Logo,
A soluo geral do sistema S = s; s; 61 s; s : s 2 R .
:
:
1
24z + 4w = 0.
z = 6 w.
Para s = 6, tem-se a seguinte soluo para a equao qumica: x = 6; y = 6; z = 1; w = 6:

14 (a)

2 3 j 1
5 7 j 3

2x + 3y = 1
,
1
y = 12
2

Logo,

2
0

5
L +L2 !L2
2 1

3
1
2

j 1
.
j 12

x=2
y = 1.

A soluo geral do sistema S = f(2; 1)g.


2 4 j 10
3 6 j 15

(b)

3
L +L2 !L2
2 1

2 4 j 10
. Logo, 2x + 4y = 10 , x = 5
0 0 j 0

A soluo geral do sistema S = f(5


4
2 j 5
6 3 j 1
impossvel). S = ?.
(c)

2
4
(d) 3
5
!

1
2
3

11
L +L3 !L3
7 2

3
L +L2 !L2
2 1

3
3 j 5
2 j 5 5
1 j 16

2
4 0
0

4
0

1
7=2
0

3
L +L2 !L2
2 1
5
L +L3 !L3
2 1

2y.

2s; s) : s 2 Rg.
2 j
0 j

2
4 0
0

3
3
j
5
13=2 j
5=2 5.
26=7 j 52=7
161

5
17
2

. Logo, o sistema no tem soluo (

1
3 j
7=2 13=2 j
11=2 13=2 j

3
5
5=2 5
7=2

11
L +L3 !L3
7 2

8
8
< 2x + y 3z = 5
< x=1
7
13
5
y
+
z
=
y = 3 A soluo geral do sistema S = f(1; 3; 2)g.
Logo,
,
2
2
: 226
:
52
z= 7
z = 2.
7
2

2
4
(e) 1
4

3
2 j 5
3 j 2 5
4 j 1

3
2
1

2L2 +L3 !L3

2
4 0
0

3
7=2
0

1
L +L2 !L2
2 1

2L1 +L3 !L3

2
4 0
0

3
7=2
7

2 j
4 j
8 j

3
5
1=2 5.
8

2 j
4 j
0 j

3
5
1=2 5
9

2L2 +L3 !L3

Logo, o sistema no tem soluo ( impossvel). S = ?.


2

3
1 2 3 j 3
(f) 4 2 3 8 j 4 5
3 2 17 j 1
Logo,

2L1 +L2 !L2


3L1 +L3 !L3

x + 2y + 3z = 3
,
y + 2z = 2

1
4 0
0

2 3 j
1 2 j
4 8 j

2
(g) 4 1
3

3
2
3 j 3
1
2 j 5 5 ! 4 2
L1 $L2
2 j 7
3

2L1 +L2 !L2


3L1 +L3 !L3

Logo,

1
4 0
0

x 2y = 5
,
7y = 7

1 2
(h) 4 2 4
3 6
!

2 j
7 j
8 j

1
L +L3 !L3
3 2

3
5
7 5
8

1
6
0

1
4 0
0

2 3 j
1 2 j
0 0 j

1; 2s + 2; s) : s 2 Rg.

3
2 j 5
3 j 3 5
2 j 7
!

8
L +L3 !L3
7 2

2L1 +L2 !L2


3L1 +L3 !L3

1
4 0
0

2 j
7 j
0 j

3
5
7 5.
0

x=3
A soluo geral do sistema S = f(3; 1)g.
y = 1.

3
1 3 j 3
4 3 j 9 5
1 8 j 10

1 2
4 0 0
0 0

4L2 +L3 !L3

x = 7z 1
y = 2z + 2.

A soluo geral do sistema S = f( 7s


2

3
3
2 5
8

2L1 +L2 !L2


3L1 +L3 !L3

3
3 j 3
3 j 3 5.
0 j 0

1 2
4 0 0
0 0

162

1
6
2

3
3 j 3
3 j 3 5
1 j 1

1
L +L3 !L3
3 2

3
3
2 5.
0

Logo,

x = 2y 52 w +
z = 12 w + 12 .

x + 2y
z + 3w = 3
,
6z 3w = 3

A soluo geral do sistema S =


2

1
4
(i) 3
2
!

5 4
1 2
2 3

1
L +L3 !L3
2 2

3
13 j 3
5 j 2 5
4 j 1

1
4 0
0

5
16
0

3L1 +L2 !L2


2L1 +L3 !L3

+ 72 ; s; 21 t + 12 ; t : s; t 2 R .

1
4 0
0

5
16
8

3
3
7 5
5

13 j
44 j
22 j

4
10
5

3
3
7 5.

13 j
44 j
0 j

4
10
0

5
t
2

2s

7
2

1
L +L3 !L3
2 2

3
2

Logo, o sistema no tem soluo ( impossvel). S = ?.


2

0 0
2
6 2 0
6
(j) 6
4 2 2
5
0 100 150
!

L1 +L2 !L2

L3 +L4 !L4

2
6 0
6
4 0
0

2
2
0
2

5
1
2
3

2
6 0
6
4 0
0

2
2
0
0

5
1
2
0

3
j 4
j 7 7
7
j 4 5
j 50

3
9
2
200

j
j
j
j

2
7
3
4
2
7
3
0

j
j
j
j

L1 $L3
1
L !L4
50 4

2
0
0
2

5
6
2
3

2
9
3
4

j
j
j
j

3
4
7 7
7
4 5
1

2
6 0
6
4 0
0

2
2
0
0

5
1
2
2

2
7
3
3

j
j
j
j

2
6 2
6
4 0
0

3
4
3 7
7
!
4 5 L2 +L4 !L4
1

3
4
3 7
7.
4 5
0

8
x1 =
>
8
>
>
>
< 2x1 + 2x2 5x3 + 2x4 = 4
<
x2 =
2x2 x3 + 7x4 = 3
Logo,
,
:
>
>
2x3 + 3x4 = 4
>
>
:
x3 =

19
2

L1 +L2 !L2

3
4
3 7
7
4 5
4

9x4

17
x
4 4

5
2

3
x
2 4

+2

A soluo geral do sistema dada por


S=

1
4
3
(k)
3

2
1
6

3
2
9

19
2

1 j
5 j
3 j

9s;

3
1
2 5
6

17
s
4

5
;
2

3L1 +L2 !L2


3L1 +L3 !L3

163

3
s + 2; s
2

1
4 0
0

2
5
0

:s2R :

3
7
0

1 j
8 j
0 j

L3 +L4 !L4

3
1
1 5.
3

Logo, o sistema no tem soluo ( impossvel). S = ?.


2

3
2 3
1 1
1
1 5 e B = 4 1 5.
15. (a) Sejam A = 4 1
1 1
1
2

2
3
3
1 1 j 1
1 1
j 1
1 j 1 5 ! 4 1
1 j 1 5
[A j B] = 4 1
L1 $L3
1 1
j 1
1 1 j 1
!

L1 +L2 !L2
L1 +L3 !L3

1
1
4 0
1 1
0 1
1

j
1
j
0
j 1

L2 +L3 !L3

L1 +L2 !L2
L1 +L3 !L3

1
4 0
0

1
1
0

(1

j
1
1
j
0
) ( + 2) j 1

Se = 1 ento car A = car [A j B] = 1 < 3 = no de incgnitas do sistema. Logo o


sistema possvel e indeterminado, tendo-se x + y + z = 1. A soluo geral deste sistema
ento dada por
S1 = f(1 s t; s; t) : s; t 2 Rg :

Se

2 ento car A < car [A j B]. Logo, o sistema no tem soluo ( impossvel).
| {z }
|
{z
}
=2

= ?.

=3

Se 6= 1 e 6= 2 ento car A = car [A j B] = 3 = no de incgnitas do sistema. Logo


o sistema possvel e determinado, tendo-se
8
8
< x+y+ z =1
< x = 1= ( + 2)
(
1) y + (1
)z = 0 ,
y = 1= ( + 2)
:
:
(1
) ( + 2) z = 1
z = 1= ( + 2) .
A soluo geral do sistema ento dada por S =

(b) Sejam A =

[A j B] =

1 2
2
8

1 2
j 1
2
8 j 3

eB=

1
;
+2

1
;
+2

j 1
.
j 1

1
.
3

2L1 +L2 !L2

1
0

164

2
4 8

1
+2

5.

Se =
6 4 ento car A = car [A j B] = 2 < 3 = no de incgnitas do sistema. Logo o
sistema possvel e indeterminado, tendo-se
8
2
>
>
( + 4) z
>
< x=1
4
x + 2y + z = 1
,
(
4) y + (8 2 ) z = 1
>
>
1
>
: y=
+ 2z.
4
A soluo geral deste sistema ento dada por
S =

Se

+ 2s; s

:s2R :

= 4 ento car A < car [A j B]. Logo, o sistema no tem soluo ( impossvel).
| {z }
|
{z
}
=1

S4 = ?.

( + 4) s;

=2

1 1
(c) Sejam A = 4 3 4
2 3
!

3L1 +L2 !L2


2L1 +L3 !L3

1 1
4 0 1
0 1

1 1
2 5 e B = 4 5. [A j B ] = 4 3 4
1
1
2 3

2 3
1 2

j
j
j

2
3

6 5

L2 +L3 !L3

1 1
4 0 1 2 3
0 0
3+

3
j 2
5
2 j
1 j 1

3
j
2
j
6 5.
j 3

3L1 +L2 !L2


2L1 +L3 !L3

Se = 3 ento car A = car [A j B ] = 2 < 3 = no de incgnitas do sistema. Logo o


sistema possvel e indeterminado, tendo-se
8
< x = 5 10z
x + y + 3z = 2
,
y 7z = 3
:
y = 3 + 7z.
A soluo geral deste sistema ento dada por
S3 = f(8

+ (2

4 ) s;

6 + (3

2) s; s) : s 2 Rg :

Se 6= 3 ento car A = car [A j B ] = 3 = no de incgnitas do sistema. Logo o sistema


possvel e determinado, tendo-se
8
8
< x+y+ z =2
< x=6+3
y + (2 3 ) z =
6 ,
y= 4 2
:
:
( 3 + )z = 3
z = 1.
165

A soluo geral do sistema ento dada por


S = f(6 + 3 ; 4

3
2
3
1 1
1
5.
1 5eB =4
(d) Sejam A = 4 1
2
1 1
2

1 1
j
1 j
! 4 1
L1 $L3
1 1 j
!

L2 +L3 !L3

1
4 0
0

1
1
0

(1

3
5

L1 +L2 !L2
L1 +L3 !L3

2 ; 1)g :

1 1 j
4
1 j
[A j B ] = 1
1 1
j

1
1
4 0
1 1
0 1
1

2
j
1
j
(1
)
) ( + 2) j (1 + ) (1

j
j
j 1

2
3

3
5

1
2

3
5

L1 $L3

L2 +L3 !L3

5.

Se = 1 ento car A = car [A j B ] = 1 < 3 = no de incgnitas do sistema. Logo o


sistema possvel e indeterminado, tendo-se x + y + z = 1. A soluo geral deste sistema
ento dada por S1 = f(1 s t; s; t) : s; t 2 Rg.
Se

2 ento car A < car [A j B ]. O sistema no tem soluo ( impossvel).


| {z }
|
{z
}
=2

= ?.

=3

Se 6= 1 e 6= 2 ento car A = car [A j B ] = 3 = no de incgnitas do sistema. Logo


o sistema possvel e determinado, tendo-se
8
8
< x+y+ z = 2
< x = ( + 1) = ( + 2)
y = 1= ( + 2)
(
1) y + (1
) z = (1
)
,
:
:
2
(1
) ( + 2) z = (1 + ) (1
)
z = (1 + )2 = ( + 2) .
A soluo geral do sistema ento dada por
(
!)
+1
1
(1 + )2
S =
;
;
:
+2 +2
+2

8
<

x+y + z =1
2x + y 2 z =
(e)
:
x+ y +z = 1+2

1 1
4
2
Sejam A =
166

2
1

5eB=4

1
1+2

5.

1 1
[A j B] = 4 2

2
1

j
j
j

1
1+2

3
5

2L1 +L2 !L2


L1 +L3 !L3

1
4 0
0

1
+2
0
(1

j
0
j
) (1 + ) j

3
1
+ 2 5:
1+

Se = 1 ento car A = car [A j B] = 2 < 3 = no de incgnitas do sistema. Logo o


sistema possvel e indeterminado, tendo-se
x+y+z =1
,
3y = 3:

x=z
y = 1:

A soluo geral deste sistema ento dada por


S1 = f(s; 1; s) : s 2 Rg :
Se = 2 ento ento car A = car [A j B] = 2 < 3 = no de incgnitas do sistema.
Logo o sistema possvel e indeterminado, tendo-se
x + y 2z = 1
,
3z = 3:

x=y
z = 1:

A soluo geral deste sistema ento dada por


S
Se

= f(s

3; s; 1) : s 2 Rg :

1 ento car A < car [A j B]. Logo, o sistema no tem soluo ( impossvel).
| {z }
|
{z
}
=2

=3

= ?:

Se 6= 1 e 6= 1 e 6= 2 ento car A = car [A j B] = 3 = no de incgnitas do sistema.


Logo o sistema possvel e determinado, tendo-se
8
8
= ( + 1)
< x+y+ z =1
< x=
( + 2) y = + 2
y=1
,
:
:
(1
) (1 + ) z = 1 +
z = 1= ( + 1) .
A soluo geral do sistema ento dada por
S =

1 4
4
16. (a) Sejam A = 2 7
1 5
2

1 4
4
[A j B ] = 2 7
1 5

+1

; 1;

1
+1

3
2
3
3
10
2 5 e B = 4 10 5.

3
3 j 10
2 j 10 5
j

2L1 +L2 !L2


L1 +L3 !L3

167

2L1 +L2 !L2


L1 +L3 !L3

1
4 0
0

4
1
1

3
8

j
j
3 j

2
3
10
1
4
10 5
0
!
L2 +L3 !L3
0
10

3
j
8
j
11 j

4
1
0

3
10
10 5.
20

Se = 11 e = 20 ento car A = car [A j B ] = 2 < 3 = no de incgnitas do sistema.


Logo o sistema possvel e indeterminado, tendo-se
x + 4y + 3z = 10
,
y 8z = 10

x = 30 + 29z
y = 10 8z.

A soluo geral deste sistema ento dada por S


Se

= 11 e

impossvel). S

8s; s) : s 2 Rg.

20 ento car A < car [A j B ]. Logo, o sistema no tem soluo (


| {z }
|
{z
}

6=
;

= f( 30 + 29s; 10

=2

=3

= ?.

Se
6 11 ento car A = car [A j B ] = 3 = no de incgnitas do sistema. Logo o
=
sistema possvel e determinado, tendo-se
8
8
29 + 250) = (
11)
< x + 4y + 3z = 10
< x = (30
y 8z = 10
y = (10
8 + 50) = (
11)
,
:
:
(
11) z =
20
z=(
20) = (
11) .
A soluo geral do sistema ento dada por
S

0
6 1
(b) Sejam A = 6
4 2
1
2

2
6 1
! 6
L1 $L3 4 0
1
!

2L1 +L2 !L2


L1 +L4 !L4

2
1
0
1
2

1 1
1 3
2
3 14

1
6 0
6
4 0
0

30

1
0
0
0

0
1
2
1

29 + 250 10
;
11

3
2
2
6
1 3 7
7eB = 6 1
4 2
1 1 5
3 14
4

3
j 2
j 1 7
7 !
5 L1 $L2
j
j 4
1
3
1
5
2
2 11

1
6 2
6
4 0
1

1
2
0
1

7
7:
5

1 3
1 1
2
3 14

3
j 1
j 0 7
7
5 2L2 +L!
j
3 !L3
2L2 +L4 !L4
j 3
168

8 + 50
;
11

1
6 0
6
4 0
0

20
11

0
6 1
[A j B ] = 6
4 2
1
3
j 1
j 2 7
7
5
j
j 4
1
0
0
0

1
1
0
0

0
1
2
1

2
1 3
1 1
3 14

3
j
j 1 7
7 !
j 2 5 L1 $L3
j 4

2L1 +L2 !L2


L1 +L4 !L4

3
5
10
1

3
j 1
j 0 7
7 !
5 L1 $L2
j
j 3

1
6 0
! 6
L1 $L2 4 0
0

1
0
0
0

1
1
0
0

3
5
1
10

3
j 1
j 0 7
7
j 3 5
j

10)L3 +L4 !L4

1
6 0
6
4 0
0

1
0
0
0

1
1
0
0

j
j
j
j

3
5
1
0

1
0
3
3(

10) +

Se = 3 (
10) ento car A = car [A j B ] = 3 < 4 = no de incgnitas do sistema.
Logo o sistema possvel e indeterminado, tendo-se
8
8
< x + y + z + 3w = 1
< x=7 y
z 5w = 0
z = 15 :
,
:
:
w=3
w = 3.
A soluo geral deste sistema ento dada por S
Se

6= 3 (

impossvel). S

= f(7

s; s; 15; 3) : s 2 Rg.

10) ento car A < car [A j B ]. Logo, o sistema no tem soluo (


| {z }
|
{z
}
=3

=4

= ?.
2

(c) Sejam A = 4 1
1
2
1
4
2
[A j B ] = 1
1
1
2
1
1 1
2 2
! 4 1
L1 $L3
1
1
2
1
1
4
0
1
!
L1 +L2 !L2
0
+1
L1 +L3 !L3
2
1
1
4
0
1
!
( +1)L2 +L3 !L3
0 0

1
2
1

1
2
1

1
+1

3
0
5 e B = 4 1 5:

3
j 0
1
j 1 5 !
L1 $L3
+1 j
3

1
2
1

+1 j
1
j 1 5
j 0
1
1

L1 +L2 !L2
L1 +L3 !L3

+1 j
j 1
2
j
j
j
j

1
+1
1
0 ( 2
1)

( +1)L2 +L3 !L3

1
2

+1

5.

1
Se 6= 0 e 6=
ento car A = car [A j B ] = 3 < 4 = no de incgnitas do sistema.
2
Logo o sistema possvel e indeterminado, tendo-se
8
< x

y + z + ( + 1) w =
y+z
w=1
:
( 2
1) w = + 1 + ( 2

1)
169

8
>
< x=
,
y=z
>
: w=
(

+1
2 +1
+1
2 +1
+1
2
1)

( +1)2
( 2
1)

1
1

+ .

7
7.
5

A soluo geral do sistema ento dada por

+1
2 +1

( + 1)2
( 2
1)

;s

+1
2 +1

+1
1; s;
( 2
1)

!)

Se = 0 e = 1 ento car A = car [A j B ] = 2 < 4 = no de incgnitas do sistema.


Logo o sistema possvel e indeterminado, tendo-se
x

y+z+w =1
,
y+z =0

x=1 w
y = z.

A soluo geral deste sistema ento dada por S

Se ( = 0 e

6= 1) ou

soluo ( impossvel). S

=
;

1
4
17. (a) Sejam A = 3
1
1
4
[A j Ba;b;c ] = 3
1
!

3L1 +L2 !L2


L1 +L3 !L3

1
4 0
0

2
1
5

= f(1

2
1
5

s; t; t; s) : s; t 2 Rg.

1
ento car A < car [A j B ]. Logo, o sistema no tem
| {z }
{z
}
|
2
=2

= ?.

=3

3
2 3
3
a
5
4
2
e Ba;b;c = b 5 :
8
c

3
3 j a
2 j b 5
8 j c

3L1 +L2 !L2


L1 +L3 !L3

3
2
3 j
a
7 11 j b 3a 5
7 11 j c a

L2 +L3 !L3

1
4 0
0

3
2
3 j
a
7 11 j
b 3a 5.
0
0 j c b + 2a

Para que haja soluo necessrio que car A = car [A j Ba;b;c ], isto , necessrio que
c

1
4
(b) Sejam A = 2
3
2

1
4
[A j Ba;b;c ] = 2
3

2
3
1

2
3
1

b + 2a = 0:

3
2 3
4
a
5
4
1 e Ba;b;c = b 5 :
2
c

3
4 j a
1 j b 5
2 j c

2L1 +L2 !L2


3L1 +L3 !L3

170

2L1 +L2 !L2


3L1 +L3 !L3

1
4 0
0

3
4 j
a
9 j b 2a 5
10 j c 3a

2
7
7

L2 +L3 !L3

1
4 0
0

2
7
0

3
4 j
a
9 j b 2a 5.
1 j c b a

Como car A = car [A j Ba;b;c ], este sistema tem soluo para quaisquer valores de a; b; c.

18. (a) Sejam x = 1 + t e y = 1

t. Logo
x + y = 2:

(b) Sejam x = t, y = 1

2t e z = 1. Tem-se ento o seguinte sistema:


8
< 2x + y = 1
:

z = 1.

(c) Sejam x = 3t, y = 2t e z = t. Tem-se ento o seguinte sistema:


8
< x 3z = 0
:

(d) Sejam x = 3t

x)

Deste modo:
s=3
Com

2z = 0.

1ez=s

s, y = t + 2s

y = t + 2 (3t

1 = 7t

z=s

2x

y + 2x + 1
7

x+3
7
Tem-se ento a seguinte equao linear:
s=

3y

2t + 1 =

3y

2t + 1. Logo s = 3t

x=
e t=

x+3
7

1,t=
3y

y + 7z = 8.

171

x+3
7

y + 2x + 1
7

Isto :
5x

y + 2x + 1
:
7

y + 2x + 1
+ 1.
7

x e assim

(e) Sejam x = 2t
Assim:

3s, y = t + s

1, z = 2s + 1 e w = t

8
>
>
>
< x = 2 (w + 1)

>
>
z
>
: y =w+1+

2y

2w =

19. Pretende-se determinar

a b
c d

a b
c d

x e t = w + 1,

x+y+w =0
2x + z = 2

Tem-se ento

x + y = 0.

tal que

1 2
3 4

1 2
3 4

8
a + 2c = a + 3b
>
>
<
b + 2d = 2a + 4b
,
3a + 4c = c + 3d
>
>
:
3b + 4d = 2c + 4d

As matrizes reais que comutam com

20. Existem 16 matrizes 2


;

0 1
1 0

1 2
3 4

a b
c d

c = 23 b
d = a + 32 b

so da forma:

a
3
b
2

b
, com a; b 2 R.
a + 32 b

2 s com 0 e 1 nas respectivas entradas. 6 so invertveis:


;

1 1
0 1

;
172

1 1
1 0

1.

8
< x+y =1
:

1 0
0 1

1.

(f) Seja S = f(1 s; s t; 2s; t 1) : s; t 2 Rg.


Sejam x = 1 s, y = s t, z = 2s, w = t 1. Uma vez que s = 1
tem-se ento o seguinte sistema linear no homogneo

(g) Por exemplo:

y = 1 x (w + 1)
,
z = 2 (1 x)

2
Deste modo, obtm-se o sistema de equaes lineares:
8
< 2x + 3z 4w = 7
:

1. Logo t = w + 1 e s =

0 1
1 1

1 0
1 1

21. Como
1
A
2

A2 + 2A + 2I = 0 , A
ento A invertvel e
1

22. Sejam A; B; X 2 Mn

1
A
2

1
A
2

I A=I

I.

(R) matrizes invertveis tais que


3 4
7 9

(AB)2 =

(i)
AXB + AB = 0 , AXB =

AB , A

, A 1 A X BB

, IXI =

A 1A

(AXB) B

=A
1

BB

II , X =

( AB) B

I.

(ii)
A 1B

BXA

=0,B

, B 1 B X AA

(BXA) A

= B 1A

1
1

, IXI = (AB)2

23. A 2 M2

3A
3A

1 T

,A=

24. (i)

A 1B

B 1A

9
7

4
3

(R),

2I
, 2I

3 4
7 9

,X=

=B

1 T

=
1
3

0 1 j 1 0
1 0 j 0 1

=
5
7
7
7

L1 $L2

4 3
7 5
3
4
3
6

, 2I

, 3A

1 T

3A
1 T

7
3

2 0
0 2

=
7
3

1 0 j 0 1
. Logo
0 1 j 1 0
173

4 3
7 5

5
7

0 1
1 0

6
7
3
7

1
1

3
4

0 1
1 0

(ii)

1 0
0 1

(iv)

1 2 j 1 0
3 4 j 0 1

L2 +L1 !L1

1
0

1 2
3 4

Logo

1 0
0 1

2 1
3 1
2

3
1 2 3 j 1 0 0
(v) 4 4 5 6 j 0 1 0 5
7 8 9 j 0 0 1
!

2L2 +L3 !L3

1
4 0
0

2
3
0
3

3 j
6 j
0 j

L2 +L1 !L1

3
2

1
2

1
2

1 0
3 1

1 0 j
0 1 j

1
L !L2
2 2

3
2

= [1]

2 j
2 j

1
0

3L1 +L2 !L2

0 j
2 j

(iii) [1]

4L1 +L2 !L2


7L1 +L3 !L3

1
4
1

1
4 0
0

2
3
6

3
0 0
1 0 5.
2 1

3
1 0 0
4 1 0 5
7 0 1

3 j
6 j
12 j

2L2 +L3 !L3

1 2 3
4
Logo, 4 5 6 5 singular e como tal no invertvel.
7 8 9
Nas alneas (vi) e (vii) s se apresentam as solues:
2
3 1 2
3
0 2
2
2
1 2 3
1
2 1
14
4
4
5
5
@
1 2
2
(vi) 0 1 2
= 0 1
(vii)
3
0 0 1
0 0
1
2 1
2

3
1 0 0
0 1 0 5
0 0 1

1 0 2 j
4
(viii) 0 3 0 j
4 0 5 j
2
1 0
4
0 3
!
2
L +L1 !L1
3 3
0 0
2

3
1 0 2
Logo 4 0 3 0 5
4 0 5

0 j
0 j
3 j
1

4L1 +L3 !L3

3
0 23
0 1 0 5
4 0 1
5
3

5
3

=4 0
4
3

0
1
3

2
3

1 0
4 0 3
0 0
2
!

1
L !L2
3 2
1
L !L3
3 3

0 5.
1
3

174

2 j
0 j
3 j

31
1
2 5A
2

3
1 0 0
0 1 0 5
4 0 1

1 0 0 j
4 0 1 0 j
0 0 1 j

5
3

0
4
3

0
1
3

=4

2
3
1
3

2
3

2
L +L1 !L1
3 3

2
3

0 5.
1
3

1
3
2
3
2
3

2
3
1
3

2
3

3
5

3
1 2 1 j 1 0 0
(ix) 4 4 0 6 j 0 1 0 5
1 8 1 j 0 0 1
!

3
L +L3 !L3
4 2

2L3 +L2 !L2


L3 +L1 !L1

2L2 +L1 !L1

6
6
6 0
6
4
0

6
6
6
6
4

1 0 0 j
j
0 1 0 j
j
0 0 1 j

1 2 1
Logo 4 4 0 6 5
1 8 1

(x) Para
cos
sen

1
sen cos

(k 2 Z).
Logo

1
2

1
6

8
3

1
2

2
3

1
2

6
6
=6
6
4

2
3

1
6

1
6

1
6

8
3

1
2

2
3

j 1 0
j 0 1

sen
cos

4
3

1
2

2
3

0
8
3

7
7
7
7
5

3
1 0 0
4 1 0 5
1 0 1

2 1 j
8 2 j
0 1 j

1
4
0
!
2
L !L3
3 3
0

1
2

4
3

2 1 j
8 2 j
6 0 j

1
L !L2
8 2

6
6
6
6
4

3
L +L3 !L3
4 2

3
1 0 0
4 1 0 5
8
3

1 2 0 j
j
0 1 0 j
j
0 0 1 j

1
2

2L3 +L2 !L2


L3 +L1 !L1

2
3

11
3

1
2

2
3

1
6

1
6

8
3

1
2

2
3

3
7
7
7
7
5

2L2 +L1 !L1

7
7
7.
7
5

7
7
7.
7
5

6= k ; (k 2 Z)
2

L2 +L1 !L1

sen2

11
3

0 j
j
8 0 j
j
0 1 j

1
4 0
0

4L1 +L2 !L2


L1 +L3 !L3

3
1 0 0
4 1 0 5
4 34 1

2 1 j
8 2 j
0 23 j

1
4 0
0

1
sen2

)L1 +L2 !L2

L2 !L2

cos
sen

1
0
0 sen cos

sen
cos

cos2
sen2

j cos
j
0

sen
sen

(cos )L1 !L1


(sen )L2 !L2

0
sen cos

1 0 j
0 1 j

cos
sen

j
j

cos sen
sen cos

cos
sen2 cos

0
sen

L2 +L1 !L1

sen2 )L1 +L2 !L2

sen
sen (1 sen2 )

sen
cos

. Note que sen cos

cos
sen

sen
cos

175

j cos
j
0

, para todo o

sen

1
cos

6= 0 para todo o

6= k ; (k 2 Z)
2

L2 !L2

6= k ;
2

Se

+ 2k ; (k 2 Z) ;

cos
sen
Se

1
0

0 1
1 0

cos
sen

sen
cos

sen
cos

1 0
0 1

1 0
0 1

1
0

cos
sen

sen
cos

+ 2k ; (k 2 Z) ;

=
cos
sen

Se

0
1

= 2k ; (k 2 Z),
cos
sen

Se

sen
cos

sen
cos

1
0

0
1

0
1

cos
sen

sen
cos

3
+ 2k ; (k 2 Z),
2
cos
sen

sen
cos

Logo, para todo o

0 1
1 0

0
1

1
0

cos
sen

sen
cos

2R
cos
sen

sen
cos

cos
sen

sen
cos

(xi) Seja k 6= 0.
2

k
6 1
6
4 0
0

0
k
1
0

0
0
k
1

0
0
0
k

1
L2 +L3 !L3
k2
1
L !L1
k 1

1
L +L4 !L4
k 3
1
L !L2
k 2

j
j
j
j

1
6 0
6
4 0
0

1
6 0
6
4 0
0

1
0
0
0

0
1
0
0

0
0
1
0

3
0
0 7
7
0 5
1

0 0 0 j
k 0 0 j
0 1 0 j
0 k1 1 j
0
1
0
0

0
0
1
0

0
0
0
1

j
j
j
j

1
L +L2 !L2
k 1
1
L !L3
k 3
1
L !L4
k 4

1
k

1
k

1
k3

0
1
k

1
k2
1
k3
1
k4

0
1
1
k2

0
0
1
k

1
k2
1
k3

k 0 0 0 j
6 0 k 0 0 j
6
4 0 1 1 0 j
k
0 0 k1 1 j

3
0 0
0 0 7
7
1
5
0
k
1
0 k
0
0
1
k

1
k2

176

3
0
0 7
7.
0 5
1
k

1
L +L4 !L4
k 3
1
L !L2
k 2

1
1
k

0
0

3
0 0 0
1 0 0 7
7
0 k1 0 5
0 0 k1

1
L2 +L3 !L3
k2
1
L !L1
k 1

k
6 1
Logo 6
4 0
0

0
k
1
0

3
0
0 7
7
0 5
k

0
0
k
1

1
k

6
=6
4

1
k2
1
k3
1
k4

0
0

1
k

1
k2
1
k3

1
k

1
k2

3
0
0 7
7.
0 5
1
k

(xii) Sejam k1 ; k2 ; k3 ; k4 6= 0.
2

j
j
j
j

0 0 0 k1
6 0 0 k2 0
6
4 0 k3 0 0
k4 0 0 0

1
L !L1
k4 1
1
L !L2
k3 2
1
L !L3
k2 3
1
L !L4
k1 4

1
6 0
6
4 0
0

0
1
0
0

0
0
1
0

1
0
0
0

0
1
0
0

j
j
j
j

0
0
0
1

0
0
0
1
1
k1

3
0 0 0 k1
6 0 0 k2 0 7
7
Logo 6
4 0 k3 0 0 5
k4 0 0 0
2

5
13

6
6
6
6
(xiii) 6
6
6
6
4

6
6
6 2
6
6
6
6 2
6
4
8

2
13
7
13

2
13
2
13

7
6
2

6
13

6
13

8
13

2
13

2
13

2
6
7
2

7
13
2
13

8 j
j
2 j
j
2 j
j
5 j

3
0
0 7
7 !
0 5 L1 $L4
1 L2 $L3

0
0
1
0

0
0
1
k2

0
1
k3

0
0

0
6 0
=6
4 0
1
1
k1
8
13
2
13
2
13
5
13

7
7
7
7
7=
7
7
7
5

1 0 0 0

j
j
j
j

k4 0 0 0
6 0 k3 0 0
6
4 0 0 k2 0
0 0 0 k1

1
k4

0
0
0
1

0
0
1
0

0
1
0
0

3
1
0 7
7
0 5
0

1
L !L1
k4 1
1
L !L2
k3 2
1
L !L3
k2 3
1
L !L4
k1 4

0 7
7.
0 5
0

0
0
1
k2

1
k4

0
0

0 7
7.
0 5
0

1
k3

6
6
6 2
6
1 6
13 6
6 2
6
4
8

7
7
0 1 0 0 7
7
7 !
7 L1 $L3
0 0 1 0 7
7
5
0 0 0 1
177

7
6
2
2

6
6
6 2
6
6
6
6 5
6
4
8

6
7
2
6
7

7
7
2 7
7
7:
7
2 7
7
5
5
7
6

j
j
2 j
j
8 j
j
5 j
2

0 0 1 0

7
7
0 1 0 0 7
7
7
7
1 0 0 0 7
7
5
0 0 0 1

L1 +L2 !L2
5
L +L3 !L3
2 1
4L1 +L4 !L4

j
j
0 j
j
13 j
j
13 j

0 0

j
j
j
j
j
j
j

6
6
6 0
13 13
6
6
!
L1 +L2 !L2 6
6 0
13 39
5
2
L +L3 !L3 6
2 1
4
4L1 +L4 !L4
0 26
26
2
2 6
7 2
6
6
6 0
13 13
0
6
6
!
L2 +L3 !L3 6
13
6
13
0
2L2 +L4 !L4 6 0
2
4
0 0
0
13
2

1
L !L1
2 1
1
L !L2
13 2

2
L !L3
13 3
1
L !L4
13 4

1 3

6
6
6 0 1
6
6
6
6 0 0
6
4
0 0
2

1 3

6
6
6 0 1
6
6
!
L4 +L1 !L1 6
6
2L4 +L3 !L3 6 0 0
4
0 0
2
1 3
6
6
6 0 1
6
6
!
L3 +L2 !L2 6
6 0 0
7
L +L1 !L1 6
2 3
4
0 0
2

6
6
6
6
Logo 6
6
6
6
4

5
13
2
13
2
13
8
13

2
13
7
13
6
13
2
13

7
2

j
j
0 j
j
2 j
j
1 j
1

1
1
0
7
2

0 j
j
1 0 j
j
1 0 j
j
0 1 j
0 0 j
j
0 0 j
j
1 0 j
j
0 1 j
2
13
6
13
7
13
2
13

1 0
0 0

2
1
2

1
13

2
13

2
13
2
13

2
13

2
13

9
26

1
13

1
13

2
13

1
13

2
13

2
13

2
13

1
13

2
13

1
13

2
13

2
13

2
13

2
13

1
13

2
13

2
13

2
13

1
13

2
13
5
13

1
13

2
13

6
13

3
7
7
7
7
7
7
7
7
5

3
7
7
7
7
7
7
7
7
5
2

1
L !L1
2 1
1
L !L2
13 2

2
L !L3
13 3
1
L !L4
13 4

L4 +L1 !L1
2L4 +L3 !L3

3
7
7
7
7
7
7
7
7
5

L3 +L2 !L2
7
L +L1 !L1
2 3

2
6
6
6
6
6
6
6
6
4

3L2 +L1 !L1

B 6
B 6
B 6 2
B1 6
6
=B
B 13 6
B 6 2
B 6
@ 4
8
178

L2 +L3 !L3
2L2 +L4 !L4

1
13

8
13

7
7
0 7
7
7
7
0 7
7
5

3
13

5
13

2
13

1
13

7
13

8
13

7
7
1 0 7
7
7
7
5
0 7
2
7
5
4 1
3
1 0
7
7
1 0 7
7
7
7
3
0 7
2
7
5
2 1

0 1

2
7
6
2

2
6
7
2

1 0 0 0 j
j
0 1 0 0 j
j
0 0 1 0 j
j
0 0 0 1 j
8

31

7C
7C
C
2 7
7C
7C
7C
C
2 7
7C
5A
5

1
13

2
13

2
13

2
13

1
13

2
13

2
13

2
13

2
13

1
13

2
13

2
13

2
13

1
13

7
7
7
7
7.
7
7
7
5

6
6
6
6
= 13 6
6
6
6
4

1
13

2
13

2
13

2
13

1
13

2
13

2
13

2
13

2
13

1
13

2
13

2
13

2
13

1
13

6
6
6
6
(xiv) 6
6
6
6
4

2
6
6
6
6
6
6
6
6
4

1
2

1
2

1
2

1
2

1
2

1
2

1
2

1
2

j
j
1 j
j
1 j
j
2 j
1

1 2 2 0

7 6
7
7 6
7
7 6 2 1 2 2 7
7 6
7
7=6
7.
7 6
7
7 6 2 2 1 2 7
7 6
7
5 4
5
0 2 2 1

1
2

1
2

1
2

7
7
7
7
7=
7
7
7
5

6
6
6
6
1 6
2 6
6
6
4

1 0 0 0

7
7
0 1 0 0 7
7
7 !
7 L1 $L4
0 0 1 0 7
7
5
0 0 0 1
1

j
j
1 j
j
1 j
j
3 j

6
6
6 0 1
1
6
6
!
L1 +L2 !L2 6
6
1 1
L1 +L3 !L3 6 0
2L1 +L4 !L4 4
0 1
1
2
1
1
1 2
6
6
6 0 1
1 1
6
6
!
L2 +L3 !L3 6
6
0
0
2
L2 +L4 !L4 6 0
4
0 0
2
4
2
1
1
1 2 j 0
6
j
6
6 0 1
1
1
j 0
6
6
j
!
L3 $L4 6
6 0 0
2
4 j 1
6
4
j
0 0
0
2 j 0

j
j
j
j
j
j
j

0 0 0
0 1 0
0 0 1
1 0 0
0

1 0

1 0
1

2
6
6
6
6
6
6
6
6
4
1

7
7
1 7
7
7.
7
1 7
7
5
2
1

7
7
1 7
7
7
7 L2 +L!
3 !L3
1 7
7 L2 +L4 !L4
5
2
3
1
7
7
1 7
7
7 !
7 L3 $L4
2 7
7
5
3
3

7
7
1 7
7
7
7
3 7
7
5
2
179

2L4 +L3 !L3


1
L +L2 !L2
2 4
L4 +L1 !L1

j
j
1 j
j
1 j
j
1 j
2

0 0 0 1

7
7
0 1 0 0 7
7
7
7
0 0 1 0 7
7
5
1 0 0 0

L1 +L2 !L2
L1 +L3 !L3
2L1 +L4 !L4

2
!

2L4 +L3 !L3


1
L +L2 !L2
2 4
L4 +L1 !L1

L2 +L1 !L1

6
6
6
6
Logo 6
6
6
6
4
2

6
6
6
6
= 26
6
6
6
4

6
6
6
6
6
6
6
6
4

1
0
0

1
0
0

0
1

1
1

1
2

1
2

1
2

1
2

1
2

1
2

1
2

1
2

1
2

1
2

1
2

1
2

1
2

1
2

1
2

1
2

1
2

1
2

1
2

1
2

1
2

1
2

7
7
7
7
7=
7
7
7
5

1
2

3
7
7
7
7
7
7
7
7
5

1
L !L3
2 3
1
L !L4
2 4

7
7
0 7
7
7
!
7
1 7 L3 +L2 !L2
2 7 L3 +L1 !L1
5
1

1
2

1
2

1
2

1
2

1
2

1
2

1
2

1
3

7
7
7
7
7
7 L2 +L!
1 !L1
7
7
5

7
7
7
7
7.
7
7
7
5

0 2

B 6
B 6
B 6
B1 6
6
=B
B2 6
B 6
B 6
@ 4

2 1 1 0

7
7
0 7
7
7
7
1 7
7
5
2

1
2

1
2

1
2

1
2

1 0 0 0 j 1 21 12 0
j
0 1 0 0 j 12 1 21 21
j
0 0 1 0 j 12 21 1 12
j
0 0 0 1 j 0 12 12 1

1
2

1 0 0 j 21
j
1 0 0 j 12
j
0 1 0 j 12
j
0 0 1 j 0

1
2

1 0 j
j
1 0 j
j
2 0 j
j
0 2 j

1 0 j 0
j
1 0 j 0
j
1 0 j 12
j
0 1 j 0

6
6
6 0
6
6
6
6 0
6
4
0
2

6
6
6 0
6
6
6
6 0
6
4
0

6
6
6 0
6
6
!
6
1
L
!L
3 6
2 3
6 0
1
L !L4 4
2 4
0

L3 +L2 !L2
L3 +L1 !L1

2
1
1
1

6
7
6
7
6 1 2 1 1 7
6
7
6
7.
6
7
6 1 1 2 1 7
6
7
4
5
0 1 1 2
180

1
2
0
1

1
0
2
1

31

7C
7C
C
1 7
7C
7C
7C
C
1 7
7C
5A
2

25. (i) Seja A 2 Mn

(R) tal que


Ak = 0

para algum k 2 Nn f1g.


(I

A) I + A + ::: + Ak

ou seja, I

= I + A + ::: + Ak

A2

:::

Ak

Ak = I

Ak = I

A invertvel e
(I

(ii)

A)

= I + A + ::: + Ak 1 .

3 1 0
1 1 0
4 0 1 1 5 = @I
0 0 1
2
3 2
0
1 0
0
1 5+4 0
I +4 0 0
0 0
0
0
2

0
uma vez que 4 0
0
2

2 2
4
26. A = 5 1
1 5
2

2 2
(i) A3 = 4 5 1
1 5
(ii) Por (i): (I

1
0
0

0
4 0
0

1
0
0

31 1
1 0
0
1 5A =
(i)
0
0
32 2
3
0
1
1 1
1 5 =4 0 1
1 5
0
0 0
1

33 2
3
0
0 0 0
1 5 = 4 0 0 0 5.
0
0 0 0

3
2
3 5:
3

33 2
3
2
0 0 0
3 5 =4 0 0 0 5
3
0 0 0
A) (I + A + A2 ) = I

27. a) Usando o mtodo de eliminao de Gauss, tem-se


2

3
1 4 2 j 10
4 2 7 2 j 20 5
1 5
j 10

2L1 +L2 !L2


L1 +L3 !L3

1
4 0
0

4
1
1

2
2

3
2
j 10
1
5
4
j 0
0
!
L2 +L3 !L3
2 j 0
0

4
1
0

2
2

3
j 10
j 0 5:
4 j 0

Se 6= 4 ento o sistema possvel e determinado, existindo uma nica soluo. Se


= 4 ento o sistema possvel e indeterminado, existindo um no innito de solues.
181

b) Para

= 4, tem-se o sistema de equaes lineares


x + 4y + 2z = 10
,
y 2z = 0.

x = 10 + 6z
y = 2z.

Colocando z = s, a soluo geral do sistema dada por: S = f(10 + 6s; 2s; s) : s 2 Rg.

28. A

1 0
6 1
=6
4 0 1
1

L1 +L3 !L3
L1 +L4 !L4

L3 +L4 !L4

Se

6=

1 0
6 0 1
6
4 0
0
2

1
6 0
6
4 0
0

1e

Se ( =
Se

0
1
0
0

1 0
6 0 1
7
7 ! 6
5 L2 $L3 4 1
1

1
2

+
2
+
1

0
+1
+1
0

7
7
5

L2 +L3 !L3
L2 +L4 !L4

+
2
+

1
6 0
6
4 0
0

+
0
1
0
0

3
7
7
5
0

L1 +L3 !L3
L1 +L4 !L4

7
7
+1 0 5
+1

L3 +L4 !L4

7
7.
+1 0 5
0

= 0 ento car A = 2 e nul A = 2.

1e
1e

1
2
+

6= 0) ou ( 6=

1e

= 0) ento car A = 3 e nul A = 1.

6= 0 ento car A = 4 e nul A = 0.

Assim, A ; invertvel se e s se
car A ; = no de colunas de A ; .

6=

1 e

6= 0, uma vez que s neste caso que

29. (a) Tem-se


2

1 0
6 2
A =6
4 4 0
0

2
3
2

3
2
4 7
7
8 5
2

2L1 +L2 !L2


4L1 +L3 !L3
L1 +L4 !L4

1 0
6 0
6
4 0 0 (2
0 0

(
2)
) (2 + )
0

2
0
0
(

2)

7
7:
5

Logo, como car A + nul A = 4,


se = 0 ento car A = 1 e nul A = 3;
se = 2 ento car A = 2 e nul A = 2;
se = 2 ento car A = 3 e nul A = 1;
se 6= 0 e 6= 2 e 6= 2 ento car A = 4 e nul A = 0.
Assim, A invertvel se e s se 2 Rn f 2; 0; 2g, uma vez que s nestes casos que
car A = no de colunas de A .
182

(b)
2
1
6 2
=6
4 4
1

A1 j I
0
1
0
0

2L4 +L1 !L1


1
L +L1 !L1
3 3
1
L +L2 !L2
3 3

1
1
1
1
2

1
6 0
6
4 0
0

2
4
8
1
0
1
0
0

0
0
3
0

j
j
j
j

1
0
0
0
0
0
0
1

0
1
0
0

3
0
0 7
7
0 5
1

0
0
1
0
7
3
2
3

j
j
j
j

0
1
4 0
1 0

1
3

(A1 )

30. (a) Ba;b


2

2 2
6 3 0
! 6
L2 $L4 4 0 0
0 0

3
2
0 7
7
0 5
1

1
3

1
0

Logo

2L1 +L2 !L2


4L1 +L3 !L3
L1 +L4 !L4

6
=6
4

4
3

3
2
1
6
a 7
7 ! 6
b 5 L1 $L2 4
0
2
2
0 a
6 0
6 0 7
6
7
!
a b 5 23 L1 +L2 !L2 4 0
L3 +L4 !L4
0
a 1

0
6 2
=6
4 0
3

0
2
0
0
3

7
3
2
3

a
0
a
6

2
0
0
3

1
6 0
6
4 0
0

0
1
0
0

1
1
3
0

1
6 0
! 6
L4 !L4 4 0
1
L !L3
0
3 3

0
1
0
0

1
3
1
3

1
3

2
0
0
0

2
3
0
0

0
a
a
6
0
6
a
0

0
1
0
0

2
0
0
1
0
0
1
0

j
j
j
j

1
2
4
1

0
0
0
1

j
j
j
j

0
1
0
1

j
j
j
j

0
1
0
0

4
3

7
3
2
3

0
0
1
0
0
1
0
0

3
0
0 7
7
0 5
1
1
3
1
3

1
3

2L4 +L1 !L1


1
L +L1 !L1
3 3
1
L +L2 !L2
3 3

3
2
0 7
7
0 5
1

3
2
0 7
7:
0 5
1

3
a
1 7
7 !
b 5 L2 $L4
0
3
a
3
a 7
2
7.
b 5
1 b

Se a = 0 ou ( a 6= 0 e b = 1) ento car Ba;b = 3 e nul Ba;b = 1.


Se a 6= 0 e b 6= 1 ento car Ba;b = 4 e nul Ba;b = 0.
2

0
6 2
(b) [B1;0 j I] = 6
4 0
3
2
3 0
6 0 2
6
!
4 0 0
2
L
+L
!L
1
2
2
3
L3 +L4 !L4
0 0
2
3 0
6 0 2
6
!
L4 +L2 !L2 4 0 0
6L3 +L1 !L1
0 0

0
2
0
0

1
0
1
6

j
j
j
j

1
1
0
0

1
0
0
0

6
4
1
0

0
1
0
1

j
j
j
j

0
0
0
1

0
4
1
0

0
0
0
1

j
j
j
j

0
1
0
1

0
1
0
0
0
1
0
0
0
1
0
0

3
2
0
3 0
7
6
0 7
2 2
! 6
0 5 L1 $L4 4 0 0
1
0 0
3
0
1
2 7
0
3 7
!
1
0 5 L4 +L2 !L2
6L3 +L1 !L1
1 0
3
6 1
2 7
1
3 7
!
1
0 5 12 L2 !L2
1
1 0
L !L1
3 1
0
0
1
0

183

6
0
1
1

0
0
0
1

0
1
0
0

0
0
1
0

3
1
0 7
7
0 5
0

2
L +L2 !L2
3 1

L3 +L4 !L4

1
6 0
! 6
4 0
1
L !L2
2 2
1
0
L !L1
3 1

0
1
0
0

Logo (B1;0 )

0
2
1
0
2

6
=6
4

0
0
0
1
0
1
2

0
1

j
j
j
j

1
2

0
1

1
2

5
2

0
0

1
1

1
2

1
2

0
0

1
1
3

1
3

1
3

7
7
!
0 5 2L3 +L2 !L2
0
1
3

1
6 0
6
4 0
0

0
1
0
0

0
0
1
0

0
0
0
1

j
j
j
j

1
2

0
1

1
2

5
2

0
0

1
1

7
7.
0 5
0
1
3

(c) Como B1;0 invertvel,

B1;0 X = C , X = (B1;0 )

6 1 1
2
2
C,X=6
4 0 0
1 0

2
5
2

1
1

3 6
6
1
6
1 76
7
2 7 6
3 76
=6
0 54 3 5 6
6
6
0
1
4
32

1
3

19
3
19
3

3
2

7
7
7
7
7:
7
7
7
5

(d) Seja X = (x1 ; x2 ; x3 ; x4 ).


2

0
6 2
Ba;1 X = D , 6
4 0
3

0
2
0
0

a
0
a
6

A soluo geral de Ba;1 X = D dada por:

32
1
x1
6 x2
a 7
76
1 5 4 x3
0
x4

3
a
7 6 0 7
7=6
7
5 4 a 5.
6

(Soluo particular de Ba;1 X = D) + (Soluo geral de Ba;1 X = 0).


O vector (0; 0; 1; 0) uma soluo particular de Ba;1 X = D. Determinemos a soluo
geral de Ba;1 X = 0.
2
3
2
3
0 0 a 1
0 0 a
1
6 2 2 0 a 7
6 2 2 0
a 7
7
6
7 !
Tem-se 6
!
4 0 0 a 1 5 L1 +L3 !L3 4 0 0 0
0 5 L1 $L2
3
3
L +L4 !L4
3 0 6 0
0
3 6
a L3 $L4
2 2
2
2
3
2
3
2 2 0
a
2 2 0
a
3
6 0 0 a
6 0
1 7
3 6
a 7
2
6
7
7.
! 6
!
3
4
5
4
5
0
3 6
a
0
0
a
1
L1 $L2
L
$L
2
3
2
L3 $L4
0 0 0
0
0 0 0
0
8
8
x = 2x3
>
>
<
< 2x1 + 2x2 + ax4 = 0
a2
3x2 + 6x3 32 ax4 = 0 ,
Logo,
x2 = 2 +
x3
>
:
2
>
ax3 + x4 = 0
:
x4 = ax3
184

1
3

7
7.
0 5
0
1
3

Assim, a soluo geral de Ba;1 X = 0 dada por:


( 2s; 2 +

a2
2

s; s; as) : s 2 R

Logo, a soluo geral do sistema linear Ba;1 X = D dada por:


f(0; 0; 1; 0)g+

2s; 2 +

a2
2

Resoluo Alternativa.
2
0 0 a 1
6 2 2 0 a
[Ba;1 j D] = 6
4 0 0 a 1
3 0 6 0
2

2
6 0
! 6
L1 $L2 4 0
L3 $L4
0

2
0
3
0

0
a
6
0

j
j
3
a j
2
0
j

a
1

j
j
j
j

s; s; as

3
a
0 7
7
a 5
6

:s2R

L1 +L3 !L3
3
L +L4 !L4
2 2

3
0
a 7
7 !
6 5 L2 $L3
0

8
>
< 2x + 2y + aw = 0
3
Tem-se ento
3y + 6z
aw =
>
2
:
az + w = a

2
6 0
6
4 0
0

0
6 2
6
4 0
0
2
3
0
0

2s; 2 +

0
2
0
3
0
6
a
0

a
0
0
6

j
3
a j
2
1
j
0
j

a2
2

j
j
j
3
a j
2

1
a
0

3
0
6 7
7.
a 5
0

s; s

1; as

:s2R .

3
a
0 7
7 !
0 5 L1 $L2
6 L3 $L4

8
>
< x = 2z 2
2
y = a2 + 2 (z + 1)
6 ,
>
:
w = a az

Logo, a soluo geral do sistema linear Ba;1 X = D dada por:


2s

2;

a2
+ 2 (s + 1) ; s; a
2

as

:s2R

185

a2
2s; 2 +
2

s; s

1; as

:s2R :

Resoluo da 1a Ficha de exerccios facultativos


1. (i) Sejam A = (aij )m

(AB) C =

n
P

p
P

n
P

aik

k=1

p
P

bkl clj

aik bkl

clj

n
P

(ii) Sejam A = (aij )m

n
P

aik bkj

n,

bkl clj

B = (bij )n

aik (bkj + ckj )


n
P

(AB) =

aik bkl clj

k=1 l=1

= A (BC)

e C = (cij )n p .
n
P

aik bkj + aik ckj

aik ckj

= AB + AC

k=1

(iii) Sejam A = (aij )m


T

p
n P
P

k=1

k=1

aik bkl clj

l=1

k=1

p P
n
P

=
p
P

aik

k=1

n
P

e C = (cij )p q .

l=1 k=1

l=1

A (B + C) =

B = (bij )n

k=1

l=1

n,

n
P

aik bkj

k=1

e B = (bij )n p .
n
P

bkj aik

n
P

k=1

2. Seja A 2 Mm
Como

bki ajk

= B T AT .

k=1

(R) tal que AT A = 0. Ento AT A


!
n
X
(aik )2
AT A (i;i) =

(i;i)

= 0, para todo o i = 1; :::; n.

k=1

tem-se ai1 = ::: = ain = 0, para todo o i = 1; :::; n. Logo A = 0.

3. Sendo A =
(A

1 2
, tem-se
2 4
5I) u = 0 ,

4
2

2
1

u= 0 , u 2 f(s; 2s) : s 2 Rg .

Logo
fu 6= 0 : Au = 5ug = f(s; 2s) : s 2 Rn f0gg :

4. Para todo o n 2 N, (prova-se por induo)


186

(i)

(ii)

0
1

1
0

1 0
2 1

8
>
k+1
>
< ( 1)
>
>
:

0
1

( 1)k I,

1
0

se n = 2k

se n = 2k, k = 1; 2; 3:::.

1 0
.
2n 1

3n 2
1 1 0
1 n 1+
(iii) 4 0 1 1 5 = 4 0 1
0 0 1
0 0

+n
n
1

Note que

1+

+ (n

= 1 + (n
|

1) +

= (n

0
6 0
(iv) 6
4 0
0
2

0
6 0
6
4 0
0
(v)

1
0
0
0

1
0
0
0
0
1
0
0

32 2
0
0
7
6
0 7
0
=6
4 0
1 5
0
0

cos
sen

1
0
0
0

sen
cos

+ (n

{z

0
0
0
0

3
0
0 7
7,
1 5
0

1 n
5=4 0 1
0 0
1) +

+1=

1) =

n (n 1)
.
2

3
0
0 7
7, para todo o natural n
0 5
0
0
6 0
6
4 0
0

cos(n )
sen(n )

5.

n
1

1) + 1 = n
| + {z + n}
}

+ (n

n(n 1)
2

n 1 parcelas

1) n , 1 +

0
1
0
0

1) + (n

n 1 parcelas

3n 2
0
0 0
7
6
0 7
0 0
=6
5
4
1
0 0
0
0 0

0
1
0
0

1, k = 1; 2; 3:::

1
0
0
0

0
1
0
0

sen(n )
cos(n )

33 2
0
0
7
6
0 7
0
=6
4 0
1 5
0
0
; (com

0
0
0
0

4.

0
0
0
0

3
1
0 7
7.
0 5
0

2 R).

5.
A2 = (AB) (AB) = A (BA) B = ABB = AB = A:
B 2 = (BA) (BA) = B (AB) A = BAA = BA = B:

6. Seja A =

a b
c d

uma matriz 2

2 ortogonal, isto , tal que AAT = AT A = I.

Tem-se
T

AA = I ,

a b
c d

a b
c d

=I,
187

a2 + b2 ac + bd
ac + bd c2 + d2

1 0
0 1

e
AT A = I ,

8 2
< a + b2 = 1
ac + bd = 0
,
: 2
c + d2 = 1

a b
c d

a b
c d

a2 + c2 ab + cd
ab + cd b2 + d2

=I,

1 0
0 1

8 2
< a + c2 = 1
ab + cd = 0
,
: 2
b + d2 = 1.

Logo c = b.
Se c = b tem-se b (a + d) = 0 , (b = 0 ou a = d).
Se c = b tem-se b (a d) = 0 , (b = 0 ou a = d).
Deste modo
a b
A=
ou A =
b
a
Por outro lado, a2 + b2 = 1 , (a = cos
A=

cos
sen

sen
cos

7. (i) A e B do tipo n
(ii) A do tipo m

n e B do tipo n

e b = sen , para algum

ou A =

n; (A + B)(A

a b
b a

cos
sen

sen
cos

B) = A2 + BA

2 R). Logo

( 2 R):

AB

B2:

m, (AB)2 = ABAB:

n; (A + B)2 = A2 + BA + AB + B 2 :

(iii) A e B do tipo n

8. (i) Falsa:
1 0
2 0

0 0
3 4

=0

mas A 6= 0 e B 6= 0.

No caso de A ser invertvel, a seguinte condio verdadeira:


AB = 0 ) (A = 0 ou B = 0)
uma vez que AB = 0

A invertvel

B = 0.

(ii) Falsa:
1 2
1 2

0 0
1 1

1 2
1 2

2 2
0 0

mas

0 0
1 1

No caso de A ser invertvel, a seguinte condio verdadeira:


AB = AC ) B = C
188

6=

2 2
0 0

uma vez que


AB = AC

A invertvel

(AB) = A

(AC) ,

, A 1 A B = A 1 A C , IB = IC , B = C:

9. (i) B T AB simtrica:
(B T AB)T = B T AT (B T )T = B T AB;
pois A = AT (A simtrica) e (B T )T = B:
(ii) Se A normal (isto AH A = AAH ) e B unitria (isto B H B = BB H = I) ento
BAB H

BAB H

= BAB H B H

AH B H = BA B H B AH B H =

= BAAH B H = BAH AB H = BAH B H BAB H = BAB H

BAB H

ou seja, BAB H normal.


(iii) Como
BT B

= BT BT

= BT B

BB T

BB H

= BT

B T = BB T

as matrizes B T B e BB T so simtricas.
Alm disso, como
BH B

= BH BH

= BH B

= BH

B H = BB H

B H B e BB H so matrizes hermitianas.

10. (i) Seja A = (aij ) do tipo n


diagonais principais tem-se:

n tal que AT =
aii =

A: Assim, em relao s respectivas

aii

e logo aii = 0; para todo o i 2 N:


(ii) Seja A = (aij ) do tipo n
(A

n. A matriz A
AT )T = AT

A=

AT anti-simtrica pois:
(A

AT ):

(iii) Escrevendo A = 12 (A + AT ) + 12 (A AT ), a matriz A pode ser decomposta pela


soma de uma matriz simtrica com uma anti-simtrica. Esta decomposio nica: Sejam
A1 simtrica e A2 anti-simtrica tais que A = A1 + A2 : Logo,
AT = (A1 + A2 )T = A1
189

A2 :

Pelo que A + AT = 2A1 e A

AT = 2A2 : Assim,

1
1
A1 = (A + AT ) e A2 = (A
2
2

11. Seja X =

a b
c d

a b
c d

a b
c d

Logo,

X2 = I ,

Se b = 0, ento a =

1ed=

1ed=

a2 + bc ab + bd
ac + cd bc + d2

8 2
a + bc = 1
>
>
>
>
>
>
>
>
< ab + bd = 0

>
>
ac + cd = 0
>
>
>
>
>
>
:
bc + d2 = 1.

1 e (b = 0 ou a =

dec=

d). Logo,
1
c

I ou X =

X = I ou X =
Se b 6= 0 e c 6= 0 ento a =

1 e (c = 0 ou a =

X = I ou X =
Se c = 0 ento a =

2 tal que X 2 = I.

uma matriz do tipo 2

X2 =

X=

0
1

b
1

d). Logo,
1
0

I ou X =

1 a2
.
b

AT ):

Logo,
a
1 a2
b

b
a

Logo, todas as matrizes X que satisfazem X 2 = I so:


I;

1
c

0
1

1
0

b
1

a
1 a2
b

b
a

Observe assim que a equao matricial X 2 = I tem um nmero innito de solues em


contraste com a equao escalar x2 = 1 que tem apenas duas solues (1 e
1).

12. Seja X =

x11 x12
x21 x22

tal que
XA = AX;
190

a b
2 M2 2 (R). Tem-se ento:
c d
8
x11 a + x12 c = ax11 + bx21
>
8
>
>
>
x12 c = bx21
>
>
>
>
>
>
>
>
<
< x21 a + x22 c = cx11 + dx21
x21 (a d) = c(x22 x11 )
,
XA = AX ,
>
>
>
>
x
b
+
x
d
=
ax
+
bx
>
>
11
12
12
22
>
>
:
>
>
(x11 x22 )b = (d a)x12 .
>
>
:
x21 b + x22 d = cx12 + dx22 .

para todo o A =

Se a = 1 e b = c = d = 0,
Se b = 1 e a = c = d = 0,
Se c = 1 e a = b = d = 0,
Se d = 1 e a = b = c = 0,
Logo, a matriz X tal que

ento x21 = x12 = 0.


ento x21 = 0 e x11 = x22 .
ento x12 = 0 e x11 = x22 .
ento x21 = x12 = 0.
XA = AX; para todo o A 2 M2 2 (R), dada por:
X=

0
0

13. Sendo A uma matriz do tipo m

, com

2 R.

n, seja N (A) = fX : AX = 0g.

(i) Sendo A e B matrizes de tipos apropriados, seja u 2 N (B). Logo Bu = 0, pelo que
(AB) u = A (Bu) = A0 = 0
e assim u 2 N (AB). Deste modo, tem-se
N (B)
(ii) Atendendo alnea anterior N (A)
Vejamos que N AT A

N (AB) .
N AT A .

3
v1
6
7
N (A). Seja u 2 N AT A e Au = 4 ... 5 2 Rn . Logo
vn
AT Au = AT A u = 0

e assim
2

3
v
1
n
X
6
7
uT AT Au = 0 , (Au)T (Au) = 0 ,
(vi )2 = 0 , Au = 4 ... 5 = 0
i=1
vn

pelo que u 2 N (A).


Logo

N (A) = N AT A .
191

n com m < n tais que AB T invertvel. Como

(iii) Sejam A e B matrizes do tipo m

N BT A

f0g =
6 N (A)
m<n

ento B T A no invertvel.
Se alguma linha de B pertencesse a N (A) isso seria equivalente a alguma coluna de B T
pertencer a N (A), ou seja, matriz AB T ter alguma coluna nula, o que contradiria o facto
de AB T ser invertvel.
(iv) Seja A 2 Mm n (R) tal que para todo o B 2 Rm o sistema AX = B possvel,
ento car A = m. Assim AAT (do tipo m m) invertvel e por (ii)
N AT = N AAT = f0g .

14. Seja A = (aij )n

2 Mn

n (R)

tal que Au = 0 para qualquer u 2 Mn 1 (R).

Para cada j 2 f1; :::; ng xo, seja ej = (

ij )n 1

2 Mn 1 (R) em que

ij

1 se i = j
0 se i 6= j.

Como
Aej = 0
para todo o j 2 f1; :::; ng e por outro lado

3
a1j
6 7
Aej = 4 ... 5
anj
2 3 2 3
a1j
0
6 .. 7 6 .. 7
para todo o j 2 f1; :::; ng, ento 4 . 5 = 4 . 5 para todo o j 2 f1; :::; ng pelo que A = 0.
anj
0

15. Sendo A; B 2 Mn

e assim

6
6
AB T = 6
4

a1
a2
..
.
an

(R)
2

6
6
A=6
4

7
7
7 b1 b2
5

a1
a2
..
.
an

7
7
7
5

6
6
B=6
4

bn

192

6
6
=6
4

b1
b2
..
.
bn

3
7
7
7
5

a1 b 1 a1 b 2
a2 b1 a2 b2
..
..
.
.
an b1 an b2

...

a1 b n
a2 b n
..
.
an b n

7
7
7:
5

Como A e B so matrizes no nulas, existe i 2 f1; :::; ng


f1; :::; ng tal que bj 6= 0, tendo-se
2
a1 b 1
a1 b j
a1 b n
.
.
..
.
6 .
..
..
.
6 .
6
T
ai b j
ai b n
AB = 6 ai b1
6 .
.
..
.
..
..
4 ..
.
an b 1
an b j
an b n
Aplicando sucessivamente a operao elementar

tal que ai 6= 0 e existe j 2


3

7
7
7
7.
7
5

ak
Li + L k ! L k
ai
para todo o k = 1; :::; n com k
2
0
0
..
...
6 ..
.
6 .
6
0
6 0
6
ai b j
6 ai b1
6
0
6 0
6 .
..
.
4 ..
..
.
0
0
com ai bj 6= 0, isto ,

6= i, tem-se
3
0
.. 7
...
. 7
7
0 7
7
ai b n 7 !
7 L $L
0 7 i 1
.. 7
...
. 5
0

2
6
6
6
4

ai b 1
0
..
.
0

ai b j
0
..
..
.
.
0

ai b n
0
..
..
.
.
0

3
7
7
7
5

car(AB T ) = 1:

16. Seja A uma matriz do tipo m n tal que car A = m. Ento existe uma matriz
invertvel R, m m, produto de matrizes elementares (por aplicao do mtodo de eliminao
de Gauss) tal que
RA = I B
onde I a matriz identidade r r e B uma matriz do tipo r (n r). Aplicando o
mtodo de eliminao de Gauss agora s colunas de RA, existe uma matriz invertvel Q do
tipo n n; (produto de matrizes elementares), tal que
RAQ =

I 0

onde 0 a matriz nula. Sendo Q1 a matriz do tipo n


apenas as primeiras m colunas, tem-se
RAQ1 = I.
Pelo que AQ1 invertvel tendo-se
AQ1 R = I.
Logo existe B = Q1 R do tipo n

m tal que AB = I.
193

m que se obtm de Q considerando

17. Sejam A e B matrizes do tipo n

n.

(i) Se A fr invertvel ento


1

AB = ABI = AB AA

= A (BA) A

isto , existe S = A invertvel tal que


1

AB = S (BA) S

ou seja, AB e BA so semelhantes. Analogamente, se B fr invertvel ento AB e BA so


semelhantes.
(ii) Sendo A e B semelhantes existe S invertvel tal que
A = SBS

ou seja
S

A = BS

Ento
X 2 N (A) , AX = 0 , S

18. Seja A do tipo n

AX = 0 , BS

X=0,S

X 2 N (B) :

n.

(i) Se A fr invertvel tem-se A 1 A = AA

= I: Logo A

invertvel e (A 1 )

= A:

(ii) Se A fr invertvel tem-se A 1 A = AA 1 = I: Logo (A 1 A)T = (AA 1 )T = I T : Pelo


que
AT (A 1 )T = (A 1 )T AT = I:
Isto , AT invertvel e (AT )

= (A 1 )T

(iii) Se A fr invertvel e simtrica tem-se A 1 A = AA


(A 1 A)T = (AA 1 )T = I T ; e assim

= I

e A = AT : Logo

AT (A 1 )T = (A 1 )T AT = I:
Pelo que, como A simtrica, tem-se A(A 1 )T = I: Logo, como A invertvel, tem-se
(A 1 )T = A 1 : Isto , A 1 simtrica.

19. (i) Por exemplo I e

I so invertveis no entanto I + ( I) = 0 no invertvel.

(ii) Como A e B so invertveis por hiptese, tem-se as seguintes igualdades


B 1 (A + B) = I + B 1 A e A 1 (A + B) = A 1 B + I,
194

que so respectivamente equivalentes a


B 1 (A + B) = (A

+ B 1 )A e A 1 (A + B) = (A

+ B 1 )B.

Como por hiptese A + B invertvel tem-se


I = (A

+ B 1 )A(A + B) 1 B

e I = (A

+ B 1 ) 1 B(A + B) 1 A.

Analogamente e partindo de:


(A + B)B

= I + AB

e (A + B)A

= AB

+ I,

obtm-se
I = B(A + B) 1 A(A
1

Deste modo A

+B

+ B 1 ) e I = A(A + B) 1 B(A

+ B 1 ).

invertvel e

(A

20. Seja A 2 Mn

+ B 1)

= A(A + B) 1 B = B(A + B) 1 A.

(R) tal que A2 = A (a matriz A diz-se neste caso idempotente).

(i) Como
(I
ento I

A)2 = (I

A) (I

2A + A2 = I

A) = I

2A + A = I

A idempotente.

(ii) Como
(2A
ento 2A

I) (2A

I) = 4A2

4A + I = 4A

4A + I = I

I invertvel e
(2A

I)

= 2A

I:

(iii) Se car A = n ento A invertvel pelo que A2 = A , A


21.
1
(I + A)
2

1
(I + A) ,
2
1
1
1
, I + A2 + A +
4
4
4

22. Sendo A = (aij )n


para k 6= 0:
k i j aij

k i j bij =

(A2 ) = A 1 A , A = I.

1
1
I + A2 + A + A = (I + A) ,
4
2
1
1
1
A = I + A , A2 = I.
4
2
2

uma matriz invertvel e B = (bij )n


n
X

k i l ail k l j blj

l=1

195

ki

n
X
l=1

a inversa da A, tem-se,

ail blj

= I.

Logo a matriz (k i j aij ) invertvel e a sua inversa a matriz (k i j bij ).

a b
c d

23. Seja A =

do tipo 2

2. Suponhamos que a 6= 0, c 6= 0 e ad

tem-se:
a b j 1 0
c d j 0 1

A=

ac
0
j
0 ad bc j

cL1 !L1
aL2 !L2

ac
bc
j
0 ad bc j

!
!

ac bc j c 0
ac ad j 0 a

c 0
c a

acb
ad bc

acd
ad bc

L1 +L2 !L2

bc
L +L1 !L2
ad bc 2

1
L !L1
ac 1
1
L !L2
ad bc 2

b
ad bc
a
ad bc

d
ad bc
c
ad bc

1 0 j
0 1 j

bc 6= 0. Logo,

Logo,
1

d
c

b
a

.
ad bc
Se a = 0 e c = 0, ento A no invertvel.
Se a = 0 e c 6= 0, ento b 6= 0, caso contrrio A no seria invertvel. Neste caso, com
a = 0, c 6= 0, b 6= 0 e ad bc 6= 0 tem-se:
A

A=

!
=

0 b j 1 0
c d j 0 1

L1 !L2

1 dc j 0 1c
0 1 j 1b 0
1 0 j
0 1 j

d
bc
c
bc

c d j 0 1
0 b j 1 0

d
L +L1 !L1
c 2

b
bc

d
ad bc
c
ad bc

1 0 j
0 1 j

1 0 j
0 1 j

1
L !L1
c 1
1
L !L2
b 2

d
bc
1
b

ad

b
c

bc

b
ad bc
a
ad bc

Se a 6= 0 e c = 0 seria anlogo. Logo, A invertvel se e s se ad


A

1
c

=
.
bc 6= 0 e

Nota: O exo foi feito apenas com o recurso ao mtodo de Gauss-Jordan. Poderia ter
sido efectuada outra resoluo atendendo frmula de inverso de matrizes:
1
A 1=
(cof A)T .
jAj
Observe que ad

24. A matriz

a b
c d

bc = jAj, com A =

2
6
6
6
4

do tipo 2

k1

0
..
.

k2
...

3
0
.
..
. .. 7
7
7
...
0 5
0 kn

196

2.

invertvel se s se k1 6= 0; k2 6= 0; : : : ; kn 6= 0, e a
2
3 1 2
k1 0
0
6
. . . .. 7
6
6
. 7
6 0 k2
6
=
6 . .
7
6
.. ... 0 5
4 ..
4
0
0 kn
25. Sejam A = (aij ); B = (bij ) 2 Mn

sua inversa dada por:


3
1
0
0
k1
. 7
..
. .. 7
0 k12
7.
7
.. . .
...
.
.
0 5
0
0 k1n

(R) e

um escalar.

(i)
tr(A + B) = tr ((aij ) + (bij )) = tr(aij + bij ) = a11 + b11 + ::: + ann + bnn =
= a11 + ::: + ann + b11 + ::: + bnn = tr(A) + tr(B).
(ii)
tr( A) = tr( aij ) = a11 + ::: + ann =
(iii)

02

a11
B
6
tr(AT ) = tr @4 ...
a1n

(a11 + ::: + ann ) =

tr(A):

31
an1
.. 7C = a + ::: + a = tr(A):
..
.
11
nn
. 5A
ann

(iv)
02

32
31
a11
an1
b11
bn1
B6
.. 7 6 .. . .
. 7C
..
tr(AB) = tr @4 ...
.
. .. 5A =
. 54 .
a1n
ann
b1n
bnn
02
31
a11 b11 + ::: + an1 b1n
a11 bn1 + ::: + an1 bnn
B6
7C
..
..
..
= tr @4
5A =
.
.
.
a1n b11 + ::: + ann b1n
a1n bn1 + ::: + ann bnn
= a11 b11 + ::: + an1 b1n + ::: + a1n bn1 + ::: + ann bnn =
= b11 a11 + ::: + b1n an1 + ::: + bn1 a1n + ::: + bnn ann =
= b11 a11 + ::: + bn1 a1n + ::: + b1n an1 + ::: + bnn ann =
02
31
b11 a11 + ::: + bn1 a1n
b11 an1 + ::: + bn1 ann
B6
7C
..
..
...
= tr @4
5A = tr(BA).
.
.
b1n a11 + ::: + bnn a1n
b1n an1 + ::: + bnn ann
26. Seja A 2 Mn

(R). No pode existir X 2 Mn


AX

XA = I
197

(R) tal que

uma vez que


tr (AX

XA) = tr (AX)

tr (AX) = 0 6= 1 = tr I

tr (XA) = tr (AX)

27. Sejam A e B matrizes do tipo n

n tais que A simtrica e B anti-simtrica.

tr(AB) = tr (AB)T = tr B T AT = tr ( BA) =

(R). Tem-se
02
32
a11
am1
a11
B6 ..
7
6
.
T
.
..
.. 5 4 ...
tr(A A) = tr @4 .
a1n
amn
am1
02

B6
= tr @4

tr(AB):

tr(AB) , tr(AB) = 0:

tr(AB) =

28. Seja A = (aij ) 2 Mm

tr (BA) =

31
a1n
.. 7C =
...
. 5A
amn

31
a11 a1n + ::: + am1 amn
7C
..
..
5A =
.
.
2
2
a1n + ::: + amn

a211 + ::: + a2m1


..
.

a1n a11 + ::: + amn am1

= a211 + ::: + a2m1 + ::: + a21n + ::: + a2mn .

Logo
tr(AT A) = 0 , A = 0:

29. Sejam u; v 2 Mn

(R) tais que uT v 6=

1. Seja

A = I + uv T .
Tem-se
I

1
uv T
T
1+u v

I + uv T = I + uv T

1
uv T
T
1+u v

1
uv T
T
1+u v

=I+

1 + uT v T
uv
1 + uT v

1
uv T
1 + uT v

1
u vT u vT =
1 + uT v

=I+

1 + uT v T
uv
1 + uT v

1
uv T
T
1+u v

vT u
uv T
T
1+u v

=I+

1 + uT v T
uv
1 + uT v
=I+

1
uv T
1 + uT v

uT v
uv T =
1 + uT v

1 + uT v 1 u T v T
uv = I.
1 + uT v
198

v T u=uT v

uv T =

Logo A invertvel e
A
Sendo u; v 2 Mn

(R)

6
6
u=6
4

tem-se
uT v =

u1 u 2

e
tr uv T
2 2

Logo

6 6
6 6
= 6tr 6
4 4

u1
u2
..
.
un

un

2 02

6 B6
6 B6
= 6tr B6
4 @4

u1 v 1 u1 v 2
u2 v 1 u2 v 2
..
..
.
.
un v 1 un v 2

..

1
uv T .
1 + uT v

=I

3
7
7
7
5

2
6
6
6
4

u1
u2
..
.
un

v1
v2
..
.
vn
3

6
6
v=6
4

un v n

vn

3
7
7
7
5

7
7
7 = [u1 v1 + u2 v2 +
5

7
7
7 v1 v2
5

u1 v n
u2 v n
..
.

v1
v2
..
.

33

13

C7
C7
v n C7 =
A5

77
77
77 = [u1 v1 + u2 v2 +
55

uT v = tr uv T

199

+ un ]

+ un ] .

Resoluo da 2a Ficha de exerccios para as aulas de problemas


1. (i) (312645) par pois tem 4 inverses.
(ii) (234516) par pois tem 4 inverses.
(iii) (654321) mpar pois tem 15 inverses.
(iv) (123456) par pois tem 0 inverses.
(v) (546321) mpar pois tem 13 inverses.
(vi) (453261) par pois tem 10
inverses.
(vii) (634125) mpar pois tem 9 inverses.
(viii) (123465) mpar pois tem 1
inverso.

2. (i) (234516) par pois tem 4 inverses e (312645) par pois tem 4 inverses. Logo,
tem-se
+a23 a31 a42 a56 a14 a65
uma vez que (234516) e (312645) tm a mesma paridade.
(ii) (123456) par pois tem 0 inverses e (654321) mpar pois tem 15 inverses. Logo,
tem-se
a16 a25 a34 a43 a52 a61
uma vez que (123456) e (654321) tm paridades diferentes.
(iii) (546321) mpar pois tem 13 inverses e (453261) par pois tem 10 inverses. Logo,
tem-se
a54 a45 a63 a32 a26 a11
uma vez que (546321) e (453261) tm paridades diferentes.
(iv) (123465) mpar pois tem 1 inverso e (634125) mpar pois tem 9 inverses. Logo,
tem-se
+a16 a23 a34 a41 a62 a55
uma vez que (123465) e (634125) tm a mesma paridade.

3. (i) (123) par pois tem 0 inverses e (321) mpar pois tem 3 inverses. Atendendo
denio de determinante, tem-se
0
0 a13
0 a22 a23
a31 a32 a33

a13 a22 a31

uma vez que (123) e (321) tm paridades diferentes.

200

(ii) (1234) par pois tem 0 inverses e (4321) par pois tem 6 inverses. Atendendo
denio de determinante, tem-se
0
0
0
0
0 a23
0 a32 a33
a41 a42 a43

a14
a24
a34
a44

= a14 a23 a32 a41

uma vez que (1234) e (4321) tm a mesma paridade.


(iii) Ao efectuar n 1 + n 2 + ::: + 2 + 1 trocas de linhas: linha 1 com cada uma das
n 1 linhas que lhe esto abaixo, nova linha 1 com cada uma das n 2 linhas que lhe esto
abaixo (excluindo a nova linha n) e assim sucessivamente, tem-se
2
3
2
3
0
0
a1n
an1
ann
6
7
6
n 1
n 1
an 1 n 7
7 = ( 1)n 2 det 6 0 an 1 2
7 = ( 1)n 2 a1n :::an
det 6
4 0 an 1 2
5
4
5
an 1 n
an1
ann
0
0
a1n
atendendo a que n

1+n

2 + ::: + 2 + 1 = (n

1)

n 1
1+1
=n
.
2
2

4. Uma matriz no singular se e s se invertvel e como tal, ser singular se e s se


fr no invertvel.
(i)

1 2
3 4

=4

2 6= 0, logo a matriz invertvel.

6=

18563 18573
21472 21482
invertvel.
(ii)

p
p3
2

18563 10
21472 10

(iii)

p
1 + p2 2
2+ 3 1

(iv)

cos
sen

(v)

2 0 1
5 3 0
5 1 2

12 + 5

(vi)

2
5
2

2
3
1

sen
cos

3
1
1

=1

= cos2

(4

18563 10
2909 0

3) =

29090 6= 0, logo a matriz

2 6= 0, logo a matriz invertvel.

sen2 ) = 1 6= 0, logo a matriz invertvel.

( 15) = 8 6= 0, logo a matriz invertvel.

2 + 18 + 10

15

201

( 6) = 13 6= 0, logo a matriz invertvel.

1 2 an1

2
5
2

(vii)

1
3
2

8 12
5
1
2 1

(viii)

1 2 3
4 5 6
7 8 9

(ix)

0
1
0
0

(x)

1
1
3

1
0
1
0
2
0
0
0

(xi)

8
3
1

0
0
1
0

2
1
0
0

2
5
2

3
1
1

2
3
1

2
5
2

3
1
1

2
3
1

1 2 3
2 1 0
0 0 0

=4

1 2 3
2 1 0
4 2 0

0
1
0
1

0
1
0
0

8
2
3
0

1
0
0
0

6
0
23
5

0
0
0
1

0
0
1
0

1
0
0
0

por (vi)

por (vi)

13 6= 0, logo a matriz invertvel.

52 6= 0, logo a matriz invertvel.

= 0, logo a matriz no invertvel.

0
1
0
0

0
0
1
0

0
0
0
1

= 1 6= 0, logo a matriz invertvel.

= 30 6= 0, logo a matriz invertvel.

(xii)
1
1
2
2

3
2
1
0
=

1
1
1
2

1
1
1
0

1
0
2
2

2 [ 6 + 1 + ( 2)

3
1
1
0

1
2
1
2

1
2
1
0

( 1)

3
= 2( 1)4+1 1
1

( 2)

1 1
1 3 1
2 2 +( 2)( 1)4+3 0 1 2
1 1
2 1 1

6] + 2 [1 + 12

2] = 20 + 18 = 38 6= 0,

logo a matriz invertvel.

(xiii)

0
0
0
2

0
0
1
2

0
3
2
8

5
23
0
6

42
52
62
72

12 22 32 42
3 5 7 9
=
5 7 9 11
7 9 11 13

2
0
0
0

2
1
0
0

8
2
3
0

6
0
23
5

por (xi)

30 6= 0, logo a matriz

invertvel.
12
22
(xiv) 2
3
42

22
32
42
52

32
42
52
62

= 0, logo a matriz no invertvel.


202

12 22 32 42
3 5 7 9
2 2 2 2
2 2 2 2

12 22 32 42
3 5 7 9
2 2 2 2
0 0 0 0

0 4
0 0
(xv) 0 0
0 0
5 0
logo a matriz
a
b
0
0
0
b.

(xvi)

s se a 6=

0 0 0
0 2 0
0 0 1 = 5( 1)5+1
3 0 0
0 0 0
invertvel.

0
a
b
0
0

7
1
(xvii)
2
2

0
0
a
b
0

0
0
0
a
b

4
2
1
2

b
0
0
0
a

0
0
3
0

= 3 (6 ( 19 + 3
invertvel.

5
2
8
4

6
0
4
3
5

1
2
3
3

4
6
3
5

1
4
1
2

8
9
6
7

2
7
3
0
invertvel.

0
2
3

1
5
12

(xix)

7
0
0
0
5

4
0
0
0
3
6

1
0
5
3
7
6

2 ( 1) ( 3)

8)) =

0
3
0
0
0

3
2
8
9

0
0
0
3

0
2
0
0

0
0
1
0

7
1
2

3 ( 1)

4
2
2

3 (6 ( 19 + 3

4
1
1
1
2

2
2
1
3
3

1
0
3
0
0

2
7
0

6
0
1
p0
3

7 4 6
0 0 p1
3
5 3

5
2
4

8)) =

4
2
9
7
0
2
3

1
6
4
1
1
13
0

2 ( 1)6

8
25
30
17

7
0
0
5

6 ( 1)5

203

4
0
0
3

7 4
5 3

b, logo a matriz invertvel se e

1
3 2
7

9
8
6
7

3 ( 1)4

3
0 2 0
0 0 1 5 = 120 6= 0,
3 0 0

= 5 44( 1)1+1

= a5 + b5 6= 0 se e s se a 6=

9
5
8
6
7

(xviii)

4
0
0
0

30 =

6
4
3
5

3 (21

4
1
1
2

2
7
7

2
2
4

2
4
5

1
3 0
0

2
2
6 8
18 19

90 6= 0, logo a matriz

2
1
3
3

6
2
1

=3

25
5
17

2
1
3
3

6
4
3
5

4
1
1
2

9
8
6
7

2
7
0

6
2
3

25
5
12

78) = 171 6= 0, logo a matriz

1
6
5
1
3 p0
7
3

= 6 6= 0, logo a matriz invertvel.

2 1 0
7 3 0
(xx)
3 1
3 9 0
matriz invertvel.
n
n

0
2
1
0

= ( 1)6

1 n

..
.
..
.

(xxi)

..
2
1

2
1

2 1
2
.
. ..
2
1

1
..
.

1
1
1
1
1
1
.
.
.
.
=
.
.
0 1
n 2 n 2
0 1
n 2 n 1
invertvel.
0
6 e
6
5. (i) Seja A = 6
6 0
4 0
0
det A = 0, isto , A no

det A =

a
0
f
0
0

2 1
3 9

2 ( 1)5

1
1
1
2
2
2
.. . .
.
.
1 2
n 1 n 1
1 2
n 1
n
1

a 0 0 0
0 b 0 0
f 0 c 0
0 g 0 d
0 0 h 0
invertvel.
0
e
0
0
0

0
2
0

1
1
..
.

1
1

1
0
..
.

2 1
7 3
3 9

1
..

0 1
.. . .
.
.
0

1
.
. ..

= 30 6= 0, logo a

= 1 6= 0, logo a matriz

..

. 1
0 1

7
7
7, com a; b; c; d; e; f; g; h 2 R. Se a = 0 ou h = 0 ento
7
5

Se a 6= 0 e h 6= 0 ento

0
b
0
g
0

0
0
c
0
h

0
0
0
d
0

0
e
0
0
0

a
0
0
0
0

0
b
0
g
0

0
0
0
0
h

0
0
0
d
0

= 0,

isto , A no invertvel. Logo, A no invertvel para quaisquer a; b; c; d; e; f; g; h 2 R.


Em alternativa, pelo teorema de Laplace
det A = ( 1)5 f

(ii) Diga, para que


2
0
6 f
6
6 0
det 6
6 0
6
4 0
0

0
e
0
0

0
b
g
0

0
0
0
h

0
0
+ ( 1)7 c
d
0

0
e
0
0

valores de a; b; c; d; e; f; g; h; i; j 2 R,
3
2
a 0 0 0 0
f
0 b 0 0 0 7
7
6 0
6
g 0 c 0 0 7
7 = a ( 1)3 det 6 0
6
0 h 0 d 0 7
7
4 0
5
0 0 i 0 e
0
0 0 0 j 0
204

a
0
0
0

0
b
g
0

0
0
d
0

= 0.

invertvel a seguinte matriz


b
0
h
0
0

0
c
0
i
0

0
0
d
0
j

0
0
0
e
0

7
7
7=
7
5

f
6
0
ac ( 1)5 det 6
4 0
0

b
h
0
0

0
d
0
j

3
2
3
0
f
b
0
0 7
7 = ace ( 1)7 det 4 0 h d 5 =
e 5
0 0 j
0

acef hj,

pelo que a matriz ser invertvel se e s se a; c; e; f; h; j 2 Rn f0g.

6. Determinemos todos os valores do escalar


invertvel, isto , todos os valores prprios de A.
(i) det (A

I) =

= (1 + )
2
1
I) = 0 , ( = 2 ou = 3).

Logo, det (A

1
(ii) det (A
=

0
0
0 , ( = 0 ou = 3).

n)

n 1

Logo, det (A

7.

0
1

1
1
1

6
6
I) = det 6
4

1
..
.

1
1

1
...
..
.

2
1
4

1
2

+ 4 (1 + )

1
..
.
1

= n).

1
.
0

205

3
7
7
7
5

n n

4 (1

3 ou

=
1

3
1
0 5=

6
6
6
= det 6
6
4

3)

. Logo, det (A

1
0
..
.

)=

= 3).

3
1
0 5 = ( 1)3 (

...

I no

6.

5 = det 4

I) = 0 , ( = 0 ou

1
1

3
2
1
3
5
4
0
0
= det
0

(iv) det (A

6=

I) = 0 , ( = 0 ou

I) = det 4

= det 4

( 1)n (

2
2

0
1
2
2
+ 8 . Logo, det (A

(iii) det (A
2

I) =

para os quais a matriz A

...

1
0
...
..

I) =

1
0
..
.

7
7
7
7=
7
0 5

3
1
0 7
7
1 5

6
8. a) A = 6
4 0
1
8
< 2
3
Logo car A =
:
4

1
0

1
0

se
se
se

=1
= 1
6= 1 e

6=

L1 +L2 !L2
L1 +L4 !L4

1
6 0
6
4 0
0

0
2

7
7.
5

1
0
0

1
0

1
1

1.

A 4 4. Logo A invertvel se e s se car A = 4. Assim, A invertvel se e s se


2 Rn f 1; 1g.
b) det (A0 )n + (A0 )n+2 = det (A0 )n I + (A0 )2

= (det A0 )n det I + (A0 )2


| {z }
=1

c) (A )

(3;1)

(1

) (1 + )

9. (i) A

(ii) A

(iii) A

3
6 0
= det 6
4 1
1

1
1

0
0
2
0

3
1
0 7
7 = 20:
1 5
2
1

1+3

)2 (1 + )

( 1)

0
1

1
0

0
1

1
1
=
(cof A)T =
det A
2
2

1
1
14
T
1
=
(cof A) =
det A
1
0

0
2
0
0

1
=
(cof A )(1;3) =
det A
(1

4
2

3
1

2
3=2

3T
2
0 0
1
5
4
1 0
= 0
1 1
0

1
1=2
1
1
0

3
0
1 5
1

3T
2
3
18 0 6
3 4 2=3
1
1
=
(cof A)T = 4 24 0 6 5 = 4 0 0 1=6 5
det A
6
4 1 1
1 1 1=6

10. (i) Tem-se

det A =

3
1
0
3

2
0
9
1

0
0
2
0

2
3
0
2

= ( 2) ( 1)3+3

3
1
3
206

2 2
0 3
1 2

= ( 2) (2 + 18

( 4)

9) =

30.

Logo,
1

(2;2)

1
(cof A)T
det A
1
4
0
0

det B =

0
0
1
1

0
1
0
2

1
(cof A)(2;2) =
det A

=
(2;2)

2
0
3
2

1
0
0
0

0
0
1
0

0
1
0
2

2
8
3
1

3
0
3

1
( 1)2+2
30

0 2
2 0
0 2

0 1
= ( 1)( 1)1+1 1 0
0 2

8
3
1

4
.
5

17.

Logo,
B

1
(2;3)

1
(cof B)T
det B

=
(2;3)

1
(cof B)(3;2) =
det B

4 2
6 3
0
(ii) det (A + B) = det 6
4 0 10
3
2
det B
2
2 2 0 0
6 5 0
1 3
det (A B) = det 6
4 0 8
2 3
3 0
2 0
1
3
2
5

11. (i) x =

3
7
3
7

2
=2
1

3
4
3 7
7=
3 5
4

0
1
2
2
3

7
7 = 138 6=
5

y=

2
5
2
5

134 6=

13 =

1
3
3
7

1
( 1)3+2
17

47 =

1 0 2
4 1 0
0 2 2

30

14
.
17

17 = det A +

30 + 17 = det A

1
=
1

det B

(ii)

x=

1
1
1
1
2
1

1
0
2
1
0
2

0
1
2
0
1
2

5
= 1; y =
5

1 0 1
2 3 2
0 1
2
6 6= 0, logo D invertvel.
12. det C =

1
2
1
1
2
1

1
1
1
1
0
2

0
1
2
0
1
2

= 0 e z=

1
2
1
1
2
1

1
0
2
1
0
2

6 6= 0, logo C invertvel. det D =

207

1
1
1
0
1
2

9
7
2

5
=
5

8
3
0

1
0
0

(i) det (2C

) = 23

(ii) det C 3 (2C)

4
3

= (det C)3

C T 2C

(iii) det

1
=
det C

1 1
1
9
= (det C)2 =
3
2 det C
8
2

= det C

11

(C

1 T

1
det (C
23

) det (C

)=

1
23

1
det C

1
288
(iv) det C T 12 C

1
det C T det (C
23
02

19
= det 4 6
2

2 3
D
3

2C T

(vi) det

3
3
4 5 = 62
6

15
5
1

3
2

8 (det C)

DT

)=

1 1
1
1
=
=
det
C
2
23
23 det C
(det C)

3
2
1
9
5
4
4
7
+2
6
2

1 1
2
@
4
8 11
(v) det (C + 2D) = det
2 1
2

14.

(i)

b
e
h
y

d e f
g h i
a b c

0
0
0
1

c
f
i
z

=5

31
1
0 5A =
0

= ( 2)3 det C T

1
det

2 3
D
3

1
det (DT )

1
27
3
1
det D
27
=8
=
= .
3
2
36
4
(det D) det C
(det D) 8

13. det(2AT B 3 ) = 23 det AT det(B 3 ) = 8 det A

a
d
g
x

8
3
0

1
48

=5,

(ii)

a b c
d e f
g h i

a
2d
g

p
1
3 = 64 3.
(det B)

=5

b
c
2e 2f
h
i

208

= 10

(iii)

a+d b+e c+f


d
e
f
g
h
i

=5

2i
(iv)

3c e
c

a
6

15. (i)

b
3
1

1
2

1 b
3
1

(iii)

2h
2g
3b d 3a
b
a

c
0

3
2

1
1

1
2
1
2
1
0
3a + 1 3b + 2 3c + 1

16.

1
+1
+1
+1
+1
+1

17.

18.

1
..
.

+1
1
..
.
1

1
2
+2
+2
+2
+2

a
b
c
2a + 2 2b + 1 2c
a+1 b+2 c+1

(ii)

=1

=1

(iv)

2
+

1
1
1
2
1
0
3a + 1 3b + 2 3c + 1

a g d
b h e
c i f

(v)

3
2

1
2

2 c

= 10

1
1
1
2
1
0
3a + 1 3b + 2 3c + 1

1 2 1
2 1 0
3a 3b 3c

1
2
3
+3
+3
+3

1
2
3
4
+4
+4

:::
1
:::
.
+ 1 ..
..
.
1
..
...
.
1
:::

1
2
3
4
5
+5

1
=

..
.

+1
1

1
+1

209

a b c
=3 2 1 0
1 2 1

0
0
0
0
0

1 1
1
0
0 0
0 0
0 0

=3

1 1 1
1 1 1
1 1 1
1 1
0
1
0 0

0 0 :::
0
1
0 :::
0
..
...
. 0
..
.
.
. 0 . . 0 ..
.. . .
.
.
0
1 0 0 ::: 0

19. Sejam

6= 0 e A = (aij )n

i j

det

:::

aij =

n 2

n 1

:::

n 2

n 1

a12
a22

a13
1
a23

a32
..
.

a33

n 2

an1

a43
..
.
n 3

an2

n 3

n 3

a12
2
a22

n 2

..
.
an1

n 1 n 2

a32

n 2

a11 a12
a21 a22
..
. a32
..
.

:::

a1 n
1
n 3 a2 n
1

an

a1 n
a2 n
an

..
.
1

a1n
2 a2n

n 1

2n 1

an 1 n
an n

an3 : : :

n 2

a13 : : :
a23 : : :
.
3
a33 . .
.
3
a43 . .
..
...
.

n 3

an2

an 1 n
ann
a1n
a2n

1
1

..
.

2n 1

a n 1 n 1 an 1 n
an n 1
ann

a13 : : : a1 n 1
a1n
a23 : : : a2 n 1
a2n
.
a33 . .
..
.
a43 . . an 2 n 1
.
.. . .
. an 1 n 1 an 1 n
.
an3 : : : an n 1
ann

an1 an2

20. Seja

an3 : : :

n 2

a11
1
a21
..
.

:::
:::
...
..
.
..
.

n 1

n 1

a11
a21
..
.

n 1

n.

= det A.

3
1 a a2
A = 4 1 b b2 5 ,
1 c c2
2

com a; b; c 2 R. A matriz A invertvel se e s se det A 6= 0. Tem-se


1 a a2
1 b b2
1 c c2

1
a
a2
0 b a b 2 a2
0 c a c 2 a2

1
a
a2
0 b a b 2 a2
0 c a c 2 a2

1
a
0 b a
0
0
c2

det A =

=0

se a = b ou a = c. Se a 6= b e a 6= c ento
det A =

1 a a2
1 b b2
1 c c2

1
a
0 b a
0
0
(c

a2
b 2 a2
a) [(c + a) (b + a)]

se b = c.
210

a2
b2 a2
a
(c a)(b + a)
2

1
a
a2
0 b a
b 2 a2
0
0
(c a) (c b)

=0

Logo, a matriz A invertvel se e s se a 6= b; a 6= c e b 6= c.


21. (i)
1 1 1
x1 y1 y1
x2 x2 y2

(ii)

1
0
0
x1 y1 x1 y1 x1
x2
0
y2 x2

(x1 )3
6 (x2 )3
det 6
4 (x3 )3
(x4 )3
2

= (x4

(x1 )2
(x2 )2
(x3 )2
(x4 )2

x1
x2
x3
x4

= (y1

x1 ) ( 1)2+2 det

3
2
1
6
1 7
7 = det 6
4
1 5
1

1
1
1
1

1
x1
(x1 )2
6 0 x2 x1 (x2 )2 (x1 )2
= det 6
4 0 x3 x1 (x3 )2 (x1 )2
0 x4 x1 (x4 )2 (x1 )2
2
1 x1
6 0 1
x1 ) (x3 x1 ) (x2 x1 ) det 6
4 0 1
0 1
2

x1
x2
x3
x4

= (x1

22. (i)

x1 ) (x3

x2 ) (x1

b+c c+a b+a


a
b
c
1
1
1

x1 ) (x2

x3 ) (x1

x1 ) (x3

x4 ) (x2

x1 ) (y2

3
(x1 )3
(x2 )3 7
7=
(x3 )3 5
(x4 )3
3
(x1 )3
(x1 )3 7
7=
(x1 )3 5
(x1 )3

3
(x1 )2
(x1 )3 (x1 )3
x2 + x1 x21 + x1 x2 + x22 7
7=
x3 + x1 x21 + x1 x3 + x23 5
x4 + x1 x21 + x1 x4 + x24

x2 ) (x4

3
(x1 )3 (x1 )3
7
x21 + x1 x2 + x22
7
2
2 5 =
x1 (x3 x2 ) + x3 x2
x1 (x4 x2 ) + x24 x22

3
x1 (x1 )2
(x1 )3 (x1 )3
1 x2 + x1 x21 + x1 x2 + x22 7
7=
0
1
x1 + x3 + x2 5
0
1
x1 + x4 + x2
3
x1 (x1 )2
(x1 )3 (x1 )3
1 x2 + x1 x21 + x1 x2 + x22 7
7=
0
1
x1 + x3 + x2 5
0
0
x4 x3

x3 ) (x2

x2 ) (x4

x4 ) (x3

a+b+c a+b+c a+b+c


a
b
c
1
1
1
211

= (y1

(x1 )2
(x2 )2
(x3 )2
(x4 )2

(x1 )3
(x2 )3
(x3 )3
(x4 )3

1 x1 (x1 )2
6 0 1 x2 + x1
= (x4 x1 ) (x3 x1 ) (x2 x1 ) det 6
4 0 0 x3 x2
0 0 x4 x2
2
1
6 0
= (x4 x1 ) (x3 x1 ) (x2 x1 ) (x3 x2 ) (x4 x2 ) det 6
4 0
0
2
1
6 0
= (x4 x1 ) (x3 x1 ) (x2 x1 ) (x3 x2 ) (x4 x2 ) det 6
4 0
0
= (x4

1
0
x2 y2 x2

x3 ) =
x4 )

0 0 0
a b c
1 1 1

=0

x2 ) .

a1 + b 1 a1
(ii) a2 + b2 a2
a3 + b 3 a3

b1 c 1
b2 c 2
b3 c 3

(iii)

a1 b 1 a1 + b 1 + c 1
a2 b 2 a2 + b 2 + c 2
a3 b 3 a3 + b 3 + c 3

23.

a1 + b1 c1 + d1
a2 + b2 c2 + d2
= a1 c 2

24.

0 0
2 1
0 0

2
1
5

2a1 a1
2a2 a2
2a3 a3

b1 c1
b2 c 2
b3 c 3

= (a1 + b1 ) (c2 + d2 )
a2 c1 + a1 d2

a1 c 1
a2 c 2

0 0 2
2 1 1
0 0 0

2a1
2a2
2a3

b1 c1
b2 c2
b3 c3

a1 b 1 c 1
2 a2 b 2 c 2
a3 b 3 c 3

a1 b 1 c 1
a2 b 2 c 2
a3 b 3 c 3

b2 c 1 + b1 c 2

a1 c 1
b2 d2

(a2 + b2 ) (c1 + d1 ) =

=0 e

a2 d1 + b1 d2

b1 d1
a2 c 2

4 2
2 1
0 0

2
1
5

b2 d1 =

b1 d1
:
b2 d2

0 0
2 1
0 0

0
1
5

=0

2x x 1 2
1 x 1
1
25. O coeciente de x3 na expresso
1, uma vez que as permutaes
3 2 x 1
1 1 1 x
(a das linhas) e (2134) (a das colunas):
correspondentes ao produto x
| 1 {z x x} so (1234)
| {z }
| {z }
=x3

26. (i)

1
0
1

x
x
(ii)
x
x

x
4
x
x

(iii)

x
1
1
1

1
x
1
1

x 1
1 1
0 2
x
x
4
x
1
1
x
1

x
x
x
4
1
1
1
x

=0,

=0,

=0,

1
0
1

par

x 0
1 1
0 1

=0,

mpar

1+x=0,x=1

x
x
x
x
0 4 x
0
0
0
0
4 x
0
0
0
0
4 x
x
1
1
1
0 x 1
0
1 x
0
0
x 1 1 x
1
1
1
x
212

= 0 , x (4

=0,

x)3 = 0 , (x = 0 ou x = 4)

x 1 x 1 x 1
0 x 1
0
1
0
0
x 1 1
1
0
0
x

x
x
x
1

=0,

0
0
0
3 2x x2
0 x 1
0
1 x
0
0
x 1
1 x
1
0
0
0
1
0
0
0 x 1
0
0
0
x 1
0
0
0
3

5 3 3
7 1 5
8 7 1

0
0
0
2x

0 1 x 1 x 1 x2
0 x 1
0
1 x
0
0
x 1 1 x
1
0
0
x 1

=0,

=0,

0
0
0
3
0 x 1
0
0
0
x 1
1
0
0

(x

1)2 (3

28.

1 8
0 10
7 4

5 53 3
7 71 5
8 87 1

coluna mltipla de 5, logo

2
1
3
2
1
3

2x
0
0
0

x2
=0,

x2 ) = 0 , (x = 1 ou x =

x2

5 53 533
27.
=
= 7 71 715 . Como
8 87 871
5 3
a
13 ento a 3 coluna tambm mltipla de 13. Logo 7 1
8 7

2
1
3

2x

=0,

533; 715 e 871 so mltiplos de

1
8 + ( 3) 1
0
10 + ( 3) 0
=
7 4 + ( 3) ( 7)
1 5
0 10 mltiplo de 5.
7 25

3
5
1

mltiplo de 13.

2
1
3

1 5
0 10 . Como a 3a
7 25

29. Seja A = (aij )n n com n mpar e tal que aij + aji = 0, para todos os i; j = 1; :::; n:
Mostre que A no invertvel.
Dem. (aij + aji = 0, para todos os i; j = 1; :::; n), AT =
det A = det AT = det ( A) = ( 1)n det A

n mpar

A. Logo
det A , det A = 0:

Pelo que A no invertvel.

30. Se A (real) fr ortogonal ento det A = 1 ou det A =


Dem. Sendo A ortogonal, tem-se
AT A = AAT = I
213

1:

3)

pelo que
(det A)2 = det A det A = det AT det A = det AT A = det I = 1 ,
, (det A = 1 ou

det A =

1) .

Sendo A unitria, tem-se


AH A = AAH = I
pelo que
jdet Aj2 = det A det A = det AH det A = det AH A = det I = 1 ,
, jdet Aj = 1.

214

Resoluo da 3a Ficha de exerccios para as aulas de problemas


1. (i) Seja U = f(x; y) 2 R2 : x

0g. Por exemplo:

(1; 1) 2 U , mas ( 1)(1; 1) = ( 1; 1) 2


= U.
Logo, U no subespao de R2 .
(ii) Seja U = f(x; y) 2 R2 : xy = 0g. Por exemplo:
(1; 0); (0; 1) 2 U , mas (1; 0) + (0; 1) = (1; 1) 2
= U.
Logo, U no subespao de R2 .
(iii) Seja U = f(x; y) 2 R2 : y = x2 g. Por exemplo:
(1; 1) 2 U , mas 2(1; 1) = (2; 2) 2
= U.
Logo, U no subespao de R2 .
2. Atendendo s respectivas dimenses, os seguintes subespaos de R2 , com as operaes
usuais, so todos os subespaos de R2 .
(i) f(0; 0)g subespao de R2 .
(ii) Seja Vk = f(x; kx) : x 2 Rg com k 2 R (xo). Vk 6= ? pois (0; 0) 2 Vk . Sejam
(x1 ; kx1 ); (x2 ; kx2 ) 2 Vk e 2 R. Tem-se
(x1 ; kx1 ) + (x2 ; kx2 ) = (x1 + x2 ; k (x1 + x2 )) 2 Vk
e, com (x; kx) 2 Vk ,

(x; kx) = ( x; k ( x)) 2 Vk .

Logo, para todo o k 2 R, Vk subespao de R2 .


Em alternativa, uma vez que

Vk = L (f(1; k)g) ,
para todo o k 2 R, conclui-se que Vk subespao de R2 (para todo o k 2 R).
(iii) Seja U = f(0; a) : a 2 Rg. U 6= ? pois (0; 0) 2 U . Sejam (0; a1 ) ; (0; a2 ) 2 U e
2 R. Tem-se
(0; a1 ) + (0; a2 ) = (0; a1 + a2 ) 2 U
e, com (0; a) 2 U ,
Logo, U subespao de R2 .
Em alternativa, uma vez que

(0; a) = (0; a) 2 U .
U = L (f(0; 1)g) ,

conclui-se que U subespao de R2 .


215

(iv) R2 subespao de R2 .
3. Uk subespao de R3 se e s se k = 0.

R3 .

4. (i) Seja U = f(x; y; z) 2 R3 : z = 2g. Ora (0; 0; 0) 2


= U . Logo, U no subespao de
(ii) Seja U = f(x; y; z) 2 R3 : x + y

z = 0g. Tem-se

U = f(x; y; x + y) : x; y 2 Rg .
Uma vez que
(x; y; x + y) = (x; 0; x) + (0; y; y) = x(1; 0; 1) + y(0; 1; 1),
para quaisquer x; y 2 R, tem-se:
U = L (f(1; 0; 1); (0; 1; 1)g) .
Logo, U subespao de R3 .
Alternativamente, note que U = N (A) subespao de R3 , com A =
3

R.

1 1

1 :

(iii) Seja U = f(x; y; z) 2 R3 : x > 0g. Ora (0; 0; 0) 2


= U . Logo, U no subespao de

(iv) Seja U = f(0; 0; z) : z 2 Rg. Uma vez que (0; 0; z) = z(0; 0; 1), para qualquer z 2 R,
tem-se:
U = L (f(0; 0; 1)g) .
Logo, U subespao de R3 .
(v) Seja U = f(x; y; z) 2 R3 : y = 2x e z = 3xg. Tem-se U = f(x; 2x; 3x) : x 2 Rg.
Uma vez que (x; 2x; 3x) = x(1; 2; 3), para qualquer x 2 R, tem-se:
U = L (f(1; 2; 3)g) .
Logo, U subespao de R3 .
Alternativamente, note que U = N (A) subespao de R3 , com A =

2 1 0
3 0 1

(vi) Seja U = f(x; y; z) 2 R3 : x + y = 1g. Ora (0; 0; 0) 2


= U . Logo, U no subespao
3
de R .
(vii) Seja U = f(x; y; z) 2 R3 : x + y + z = 0 e x

U = f(0; y; y) : y 2 Rg .
Uma vez que
(0; y; y) = y(0; 1; 1),
para qualquer y 2 R, tem-se:

U = L (f(0; 1; 1)g) .
216

z = 0g. Tem-se

Logo, U subespao de R3 .
Alternativamente, note que U = N (A) subespao de R3 , com A =

1
1

1
1

1
1

(viii) Seja U = f(x; y; z) 2 R3 : x = y ou y = zg. Tem-se:


U = (x; y; z) 2 R3 : x = y [ (x; y; z) 2 R3 : y = z
Por exemplo:
(1; 1; 2); (1; 2; 2) 2 U , mas (1; 1; 2) + (1; 2; 2) = (2; 3; 4) 2
= U.
Logo, U no subespao de R3 .
(ix) Seja U = f(x; y; z) 2 R3 : x

y = 0 e 2y + z = 0g. Tem-se

U = f(x; x; 2x) : x 2 Rg .
Uma vez que
(x; x; 2x) = x(1; 1; 2),
para qualquer x 2 R, tem-se:

U = L (f(1; 1; 2)g) .

Logo, U subespao de R3 .
Alternativamente, note que U = N (A) subespao de R3 , com A =

1
0

1 0
2 1

(x) Seja U = f(x; y; z) 2 R3 : xy = 0g. Por exemplo:


(1; 0; 1); (0; 1; 0) 2 U , mas (1; 0; 1) + (0; 1; 0) = (1; 1; 1) 2
= U.
Logo, U no subespao de R3 .
O conjunto de todos os polinmios reais de grau igual a n:
U = fa0 + a1 t +

+ an tn 2 Pn : a0 ; a1 ; :::; an 2 R e an 6= 0g ,

com as operaes usuais, no um espao linear. Por exemplo: o polinmio nulo p(t) = 0 2
=
U.
5. Seja P2 o espao linear de todos os polinmios reais de varivel real e de grau menor
ou igual a 2, com as operaes usuais:
(i) Seja U = fa0 + a1 t + a2 t2 2 P2 : a0 = 0g. Tem-se
U = a1 t + a2 t2 : a1 ; a2 2 R = L

t; t2

Logo, U subespao de P2 .
(ii) Seja U = fa0 + a1 t + a2 t2 2 P2 : a2 = 2a0 e a1 = 0g. Tem-se
U = a0 + 2a0 t2 : a0 2 R .
217

Uma vez que


a0 + 2a0 t2 = a0 (1 + 2t2 ),
para qualquer a0 2 R, tem-se:

U =L

1 + 2t2

Logo, U subespao de P2 .
(iii) Seja U = fa0 + a1 t + a2 t2 2 P2 : a1 = 1g. Por exemplo: o polinmio nulo p(t) =
02
= U . Logo, U no subespao de P2 .
(iv) Seja U = fa0 + a1 t + a2 t2 2 P2 : a2 a1 = 2g. Por exemplo: o polinmio nulo
p(t) = 0 2
= U . Logo, U no subespao de P2 .
(v) Seja U = fa0 + a1 t + a2 t2 2 P2 : a2

a1 + 2a0 = 0g. Tem-se


2a0 ) t2 : a0 ; a1 2 R .

U = a0 + a1 t + (a1
Uma vez que
a0 + a1 t + (a1

2a0 ) t2 = a0 (1

2t2 ) + a1 (t + t2 ),

para quaisquer a0 ; a1 2 R, tem-se:


U =L

2t2 ; t + t2

Logo, U subespao de P2 .
6. Seja M2 3 (R) o espao linear de todas as matrizes do tipo 2
a b c
(i) Seja U =
2 M2 3 (R) : b = a + c . Tem-se
d 0 0
a a+c c
d
0
0

U=

3 com entradas reais.

: a; c; d 2 R .

Uma vez que


a a+c c
d
0
0

=a

1 1 0
0 0 0

+c

0 1 1
0 0 0

+d

0 0 0
1 0 0

para quaisquer a; c; d 2 R, tem-se:


U =L

1 1 0
0 0 0

0 1 1
0 0 0

0 0 0
1 0 0

Logo, U subespao de M2 3 (R).


(ii) Seja U =

a b c
d 0 f

2 M2 3 (R) : b < 0 . Por exemplo: a matriz nula


0 0 0
0 0 0
218

2
= U.

Logo, U no subespao de M2 3 (R).


(iii) Seja U =

a b c
d e f

2 M2 3 (R) : a =

U=

2c b
c
d e 2e + d

2c e f = 2e + d . Tem-se

: b; c; d; e 2 R .

Uma vez que


2c b
c
d e 2e + d

=b

0 1 0
0 0 0

+c

2 0 1
0 0 0

+d

0 0 0
1 0 1

+e

0 0 0
0 1 2

para quaisquer b; c; d; e 2 R, tem-se:


U =L

0 1 0
0 0 0

2 0 1
0 0 0

Logo, U subespao de M2 3 (R).

0 0 0
1 0 1

0 0 0
0 1 2

1
1
. Tem-se C(A) = L (f(1; 0)g) e L(A) = L (f(1; 1)g).
0
0
Seja u = (x; y) 2 R2 . Atendendo a que
7. (i) Seja A =

1
0

1
0

x
y

=0,x

y = 0,

o ncleo de A dado por:


N (A) = u 2 R2 : Au = 0 = (x; y) 2 R2 : x y = 0 =
= f(x; x) : x 2 Rg = fx(1; 1) : x 2 Rg = L (f(1; 1)g) .
1 2 3
. Tem-se C(A) = L (f(1; 0)g) e L(A) = L (f(1; 2; 3)g).
0 0 0
Seja u = (x; y; z) 2 R3 . Atendendo a que
2 3
x
1 2 3 4 5
y = 0 , x + 2y + 3z = 0,
0 0 0
z

(ii) Seja A =

o ncleo de A dado por:

N (A) = u 2 R3 : Au = 0 = (x; y; z) 2 R3 : x + 2y + 3z = 0 =
= f( 2y 3z; y; z) : y; z 2 Rg = fy( 2; 1; 0) + z( 3; 0; 1) : y; z 2 Rg
= L (f( 2; 1; 0); ( 3; 0; 1)g) .

219

0 0 0
. Tem-se C(A) = f(0; 0)g e L(A) = f(0; 0; 0)g.
0 0 0
O ncleo de A dado por: N (A) = R3 .
2
3
2 1 1
(iv) Seja A = 4 0 0 1 5. Tem-se
0 0 0
(iii) Seja A =

C(A) = L (f(2; 0; 0); (1; 1; 0)g)

Seja u = (x; y; z) 2 R3 .
2
2
4 0
0

o ncleo de A dado por:

e L(A) = L (f(2; 1; 1); (0; 0; 1)g) :

Atendendo a que
8
32 3
1 1
x
< 2x + y + z = 0
5
4
5
0 1
y =0,
:
0 0
z
z=0

N (A) = u 2 R3 : Au = 0 = (x; y; z) 2 R3 : 2x + y + z = 0 e z = 0 =
= f(x; 2x; 0) : x 2 Rg = fx(1; 2; 0) : x 2 Rg = L (f(1; 2; 0)g) .
2

3
1 0
(v) Seja A = 4 2 3 5. Tem-se
2 1

C(A) = L (f(1; 2; 2); (0; 3; 1)g)

e L(A) = L (f(1; 0); (2; 3)g) ,

pois
1
4
(2; 1) = (1; 0) + (2; 3).
3
3
2
Seja u = (x; y) 2 R . Atendendo a que
8
x=0
>
>
2
3
>
>
1 0
<
4 2 3 5 x =0,
2x + 3y = 0
y
>
>
2 1
>
>
:
2x + y = 0

o ncleo de A dado por:

N (A) = u 2 R2 : Au = 0 = (x; y) 2 R2 : x = 0 e 2x + 3y = 0 e 2x + y = 0 = f(0; 0)g .


2

3
1 2
(vi) Seja A = 4 2 4 5. Tem-se
2 4

C(A) = L (f(1; 2; 2)g)

e L(A) = L (f(1; 2)g) .


220

Seja u = (x; y) 2 R2 . Atendendo a que


2
3
1 2
4 2 4 5 x
y
2 4

=0,

o ncleo de A dado por:

8
< x + 2y = 0
:

2x + 4y = 0

N (A) = u 2 R2 : Au = 0 = (x; y) 2 R2 : x + 2y = 0 =
= f( 2y; y) : y 2 Rg = fy( 2; 1) : y 2 Rg = L (f( 2; 1)g) .
2

3
0 0
(vii) Seja A = 4 0 0 5. Tem-se
0 0

C(A) = f(0; 0; 0)g

e L(A) = f(0; 0)g .

O ncleo de A dado por:


N (A) = R2 .
2

3
1 0 1
(viii) Seja A = 4 2 3 0 5. Tem-se
2 1 0

C(A) = L (f(1; 2; 2); (0; 3; 1); (1; 0; 0)g)

e L(A) = L (f(1; 0; 1); (2; 3; 0); (2; 1; 0)g) .

Seja u = (x; y; z) 2 R3 . Atendendo a que


2

32 3
2
1 0 1
x
1 0
4 2 3 0 54 y 5 = 0 , 4 0 1
2 1 0
z
0 0
o ncleo de A dado por:

8
x+z =0
>
>
32 3
>
>
1
x
<
5
4
5
2
y =0,
y 2z = 0
>
>
4
z
>
>
:
4z = 0

N (A) = u 2 R3 : Au = 0 = f(0; 0; 0)g .


Observao: Como N (A) = f(0; 0; 0)g e sendo A quadrada 3 3, tem-se L(A) = C(A) = R3 .
8. Seja

Uma vez que


2

82
9
3
< a 0
=
U = 4 b c 5 2 M3 2 (R) : a; b; c; d 2 R .
:
;
0 d

3
2
3
2
3
2
3
2
3
a 0
1 0
0 0
0 0
0 0
4 b c 5 = a4 0 0 5 + b4 1 0 5 + c4 0 1 5 + d4 0 0 5,
0 d
0 0
0 0
0 0
0 1
221

com a; b; c; d 2 R, tem-se
082
3 2
3 2
3 2
391
0 0
0 0
0 0 =
< 1 0
U = L@ 4 0 0 5;4 1 0 5;4 0 1 5;4 0 0 5 A.
:
;
0 0
0 0
0 0
0 1
9. Considere, no espao linear R3 , os vectores v1 = (1; 2; 1), v2 = (1; 0; 2) e v3 = (1; 1; 0).
Tem-se
(i) (3; 3; 0) = 0(1; 2; 1) + 0(1; 0; 2) + 3(1; 1; 0)
(ii) (2; 1; 5) = 1(1; 2; 1) + 2(1; 0; 2) + ( 1)(1; 1; 0)
(iii) ( 1; 2; 0) = 2(1; 2; 1) + ( 1)(1; 0; 2) + ( 2)(1; 1; 0)
(iv) (1; 1; 1) = 13 (1; 2; 1) + 31 (1; 0; 2) + 31 (1; 1; 0).
10. Considere, no espao linear R4 , os vectores
v3 = (0; 1; 2; 1). Tem-se
2
3
2
1 1 0 j
1 j 2 j 1 j 0
6 0
6
1 1 j 4 j 0 j 1 j 1 7
6
7
6
!
4 0 0 2 j 2 j 2 j
2 j 1 5 L1 +L4 !L4 4
1 0 1 j 2 j 2 j 2 j 0
!

L2 +L4 !L4

1
6 0
6
4 0
0

1
1
0
0

0
1
2
0

j
j
j
j

1
4
2
1

j
j
j
j

2
0
2
0

j
j
j
j

v1 = (1; 0; 0; 1), v2 = (1; 1; 0; 0) e


1
0
0
0

1
1
0
1
1
1
2
0

Logo, (2; 0; 2; 2); (1; 1; 2; 2) 2 L (fv1 ; v2 ; v3 g), com

0
1
2
1
j
j
j
j

j
j
j
j

1
4
2
3

j
j
j
j

2
0
2
0

3
0
1 7
7 : (*)
1 5
1

j
j
j
j

1
1
2
1

j
j
j
j

3
0
1 7
7
1 5
0

(2; 0; 2; 2) = (1; 0; 0; 1) + (1; 1; 0; 0) + (0; 1; 2; 1)


(1; 1; 2; 2) = 3(1; 0; 0; 1) + ( 2)(1; 1; 0; 0) + ( 1)(0; 1; 2; 1).
Atendendo a (*), ( 1; 4; 2; 2); (0; 1; 1; 0) 2
= L (fv1 ; v2 ; v3 g).
11. Tem-se
2

3
4 0
2

2 j
1 j
5 j

3
1
2 5
k

2
L +L3 !L3
3 1

3
4 0
0

3
2
j
1
1
j
2 5
11=3 j k + 2=3

11
L +L3 !L3
3 2

3
4 0
0

3
2 j
1
1 j
2 5:
0 j k+8

Logo, 8 o nico valor de k para o qual o vector u = (1; 2; k) 2 R3 combinao


linear dos vectores
v = (3; 0; 2) e w = (2; 1; 5):

222

L2 +L4 !L4

12. Considere, no espao linear P2 , os vectores p1 (t) = 2 + t + 2t2 , p2 (t) =


p3 (t) = 2 5t + 5t2 e p4 (t) = 2 3t t2 . O vector

2t + t2 ,

q(t) = 2 + t + t2
pertence expanso linear L (fp1 (t); p2 (t); p3 (t); p4 (t)g)? Podem os vectores p1 (t), p2 (t),
p3 (t) e p4 (t) gerar P2 ? Tem-se
2
2
3
3
2 0
2
2 j 2
2 0
2
2 j 2
4 1
4 0
2
6
2 j 0 51
2
5
3 j 1 5 1 !
!
L
+L
!L
1
2
2
2
0 1
3
1 j
1 2 L2 +L3 !L3
2 1
5
1 j 1
2

2
4
! 0
0

L1 +L3 !L3

0
2
0

3
2
0 5 . (**)
1

2 j
2 j
0 j

2
6
0

Atendendo a (**), q(t) = 2 + t + t2 2


= L (fp1 (t); p2 (t); p3 (t); p4 (t)g). Logo,
fp1 (t); p2 (t); p3 (t); p4 (t)g no pode gerar P2 :
13. (i) Seja U = f(1; 0; 0); (0; 1; 0); (0; 0; 1)g. Seja (x; y; z) 2 R3 . Tem-se
(x; y; z) = x(1; 0; 0) + y(0; 1; 0) + z(0; 0; 1).
Logo, U gera R3 .
(ii) Seja U = f(1; 1; 1); (0; 1; 1); (0; 0; 1)g. Seja (x; y; z) 2 R3 . Tem-se
(x; y; z) = x(1; 1; 1) + (y

x) (0; 1; 1) + (z

y) (0; 0; 1).

Logo, U gera R3 .
(iii) Seja U = f(1; 1; 1) ; ( 1; 1; 1); (1; 1; 1); ( 1; 1; 1)g. Seja (x; y; z) 2 R3 . Determinemos os valores dos escalares 1 ; 2 ; 3 ; 4 para os quais se tem
2 3
2 3
2
3
2
3
2
3
x
1
1
1
1
4 y 5 = 14 1 5 + 24 1 5 + 34 1 5 + 44 1 5.
z
1
1
1
1
Ora a ltima igualdade equivalente a
2
2

1
4 1
1

x
1
4 y 5=4 1
z
1
1
1
1

1
1
1

3
1 j x
1 j y 5
1 j z

1
1
1

1
1
1

L1 +L2 !L2
L1 +L3 !L3

223

1 6
1 56
4
1

1
4 0
0

1
2
0

1
2
3
4

1
2
2

7
7.
5

3
1 j
x
0 j y x 5.
2 j z x

Logo

8
>
>
>
>
>
>
>
>
<

e assim
2 3
x
4 y 5=
z

>
>
>
>
>
>
>
>
:

= 12 x + 12 y + s

= 12 y

1
z
2

+s

= 12 x

1
z
2

+s

= s, s 2 R
2

3
1
1
1
1
x + y + s 4 1 5+ y
2
2
2
1

com s 2 R. Logo, U gera R3 .


14.
3
1

1
1

1 A+ 2 B+ 3 C

=
1

+2

8
>
>
<

3
3

Logo
3
1

1
1

1 1
1 0

=3

3
1
1
1
z + s 4 1 5+ x
2
2
1

0 0
1 1

3 2
3
1
1
1
z + s 4 1 5+s 4 1 5 ,
2
1
1

8
=3
<
1+2 3 = 1
,
,
>
:
1+ 2 = 1
>
:
=
1
2
3
1

0
0

2
1

1
2
3

=3
= 2
= 1.

1 1
0 0
0 2
;
;
.
1 0
1 1
0
1
Existe D 2 M2 2 (R) tal que D 2
= U uma vez que
Seja U = L

M2

Seja
que

a b
c d

2 U . Tem-se

(R)

e dim
| {z U}

a b
c d
a b
c d

a b
c d

= A+ B+ C ,
2

1
6 1
6
4 1
0

0
0
1
1

0
2
0
1

j
j
j
j

3
a
b 7
7
c 5
d

< dim M2
|
{z

a b
c d

=4

(R).
}

2 U se e s se existirem escalares ; ;

2 R tais

= A + B + C.

L1 +L2 !L2
L1 +L3 !L3

1
6 0
6
4 0
0
224

1+2

0
0
1
1

0
2
0
1

3
3

3
j
a
j b a 7
7
j c a 5
j
d

8
>
>
<

=a
+
2 3=b
1
,
>
1+ 2 = c
>
:
2
3 = d
!

L3 +L4 !L4
1
L
+L4 !L4
2 2

L3 +L4 !L4
1
L
+L4 !L4
2 2

1
6 0
6
4 0
0

0
0
1
0

0
2
0
0

j
a
j
b a
j
c a
1
j d + 2 (b + a)

1
7
6 0
7 ! 6
5 L2 $L3 4 0
c
0

0
1
0
0

0
0
2
0

j
a
j
c a
j
b a
1
j d + 2 (b + a)

Logo, para que o sistema linear anterior seja possvel necessrio que se tenha
d+

1
(b + a)
2

3
c.

7
7
5

c = 0.

Deste modo podemos escrever


a b
c d

U=

e assim, sendo V =

a b
c d

2 M2

2 M2

M2

(R) : d +

1
(b + a)
2

(R) : d + 12 (b + a)
2

(R) = U

c=0

c 6= 0 , tem-se

V.

Ou seja, qualquer vector de V que no seja o vector nulo, esse vector no pertence a U . Por
exemplo
1 1
1 1
0 0
0 2
2
=U =L
;
;
.
1 1
1 0
1 1
0
1

15. Sejam
u = (2; 1; 0);

v = (1; 1; 2) e w = (0; 3; 4):

O vector (a; b; c) de R3 pertencer a L (fu; v; wg) se existirem ; ;

2 R tais que

(a; b; c) = (2; 1; 0) + (1; 1; 2) + (0; 3; 4),


isto , se o seguinte sistema (nas variveis , e ) fr possvel e determinado:
8
< 2 + =a
+3 =b
:
2
4 = c.
Considerando
2
2
4 1
0

ento a matriz aumentada deste sistema, tem-se:


3
2
2
1
0 j
1
0 j a
4
5
0
3=2 3 j b
1 3 j b
!
1
L1 +L2 !L2
2
0
2
4 j
2
4 j c
2
2
1
0 j
a
4
0
3=2 3 j
b a2
!
4
L
+L
!L
3
3
3 2
0
0
0 j c + 43 b 32 a

Assim, o vector (a; b; c) de R3 pertencer a L (fu; v; wg) se:


4
c+ b
3

2
a = 0.
3
225

3
a
a=2 5
c
3
5.

4
L +L3 !L3
3 2

Observao: Deste modo, tem-se L (fu; v; wg) 6= R3 . De facto, uma vez que
1
v= u
2

1
w
2

tem-se L (fu; v; wg) = L (fu; wg) e como tal fu; v; wg no pode gerar R3 .
16. Sejam
A=

1 1 5
2 3 13

A=

1 1 5
2 3 13

Tem-se

1
e B=4 4
3

1
3
1

3
1
1 5:
3

1 1 5
0 1 3

2L1 +L2 !L2

= A0

e
2

1
4
B= 4
3

1
3
1

3
1
1 5
3

4L1 +L2 !L2


3L1 +L3 !L3

1
4 0
0

1
1
2

Atendendo ao mtodo de eliminao de Gauss:

3
1
3 5
6

2L2 +L3 !L3

1
4 0
0

L(A) = L(A0 ) e L(B) = L(B 0 ).


Alm disso, uma vez que
(1; 1; 1) = (1; 1; 5)

2(0; 1; 3),

tem-se
L(A) = L(A0 ) = L(B 0 ) = L(B).
Finalmente, como se tem sempre
C(AT ) = L(A) e L(B) = C(B T ),
conclui-se que C(AT ) = C(B T ).
17. (i) Seja U = f(x; y; z; w) 2 R4 : x = 0 e y =

zg. Tem-se

U = f(0; z; z; w) : z; w 2 Rg .
Atendendo a que
(0; z; z; w) = z(0; 1; 1; 0) + w(0; 0; 0; 1),
tem-se
U = L (f(0; 1; 1; 0); (0; 0; 0; 1)g) .
(ii) Seja U = f(x; y; z; w) 2 R4 : x + y + z + w = 0g. Tem-se
U = f( y

w; y; z; w) : y; z; w 2 Rg .
226

1
1
0

3
1
3 5 = B0.
0

Atendendo a que
( y

w; y; z; w) = y( 1; 1; 0; 0) + z( 1; 0; 1; 0) + w( 1; 0; 0; 1),

tem-se
U = L (f( 1; 1; 0; 0); ( 1; 0; 1; 0); ( 1; 0; 0; 1)g) .

(iii) Seja U = f(x; y; z; w) 2 R4 : x + 2y


Observe-se que

Tem-se
2

z = 0 e x + y + 2w = 0 e y
2

1 2
4
U = N (A), com A = 1 1
0 1

1 2
A=4 1 1
0 1

3
1 0
0 2 5
1 1

L1 +L2 !L2

Logo, U = N (A) = N (A0 ). Assim,

1
4 0
0

U = (x; y; z; w) 2 R4 : x + 2y

3
1 0
0 2 5.
1 1

3
2
1 0
1
1 2 5
! 4 0
L2 +L3 !L3
1 1
0

2
1
1

z=0 e

z + w = 0g.

2
1
0

3
1 0
1 2 5 = A0 .
0 3

y + z + 2w = 0 e 3w = 0 =

= f( z; z; z; 0) : z 2 Rg = fz( 1; 1; 1; 0) : z 2 Rg = L (f( 1; 1; 1; 0)g) .


18. (i) Seja U = L (f1 t2 ; 1 + tg) um subespao de P2 . Seja p (t) 2 U , com p (t) =
a0 + a1 t + a2 t2 . Ento, existiro ; 2 R tais que
p (t) = a0 + a1 t + a2 t2 =
Tem-se
2
1
4 0
1

ento a matriz aumentada


3
2
3
1 j a0
1 1 j
a0
4 0 1 j
5
1 j a1 5
a1
!
L1 +L3 !L3
0 j a2
0 1 j a0 + a2

t2 + (1 + t) .

L2 +L3 !L3

1 1 j
a0
4 0 1 j
a1
0 0 j a0 + a2

Logo, para que o sistema linear anterior seja possvel preciso que a0 + a2
U = p (t) = a0 + a1 t + a2 t2 2 P2 : a0 + a2

a1

5.

a1 = 0. Assim,

a1 = 0 .

(ii) Seja U = L (f(1; 0; 1); (0; 1; 0); ( 2; 1; 2)g). Seja (x; y; z) 2 U . Ento, existiro
; ; 2 R tais que
(x; y; z) = (1; 0; 1) + (0; 1; 0) + ( 2; 1; 2).
Tem-se ento a matriz aumentada
2
3
1 0
2 j x
4 0 1 1 j y 5
1 0
2 j z

L1 +L3 !L3

227

1 0
4 0 1
0 0

3
2 j
x
1 j
y 5.
0 j z x

Assim,
U = (x; y; z) 2 R3 : z

x=0 .

Observao extra: U = L (f(1; 0; 1); (0; 1; 0); ( 2; 1; 2)g) = L (f(1; 0; 1); (0; 1; 0)g),
uma vez que
( 2; 1; 2) = ( 2)(1; 0; 1) + (0; 1; 0).
(iii) Seja V = L (f(0; 1; 0); ( 2; 1; 2)g). Seja (x; y; z) 2 V . Ento, existiro ;
tais que
(x; y; z) = (0; 1; 0) + ( 2; 1; 2).
Tem-se ento
2
0
4 1
0
Assim,

a matriz aumentada
2
3
2 j x
1
1 j y 5
! 4 0
L1 !L2
2 j z
0

3
1 j y
2 j x 5
2 j z

V = (x; y; z) 2 R3 : z

L2 +L3 !L3

1
4 0
0

2R

3
1 j
y
2 j
x 5.
0 j z x

x=0 .

Observao extra: V = L (f(0; 1; 0); ( 2; 1; 2)g) = L (f(1; 0; 1); (0; 1; 0)g), uma vez
que
1
2

( 2; 1; 2) = ( 2)(1; 0; 1) + (0; 1; 0) e (1; 0; 1) =

1
( 2; 1; 2) + (0; 1; 0).
2

(iv) Seja W = L (f(1; 1; 2); (2; 1; 1)g). Seja (x; y; z) 2 V . Ento, existiro ;
que
(x; y; z) = (1; 1; 2) + (2; 1; 1).
Tem-se ento a matriz aumentada
2
3
2
3
1 2 j x
1 2 j
x
4 1 1 j y 5
4 0
1 j y x 5
!
L1 +L2 !L2
2 1 j z
0
3 j z 2x
2L1 +L3 !L3

3L2 +L3 !L3

Assim,

W = (x; y; z) 2 R3 : x

1
4 0
0

2 R tais

3
2 j
x
5.
1 j
y x
0 j z 3y + x

3y + z = 0 .

Observao extra: W = L (f(3; 1; 0); ( 1; 0; 1)g) = L (f(1; 1; 2); (2; 1; 1)g), uma vez
que
(3; 1; 0) = 2(2; 1; 1) + ( 1)(1; 1; 2), ( 1; 0; 1) = (1; 1; 2) + ( 1)(2; 1; 1)
e
(1; 1; 2) = (3; 1; 0) + 2( 1; 0; 1), (2; 1; 1) = (3; 1; 0) + ( 1; 0; 1).
(v) Seja U = L (f(1; 0; 1; 1)g). Seja (x; y; z; w) 2 U . Ento, existir
(x; y; z; w) = (1; 0; 1; 1).
228

2 R tal que

Tem-se ento a matriz aumentada


2
1 j x
6 0 j y
6
4 1 j z
1 j w
Assim,

3
7
7
5

L1 +L3 !L3
L1 +L4 !L4

3
j
x
j
y 7
7.
j x+z 5
j w x

1
6 0
6
4 0
0

U = (x; y; z; w) 2 R4 : y = 0 e x + z = 0 e w

x=0 .

(vi) Seja U = L (f(1; 2; 5; 3); (2; 4; 6; 2); (3; 6; 11; 1); (0; 0; 1; 2)g). Como
1
1
(3; 6; 11; 1) = (1; 2; 5; 3)+(2; 4; 6; 2) e (0; 0; 1; 2) = (1; 2; 5; 3) (2; 4; 6; 2)
2
4
ento
U = L (f(1; 2; 5; 3); (2; 4; 6; 2)g) .
Seja (x; y; z; w) 2 U . Ento, existiro ;

2 R tais que

(x; y; z; w) = (1; 2; 5; 3) + (2; 4; 6; 2).


Tem-se ento a matriz
2
1
2
6 2
4
6
4 5
6
3 2
!

2L3 +L4 !L4

Assim,

aumentada
3
j x
j y 7
7
!
j z 5 2L1 +L2 !L2
5L1 +L3 !L3
j w
3L +L !L

1
6 0
6
4 0
0

2
0
4
0

1
6 0
6
4 0
0

2
0
4
8

3
x
7
2x + y
7 !
5 L2 $L3
5x + z
7x + 2z + w

j
j
j
j

U = (x; y; z; w) 2 R4 : 2x + y = 0 e

j
j
j
j

3
x
2x + y 7
7
!
.
5x + z 5 2L3 +L4 !L4
3x + w

1
6 0
6
4 0
0

2
4
0
0

j
j
j
j

3
x
7
5x + z
7:
5
2x + y
7x + 2z + w

7x + 2z + w = 0 .

19. Podemos colocar os vectores do conjunto f( 1 ; 1 ; 3); ( 2 ; 2 ; 9)g como colunas de


uma matriz A e de seguida aplicar a essa matriz o mtodo de eliminao de Gauss. Se
1 6= 0, tem-se
2
3
1

A=4

3
5

1 L +L !L
1
2
2
1
3
L1 +L3 !L3
1

6
6
6
6 0
6
6
6
4
0

3
2
1

+9

7
7
7
7
7 = A0 .
7
7
5

As colunas da matriz A correspondentes s colunas da matriz em escada A0 que contm


os pivots, formam um conjunto de vectores linearmente independente. Logo, o conjunto
229

f(
Se

1;
1

1 ; 3); ( 2 ;

2 ; 9)g

= 0, tem-se
2
0
4 1
3

2
2

linearmente independente se
3
5

! 4

L1 $L3

3
1

9
2
2

3
5

1 L +L !L
1
2
2
3

6= 0 e
2

3
4 0
0

6=

9
3

ou

2
1

6= 3 .

5.

Logo, o conjunto f( 1 ; 1 ; 3); ( 2 ; 2 ; 9)g linearmente independente se 1 = 0 e ( 2 6= 3 1


ou 2 6= 0). Assim, o conjunto f( 1 ; 1 ; 3); ( 2 ; 2 ; 9)g linearmente independente se e s se
1

6= 0 e

6=

ou

2
1

6= 3

ou (

=0 e (

6= 3

ou

6= 0)) .

20. (i) Podemos colocar os vectores do conjunto f(4; 2; 1); (2; 6; 5); (1; 2; 3)g como
colunas de uma matriz A e de seguida aplicar a essa matriz o mtodo de eliminao de
Gauss:
2
3
2
3
4 2
1
1
5 3
2 5 ! 4 2 6
2 5
A=4 2 6
!
L1 $L3
2L1 +L2 !L2
1
5 3
4 2
1
4L1 +L3 !L3
2
3
2
3
2
3
1
5 3
1
5 3
1
5 3
4 0 2
1 5
1 5 = A0 .
8 5 1 ! 4 0 2
! 4 0 16
!
L2 +L3 !L3
L
!L
2
2
8
0 22
11
0 2
1
0 0
0
1
11

L3 !L3

As colunas da matriz A correspondentes s colunas da matriz em escada A0 que contm


os pivots, formam um conjunto de vectores linearmente independente. Logo, o conjunto
f(4; 2; 1); (2; 6; 5); (1; 2; 3)g linearmente dependente, mas o conjunto f(4; 2; 1); (2; 6; 5)g
linearmente independente. Procuremos ento ; 2 R tais que
(1; 2; 3) = (4; 2; 1) + (2; 6; 5).
Atendendo ao que j se fez e considerando a 3a coluna como o termo independente do sistema,
tem-se
8
4 +2 =1
>
8
8
>
>
>
5 =3
<
< = 12
<
2 +6 = 2 ,
,
>
:
:
>
2
=
1
= 12 .
>
>
:
5 =3
Pelo que

1
(1; 2; 3) = (4; 2; 1)
2

1
(2; 6; 5).
2

(ii) Podemos colocar os vectores do conjunto


f(1; 2; 1); (3; 2; 5)g
230

como colunas de uma matriz e de seguida


Gauss.
2
2
3
1 3
4
A=4 2 2 5
!
2L1 +L2 !L2
1 5
L1 +L3 !L3

aplicar a essa matriz o mtodo de eliminao de


2
3
3
1 3
1 3
4 0
0
4 5
4 5 = A0
!
2L2 +L3 !L3
0 8
0 0

As colunas da matriz A correspondentes s colunas da matriz em escada A0 que contm


os pivots, formam um conjunto de vectores linearmente independente. Logo, o conjunto
f(1; 2; 1); (3; 2; 5)g linearmente independente.

(iii) Podemos colocar os vectores do conjunto f(1; 2; 3); (1; 1; 1); (1; 0; 1)g como colunas
de uma matriz A e de seguida aplicar a essa matriz o mtodo de eliminao de Gauss.
2
2
2
3
3
3
1 1 1
1 1
1
1 1
1
4 0
4 0
1
2 5
1
2 5 = A0 .
A=4 2 1 0 5
!
!
2L1 +L2 !L2
2L2 +L3 !L3
0
2
2
0 0
2
3 1 1
3L1 +L3 !L3

As colunas da matriz A correspondentes s colunas da matriz em escada A0 que contm


os pivots, formam um conjunto de vectores linearmente independente. Logo, o conjunto
f(1; 2; 3); (1; 1; 1); (1; 0; 1)g linearmente independente.
Observao extra: encontrmos trs vectores de R3 linearmente independentes. Como
a dimenso de R3 3, ento o conjunto f(1; 2; 3); (1; 1; 1); (1; 0; 1)g desde logo uma base de
R3 , sem ser preciso vericar se gera R3 .
(iv) O conjunto f(1; 0; 1); (0; 0; 0); (0; 1; 1)g contm o vector nulo, logo linearmente dependente. Facilmente se v que f(1; 0; 1); (0; 1; 1)g linearmente independente. Facilmente
tambm se v que
(0; 0; 0) = 0(1; 0; 1) + 0(0; 1; 1).
(v) Como a dimenso de R3 3, ento qualquer conjunto de vectores de R3 com mais
do que trs vectores linearmente dependente. O conjunto
f(1; 1; 0); (0; 2; 3); (1; 2; 3); (x; y; z)g
formado por quatro vectores de R3 , logo linearmente dependente para quaisquer x; y; z 2
R.
Resoluo alternativa para vericar a dependncia linear: Podemos colocar os
vectores do conjunto f(1; 1; 0); (0; 2; 3); (1; 2; 3); (x; y; z)g como colunas de uma matriz A e
de seguida aplicar a essa matriz o mtodo de eliminao de Gauss.
2
3
2
3
1 0 1 x
1 0 1
x
4 0 2 1 y x 5
A=4 1 2 2 y 5
!
!
3
L1 +L2 !L2
L +L3 !L3
2 2
0 3 3 z
0 3 3
z
2
3
1 0 1
x
4 0 2 1
5 = A0 .
y x
!
3
L
+L
!L
3
3
3
3
2 2
0 0 2 z 2 (y x)
231

As colunas da matriz A correspondentes s colunas da matriz em escada A0 que contm os


pivots, formam um conjunto de vectores linearmente independente. Logo, o conjunto
f(1; 1; 0); (0; 2; 3); (1; 2; 3); (x; y; z)g
linearmente dependente para quaisquer x; y; z 2 R, mas o conjunto f(1; 1; 0); (0; 2; 3); (1; 2; 3)g
linearmente independente.
Observao extra: encontrmos trs vectores de R3 linearmente independentes. Como
a dimenso de R3 3, ento o conjunto f(1; 1; 0); (0; 2; 3); (1; 2; 3)g desde logo uma base de
R3 , sem ser preciso vericar se gera R3 .
Procuremos ento ; ; 2 R tais que
(x; y; z) = (1; 1; 0) + (0; 2; 3) + (1; 2; 3).
Atendendo ao que j se fez e considerando a 4a coluna como o termo independente do sistema,
tem-se
8
8
8
+
=
x
+
=
x
= x 32 z + y
>
>
>
>
>
>
>
>
>
>
>
>
<
<
<
+2 + =y ,
2 + =y x
= (y x) 13 z
,
>
>
>
>
>
>
>
>
>
>
>
>
:
: 3
:
3
3 +3 =z
=
z
(y
x)
= 23 z y + x.
2
2
Pelo que

(x; y; z) =

2
z + y (1; 1; 0) + (y
3

x)

1
z (0; 2; 3) +
3

2
z
3

y + x (1; 2; 3).

21. Podemos colocar os vectores do conjunto f(a2 ; 0; 1); (0; a; 2); (1; 0; 1)g como colunas
de uma A matriz e de seguida aplicar a essa matriz o mtodo de eliminao de Gauss.
2 2
3
2
3
a 0 1
1 2 1
A=4 0 a 0 5 ! 4 0 a 0 5 2 !
L1 $L3
a L1 +L3 !L3
1 2 1
a2 0 1
2
3
2
3
1
2
1
1 2
1
4 0
4 0 a
a
0 5
0 5 = A0 .
!
!
2aL2 +L3 !L3
a2 L1 +L3 !L3
2
2
0
2a 1 a
0 0 1 a2

As colunas da matriz A correspondentes s colunas da matriz em escada A0 que contm os


pivots, formam um conjunto de vectores linearmente independente. Logo, o conjunto
Sa = (a2 ; 0; 1); (0; a; 2); (1; 0; 1)
linearmente independente se e s se a 2
= f 1; 0; 1g. Logo, uma vez que dim R3 = 3 e Sa
tem 3 vectores, Sa ser uma base de R3 se e s se a 2
= f 1; 0; 1g.

232

22. Sejam U = L (f(1; 1; 0; 0); (0; 1; 1; 0)g) e Vk = L (f(2; k; 1; 0); (0; 0; 0; 1)g) subespaos
de R4 : Determine os valores de k para os quais dim (U \ Vk ) = 1. Coloquemos os vectores
geradores de U e de V como colunas da matriz:
2
3
2
3
1 0 2 0
1 0 2 0
6 1 1 k 0 7
6
7
6
7 ! 6 0 1 1 0 7
!
4 0 1 1 0 5 L2 $L
4
1 1 k 0 5 L1 +L3 !L3
3
0 0 0 1
0 0 0 1
!

L1 +L3 !L3

3
0
2
0
1
1
0 7
7
1 k 2 0 5
0
0
1

1
6 0
6
4 0
0

L2 +L3 !L3

Note que U + Vk = L (U [ Vk ). Como


dim (U \ Vk ) = dim U + dim Vk

1
6 0
6
4 0
0

dim (U + Vk ) = 2 + 2

3
0
2
0
1
1
0 7
7.
0 k 3 0 5
0
0
1

dim (U + Vk ) = 4

dim (U + Vk )

e
dim (U + Vk ) =

3 se k = 3
4 se k 6= 3

ento dim (U \ Vk ) = 1 se e s se k = 3.
23. (i) Seja (x; y; z) 2 R3 . Podemos colocar os vectores do conjunto
f(1; 0; 2); (0; 1; 2); (x; y; z)g
como colunas
de Gauss.
2
1 0
A=4 0 1
2 2

de uma matriz A e de seguida aplicar a essa matriz o mtodo de eliminao


3
x
y 5
z

2L1 +L3 !L3

3
1 0
x
4 0 1
5
y
0 2 z 2x

2L2 +L3 !L3

1 0
4 0 1
0 0 z

x
y
2x

2y

5 = A0 .

As colunas da matriz A correspondentes s colunas da matriz em escada A0 que contm


os pivots, formam um conjunto de vectores linearmente independente. Qualquer conjunto
f(1; 0; 2); (0; 1; 2); (x; y; z)g em que z 2x 2y 6= 0 constitui uma base de R3 .
(ii) Seja (x; y; z) 2 R3 . Podemos colocar os vectores do conjunto
f(2; 1; 1); ( 4; 2; 1); (x; y; z)g
como colunas de uma matriz A e de seguida aplicar a essa matriz o mtodo de eliminao
de Gauss.
2
3
2
3
2
3
2
4
x
2
4
x
2
4 x
4 0 0 y + x 5 ! 4 0 3 z x 5 = A0 .
A=4 1 2 y 5 1
!
2
2
L2 $L3
L +L2 !L2
x
x
2 1
1
1 z
0
3
z
0
0
y
+
2
2
1
2

L1 +L3 !L3

233

As colunas da matriz A correspondentes s colunas da matriz em escada A0 que contm os


pivots, formam um conjunto de vectores linearmente independente. Logo, qualquer conjunto
f(2; 1; 1); ( 4; 2; 1); (x; y; z)g
em que y +

x
2

6= 0 constitui uma base de R3 .

(iii) Seja (x; y; z) 2 R3 . Podemos colocar os vectores do conjunto


f( 1; 2; 1); (1; 0; 1); (x; y; z)g
como colunas de uma matriz
de Gauss.
2
1
4
2
A=
1

A e de seguida aplicar a
2
3
1 x
1
4
5
0 y
0
!
2L1 +L2 !L2
1 z
0
L1 +L3 !L3

essa matriz o mtodo de eliminao


3
1
x
2 y + 2x 5 = A0 .
0 z+x

As colunas da matriz A correspondentes s colunas da matriz em escada A0 que contm os


pivots, formam um conjunto de vectores linearmente independente. Logo, qualquer conjunto
f( 1; 2; 1); (1; 0; 1); (x; y; z)g
em que z + x 6= 0 constitui uma base de R3 .
24. (i) Seja
S = cos2 t; sen2 t; cos 2t .
O conjunto S linearmente dependente, pois:
cos 2t = cos2 t

sen2 t.

Mas, o conjunto
S 0 = cos2 t; sen2 t
linearmente independente pois se tivermos ;

2 R tais que

cos2 t + sen2 t = 0,
para todo o t 2 R, ento se zermos t = 2 obtemos = 0 e a seguir se zermos t = 0
obtemos = 0. Logo, = = 0. Pelo que, o conjunto S 0 = fcos2 t; sen2 tg uma base de
L(S), pois gera L(S) e linearmente independente. E ento,
dim L(S) = 2.
(ii) Seja
S = 2; sen2 t; cos2 t .
O conjunto S linearmente dependente, pois:
2 = 2 cos2 t + 2 sen2 t.

234

Mas, o conjunto
S 0 = cos2 t; sen2 t
linearmente independente pois se tivermos ;

2 R tais que

cos2 t + sen2 t = 0,
para todo o t 2 R, ento se zermos t = 2 obtemos = 0 e a seguir se zermos t = 0
obtemos = 0. Logo, = = 0. Pelo que, o conjunto S 0 = fcos2 t; sen2 tg uma base de
L(S), pois gera L(S) e linearmente independente. E ento,
dim L(S) = 2.
(iii) Seja
S = et ; e t ; cosh t .
O conjunto S linearmente dependente, pois:
cosh t =

et + e t
.
2

Mas, o conjunto
S 0 = et ; e
linearmente independente pois se tivermos ;
et + e

2 R tais que
t

= 0,

para todo o t 2 R, ento se zermos t = 0 obtemos + = 0 e a seguir se zermos t = 1


obtemos e1 + e 1 = 0. Logo, = = 0. Pelo que, o conjunto S 0 = fet ; e t g uma base
de L(S), pois gera L(S) e linearmente independente. E ento,
dim L(S) = 2.
(iv) Seja
S = 1; t; t2 ; (t + 1)2 .
O conjunto S linearmente dependente, pois:
dim P2 = 3 e S tem 4 vectores.
Mas, o conjunto
S 0 = 1; t; t2
linearmente independente pois trata-se da base cannica de P2 . Logo,
L(S) = P2 e dim L(S) = dim P2 = 3.
25. Seja V o espao linear de todas as funes reais de varivel real. Sejam f; g; h 2 V ,
com f (t) = sen t, g (t) = cos t e h (t) = t. Vejamos que o conjunto ff; g; hg linearmente
independente. Sejam ; ; 2 R tais que
f + g + h = 0.
235

Note que
f + g + h = 0 , f (t) + g (t) + h (t) = 0, para todo o t 2 R ,
,
sen t + cos t + t = 0, para todo o t 2 R.
Para t = 0, t = , t =
8
>
>
>
>
>
<
>
>
>
>
>
:

tem-se respectivamente as seguintes equaes

sen 0 + cos 0 + 0 = 0
sen + cos +
sen

+ cos

=0

=0

8
>
>
>
>
>
<
>
>
>
>
>
:

=0
+
+

=0 ,
2

=0

8
<
:

=0
=0
= 0.

Logo = = = 0, e assim o conjunto ff; g; hg linearmente independente.


Observao. Como ff; gg ff; g; hg, as funes sen t e cos t so linearmente independentes.
26. (i) Podemos colocar os vectores do conjunto f(1; 3); (1; 1)g como colunas de uma
matriz A e de seguida aplicar a essa matriz o mtodo de eliminao de Gauss:
A=

1
3

1
1

3L1 +L2 !L2

1
0

1
4

= A0 .

As colunas da matriz A correspondentes s colunas da matriz em escada A0 que contm


os pivots, formam um conjunto de vectores linearmente independente. Logo, o conjunto
formado pelos vectores das colunas 1 e 2 da matriz A:
f(1; 3); (1; 1)g
linearmente independente. Temos assim, dois vectores de R2 linearmente independentes.
Como a dimenso de R2 2, ento o conjunto B = f(1; 3); (1; 1)g desde logo uma
base de R2 . (No foi preciso vericar se B gera R2 ). Isto , B base de L(B) = R2 e
dim L(B) = dim R2 = 2.
Determinemos agora as coordenadas do vector (0; 1) em relao base
B = f(1; 3); (1; 1)g
de R2 . Isto , queremos encontrar ;

2 R tais que

(0; 1) = (1; 3) + (1; 1).


Formando a matriz aumentada do sistema, tem-se
1
3
Logo,

1 j
1 j
8
<
:

0
1
+

3L1 +L2 !L2

=0

4 =

236

8
<
:

1 j
4 j

1
0

1
4

=
=

1
4

0
1

e assim,
1
1
(1; 3) + (1; 1).
4
4
2
Finalmente e ainda em relao base B de R , o vector cujas coordenadas so (0; 1) nessa
base, dado por:
0(1; 3) + ( 1)(1; 1) = ( 1; 1).
(0; 1) =

(ii) O conjunto S = f(0; 0); (1; 2)g contm o vector nulo, logo o conjunto linearmente
dependente, pelo que no pode ser base de R2 . No entanto, S 0 = f(1; 2)g linearmente
independente e S 0 base de L(S 0 ) = L(S). Logo, dim L(S) = 1.
(iii) O conjunto S = f(2; 4)g no pode ser base de R2 uma vez que tem s um vector e
qualquer base de R2 tem sempre dois vectores (pois dim R2 = 2). No entanto, S = f(2; 4)g
linearmente independente e S base de L(S). Logo, dim L(S) = 1.
(iv) Facilmente se v que o conjunto B = f( 5; 0); (0; 2)g linearmente independente.
Temos assim, dois vectores de R2 linearmente independentes. Como a dimenso de R2 2,
ento o conjunto B = f( 5; 0); (0; 2)g desde logo uma base de R2 . (No foi preciso vericar
se B gera R2 ).
Determinemos agora as coordenadas do vector (0; 1) em relao base
B = f( 5; 0); (0; 2)g
de R2 . Isto , queremos encontrar ;

2 R tais que

(0; 1) = ( 5; 0) + (0; 2).


Facilmente se v que

1
2

= 0. Isto ,

(0; 1) = 0( 5; 0) +

1
2

(0; 2).

Finalmente e ainda em relao base B de R2 , o vector cujas coordenadas so (0; 1) nessa


base, dado por:
0( 5; 0) + ( 1)(0; 2) = (0; 2).
(v) Como a dimenso de R2 2, ento qualquer conjunto de vectores de R2 com mais do
que 2 vectores linearmente dependente. O conjunto S = f(1; 2); (2; 3); (3; 2)g formado
por trs vectores de R2 , logo linearmente dependente e como tal no pode ser uma base
de R2 . No entanto, podemos colocar os vectores do conjunto S = f(1; 2); (2; 3); (3; 2)g
como colunas de uma matriz A e de seguida aplicar a essa matriz o mtodo de eliminao
de Gauss:
1 2 3
1 2
3
A=
!
= A0 .
2
3 2
0
7
4
2L1 +L2 !L2
As colunas da matriz A correspondentes s colunas da matriz em escada A0 que contm
os pivots, formam um conjunto de vectores linearmente independente. Logo, o conjunto
formado pelos vectores das colunas 1 e 2 da matriz A:
B = f(1; 2); (2; 3)g
237

linearmente independente. Temos assim, dois vectores de R2 linearmente independentes.


Como a dimenso de R2 2, ento o conjunto B = f(1; 2); (2; 3)g desde logo uma base
de R2 . (No foi preciso vericar se B gera R2 ).
Determinemos agora as coordenadas do vector (0; 1) em relao base
B = f(1; 2); (2; 3)g
de R2 . Isto , queremos encontrar ;

2 R tais que

(0; 1) =

f(1; 2) + (2; 3)g .

Formando a matriz aumentada do sistema, tem-se


1
2
Logo,

e assim,

2 j
3 j
8
<
:

0
1

2L1 +L2 !L2

+2 =0
7 =

8
<
:

1
0

2 j
7 j

2
7

=
=

0
1

1
7

1
2
(1; 2) + (2; 3).
7
7
2
Finalmente e ainda em relao base B de R , o vector cujas coordenadas so (0; 1) nessa
base, dado por:
0(1; 2) + ( 1)(2; 3) = ( 2; 3).
(0; 1) =

(vi) Bc2 = f(1; 0); (0; 1)g a base cannica de R2 . As coordenadas do vector (0; 1) em
relao base Bc2 so precisamente 0 e 1. Ainda em relao base Bc2 , o vector cujas
coordenadas nessa base so (0; 1) precisamente o vector (0; 1).
27. (i) O conjunto f(1; 2; 3); (0; 0; 0); (0; 1; 2)g contm o vector nulo, logo o conjunto
linearmente dependente, pelo que no pode ser base. Mas,
L (f(1; 2; 3); (0; 0; 0); (0; 1; 2)g) = L (f(1; 2; 3); (0; 1; 2)g)
e facilmente se v que o conjunto f(1; 2; 3); (0; 1; 2)g linearmente independente. Logo,
dim L (f(1; 2; 3); (0; 0; 0); (0; 1; 2)g) = 2
e o conjunto f(1; 2; 3); (0; 1; 2)g uma base de L (f(1; 2; 3); (0; 0; 0); (0; 1; 2)g).
(ii) Facilmente se v que o conjunto f(1; 2; 0); (0; 1; 1)g linearmente independente.
Logo, o conjunto f(1; 2; 0); (0; 1; 1)g uma base de L (f(1; 2; 0); (0; 1; 1)g) e
dim L (f(1; 2; 0); (0; 1; 1)g) = 2.

238

(iii) Podemos colocar os vectores do conjunto f(3; 2; 2); ( 1; 2; 1); (0; 1; 0)g como colunas
de uma matriz A e de seguida aplicar a essa matriz o mtodo de eliminao de Gauss:
2
3
2
2
3
3
3
1 0
3
1 0
3
1
0
4 0 8=3 1 5
4 0 8=3
1 5 = A0 .
A=4 2 2 1 5 2 !
!
5
L +L2 !L2
L +L3 !L3
3 1
8 2
2 1 0
0 5=3 0
0 0
5=8
2
3

L1 +L3 !L3

As colunas da matriz A correspondentes s colunas da matriz em escada A0 que contm


os pivots, formam um conjunto de vectores linearmente independente. Logo, o conjunto
f(3; 2; 2); ( 1; 2; 1); (0; 1; 0)g linearmente independente. Temos assim, trs vectores de R3
linearmente independentes. Como a dimenso de R3 3, ento o conjunto f(3; 2; 2); ( 1; 2; 1); (0; 1; 0)g
desde logo uma base de R3 . Vamos agora escrever o vector ( 1; 1; 2) como combinao
linear dos vectores desta base. Isto , procuremos ; ; 2 R tais que
( 1; 1; 2) = (3; 2; 2) + ( 1; 2; 1) + (0; 1; 0).
Temos ento
2
3
1 0 j
4 2 2 1 j
2 1 0 j

3
1
1 5
2

2
L +L2 !L2
3 1
2
L +L3 !L3
3 1

Logo,

8
3
>
>
>
>
<
>
>
>
>
:

Pelo que
( 1; 1; 2) =

3
1
5=3 5
4=3

3
1 0 j
4 0 8=3 1 j
0 5=3 0 j
=

8
3

3
5

+
5
8

1
5
3

19
8

(3; 2; 2) +

8
>
>
>
>
<
>
>
>
>
:
4
5

5
L +L3 !L3
8 2

3
5

4
5

19
.
5

( 1; 2; 1) +

3
1
4 0 8=3
0 0

0
j
1
j
5=8 j

19
(0; 1; 0).
5

Finalmente e ainda em relao base f(3; 2; 2); ( 1; 2; 1); (0; 1; 0)g de R3 , o vector cujas
coordenadas so ( 1; 1; 2) nessa base, dado por:
( 1)(3; 2; 2) + ( 1; 2; 1) + ( 2)(0; 1; 0) = ( 4; 2; 1).
(iv) Facilmente se v que o conjunto f(1; 1; 1); (0; 1; 1); (0; 0; 1)g linearmente independente. Temos ento trs vectores de R3 linearmente independentes. Como a dimenso de
R3 3, ento o conjunto f(1; 1; 1); (0; 1; 1); (0; 0; 1)g desde logo uma base de R3 . Vamos
agora escrever o vector ( 1; 1; 2) como combinao linear dos vectores desta base. Isto ,
procuremos ; ; 2 R tais que
( 1; 1; 2) = (1; 1; 1) + (0; 1; 1) + (0; 0; 1).
Temos ento:

8
<
:

= 1
+ =1
+ + =

2
239

8
<
:

= 1
=2
= 3.

3
1
5=3 5 .
19=8

Pelo que
( 1; 1; 2) = ( 1)(1; 1; 1) + 2(0; 1; 1) + ( 3)(0; 0; 1).
Finalmente e ainda em relao base B de R2 , o vector cujas coordenadas so ( 1; 1; 2)
nessa base, dado por:
( 1)(1; 1; 1) + (0; 1; 1) + ( 2)(0; 0; 1) = ( 1; 0; 2).
(v) Como a dimenso de R3 3, ento qualquer conjunto de vectores de R3 com mais
do que trs vectores linearmente dependente. O conjunto
f(1; 1; 1); (2; 3; 4); (4; 1; 1); (0; 1; 1)g
formado por quatro vectores de R3 , logo linearmente dependente. Vamos procurar o
nmero mximo de vectores linearmente independentes que, em conjunto, geram
L (f(1; 1; 1); (2; 3; 4); (4; 1; 1); (0; 1; 1)g) .
Podemos colocar os vectores do conjunto f(1; 1; 1); (2; 3; 4); (4; 1; 1); (0; 1; 1)g como linhas de uma A matriz e de seguida aplicar a essa matriz o mtodo de eliminao de Gauss:
2
3
3
2
1 1
1
1 1
1
6 2 3 4 7
6 0 1
6 7
7
6
7
A=6
!
!
4 4 1
1 5 2L1 +L2 !L2 4 0
3 3 5 3L2 +L3 !L3
4L1 +L3 !L3
L2 +L4 !L4
0 1
1
0 1
1
2
3
2
3
2
3
1 1
1
1 1
1
1 1
1
6 0 1 6 7
6 0 1 6 7
6 0 1 6 7
0
6
7
6
7
6
7
!
!
!
5 L3 +L4 !L4 4 0 0 1 5 = A .
1
3L2 +L3 !L3 4 0 0 21 5 1 L3 !L3 4 0 0
21
L2 +L4 !L4
1
0 0
1
0 0 0
0 0
7
L !L4
7 4

As linhas no nulas da matriz em escada A0 so linearmente independentes. Logo, o conjunto


f(1; 1; 1); (0; 1; 6); (0; 0; 1)g formado por trs vectores de R3 , linearmente independentes.
Atendendo a que a dimenso de R3 3, o conjunto
f(1; 1; 1); (0; 1; 6); (0; 0; 1)g

desde logo uma base de R3 . Uma vez que L(A) = L(A0 ) temos ento:
L (f(1; 1; 1); (2; 3; 4); (4; 1; 1); (0; 1; 1)g) = L (f(1; 1; 1); (0; 1; 6); (0; 0; 1)g) = R3 .
Logo,
dim L (f(1; 1; 1); (2; 3; 4); (4; 1; 1); (0; 1; 1)g) = 3.
Vamos agora escrever o vector ( 1; 1; 2) como combinao linear dos vectores da base
f(1; 1; 1); (0; 1; 6); (0; 0; 1)g :
Isto , procuremos ; ;

2 R tais que

( 1; 1; 2) = (1; 1; 1) + (0; 1; 6) + (0; 0; 1).


240

Temos ento:

Pelo que

8
<
:

= 1
+ =1
+6 +

2,

8
<
:

= 1
=2
= 15.

( 1; 1; 2) = ( 1)(1; 1; 1) + 2(0; 1; 6) + ( 15)(0; 0; 1).


Finalmente e ainda em relao base f(1; 1; 1); (0; 1; 6); (0; 0; 1)g de R3 , o vector cujas
coordenadas so ( 1; 1; 2) nessa base, dado por:
( 1)(1; 1; 1) + (0; 1; 6) + ( 2)(0; 0; 1) = ( 1; 0; 5).
(vi) Bc3 = f(1; 0; 0); (0; 1; 0); (0; 0; 1)g a base cannica de R3 . As coordenadas do vector
( 1; 1; 2) em relao base Bc3 so precisamente 1; 1 e 2. Ainda em relao base Bc3 ,
o vector cujas coordenadas nessa base so ( 1; 1; 2) precisamente o vector ( 1; 1; 2).
28. (i) Podemos colocar os vectores do conjunto f(1; 0; 0; 1); (0; 1; 0; 0); (1; 1; 1; 1); (0; 1; 1; 1)g
como colunas de uma matriz e de seguida aplicar a essa matriz o mtodo de eliminao de
Gauss:
2
3
2
3
1 0 1 0
1 0 1 0
6 0 1 1 1 7
6 0 1 1 1 7
6
7
6
7
!
4 0 0 1 1 5 L1 +L4 !L4 4 0 0 1 1 5 .
1 0 1 1
0 0 0 1

Logo, o conjunto f(1; 0; 0; 1); (0; 1; 0; 0); (1; 1; 1; 1); (0; 1; 1; 1)g linearmente independente.
Temos assim, quatro vectores de R4 linearmente independentes. Como a dimenso de R4
4, ento o conjunto f(1; 0; 0; 1); (0; 1; 0; 0); (1; 1; 1; 1); (0; 1; 1; 1)g desde logo uma base de R4
e
dim L (f(1; 0; 0; 1); (0; 1; 0; 0); (1; 1; 1; 1); (0; 1; 1; 1)g) = dim R4 = 4.

(ii) Podemos colocar os vectores do conjunto f(1; 1; 0; 2); (3; 1; 2; 1); (1; 0; 0; 1)g como
colunas de uma matriz e de seguida aplicar a essa matriz o mtodo de eliminao de Gauss:
2
3
2
3
1
3 1
1 3
1
6 1
6 0 2
1 0 7
1 7
6
7
6
7
!
!
4 0
2 0 5 L1 +L2 !L2 4 0 2
0 5 L2 +L3 !L3
5
2L1 +L4 !L4
0
5
1 2 L2 +L4 !L4
2
1 1
2
3
2
3
1 3 1
1 3 1
6 0 2 1 7
6 0 2 1 7
6
7.
6
7
!
!
1 5 32 L3 +L4 !L4 4 0 0
1 5
L2 +L3 !L3 4 0 0
5
L +L4 !L4
0 0 32
0 0 0
2 2

Logo, o conjunto f(1; 1; 0; 2); (3; 1; 2; 1); (1; 0; 0; 1)g linearmente independente e assim
uma base do subespao de R4 :
L (f(1; 1; 0; 2); (3; 1; 2; 1); (1; 0; 0; 1)g)

tendo-se
dim L (f(1; 1; 0; 2); (3; 1; 2; 1); (1; 0; 0; 1)g) = 3.
241

Atendendo ainda ao mtodo de eliminao de Gauss, uma base de R4 que inclui pelo
menos dois vectores do conjunto apresentado:
f(1; 1; 0; 2); (1; 0; 0; 1); (0; 0; 1; 0); (0; 0; 0; 1)g
uma vez que

1
6 1
6
4 0
2

1
0
0
1

0
0
1
0

3
2
0
1
7
6
0 7
0
!6
0 5 ::: 4 0
1
0
|

1 0
1 0
0 1
0 0
{z

car=4

(iii) Podemos colocar os vectores do conjunto

3
0
0 7
7.
0 5
1
}

f(1; 0; 0; 1); (0; 1; 1; 0); (0; 1; 0; 1); (1; 0; 1; 0); (0; 0; 1; 1)g
como colunas de uma
Gauss:
2
1
6 0
A=6
4 0
1
!

L2 +L3 !L3

matriz e de seguida aplicar a essa matriz o mtodo de eliminao de


3
2
3
0 0 1 0
1 0 0 1 0
6 0 1 1 0 0 7
1 1 0 0 7
7
6
7
!
!
5
1 0 1 1
1 1 5 L2 +L3 !L3
L1 +L4 !L4 4 0 1 0
0 0 1
1 1
0 1 0 1
2
3
2
3
1 0 0
1 0
1 0 0 1 0
6 0 1 1
6 0 1 1 0 0 7
0 0 7
6
7
7 = A0 .
! 6
4 0 0
5
1 1 1 5
1 1 1 L3 +L4 !L4 4 0 0
0 0 1
1 1
0 0 0 0 2

As colunas da matriz A correspondentes s colunas da matriz A0 que contm os pivots,


formam um conjunto de vectores linearmente independente. Logo, os vectores das colunas
1; 2; 3 e 5 da matriz A:
f(1; 0; 0; 1); (0; 1; 1; 0); (0; 1; 0; 1); (0; 0; 1; 1)g

so uma base de R4 , por serem quatro vectores linearmente independentes de um espao


linear de dimenso 4. E
dim L (f(1; 0; 0; 1); (0; 1; 1; 0); (0; 1; 0; 1); (0; 0; 1; 1)g) = dim R4 = 4.
(iv) Facilmente se v que o conjunto f(1; 0; 0; 2); (1; 0; 2; 0); (1; 2; 0; 0); (3; 0; 0; 0)g linearmente independente. Temos ento quatro vectores de R4 linearmente independentes.
Como a dimenso de R4 4, ento o conjunto
f(1; 0; 0; 2); (1; 0; 2; 0); (1; 2; 0; 0); (3; 0; 0; 0)g
desde logo uma base de R4 e
dim L (f(1; 0; 0; 2); (1; 0; 2; 0); (1; 2; 0; 0); (3; 0; 0; 0)g) = dim R4 = 4.
(v) Podemos colocar os vectores do conjunto
f(1; 2; 5; 3); (2; 4; 6; 2); (3; 6; 11; 1); (0; 0; 5; 5)g
242

como colunas de uma matriz e de


Gauss:
2
1
2
3
6 2
4
6
A=6
4 5
6 11
3 2
1
2
1 2
6 0 8
6
!
4
3L1 +L2 !L2 4 0
5L1 +L3 !L3
0
0
2L +L !L
1

seguida aplicar a essa matriz o mtodo de eliminao de


3
2
3
0
1
2
3 0
6
0 7
1 5 7
7 ! 6 3 2
7
!
5
4
5 L2 $L4
5
6 11 5 5 3L1 +L2 !L2
5L1 +L3 !L3
5
2
4
6 0
2L1 +L4 !L4
2
3
3
1 2 3 0
3 0
6 0 8 8 5 7
8 5 7
7
6
7 = A0 .
!
5
4 5 21 L2 +L3 !L3 4 0 0 0 15 5
0 0 0 0
0 0

As colunas da matriz A correspondentes s colunas da matriz A0 que contm os pivots,


formam um conjunto de vectores linearmente independente. Logo, os vectores das colunas
1; 2 e 4 da matriz A formam um conjunto linearmente independente:
f(1; 2; 5; 3); (2; 4; 6; 2); (0; 0; 5; 5)g .
Assim, o conjunto f(1; 2; 5; 3); (2; 4; 6; 2); (0; 0; 5; 5)g uma base de
L (f(1; 2; 5; 3); (2; 4; 6; 2); (0; 0; 5; 5)g) ,
tendo-se
dim L (f(1; 2; 5; 3); (2; 4; 6; 2); (0; 0; 5; 5)g) = 3.

Atendendo ainda ao mtodo de eliminao de Gauss, uma base de R4 que inclui pelo
menos dois vectores do conjunto inicial:
f(1; 2; 5; 3); (0; 1; 0; 0); (2; 4; 6; 2); (0; 0; 5; 5)g
uma vez que

1
6 2
6
4 5
3

0
1
0
0

2
4
6
2

3
2
0
1
7
6
0 7
0
!6
5 5 ::: 4 0
5
0
|

(vi) Podemos colocar os vectores do conjunto

0
1
0
0

3
2 0
0 0 7
7.
4 5 5
0 15
{z
}

car=4

f(2; 1; 1; 2); ( 1; 1; 1; 2); (4; 2; 2; 2); (5; 2; 2; 2)g


como colunas de uma matriz e de seguida aplicar a essa matriz o mtodo de eliminao de
Gauss:
3
2
3
2
2
1 4
5
1
1
2
2
6
6 1
1 4
5 7
1
2
2 7
7 ! 6 2
7
!
A=6
4 1 1
2
2 2 5 2L1 +L2 !L2
2
2 5 L1 $L2 4 2
2L1 +L3 !L3
L3 $L4
2
2
2 2
1 1
2
2
L1 +L4 !L4
2
3
2
3
1
1
2
2
1
1
2
2
6 0 1
6 0 1
8
9 7
8
9 7
6
7
6
7 = A0 .
!
!
4
2
6 5 4L2 +L3 !L3 4 0 0
30
30 5
2L1 +L2 !L2 4 0
2L1 +L3 !L3
0 0
0
0
0 0
0
0
L +L !L
1

243

As colunas da matriz A correspondentes s colunas da matriz A0 que contm os pivots,


formam um conjunto de vectores linearmente independente. Logo, os vectores das colunas
1; 2 e 3 da matriz A formam um conjunto linearmente independente:
f(2; 1; 1; 2); ( 1; 1; 1; 2); (4; 2; 2; 2)g .
Assim, o conjunto f(2; 1; 1; 2); ( 1; 1; 1; 2); (4; 2; 2; 2)g uma base de
L (f(2; 1; 1; 2); ( 1; 1; 1; 2); (4; 2; 2; 2)g) ,
tendo-se
dim L (S) = dim L (f(2; 1; 1; 2); ( 1; 1; 1; 2); (4; 2; 2; 2)g) = 3.
Uma base de R4 que inclui pelo menos dois vectores do conjunto
f(2; 1; 1; 2); ( 1; 1; 1; 2); (4; 2; 2; 2)g :
f(2; 1; 1; 2); ( 1; 1; 1; 2); (4; 2; 2; 2); (0; 0; 0; 1)g .
Vejamos que (8; 3; 3; 5) 2 L (S) e determinemos uma base de L (S) que inclua o vector
(8; 3; 3; 5). Isto , procuremos ; ; 2 R tais que
(8; 3; 3; 5) = (2; 1; 1; 2) + ( 1; 1; 1; 2) + (4; 2; 2; 2).
Temos ento:
2

2
6 1
6
4 1
2
!

2L1 +L2 !L2


2L1 +L3 !L3
L1 +L4 !L4

Logo,

Pelo que

1
1
1
2

4
2
2
2

1
6 0
6
4 0
0

1
1
4
0

3
8
3 7
7 !
3 5 L1 $L2
L3 $L4
5

j
j
j
j
2
8
2
0

3
j
3
j 14 7
7
j 11 5
j 0

8
>
>
>
>
<
>
>
>
>
:

1
6 2
6
4 2
1

1
1
2
1

2L1 +L2 !L2


2L1 +L3 !L3
L1 +L4 !L4

2
4
2
2

1
6 0
6
4 0
0

j
j
j
j

3
3
8 7
7
5 5
3

1
1
0
0

2
8
30
0

2L1 +L2 !L2


2L1 +L3 !L3
L1 +L4 !L4

j
j
j
j

3
3
14 7
7 . (*)
45 5
0

=2
=2
=

3
2

3
(8; 3; 3; 5) = 2(2; 1; 1; 2) + 2( 1; 1; 1; 2) + (4; 2; 2; 2).
2
Atendendo a (*), o conjunto
f(2; 1; 1; 2); ( 1; 1; 1; 2); (8; 3; 3; 5)g
uma base de L (S) que inclui o vector (8; 3; 3; 5):
244

Atendendo ainda ao mtodo de eliminao de Gauss, uma base de R4 que inclui pelo
menos dois vectores do conjunto inicial:
f(2; 1; 1; 2); ( 1; 1; 1; 2); (0; 0; 1; 0); (8; 3; 3; 5)g
uma vez que

2
6 1
6
4 1
2

1
1
1
2

3
2
8
2
7
6
3 7
0
!6
5
4
0
3
:::
5
0
|

0
0
1
0

1
1=2
0
0

{z

0
0
4
0

car=4

3
8
7 7
7.
0 5
45
}

29. (i) Podemos colocar os coecientes dos vectores do conjunto


2+t

t2 ; 2t + 2t2 ; t2

como colunas de uma matriz


de Gauss:
2
2
4
1
A=
1
2
!

L1 +L2 !L2
2L1 +L3 !L3

A e de seguida aplicar
3
2
0 0
1
5
4
2 0
1
!
L1 $L3
2
1
2
3
1 2
1
4 0 4
1 5
!
L1 +L2 !L2
0 4
2 2L1 +L3 !L3

a essa matriz o mtodo de eliminao


3
2
1
2 0 5
!
L1 +L2 !L2
0 0
2L1 +L3 !L3
2
3
1 2
1
4 0 4
1 5 = A0 .
0 0
1

As colunas da matriz A correspondentes s colunas da matriz em escada A0 que contm os


pivots, formam um conjunto de vectores linearmente independente. Logo, o conjunto
2+t

t2 ; 2t + 2t2 ; t2 ,

formado por trs vectores de P2 , linearmente independente. Como a dimenso de P2 3,


ento o conjunto
2 + t t2 ; 2t + 2t2 ; t2
desde logo uma base de P2 tendo-se
L

t2 ; 2t + 2t2 ; t2

2+t

= P2

e
dim L

2+t

Vamos agora escrever o vector 1

t2 ; 2t + 2t2 ; t2

t como combinao linear dos vectores da base


2+t

Isto , procuremos ; ;
1

= dim P2 = 3.

t2 ; 2t + 2t2 ; t2 :

2 R tais que
t = (2 + t

t2 ) + (2t + 2t2 ) + ( t2 ).

245

Temos ento:

8
2 =1
>
>
>
>
<
+2 =
>
>
>
>
:
+2

Pelo que

,
= 0,

8
>
>
>
>
<
>
>
>
>
:

1
2

3
4

2.

3
1
(2t + 2t2 ) 2( t2 ).
t = (2 + t t2 )
2
4
Finalmente e ainda em relao base f2 + t t2 ; 2t + 2t2 ; t2 g de P2 , o vector cujas coordenadas so ( 1; 3; 2) nessa base, dado por:
1

t2 ) + 3(2t + 2t2 ) + 2( t2 ) =

( 1)(2 + t

2 + 5t + 5t2 .

(ii) Podemos colocar os coecientes dos vectores do conjunto


t2 ; 1

2t
como colunas de uma matriz
de Gauss:
2
0
1
4
2
0
A=
1
2
2
2
4
0
!
1
L
+L
!L
1
3
3
2
0

2t2 ; 2 + t; 1

4t

A e de seguida aplicar a essa matriz o mtodo de eliminao


3
2
1
2
5
4
4
0
!
L1 $L2
0
1
3
0 1
4
1 2 1 5
!
2L2 +L3 !L3
2
2 21
2
1
0

0 1
1 2
2 0
2
2 0
4 0 1
0 0

3
4
1 5
0
1
2
9
2

1
L +L3 !L3
2 1

3
4
1 5 = A0 .
0

As colunas da matriz A correspondentes s colunas da matriz A0 que contm os pivots,


formam um conjunto de vectores linearmente independente. Logo, o conjunto dos vectores
correspondentes s colunas 1; 2 e 3 da matriz A:
2t

t2 ; 1

2t2 ; 2 + t

uma base de
L

t2 ; 1

2t

2t2 ; 2 + t; 1

4t

Como a dimenso de P2 3, ento o conjunto


2t

t2 ; 1

2t2 ; 2 + t

desde logo uma base de P2 tendo-se


L

2t

t2 ; 1

2t2 ; 2 + t; 1

4t

=L

2t

t2 ; 1

2t2 ; 2 + t

= P2

e
dim L

2t

t2 ; 1

2t2 ; 2 + t; 1

4t

= dim P2 = 3.

Vamos agora escrever o vector 1 t como combinao linear dos vectores da base f2t
Isto , procuremos ; ; 2 R tais que
1

t = (2t

t2 ) + (1
246

2t2 ) + (2 + t).

t2 ; 1

2t2 ; 2 + tg.

Temos ento:

8
>
>
>
>
<
>
>
>
>
:

Pelo que

+2 =1
2 +

2 = 0,

8
>
>
>
>
<
>
>
>
>
:

1
3

1+4

2 .

8
>
>
>
>
<
>
>
>
>
:

1
3

1
3
2
.
3

2
1
1
(2t t2 ) + (1 2t2 ) + (2 + t).
3
3
3
Finalmente e ainda em relao base f2t t2 ; 1 2t2 ; 2 + tg de P2 , o vector cujas coordenadas so ( 1; 3; 2) nessa base, dado por:
1

t=

( 1)(2t

t2 ) + 3(1

2t2 ) + 2(2 + t) = 7

5t2 .

(iii) Podemos colocar os coecientes dos vectores do conjunto


1 + t2 ; t

t2 ; 1

t + 2t2 ; 1 + t

como colunas de uma matriz A e de seguida aplicar a essa matriz o


de Gauss:
2
3
2
3
2
1 0
1 1
1 0
1
1
4 0 1
1 1 5
1 1 5
!
! 4
A=4 0 1
L1 +L3 !L3
L2 +L3 !L3
1
1 2 0
0
1 1
1

mtodo de eliminao
1 0
0 1
0 0

3
1 1
1 1 5 = A0 .
0 0

As colunas da matriz A correspondentes s colunas da matriz A0 que contm os pivots,


formam um conjunto de vectores linearmente independente. Logo, o conjunto dos vectores
correspondentes s colunas 1; 2 da matriz A:
1 + t2 ; t

t2

uma base de
L

1 + t2 ; t

t2 ; 1

t + 2t2 ; 1 + t

tendo-se
L

1 + t2 ; t

t2 ; 1

t + 2t2 ; 1 + t

1 + t2 ; t

=L

t2

e
dim L

1 + t2 ; t

t2 ; 1

t + 2t2 ; 1 + t

= dim L

(iv) Facilmente se v que o conjunto f 1 + 2t + t2 ; 2


Logo, ele prprio uma base de

1 + t2 ; t

= 2.

tg linearmente independente.

1 + 2t + t2 ; 2

dim L

1 + 2t + t2 ; 2

= 2.

e tem-se

247

t2

(v) Podemos colocar os coecientes dos vectores do conjunto


1 + 2t

t2 ; 3 + t2 ; 5 + 4t

t2 ; 2 + 2t

t2

como colunas de uma matriz A e de seguida aplicar a essa matriz o mtodo de eliminao
de Gauss:
2
2
3
3
1 3 5
2
1 3
5
2
4 0
2 5
6
6 6 51 !
A=4 2 0 4
!
2L1 +L2 !L2
1 1
1
1
0 4
4
3 6 L2 !L2
L1 +L3 !L3
2
2
3
3
1 3
5
2
1 3
5
2
4 0
1
1 1 5
1
1 1 5 = A0 .
! 4 0
!
1
4L
+L
!L
2
3
3
L !L2
6 2
0 4
4
3
0 0
0
1

As colunas da matriz A correspondentes s colunas da matriz A0 que contm os pivots,


formam um conjunto de vectores linearmente independente. Logo, o conjunto dos vectores
correspondentes s colunas 1; 2 e 4 da matriz A:
t2 ; 3 + t2 ; 2 + 2t

1 + 2t

t2

uma base de
L

1 + 2t

t2 ; 3 + t2 ; 5 + 4t

t2 ; 2 + 2t

t2

Como a dimenso de P2 3, ento o conjunto


t2 ; 3 + t2 ; 2 + 2t

1 + 2t

t2

desde logo uma base de P2 tendo-se


L

1 + 2t
=L

t2 ; 3 + t2 ; 5 + 4t

1 + 2t

t2 ; 2 + 2t

t2 ; 3 + t2 ; 2 + 2t

t2

t2

= P2

e
dim L

1 + 2t

t2 ; 3 + t2 ; 5 + 4t

Vamos agora escrever o vector 1

1
Temos ento:

t2

= dim P2 = 3.

t como combinao linear dos vectores da base

1 + 2t
Isto , procuremos ; ;

t2 ; 2 + 2t

t2 ; 3 + t2 ; 2 + 2t

t2 :

2 R tais que

t = (1 + 2t
2

1 3
4 2 0
1 1

t2 ) + (3 + t2 ) + ( 2 + 2t
32
2
2 54
1

3
1
5 = 4 1 5.
0

t2 ).

Aplicando ento o mtodo de eliminao de Gauss matriz aumentada do sistema anterior,


temos:
2
3
2
3
1 3
2 j 1
1 3
2 j 1
4 2 0 2 j
4 0
1 5
6 6 j
3 51 !
!
2L1 +L2 !L2
L !L2
3 2
1 1
1 j 0
0 4
3 j 1
L1 +L3 !L3
248

Logo,

1
4 0
!
1
L !L2
3 2
0

2
3
1
1
4 0
1 5
!
2L2 +L3 !L3
1
0

2 j
2 j
3 j

3
2
4

8
>
>
>
>
<
>
>
>
>
:

Pelo que
1

1
t = (1 + 2t
2

t2 ) +

2 j
2 j
1 j

1
2

1.
(3 + t2 ) + ( 1) ( 2 + 2t

Finalmente e ainda em relao base f1 + 2t t2 ; 3 + t2 ; 2 + 2t


coordenadas so ( 1; 3; 2) nessa base, dado por:
( 1)(1 + 2t

3
1
1 5.
1

1
2

1
2

3
2
0

t2 ) + 3(3 + t2 ) + 2( 2 + 2t

t2 ).

t2 g de P2 , o vector cujas

t2 ) = 4 + 2t + 2t2 .

(vi) O conjunto f1; t; t2 g a base cannica de P2 . As coordenadas do vector 1+3t+2t2


em relao a essa base so precisamente 1; 3 e 2. Ainda em relao base f1; t; t2 g, o vector
cujas coordenadas nessa base so ( 1; 3; 2) precisamente o vector 1 + 3t + 2t2 .
30. Como o espao linear M2 2 (R) tem dimenso 4, ento para vericar que as matrizes
1 1
0 0

0 0
1 1

1 0
0 1

0 1
1 1

formam uma base de M2 2 (R) basta ver que so linearmente independentes. Sejam ; ; ; 2
R tais que
1 1
0 0
1 0
0 1
+
+
+
= 0,
0 0
1 1
0 1
1 1
onde 0 a matriz nula

0 0
. Queremos provar que
0 0

Temos ento:
+
+
isto ,

ou ainda

+
+
8
>
>
<

1
6 1
6
4 0
0

0
0
1
1

>
>
:

1
0
0
1

0 0
0 0

+ =0
+ =0
+ =0
+ + = 0,
32
0
6
1 7
76
5
1 4
1
249

3
0
7 6 0 7
7 = 6 7.
5 4 0 5
0

= 0.

Aplicando ento o mtodo


homogneo anterior, temos:
2
1 0
6 1 0
6
4 0 1
0 1
2

1
6 0
! 6
L2 $L3 4 0
0

0
1
0
1

1
0
1
1

de eliminao de Gauss matriz dos coecientes do sistema


3
0
1 7
7
1 5
1

1
0
0
1

3
0
1 7
7
1 5
1

L1 +L2 !L2

L2 +L4 !L4

1
6 0
6
4 0
0

1
6 0
6
4 0
0

0
1
0
0

0
0
1
1

1
0
1
1

3
0
1 7
7 !
1 5 L2 $L3
1

1
1
0
1

3
0
1 7
7
!
1 5 L3 +L4 !L4
0

1
6 0
6
4 0
0

0
1
0
0

3
0
1 7
7.
1 5
1

1
0
1
0

Logo, a nica soluo do sistema : ( ; ; ; ) = (0; 0; 0; 0). Assim, o conjunto


1 1
0 0

0 0
1 1

1 0
0 1

0 1
1 1

uma base de M2 2 (R).


1 3
0 11
2
5
4 1
3
2
;
;
,
;
. Seja W um
1 2
5 3
3 1
1 5
2 3
subespao de M2 2 (R) gerado por S. Determinemos uma base para W que inclua vectores
de S.
Sejam 1 ; 2 ; 3 ; 4 ; 5 2 R tais que
31. Seja S =

0 0
0 0

Temos ento:
2
1
0
6 3 11
6
4 1
5
2
3

1 3
1 2

2
5
3
1

3
3
2 7
7
2 5
3

4
1
1
5

0 11
5 3

3L1 +L2 !L2


L1 +L3 !L3
2L1 +L4 !L4

5
L +L3 !L3
11 2
3
L +L4 !L4
11 2

2
3

1 0
6 0 11
6
4 0
5
0 3

1 0
6 0 11
6
4 0 0
0 0

2
11
0
0

5
1

2
11
5
3
4
11
0
0

4
11
5
3
3
3
11 7
7
0 5
0

4 1
1 5
3
3
11 7
7
5 5
3

3
2

2
3

5
L +L3 !L3
11 2
3
L +L4 !L4
11 2

pelo que sendo as 2 primeiras colunas da matriz em escada anterior independentes, o conjunto
de matrizes
1 3
0 11
;
1 2
5 3
uma base de W , atendendo tambm a que
2
3

5
1

4 1
1 5

3
2

2
3

2L
250

1 3
1 2

0 11
5 3

32. A dimenso do espao linear M3 2 (R) 6. Assim, para encontrar uma base de
M3 2 (R), basta encontrar 6 matrizes do tipo 3 2 que sejam linearmente independentes. O
seguinte conjunto de 6 matrizes do tipo 3 2:
82
3 2
3 2
3 2
3 2
3 2
39
0 1
0 0
0 0
0 0
0 0 =
< 1 0
4 0 0 5;4 0 0 5;4 1 0 5;4 0 1 5;4 0 0 5;4 0 0 5
:
;
0 0
0 0
0 0
0 0
1 0
0 1
linearmente independente. Logo, uma base de M3 2 (R). (Chama-se a esta base, a base
cannica de M3 2 (R).)
33. (i) Uma matriz diagonal do tipo 3
2
3
a 0 0
4 0 b 0 5
0 0 c

E tem-se

3 tem a seguinte forma:


com a; b; c 2 R.

3
2
3
2
3
2
3
a 0 0
1 0 0
0 0 0
0 0 0
4 0 b 0 5 = a4 0 0 0 5 + b4 0 1 0 5 + c4 0 0 0 5.
0 0 c
0 0 0
0 0 0
0 0 1

Isto , o subespao formado por todas as matrizes diagonais do tipo 3


conjunto
82
3 2
39
3 2
0 0 0
0 0 0 =
< 1 0 0
D = 4 0 0 0 5;4 0 1 0 5;4 0 0 0 5 .
;
:
0 0 0
0 0 1
0 0 0

3, gerado pelo

Alm disso, este conjunto linearmente independente. Temos ento que o conjunto D uma
base do subespao formado por todas as matrizes diagonais do tipo 3 3. Logo, o subespao
tem dimenso 3.
(ii) Uma matriz simtrica do tipo 3
2
3
a b c
4 b d e 5
c e f

E tem-se

3 tem a seguinte forma:


com a; b; c; d; e; f 2 R.

3
2
3
2
3
2
a b c
1 0 0
0 1 0
4 b d e 5 = a4 0 0 0 5 + b4 1 0 0 5 + c4
c e f
0 0 0
0 0 0
2
3
2
3
2
0 0 0
0 0 0
0 0
4
5
4
5
4
+d 0 1 0 + e 0 0 1 + f 0 0
0 0 0
0 1 0
0 0

Isto , o subespao formado


conjunto
82
3 2
0 1
< 1 0 0
S = 4 0 0 0 5;4 1 0
:
0 0 0
0 0

3
0 0 1
0 0 0 5+
1 0 0
3
0
0 5.
1

por todas as matrizes simtricas do tipo 3

3, gerado pelo

3 2
3 2
3 2
3 2
39
0
0 0 1
0 0 0
0 0 0
0 0 0 =
0 5;4 0 0 0 5;4 0 1 0 5;4 0 0 1 5;4 0 0 0 5
;
0
1 0 0
0 0 0
0 1 0
0 0 1
251

Alm disso, este conjunto linearmente independente. Temos ento que o conjunto S uma
base do subespao formado por todas as matrizes simtricas do tipo 3 3. Logo, o subespao
tem dimenso 6.
34. (i)
3
6

A=

1
2

2L1 +L2 !L2

3 1
0 0

= A0 .

As colunas da matriz A correspondentes s colunas da matriz A0 que contm os pivots,


formam um conjunto de vectores linearmente independente. Logo,
C(A) = L (f(3; 6)g)
e o conjunto f(3; 6)g uma base de C(A). Por outro lado,
L(A) = L (f(3; 1)g) ,
e o conjunto f(3; 1)g uma base de L(A). Desta forma:
carA = dim C(A) = dim L(A) = 1.
Por denio:
N (A) = u 2 R2 : Au = 0 .
Temos ento, pelo mtodo de eliminao de Gauss,
Au = 0 , A0 u = 0.
A equao
3 1
0 0

u1
u2

0
0

equivalente equao
3u1 + u2 = 0.
Logo,
N (A) = f(u1 ; 3u1 ) : u1 2 Rg = L (f(1; 3)g) .
O conjunto S = f(1; 3)g linearmente independente. Como S linearmente independente
e gera N (A), temos ento que S uma base de N (A) e:
nulA = dim N (A) = 1.
(ii)
A=

3 0
1 0

6 0
2 0

1
L +L2 !L2
3 1

3 0
0 0

6 0
0 0

= A0 .

As colunas da matriz A correspondentes s colunas da matriz A0 que contm os pivots,


formam um conjunto de vectores linearmente independente. Logo,
C(A) = L (f(3; 1)g)
252

e o conjunto f(3; 1)g uma base de C(A). Por outro lado,


L(A) = L (f(3; 0; 6; 0)g) ,
e o conjunto f(3; 0; 6; 0)g uma base de L(A). Desta forma:
carA = dim C(A) = dim L(A) = 1.
Por denio:
N (A) = u 2 R4 : Au = 0 .
Temos ento, pelo mtodo de eliminao de Gauss,
Au = 0 , A0 u = 0.
A equao
3 0
0 0

6 0
0 0

equivalente equao
3u1

3 2
0
u1
6 u2 7 6 0
7 6
6
4 u3 5 = 4 0
u4
0
2

3
7
7
5

6u3 = 0,

ou seja a
u1 = 2u3 .
Logo,
N (A) = f(2u3 ; u2 ; u3 ; u4 ) : u2 ; u3 ; u4 2 Rg .
Como
(2u3 ; u2 ; u3 ; u4 ) = u3 (2; 0; 1; 0) + u2 (0; 1; 0; 0) + u4 (0; 0; 0; 1),
tem-se:
N (A) = L (f(2; 0; 1; 0); (0; 1; 0; 0); (0; 0; 0; 1)g) .
O conjunto S = f(2; 0; 1; 0); (0; 1; 0; 0); (0; 0; 0; 1)g linearmente independente. Como S
linearmente independente e gera N (A), temos ento que S uma base de N (A) e:
nulA = dim N (A) = 3.
(iii)

3
0 1 0 0
A = 4 0 0 1 0 5.
0 0 0 1

As colunas da matriz A que contm os pivots, formam um conjunto de vectores linearmente


independente. Logo,
C(A) = L (f(1; 0; 0); (0; 1; 0); (0; 0; 1)g)
e o conjunto f(1; 0; 0); (0; 1; 0); (0; 0; 1)g uma base de C(A). Por outro lado,
L(A) = L (f(0; 1; 0; 0); (0; 0; 1; 0); (0; 0; 0; 1)g) ,
253

e o conjunto f(0; 1; 0; 0); (0; 0; 1; 0); (0; 0; 0; 1)g uma base de L(A). Desta forma:
carA = dim C(A) = dim L(A) = 3.
Por denio:
N (A) = u 2 R4 : Au = 0 .
A equao

equivalente ao sistema

3 2
u1
0
0 1 0 0 6
7
6
4 0 0 1 0 5 6 u2 7 = 6 0
4 u3 5 4 0
0 0 0 1
u4
0
2

8
< u2 = 0
u3 = 0
:
u4 = 0.

Logo,

3
7
7
5

N (A) = f(u1 ; 0; 0; 0) : u1 2 Rg = L (f(1; 0; 0; 0)g) .


O conjunto S = f(1; 0; 0; 0)g linearmente independente. Como S linearmente independente e gera N (A), temos ento que S uma base de N (A) e:
nulA = dim N (A) = 1.
(iv)
2

1 1
4
1 2
A=
0 1

3
2
2
1 1
5
4
1
0 3
!
L1 +L2 !L2
1
0 1

3
2
1 5
1

1
L +L3 !L3
3 2

1 1
4 0 3
0 0

3
2
1 5 = A0 .
2
3

As colunas da matriz A correspondentes s colunas da matriz A0 que contm os pivots,


formam um conjunto de vectores linearmente independente. Logo,
C(A) = L (f(1; 1; 0); (1; 2; 1); ( 2; 1; 1)g)
e o conjunto f(1; 1; 0); (1; 2; 1); ( 2; 1; 1)g uma base de C(A). Por outro lado,
L(A) = L (f(1; 1; 2); ( 1; 2; 1); (0; 1; 1)g) = L

(1; 1; 2); (0; 3; 1); (0; 0;

e quer o conjunto f(1; 1; 2); ( 1; 2; 1); (0; 1; 1)g ; quer o conjunto


(1; 1; 2); (0; 3; 1); (0; 0;

2
) ,
3

so bases para L(A). Desta forma:


carA = dim C(A) = dim L(A) = 3.
Por denio:
N (A) = u 2 R3 : Au = 0 .
254

2
)
3

Como se tem sempre:


no de colunas de A = carA + nulA,
ento
N (A) = f0g
e
nulA = dim N (A) = 0.
Alternativamente poderamos vericar que se tem mesmo
N (A) = f0g .
Pelo mtodo de eliminao de Gauss, temos
Au = 0 , A0 u = 0.
A equao

equivalente ao sistema

ou seja a

1 1
4 0 3
0 0

3 2 3
32
0
u1
2
1 5 4 u2 5 = 4 0 5
2
0
u3
3

8
< u1 + u2 2u3 = 0
3u2 u3 = 0
: 2
u =0
3 3
u1 = u2 = u3 = 0.

Logo,
N (A) = f(0; 0; 0)g
e como tal
nulA = dim N (A) = 0.
(v)

1
6 0
A=6
4 0
0

0
1
0
0

3
0
0 7
7.
1 5
0

As colunas da matriz A que contm os pivots, formam um conjunto de vectores linearmente


independente. Logo,
C(A) = L (f(1; 0; 0; 0); (0; 1; 0; 0); (0; 0; 1; 0)g)
e o conjunto f(1; 0; 0; 0); (0; 1; 0; 0); (0; 0; 1; 0)g uma base de C(A). Por outro lado,
L(A) = L (f(1; 0; 0); (0; 1; 0); (0; 0; 1)g) = R3 ,
e o conjunto f(1; 0; 0); (0; 1; 0); (0; 0; 1)g uma base de L(A). Desta forma:
carA = dim C(A) = dim L(A) = 3.
255

Por denio:
N (A) = u 2 R3 : Au = 0 .
A equao

1
6 0
6
4 0
0

equivalente ao sistema

0
1
0
0

3
3 2 3
0 2
u1
0
7
0 74
5
4
u2 = 0 5
1 5
u3
0
0
8
< u1 = 0
u2 = 0
:
u3 = 0.

Logo,

N (A) = f(0; 0; 0)g


(vi)

3
1 3 0 2
A=4 0 2 2 0 5
1 3 0 2

e nulA = dim N (A) = 0.

L1 +L3 !L3

3
1 3 0 2
4 0 2 2 0 5 = A0 .
0 0 0 0

As colunas da matriz A correspondentes s colunas da matriz A0 que contm os pivots,


formam um conjunto de vectores linearmente independente. Logo,
C(A) = L (f( 1; 0; 1); (3; 2; 3)g)
e o conjunto f( 1; 0; 1); (3; 2; 3)g uma base de C(A). Por outro lado,
L(A) = L(A0 ) = L (f( 1; 3; 0; 2); (0; 2; 2; 0)g) ,
e o conjunto f( 1; 3; 0; 2); (0; 2; 2; 0)g uma base de L(A). Desta forma:
carA = dim C(A) = dim L(A) = 2.
Por denio:
N (A) = u 2 R4 : Au = 0 .
Temos ento, pelo mtodo de eliminao de Gauss,
Au = 0 , A0 u = 0.
A equao

equivalente ao sistema

3 2
u1
0
1 3 0 2 6
7
6
4 0 2 2 0 5 6 u2 7 = 6 0
4 u3 5 4 0
0 0 0 0
u4
0
2

u1 + 3u2 + 2u4 = 0
2u2 + 2u3 = 0

256

3
7
7
5

ou seja a
u1 = 3u2 + 2u4
u 3 = u2 .
Logo,
N (A) = f(3u2 + 2u4 ; u2 ; u2 ; u4 ) : u2 ; u4 2 Rg .
Como
(3u2 + 2u4 ; u2 ; u2 ; u4 ) = (3u2 ; u2 ; u2 ; 0) + (2u4 ; 0; 0; u4 ) = u2 (3; 1; 1; 0) + u4 (2; 0; 0; 1),
tem-se:
N (A) = L (f(3; 1; 1; 0); (2; 0; 0; 1)g) .
O conjunto S = f(3; 1; 1; 0); (2; 0; 0; 1)g linearmente independente. Como S linearmente
independente e gera N (A), temos ento que S uma base de N (A) e:
nulA = dim N (A) = 2.
(vii)
2
1
6 2
A=6
4 3
1

2
3
4
1

3
2
1
1

3
1
0 7
7
1 5
1

2L1 +L2 !L2


3L1 +L3 !L3
L1 +L4 !L4

1
6 0
6
4 0
0

2
1
2
1

3
4
8
4

3
1
2 7
7
4 5
2

2L2 +L3 !L3


L2 +L4 !L4

1
6 0
6
4 0
0

2
1
0
0

3
4
0
0

3
1
2 7
7 = A0 .
0 5
0

As colunas da matriz A correspondentes s colunas da matriz A0 que contm os pivots,


formam um conjunto de vectores linearmente independente. Logo,
C(A) = L (f(1; 2; 3; 1); (2; 3; 4; 1)g)
e o conjunto f(1; 2; 3; 1); (2; 3; 4; 1)g uma base de C(A). Por outro lado,
L(A) = L (f(1; 2; 3; 1); (0; 1; 4; 2)g) ,
e o conjunto f(1; 2; 3; 1); (0; 1; 4; 2)g uma base de L(A). Desta forma:
carA = dim C(A) = dim L(A) = 2.
Por denio:
N (A) = u 2 R4 : Au = 0 .
Temos ento, pelo mtodo de eliminao de Gauss,
Au = 0 , A0 u = 0.
A equao

1
6 0
6
4 0
0

2
1
0
0

3
4
0
0

32
1
u1
7
6
2 7 6 u2
0 5 4 u3
0
u4
257

3
0
7 6 0 7
7=6 7
5 4 0 5
0

equivalente ao sistema
u1 + 2u2 + 3u3 u4 = 0
u2 4u3 + 2u4 = 0
ou seja a
u1 =
u2 =

2u2 3u3 + u4
4u3 + 2u4

e ainda a
u1 = 5u3 3u4
u2 = 4u3 + 2u4 .
Logo,
N (A) = f(5u3

3u4 ; 4u3 + 2u4 ; u3 ; u4 ) : u3 ; u4 2 Rg .

Como
(5u3

3u4 ; 4u3 + 2u4 ; u3 ; u4 ) = (5u3 ; 4u3 ; u3 ; 0) + ( 3u4 ; 2u4 ; 0; u4 )


= u3 (5; 4; 1; 0) + u4 ( 3; 2; 0; 1),

tem-se:
N (A) = L (f(5; 4; 1; 0); ( 3; 2; 0; 1)g) .
O conjunto S = f(5; 4; 1; 0); ( 3; 2; 0; 1)g linearmente independente. Como S linearmente independente e gera N (A), temos ento que S uma base de N (A) e:
nulA = dim N (A) = 2.
35. Sejam U e V subespaos de W tais que dim U = 4; dim V = 5 e dim W = 7. Tem-se
dim (U \ V ) = dim U + dim V

dim (U + V ) = 9

dim (U + V ) :

Como U + V subespao de W , tem-se


5 = dim V

dim (U + V )

dim W = 7

e assim dim (U + V ) 2 f5; 6; 7g. Logo,


dim (U \ V ) 2 f2; 3; 4g :
36. Determine bases e calcule as dimenses de U + V e U \ V , dizendo em que casos
U + V a soma directa U V (determine-a) dos subespaos U e V .
(i) Em R3 , considere os subespaos:
U = L (f(1; 1; 1); (0; 1; 1)g)

e V = L (f(1; 1; 2); ( 1; 1; 1)g) .

Logo, U + V = L (U [ V ) = L (f(1; 1; 1); (0; 1; 1); (1; 1; 2); ( 1; 1; 1)g). Facilmente se verica que f(1; 1; 1); (0; 1; 1); ( 1; 1; 1)g uma base de U + V , ou melhor de R3 . Logo,
dim (U + V ) = 3 e
dim(U \ V ) = dim U + dim V

dim (U + V ) = 2 + 2
258

3 = 1.

Seja (x; y; z) 2 U . Tem-se


2
2
3
3
1 0 j x
1 0 j
x
4 1 1 j y 5
4 0 1 j x+y 5
!
L1 +L2 !L2
1 1 j z
0 1 j z x
L1 +L3 !L3

L2 +L3 !L3

Logo

U = (x; y; z) 2 R3 : z
Seja (x; y; z) 2 V . Tem-se
2
2
3
1
1 j x
1
4 1 1 j y 5
4
0
!
L1 +L2 !L2
2 1 j z
0
2L1 +L3 !L3

Logo

2x

3
1 j
x
2 j y x 5
3 j z 2x

V = (x; y; z) 2 R3 : 2z

3
1 0 j
x
4 0 1 j
5.
x+y
0 0 j z 2x y

y=0 .

3
L +L3 !L3
2 2

3y

1
4 0
0

3
1 j
x
5.
2 j
y x
1
3
0 j z 2y 2x

x=0 .

Deste modo
U \ V = (x; y; z) 2 R3 : z

2x

y = 0 e 2z

3y

x = 0 = L (f(1; 3; 5)g)

e como tal, f(1; 3; 5)g uma base de U \ V , tendo-se dim (U \ V ) = 1


Neste caso, como U \ V 6= f0g ento U + V no a soma directa dos subespaos U e V .
(ii) Sejam U = f(x; y; z) 2 R3 : x + y z = 0 e x + y = 0g ; V = L (f(1; 1; 1)g).
Tem-se (1; 1; 1) 2
= U pois 1 + 1 1 6= 0. Logo
U \ V = f0g

dim (U \ V ) = 0.

Por outro lado, como


U = ( y; y; 0) 2 R3 : y 2 R = L (f( 1; 1; 0)g) ,
tem-se
U + V = L (f( 1; 1; 0); (1; 1; 1)g)
e sendo f( 1; 1; 0); (1; 1; 1)g uma base de U + V , dim (U + V ) = 2.
Alm disso, como U \ V = f0g,
U +V =U

V = L (f( 1; 1; 0); (1; 1; 1)g) .

(iii) Em R3 , considere os subespaos:


U = L (f(1; 0; 1); ( 1; 1; 2)g)

e V = f(x; y; z) : x + y + 3z = 0g .

Seja v 2 U , ento
v = (1; 0; 1) + ( 1; 1; 2) = (
com ;

; ;

2 R. Para que v esteja tambm em V preciso que:


+

+ 3 ( + 2 ) = 0.
259

+ 2 ),

isto ,
4 +6 =0 ,

3
.
2

Assim,
5
1
; ;
2
2

v = (1; 0; 1) + ( 1; 1; 2) =

5
1
; 1;
2
2

Logo,
5
1
; 1;
2
2

U \V =
5
; 1; 12
2

e como tal,
Tem-se

2R

5
1
; 1;
2
2

=L

uma base de U \ V , tendo-se dim (U \ V ) = 1


V = L (f( 1; 1; 0); ( 3; 0; 1)g) .

Logo,
U + V = L (U [ V ) = L (f(1; 0; 1); ( 1; 1; 2); ( 1; 1; 0); ( 3; 0; 1)g) :
Facilmente se verica que f(1; 0; 1); ( 1; 1; 2); ( 1; 1; 0)g uma base de U + V , ou melhor de
R3 . Logo, dim (U + V ) = 3:
Neste caso, como U \ V 6= f0g ento U + V no a soma directa dos subespaos U e V .
(iv) Em R3 , considere os subespaos:
U = (x; y; z) 2 R3 : x = y = z

e V = (x; y; z) 2 R3 : x = 0 .

Tem-se U = L (f(1; 1; 1)g) e V = L (f(0; 1; 0); (0; 0; 1)g).


Como f(1; 1; 1); (0; 1; 0); (0; 0; 1)g uma base de U + V = L (U [ V ) ento
V = R3 .

dim (U + V ) = 3 e U + V = U
Como U \ V = f0g ento dim (U \ V ) = 0.
(v) Em P2 , considere os subespaos:
U =L

1 + t; 1

t2

e V = a0 + a1 t + a2 t2 2 P2 : a2

Seja p (t) 2 U . Ento existem ;

a1 + a0 = 0 .

2 R tais que

p (t) = a0 + a1 t + a2 t2 =

(1 + t) +

t2 .

Atendendo a
2

1
4 1
0

3
1 j a0
0 j a1 5
1 j a2

Logo, tem-se

L1 +L2 !L2

1
4 0
0

3
1 j
a0
1 j a1 a0 5
1 j
a2

L2 +L3 !L3

1
4 0
0

U =V
pelo que
U +V =U =V

e U \V =U =V.
260

3
1 j
a0
5.
1 j
a1 a0
0 j a2 a1 + a0

Assim, f1 + t; 1

t2 g uma base de U; de V , de U + V e de U \ V , tendo-se


dim (U + V ) = dim (U \ V ) = 2.

Neste caso, como U \ V 6= f0g ento U + V no a soma directa dos subespaos U e V .


(vi) Em P3 , considere os subespaos:
U =L

1 + t; 1

t3

1 + t + t2 ; t

e V =L

t3 ; 1 + t + t3

Logo
U + V = L (U [ V ) = L

t3 ; 1 + t + t2 ; t

1 + t; 1

t3 ; 1 + t + t3

Vejamos quais dos vectores do conjunto


1 + t; 1

t3 ; 1 + t + t2 ; t

so linearmente independentes. Coloquemos ento


unas de uma matriz:
2
3
2
1 1 1 0 1
1 1 1
6 1 0 1 1 1 7
6 0
1 0
7
6
A=6
4 0 0 1 0 0 5 L1 +L!
4
0 0 1
2 !L2
0
1 0
1 1
0
1 0

t3 ; 1 + t + t3
os coecientes desses vectores como col0
1
0
1

3
1
0 7
7
0 5
1

L2 +L4 !L4

1
6 0
6
4 0
0

1
1
0
0

1
0
1
0

0
1
0
2

3
1
0 7
7 = A0 . (*)
0 5
1

As colunas da matriz A correspondentes s colunas da matriz A0 que contm os pivots,


formam um conjunto de vectores linearmente independente. Logo, o conjunto
1 + t; 1

t3 ; 1 + t + t2 ; t

t3

uma base de U + V , tendo-se dim (U + V ) = 4 e deste modo U + V = P3 .


Por outro lado, tambm se conclui de (*) que o conjunto
t3

1 + t; 1
base de
2
1 0
6 0 1
6
4 1 0
0
2

U , tendo-se dim U = 2, e como


2
3
3
1 0
1
1
6 0 1
0 7
0 7
7
6
7
!
!
0 5 L1 +L3 !L3 4 0 0
1 5 2L2 +L4 !L4
L1 +L3 !L3
0
2 1
1

o conjunto

1 + t + t2 ; t
base de V , tendo-se dim V = 3.
Logo,
dim(U \ V ) = dim U + dim V
2
6
6
4

1
6 0
6
4 0
0

0
1
0
0

3
1
0 7
7
!
1 5 L3 +L4 !L4
1

1
6 0
6
4 0
0

0
1
0
0

3
1
0 7
7
1 5
0

t3 ; 1 + t + t3

dim (U + V ) = 2 + 3

4 = 1.

Neste caso, como U \ V 6= f0g ento U + V no a soma directa dos subespaos U e V .


Determinemos U \ V . Seja p (t) = a0 + a1 t + a2 t2 + a3 t3 2 U . Tem-se
3
2
3
2
3
1 1 j a0
1 1 j
a0
1 1 j
a0
6 0
6 0
7
1 0 j a1 7
1 j a1 a0 7
1 j
a1 a0
7
6
7
6
7.
!
!
5
4
5
4
5
0 0 j a2
0 0 j
a2
0 0 j
a2
L1 +L2 !L2
L2 +L4 !L4
0
1 j a3
0
1 j
a3
0 0 j a3 + a0 a1
261

Logo
U = a0 + a1 t + a2 t2 + a3 t3 2 P3 : a2 = 0 e a3 + a0

Seja q (t) = a0 + a1 t + a2 t2 + a3 t3 2 V . Tem-se


2
2
3
1 0
1 0 1 j a0
6 0 1
6 1 1 1 j a1 7
6
6
7
4 0 0
4 1 0 0 j a2 5 L1 +L!
2 !L2
L1 +L3 !L3
0
1
0
1 1 j a3
2
2
3
1 0 1 j
a0
6
6 0 1 0 j
7
a1 a0
6
7
!
! 6
5 L3 +L4 !L4 4
1 j
a2 a0
L2 +L4 !L4 4 0 0
0 0 1 j a1 a0 + a3

1
0
1
1
1
0
0
0

0
1
0
0

a1 = 0 .

3
j
a0
j a1 a0 7
7
!
j a2 a0 5 L2 +L4 !L4
j
a3
1
0
1
0

Logo

V = a0 + a1 t + a2 t2 + a3 t3 2 P3 : a1 + a2

3
j
a0
7
j
a1 a0
7.
5
j
a2 a0
j a1 + a2 2a0 + a3

2a0 + a3 = 0 .

Deste modo
U \V = a0 + a1 t + a2 t2 + a3 t3 2 P3 : a2 = 0 e a0

a1 + a3 = 0 e
02
0
0
2
3
@
4
1
1
= a0 + a1 t + a2 t + a3 t 2 P3 : (a0 ; a1 ; a2 ; a3 ) 2 N
2 1

Atendendo
2
0
0
4 1
1
2 1
tem-se

a que

3
2
1 0
0
5
4
0 1
1
!
2L2 +L3 !L3
1 1
0

3
2
0 1 0
1
5
4
1 0 1
0
!
L1 $L2
1 1 3
0

U \V = a0 + a1 t + a2 t2 + a3 t3 2 P3 : a0

a1 + a3 = 0 e

2a0 + a1 + a2 + a3 = 0 =
31
1 0
0 1 5A .
1 1

3
2
1 0 1
1
5
4
0 1 0
0
!
L2 $L3
1 1 3
0

3
1 0 1
1 1 3 5
0 1 0

a1 + a2 + 3a3 = 0 e a2 = 0 =

= a0 + a1 t + a2 t2 + a3 t3 2 P3 : a0 = 2a3 e a1 = 3a3 e a2 = 0 =

= 2a3 + 3a3 t + a3 t3 2 P3 : a3 2 R = a3 2 + 3t + t3 2 P3 : a3 2 R = L

2 + 3t + t3

e como tal, f2 + 3t + t3 g uma base de U \ V , tendo-se dim (U \ V ) = 1.


(vii) Em R4 , considere os subespaos:

U = L (f(2; 2; 1; 2); ( 1; 1; 1; 3); (0; 0; 6; 8); ( 1; 1; 5; 5)g)


e
V = L (f(0; 0; 0; 1); (0; 1; 2; 3); (0; 2; 4; 8)g) .
Atendendo a que
2
2
1 0
6 2 1
0
A=6
4 1
1
6
2 3
8

1
1
5
5

0
0
0
1

0
1
2
3

3
0
2 7
7
4 5
8

L1 +L2 !L2
1
L +L3 !L3
2 1
L1 +L4 !L4

262

2
1
6 0 0
6
4 0 3=2
0 2

0
0
6
8

1
0
9=2
6

0
0
0
1

0
1
2
3

3
0
2 7
7 !
4 5 L2 $L4
8

2
1
6 0 2
! 6
L2 $L4 4 0 3=2
0 0

0
8
6
0

1
6
9=2
0

0
1
0
0

0
3
2
1

3
0
8 7
7
4 5
2

3
L +L3 !L3
4 2

2
6 0
6
4 0
0

1
2
0
0

0
8
0
0

1 0
6
1
0 3=4
0
0

0
3
1=4
1

3
0
8 7
7 = A0 (*).
2 5
2

As colunas da matriz A correspondentes s colunas da matriz A0 que contm os pivots,


formam um conjunto de vectores linearmente independente. Logo, o conjunto
f(2; 2; 1; 2); ( 1; 1; 1; 3); (0; 1; 2; 3); (0; 2; 4; 8)g
uma base de U + V , tendo-se dim (U + V ) = 4 e deste modo U + V = R4 .
Por outro lado, tambm se conclui de (*) que o conjunto
f(2; 2; 1; 2); ( 1; 1; 1; 3)g

base de U ,
2
0
6 1
6
4 3=4
0

tendo-se dim U = 2,
3
2
0
0
6
3
8 7
7 ! 6
1=4
2 5 L1 $L2 4
1
2
2
!

3L1 +L3 !L3

o conjunto

e como
1
0
3=4
0

1
6 0
6
4 0
0

3
1
8
0

3
2
3
8
1
7
6
0
0 7
0
! 6
1=4
2 5 L2 $L4 4 3
4L3 !L3
1
2
0
3
2
8
1 3
7
6
2 7
6 0 1
!
16 5 8L2 +L3 !L3 4 0 0
0
0 0

3
1
1
0
3
8
2 7
7
0 5
0

3
8
2 7
7
!
8 5 3L1 +L3 !L3
0

f(0; 0; 0; 1); (0; 1; 2; 3)g

base de V , tendo-se dim V = 2.


Logo,
dim(U \ V ) = dim U + dim V

dim (U + V ) = 2 + 2

4 = 0.

Neste caso, como U \ V = f0g ento

V = R4 :

U +V =U
(viii) Em R4 , considere os subespaos:

U = (x; y; z; w) 2 R4 : x + 2y + 3z = 0 e y + 2z + 3w = 0
e
V = L (f(2; 5; 4; 1); (0; 9; 6; 1); ( 4; 1; 2; 1)g) .
Seja (x; y; z; w) 2 V . Ento existem ; ;

2 R tais que

(x; y; z; w) = (2; 5; 4; 1) + (0; 9; 6; 1) + ( 4; 1; 2; 1).


Atendendo a
2
2
6 5
6
4 4
1

0
9
6
1

4
1
2
1

j
j
j
j

3
x
y 7
7 !
z 5 L1 $L4
w

1
6 5
6
4 4
2
263

1
9
6
0

1
1
2
4

j
j
j
j

3
w
y 7
7
z 5
x

5L1 +L2 !L2


4L1 +L3 !L3
2L1 +L4 !L4

5L1 +L2 !L2


4L1 +L3 !L3
2L1 +L4 !L4

1
6 0
6
4 0
0

1
4
2
2

1
4
2
2

Logo, tem-se

V =

3
j
w
j y 5w 7
7
j z + 4w 5
j x 2w

1
L +L3 !L3
2 2
1
L +L4 !L4
2 2

1
6 0
6
4 0
0

1
4
0
0

3
j
w
7
j
y 5w
7 (*)
1
3
j 2w + 2y + z 5
j x 29 w + 12 y

1
4
0
0

1
3
(x; y; z; w) 2 R4 : w + y + z = 0 e x
2
2

9
1
w+ y=0
2
2

= (x; y; z; w) 2 R4 : y + 2z + 3w = 0 e x + 2y + 3z = 0 = U
pelo que
e U \V =U =V.

U +V =U =V

Atendendo ainda a (*), o conjunto f(2; 5; 4; 1); (0; 9; 6; 1); ( 4; 1; 2; 1)g linearmente
dependente, sendo linearmente independente o seguinte seu subconjunto
f(2; 5; 4; 1); (0; 9; 6; 1)g .
Assim, f(2; 5; 4; 1); (0; 9; 6; 1)g uma base de U; de V , de U + V e de U \ V , tendo-se
dim (U + V ) = dim (U \ V ) = 2.
Neste caso, como U \ V 6= f0g ento U + V no a soma directa dos subespaos U e V .
(ix) Seja U o subespao de R5 gerado por
f(1; 1; 1; 2; 0); (1; 2; 2; 0; 3); (1; 1; 2; 2; 1)g .
Seja V o subespao de R5 gerado por
f(1; 2; 3; 0; 2); (1; 1; 3; 2; 4); (1; 1; 2; 2; 5)g .
Atendendo a
2
1
1
6 1
2
6
6
2
A=6 1
4 2 0
0
3
2
1
6 0
6
6 0
!
L2 +L3 !L3 6
4 0
2L2 +L4 !L4
3L2 +L5 !L5
0

que
1
1
2
2
1

1
2
3
0
2

1
1
3
2
4

1
1
2
2
5

1
1
0
0
0

1
0
1
0
1

1
1
1
0
1

1
0
2
4
4

3
L +L5 !L5
2 4

1
6 0
6
6 0
6
4 0
0

3
7
7
7
7
5

L1 +L2 !L2
L1 +L3 !L3
2L1 +L4 !L4

1
0
1
4
5

1
1
0
0
0

6
6
6
6
4

1
0
0
0
0
2

7
7
7
7 L3 +L!
5 !L5
5
1
0
1
0
0

1
1
1
0
0

264

6
6
6
6
4

1
0
2
4
0

1
1
1
2
3
1
0
0
0
0
1
0
1
4
0

1
0
1
0
1
1
1
0
0
0
3

1
1
2
2
2
1
0
1
0
0

1
0
2
4
4
1
1
1
0
0

7
7
7 = A0 (*).
7
5

1
0
1
4
5
1
0
2
4
6

3
7
7
7
7
5

1
0
1
4
6

L2 +L3 !L3
2L2 +L4 !L4
3L2 +L5 !L5

3
7
7
7
7
5

3
L +L5 !L5
2 4

As colunas da matriz A correspondentes s colunas da matriz A0 que contm os pivots,


formam um conjunto de vectores linearmente independente. Logo, o conjunto
f(1; 1; 1; 2; 0); (1; 2; 2; 0; 3); (1; 1; 2; 2; 1); (1; 1; 3; 2; 4)g
uma base de U + V , tendo-se dim (U + V ) = 4.
Por outro lado, tambm se conclui de (*) que o conjunto
f(1; 1; 1; 2; 0); (1; 2; 2; 0; 3); (1; 1; 2; 2; 1)g
base de U , tendo-se dim U
2
1
1
1
6 1 0
0
6
6 1
2
1
6
4 0
4
4
0
0
0
o conjunto

= 3, e como
2
3
7
7
7
7 L1 +L!
2
5 L +L2 !L
!L
1

6
6
6
6
4

1
0
0
0
0

1
1
1
4
0

1
1
0
4
0

3
7
7
7
7
5

L2 +L3 !L3
4L2 +L4 !L4

2
6
6
6
6
4

1
0
0
0
0

1
1
0
0
0

1
1
1
0
0

3
7
7
7
7
5

f(1; 2; 3; 0; 2); (1; 1; 3; 2; 4); (1; 1; 2; 2; 5)g


base de V , tendo-se dim V = 3.
Logo,
dim(U \ V ) = dim U + dim V

dim (U + V ) = 3 + 3

4 = 2.

Neste caso, como U \ V 6= f0g ento U + V no a soma directa dos subespaos U e V .


Determinemos uma base para U \ V .
Atendendo a
3
2
2
3
1
1
1 j x1
1 1
1 j
x1
6 1
6 0
2
1 j x2 7
1 0 j x1 + x2 7
7
6
6
7
7
6 1
6
7
2
2
j
x
0
1
1
j
x
+
x
!
3
1
3
7 L1 +L2 !L2 6
6
7 L +L!!L
4 2 0
2 j x4 5 L1 +L3 !L3 4 0 2
0 j 2x1 + x4 5 2L22+L43!L43
3L2 +L5 !L5
0
3 1 j x5 2L1 +L4 !L4 0
3 1 j
x5
3
2
2
1 1
1 j
x1
1 1
1 j
x1
7
6
6 0
1
0
j
x
+
x
0
1
0
j
x
1
2
1 + x2
7
6
6
7
6
6
0 0
1 j
x2 + x3
1 j
x2 + x3
!
7 L3 +L!
6 0 0
L2 +L3 !L3 6
5 !L5
4
5
4
0 0
0 j 4x1 + 2x2 + x4
0 0
0 j
4x1 + 2x2 + x4
2L2 +L4 !L4
3L2 +L5 !L5
0 0
1 j
3x1 3x2 + x5
0 0
0 j
3x1 4x2 + x3 + x5
tem-se

U = (x1 ; x2 ; x3 ; x4 ; x5 ) 2 R5 : 4x1 + 2x2 + x4 = 0 e


Por outro lado,
2
1
6 2
6
6 3
6
4 0
2

3x1

4x2 + x3 + x5 = 0 .

atendendo a
1
1
3
2
4

1
1
2
2
5

j
j
j
j
j

x1
x2
x3
x4
x5

7
7
7
7 2L1 +L!
2
5 3L +L2 !L
!L
1

2L1 +L5 !L5

2
6
6
6
6
4

1
0
0
0
0

265

1
1
0
2
2

1
1
1
2
3

j
x1
j 2x1 + x2
j 3x1 + x3
j
x4
j 2x1 + x5

3
7
7
7
7
5

2L2 +L4 !L4


L3 +L5 !L5

3
7
7
7
7
5

2L2 +L4 !L4


L3 +L5 !L5

tem-se

1
0
0
0
0

6
6
6
6
4

1
1
0
0
2

j
j
j
j
j

1
1
1
0
2

x1
2x1 + x2
3x1 + x3
4x1 2x2 + x4
5x1 + x3 + x5

V = (x1 ; x2 ; x3 ; x4 ; x5 ) 2 R5 :

4x1

7
7
7
7 2L2 +L!
5 !L5
5

2
6
6
6
6
4

1
0
0
0
0

1
1
0
0
0

j
x1
j
2x1 + x2
j
3x1 + x3
j
4x1 2x2 + x4
j 9x1 + 2x2 + x3 + x5

1
1
1
0
0

3
7
7
7
7
5

2x2 + x4 = 0 e 9x1 + 2x2 + x3 + x5 = 0 .

Logo
U \V =
Como
2
4
6 3
6
4 4
9

2
4
2
2

tem-se

pelo que

(x1 ; x2 ; x3 ; x4 ; x5 ) 2 R5 : 4x1 + 2x2 + x4 = 0 e


3x1 4x2 + x3 + x5 = 0
e
4x1 2x2 + x4 = 0 e 9x1 + 2x2 + x3 + x5 = 0
3
0
1 7
7
!
0 5 L1 +L3 !L3
1 3L2 +L4 !L4
2
4 2
5
6 0
2
6
!
0
4L2 +L4 !L4 4 0
0 0

0
1
0
1

1
0
1
0

3
2 0 1 0
4 1 0 1 7
7
!
0 0 2 0 5 43 L1 +L2 !L2
10 4 0 4
2
3
1 0
4 2
3
6 0
7
1
10
4
6
7
!
4
5
2 0
0 0
4L2 !L2
3
3 0 2 L3 +L4 !L44 0 0

4
6 3
6
4 0
0
0
1
0
0

4
6 0
6
4 0
0
0
4
0
0

1
3
2
0

8
8
< 4x1 + 2x2 + x4 = 0
< x1 = 21 x2 = 15 x3
10x2 + 4x3 + 3x4 + 4x5 = 0 ,
x2 = 25 x3 + 25 x5
:
:
2x4 = 0
x4 = 0
U \V

1
x3
5

=
= L

3
0 1 0
5
1 34 1 7
2
7
0 0 2 0 5
10 4 0 4
3
0
4 7
7
0 5
0
2

1
x
5 5

1
2
2
x5 ; x3 + x5 ; x3 ; 0; x5 2 R5 : x3 ; x5 2 R
5
5
5
1 2
1 2
; ; 1; 0; 0 ;
; ; 0; 0; 1
.
5 5
5 5

Como o conjunto
1 2
; ; 1; 0; 0 ;
5 5

1 2
; ; 0; 0; 1
5 5

gera U \V e linearmente independente, ento uma base de U \V , tendo-se dim (U \ V ) =


2.
(x) Atendendo a
2
1 0 1
6 0 1 0
A=6
4 1 1 0
0 1
2

que
0
0
1
2

1
1
1
1

1
2
0
1

3
0
0 7
7
1 5
1

L1 +L3 !L3
1
L +L3 !L3
2 1
L1 +L4 !L4

266

1
6 0
6
4 0
0

0
1
1
1

1
0
1
2

0
0
1
2

1
1
2
1

1
2
1
1

3
0
0 7
7
1 5
1

L2 +L3 !L3
L2 +L4 !L4

4L2 +L4 !L4

L2 +L3 !L3
L2 +L4 !L4

1
6 0
6
4 0
0

0
1
0
0

1
0
1
2

0
0
1
2

1
1
1
0

1
2
1
3

3
0
0 7
7
!
1 5 2L3 +L4 !L4
1

1
6 0
6
4 0
0

0
1
0
0

1
0
1
0

0
0
1
0

1
1
1
2

3
0
0 7
7 = A0 (*).
1 5
3

1
2
1
5

As colunas da matriz A correspondentes s colunas da matriz A0 que contm os pivots,


formam um conjunto de vectores linearmente independente. Logo, o conjunto
f(1; 0; 1; 0); (0; 1; 1; 1); (1; 0; 0; 2) ; (1; 1; 1; 1)g
uma base de U + V , tendo-se dim (U + V ) = 4 e assim U + V = R4 .
Por outro lado, tambm se conclui de (*) que o conjunto
f(1; 0; 1; 0); (0; 1; 1; 1); (1; 0; 0; 2)g
base de U , tendo-se dim U = 3,
2
3
2
1 1 0
1
6 1 2 0 7
6 0
6
7
6
!
4 1
1 1 5 L1 +L2 !L2 4 0
L1 +L3 !L3
2
5 3
0
2L +L !L
1

e como
3
1 0
1 0 7
7
!
2 1 5 2L2 +L3 !L3
7 3 7L2 +L4 !L4

1
6 0
6
4 0
0

1
1
0
0

3
0
0 7
7
1 5
3

3L3 +L4 !L4

1
6 0
6
4 0
0

1
1
0
0

3
0
0 7
7
1 5
0

o conjunto
f(1; 1; 1; 1); (1; 2; 0; 1); (0; 0; 1; 1)g
base de V , tendo-se dim V = 3.
Logo,
dim(U \ V ) = dim U + dim V
Uma base para
Atendendo a
2
1 0
6 0 1
6
4 1 1
0 1
!

L2 +L3 !L3
L2 +L4 !L4

tem-se

1
6 0
6
4 0
0

0
1
0
0

dim (U + V ) = 3 + 3

4 = 2.

U \V.

1
0
1
2

1
0
0
2

j
j
j
j

3
x1
x2 7
7
!
x3 5 L1 +L3 !L3
x4

1
6 0
6
4 0
0

0
1
1
1

3
j
x1
7
j
x2
7
!
j x1 + x3 x2 5 2L3 +L4 !L4
j
x4 x2

U = (x1 ; x2 ; x3 ; x4 ) 2 R4 : 2x1
Por outro lado, atendendo a
2
3
1 1 0 j x1
6 1 2 0 j x2 7
6
7
4 1 0 1 j x3 5
1
1 1 j x4

L1 +L2 !L2
L1 +L3 !L3
L1 +L4 !L4

1
6 0
6
4 0
0
267

1
0
1
2
2

3
j
x1
7
j
x2
7
j x1 + x3 5
j
x4

1
6 0
6
4 0
0

0
1
0
0

1
0
1
0

L2 +L3 !L3
L2 +L4 !L4

3
j
x1
7
j
x2
7
5
j
x1 + x3 x2
j 2x1 3x2 + 2x3 + x4

3x2 + 2x3 + x4 = 0

1
1
1
2

0
0
1
1

3
j
x1
j x2 x1 7
7
!
j x3 x1 5 L2 +L3 !L3
j x4 x1 2L2 +L4 !L4

L2 +L3 !L3
2L2 +L4 !L4

tem-se

1
6 0
6
4 0
0

1
1
0
0

0
0
1
1

3
j
x1
7
j
x2 x1
7
j x2 2x1 + x3 5
j 2x2 3x1 + x4

L3 +L4 !L4

V = (x1 ; x2 ; x3 ; x4 ) 2 R4 :

1
6 0
6
4 0
0

1
1
0
0

0
0
1
0

3
j
x1
7
j
x2 x1
7
j
x2 2x1 + x3 5
j x2 x1 x3 + x4

x3 + x4 = 0 .

x1 + x2

Logo
U \V

= (x1 ; x2 ; x3 ; x4 ) 2 R4 : 2x1 3x2 + 2x3 + x4 = 0 e


x1 + x2 x3 + x4 = 0 =
4
= (x1 ; x2 ; x3 ; x4 ) 2 R : x2 = 3x4 e x1 = x3 + 4x4 =
= f( x3 + 4x4 ; 3x4 ; x3 ; x4 ) : x3 ; x4 ) 2 Rg = L (f( 1; 0; 1; 0) ; (4; 3; 0; 1)g)

Como o conjunto
f( 1; 0; 1; 0) ; (4; 3; 0; 1)g
gera U \V e linearmente independente, ento uma base de U \V , tendo-se dim (U \ V ) =
2.

37.
2

6
6
A=6
6
4

1
0
2
1
0

1
0
2
1
0
2

1
L +L3 !L3
2 2

L2 +L4 !L4

(i)

6
6
6
6
4

0
2
1
2
0
1
0
0
0
0

2
4
2
2
0

1
0
0
0
0

1
0
1
1
0
0
2
0
0
0

3
7
7
7
7
5
2
4
0
0
0

2L1 +L3 !L3


L1 +L4 !L4

1
0
1
2
0

6
6
6
6
4

1
0
0
0
0

7
7
7
7 2L3 +L!
4 !L4
5

1
0
0
0
0
2
6
6
6
6
4

1
0
0
0
0

0
2
1
2
0

2
4
2
4
0

1
0
0
0
0

0
2
0
0
0

1
0
1
2
0
2
4
0
0
0

3
7
7
7
7
5

1
0
1
0
0

1
L +L3 !L3
2 2

L2 +L4 !L4

7
7
7 = A0 .
7
5

carA = dim C(A) = dim L(A) = 3.


Como A tem 5 colunas e
no de colunas de A = carA + nulA,
ento
nulA = 2,

isto ,

dim N (A) = 2.

(ii) As colunas da matriz A correspondentes s colunas da matriz A0 que contm os


pivots, formam um conjunto de vectores linearmente independente. Logo,
C(A) = L (f(1; 0; 2; 1; 0); (0; 2; 1; 2; 0); (1; 0; 1; 1; 0)g)
268

e o conjunto f(1; 0; 2; 1; 0); (0; 2; 1; 2; 0); (1; 0; 1; 1; 0)g uma base de C(A).
Por denio:
N (A) = u 2 R5 : Au = 0 .
Temos ento, pelo mtodo de eliminao de Gauss,
Au = 0 , A0 u = 0.
A equao matricial

equivalente ao sistema

ou seja a

6
6
6
6
4

1
0
0
0
0

1
0
0
0
0

0
2
0
0
0

2
4
0
0
0

1
0
1
0
0

32
76
76
76
76
54

u1
u2
u3
u4
u5

7 6
7 6
7=6
7 6
5 4

0
0
0
0
0

8
< u1 u2 + 2u4 + u5 = 0
2u3 + 4u4 = 0
:
u5 = 0

Logo,
N (A) = f(u2

3
7
7
7
7
5

8
< u1 = u2 2u4
u3 = 2u4
:
u5 = 0.

2u4 ; u2 ; 2u4 ; u4 ; 0) : u2 ; u4 2 Rg .

Como
(u2

2u4 ; u2 ; 2u4 ; u4 ; 0) = (u2 ; u2 ; 0; 0; 0) + ( 2u4 ; 0; 2u4 ; u4 ; 0)


= u2 (1; 1; 0; 0; 0) + u4 ( 2; 0; 2; 1; 0),

tem-se:
N (A) = L (f(1; 1; 0; 0; 0); ( 2; 0; 2; 1; 0)g) .
Facilmente se verica que o conjunto S = f(1; 1; 0; 0; 0); ( 2; 0; 2; 1; 0)g linearmente independente. Como S linearmente independente e gera N (A), temos ento que S uma
base de N (A).
(iii) A soluo geral do sistema de equaes lineares homogneo Au = 0 dada por
(1; 1; 0; 0; 0) + ( 2; 0; 2; 1; 0),
com ;

2 R.

(iv) Uma soluo particular de Au = b, com b = (1; 0; 2; 1; 0), por exemplo u =


(1; 0; 0; 0; 0). Logo, a soluo geral de Au = b dada por:
(1; 0; 0; 0; 0) + (1; 1; 0; 0; 0) + ( 2; 0; 2; 1; 0).
Observao. Note que se tem sempre:
no de colunas de A = carA + nulA.
269

38. (i) Se A 2 M3 3 (R) tal que car A = 3 e car[A j B] = 3 ento


car A = dim L(A) = dim C(A) = 3.
Logo,
nul A = dim N (A) = 0.

Como carAT = dim L(AT ) = dim C(AT ) = 3 ento

nul AT = dim N (AT ) = 0.


O correspondente sistema de equaes lineares no homogneo AX = B possvel e determinado. Neste caso, na soluo geral de AX = B, no existe nenhum parmetro.
(ii) Se A 2 M3 3 (R) tal que car A = 2 e car[A j B] = 3 ento
car A = dim L(A) = dim C(A) = 2:
Logo,
nul A = dim N (A) = 1:

Como carAT = dim L(AT ) = dim C(AT ) = 2 ento

nul AT = dim N (AT ) = 1:


O correspondente sistema de equaes lineares no homogneo AX = B impossvel.
(iii) Se A 2 M3 3 (R) tal que car A = 1 e car[A j B] = 1 ento
car A = dim L(A) = dim C(A) = 1:
Logo,
nul A = dim N (A) = 2:

Como carAT = dim L(AT ) = dim C(AT ) = 1 ento

nul AT = dim N (AT ) = 2:


O correspondente sistema de equaes lineares no homogneo AX = B possvel e indeterminado. Neste caso, na soluo geral de AX = B, existem dois parmetros.
(iv) Se A 2 M5 9 (R) tal que car A = 2 e car[A j B] = 2 ento
car A = dim L(A) = dim C(A) = 2:
Logo,
nul A = dim N (A) = 7:

Como carAT = dim L(AT ) = dim C(AT ) = 2 ento

nul AT = dim N (AT ) = 3:


O correspondente sistema de equaes lineares no homogneo AX = B possvel e indeterminado. Neste caso, na soluo geral de AX = B, existem 7 parmetros.
270

(v) Se A 2 M9 5 (R) tal que car A = 2 e car[A j B] = 3 ento


car A = dim L(A) = dim C(A) = 2:
Logo,
nul A = dim N (A) = 3:

Como carAT = dim L(AT ) = dim C(AT ) = 2 ento

nul AT = dim N (AT ) = 7:


O correspondente sistema de equaes lineares no homogneo AX = B impossvel.
(vi) Se A 2 M4 4 (R) tal que car A = 0 e car[A j B] = 0 ento
car A = dim L(A) = dim C(A) = 0:
Logo,
nul A = dim N (A) = 4:

Como carAT = dim L(AT ) = dim C(AT ) = 0 ento

nul AT = dim N (AT ) = 4:


O correspondente sistema de equaes lineares no homogneo AX = B possvel e indeterminado. Neste caso, na soluo geral de AX = B, existem 4 parmetros.
(vii) Se A 2 M6 2 (R) tal que car A = 2 e car[A j B] = 2 ento
car A = dim L(A) = dim C(A) = 2:
Logo,
nul A = dim N (A) = 0:

Como carAT = dim L(AT ) = dim C(AT ) = 2 ento

nul AT = dim N (AT ) = 4:


O correspondente sistema de equaes lineares no homogneo AX = B possvel e determinado. Neste caso, na soluo geral de AX = B, no existe nenhum parmetro.
39. Queremos encontrar A tal que N (A) = L (f(2; 0; 1)g). Por denio N (A) =
fu 2 R3 : Au = 0g. Por outro lado, temos
L (f(2; 0; 1)g) = f (2; 0; 1) :

2 Rg = (u1 ; u2 ; u3 ) 2 R3 : u2 = 0 e u1 = 2u3 .

Por exemplo:
A=

1 0 2
0 1 0

verica
N (A) = L (f(2; 0; 1)g) ,
271

pois

1
6 0
Au = 0 , 6
4 0
0

0
1
0
0

3
8
3 2 3
2 2
u
0
< u1 + 2u3 = 0
1
0 7
7 4 u2 5 = 4 0 5 ,
0 5
:
u3
0
u2 = 0.
0

40. No possvel encontrar A tal que


(1; 1; 1) 2 L(A)

(1; 0; 0) 2 N (A),

pois se (1; 0; 0) 2 N (A) ento a primeira entrada de todas as linhas de A 0. Pelo que, nesse
caso, no se pode ter (1; 1; 1) 2 L(A).
41. Seja A 2 M3 3 (R) tal que nul A = 3. Uma vez que
no de
2
0
ento car A = 0. Isto , A = 4 0
0
42. Seja A 2 Mm

n (R)

colunas de A = carA + nulA,


3
0 0
0 0 5.
0 0

tal que
C(A) = N (A):

Logo, o no de linhas de A igual ao no de colunas de A. Isto , m = n. Alm disso, como


n = carA + nulA,
tem-se
n = 2 dim N (A).
Pelo que, A 2 Mn

n (R)

com n par. Exemplo:


2
0 0
6 0 0
A=6
4 0 0
0 0

1
0
0
0

3
0
1 7
7:
0 5
0

43. Seja A 2 Mn n (R) tal que car A = n. Logo, A invertvel. Isto , existe A
que AA 1 = A 1 A = I. Alm disso, se A fr tal que A2 = A, ento
A = AI = A(AA 1 ) = (AA)A
Logo, A = I.

272

= A2 A

= AA

= I.

tal

44. Sejam B1 = f(1; 2); (0; 1)g e B2 = f(1; 1); (2; 3)g duas bases ordenadas de R2 . Seja
v = (1; 5).
(i) Tem-se v = (1; 2) + 3(0; 1). Logo, 1 e 3 so as coordenadas de v em relao base B1 .
(ii) Tem-se
1
1

SB1 !B2 =
uma vez que (1; 2) =

(1; 1) + (2; 3) e (0; 1) =

2
1

2(1; 1) + (2; 3).

(iii) As coordenadas de v = (1; 5) em relao base B2 , so dadas por:


SB1 !B2

1
5

1
1

2
1

1
3

7
4

uma vez que 1 e 3 so as coordenadas de v em relao base B1 .


(iv) Tem-se v = (1; 5) =

7(1; 1) + 4(2; 3).

(v) Tem-se
1
1

SB2 !B1 =
uma vez que (1; 1) = (1; 2)

2
1

(0; 1) e (2; 3) = 2(1; 2)

(0; 1).

Observao:
SB2 !B1 = (SB1 !B2 )

e SB1 !B2 = (SB2 !B1 )

(vi) As coordenadas de v = (1; 5) em relao base B1 , so dadas por:


SB2 !B1
uma vez que

7
4

1
1

2
1

7
4

1
3

7 e 4 so as coordenadas de v em relao base B2 .

45. Sejam B1 = fv1 ; v2 g e B2 = fw1 ; w2 g duas bases ordenadas de R2 , onde


v1 = (1; 2), v2 = (0; 1).
Seja
SB2 !B1 =

2 1
1 1

a matriz de mudana da base B2 para a base B1 . Determinemos B2 .


Uma vez que
2 1
SB2 !B1 =
,
1 1
ento w1 = 2v1 + v2 = 2(1; 2) + (0; 1) = (2; 5) e w2 = v1 + v2 = (1; 2) + (0; 1) = (1; 3).
Logo,
B2 = f(2; 5); (1; 3)g .
273

46. Sejam B1 = fv1 ; v2 g e B2 = fw1 ; w2 g duas bases ordenadas de P1 , onde


w1 =

1 + t, w2 = 1 + t.

Seja
SB1 !B2 =

2 3
1 2

a matriz de mudana da base B1 para a base B2 . Determinemos B1 .


Uma vez que
2 3
SB1 !B2 =
,
1 2
ento v1 = 2 ( 1 + t)

(1 + t) =

3 + t e v2 = 3 ( 1 + t) + 2 (1 + t) =

1 + 5t. Logo,

B1 = f 3 + t; 1 + 5tg .

47. Sejam B1 = f1; 1

t; t2 g e B2 = f1; 1 + t; 1 + t + t2 g duas bases ordenadas de P2 .

(i) Sejam 1; 2 e 3 as coordenadas de um vector p(t) 2 P2 em relao base B2 . Determinemos as coordenadas do mesmo vector p(t) em relao base B1 .
Tem-se
p(t) = 1 + 2 (1 + t) + 3 1 + t + t2 = 6 + 5t + 3t2 = 1 + (1
fcil ver que

= 11,

5 e

t) + t2 .

= 3.

Resoluo alternativa: Tem-se


SB2 !B1

1
=4 0
0

uma vez que 1 = 1 + 0 (1 t) + 0t2 , 1 + t = 2


Logo, as coordenadas de p(t) em relao base
2 3 2
1
1 2
1
SB2 !B1 4 2 5 = 4 0
3
0 0

2
1
0

3
2
1 5,
1

(1 t) + 0t2 e 1 + t + t2 = 2
B1 so dadas por:
32 3 2
3
2
1
11
1 54 2 5 = 4 5 5,
1
3
3

onde 1; 2 e 3 so as coordenadas de p(t) em relao base B2 .

(ii) Determinemos a matriz SB1 !B2 de mudana da base B1 para a base B2 .


Como
1 = 1 1 + 0 (1 + t) + 0 (1 + t + t2 )
1

t=2

t2 = 0

(1 + t) + 0 (1 + t + t2 )

(1 + t) + (1 + t + t2 )
274

(1

t) + t2 .

ento

1
4
= 0
0

SB1 !B2
Alm disso, bastaria ver que

SB1 !B2 = (SB2 !B1 )

1
4
= 0
0

Logo, como
2

t + t2 na
2 3 2
1
4
SB1 !B2 1 5 = 4
1
2

3
2
1 5
1

t + t2 = 1 + (1

as coordenadas do vector 2

ou seja

2
1
0

3
0
1 5.
1

2
1
0

1
4
= 0
0

2
1
0

3
0
1 5.
1

t) + t2

base B2 so dadas por


32 3 2
3
1 2
0
1
3
0
1
1 54 1 5 = 4 2 5,
0 0
1
1
1

t + t2 = 3

2 (1 + t) + 1 + t + t2 .

48. Sejam B1 = fv1 ; v2 g e B2 = fw1 ; w2 g duas bases ordenadas de P1 , onde


w1 = t, w2 = 1

t.

Seja
SB2 !B1 =

2 3
1 2

a matriz de mudana da base B2 para a base B1 . Determinemos B1 .


Uma vez que
2 3
SB2 !B1 =
,
1 2
ento w1 = 2v1

v2 e w2 = 3v1 + 2v2 . Isto , tem-se o sistema


8
< 2v1 v2 = t

cuja matriz aumentada dada por

3v1 + 2v2 = 1

2
3

t,

1 j
t
2 j 1 t

Pelo mtodo de eliminao de Gauss:


2
3

1 j
t
2 j 1 t

3
L +L2 !L2
2 1

275

2
0

1 j
t
j 1 25 t

7
2

Logo, v2 =

2
7

5
t
7

e v1 = 12 (v2 + t) =
B1 =

1
7

+ 17 t. Logo,
1 1 2
+ t;
7 7 7

5
t .
7

49. Sejam B1 = fv1 ; v2 ; v3 g e B2 = fw1 ; w2 ; w3 g duas bases ordenadas de R3 , onde


v1 = (1; 0; 1), v2 = (1; 1; 0), v3 = (0; 0; 1).
Seja
SB1 !B2

1
4
2
=
1

3
1 2
1 1 5,
1 1

a matriz de mudana da base B1 para a base B2 . Determinemos B2 = fw1 ; w2 ; w3 g. Uma


vez que
2
3
1
1 2
1 1 5,
SB1 !B2 = 4 2
1
1 1
ento v1 = w1 + 2w2
sistema

w3 ; v2 = w1 + w2 w3 e v3 = 2w1 + w2 + w3 . Isto , tem-se o


8
< w1 + 2w2 w3 = (1; 0; 1)
w1 + w2 w3 = (1; 1; 0)
:
2w1 + w2 + w3 = (0; 0; 1),

cuja matriz aumentada dada por


2

1 2
4 1 1
2 1

3
1 j 1 0 1
1 j 1 1 0 5.
1 j 0 0 1

Pelo mtodo de eliminao de Gauss:


2
3
2
1 2
1 j (1; 0; 1)
4 1 1
4
1 j (1; 1; 0) 5
!
L1 +L2 !L2
2 1 1 j (0; 0; 1)
2L1 +L3 !L3
2
1 2
4
0
1
!
3L2 +L3 !L3
0 0
Tem-se ento o sistema

3
1 j
(1; 0; 1)
0 j (0; 1; 1) 5
3 j ( 2; 0; 1)
3
1 j
(1; 0; 1)
0 j (0; 1; 1) 5 .
3 j ( 2; 3; 2)

1
0
0

2
1
3

8
w1 + 2w2 w3 = (1; 0; 1)
>
>
>
>
<
w2 = (0; 1; 1)
>
>
>
>
:
3w3 = ( 2; 3; 2).

2
; 1; 32 ; w2 = (0; 1; 1) e w1 = (1; 0; 1) 2(0; 1; 1) +
Logo, w3 =
3
1
1
; 1; 3 . Logo,
3
1
1
2
2
B2 =
; 1;
; (0; 1; 1);
; 1;
.
3
3
3
3

276

3L2 +L3 !L3

2
;
3

1; 23 =

Note que

1
3

4 1
2

1
1
1 54 2
1
2
1
1
3
3
2
2
0
1 1
3
5
4
1
1 = 0 1
2
1
1 0
3

1
3
1
3

,4 1

em que

32

2
3

0
1
1

1
3

SB2 !B1

3 2
3
2
1 1 0
1 5=4 0 1 0 5,
1
1 0 1
3
32 2
1
1
0
3
3
1
1 5
0 54 1
1
1
0
1
3
3

3 1 2
1 2
1 1 5 =4
1 1
2
1
1
4
2
1
SB1 !B2 =
1
1

1
=4 2
1

2
3

1
1
3

1
3

1
1
0

1 5;

1
3

3
2
1 5,
1

B1 = f(1; 0; 1); (1; 1; 0); (0; 0; 1)g ,


B2 =

1
; 1;
3

1
3

; (0; 1; 1);

B1 =

1 0
0 0

0 1
0 0

1
1

2
2
; 1;
3
3

50. Sejam
0 0
1 0

0 0
0 1

e
1 1
1 1

B2 =

1
1

1 1
1 1

1
1

1
1

duas bases ordenadas de M2 2 (R). Determinemos a matriz SB1 !B2 de mudana da base B1
para a base B2 .
Queremos encontrar a1 ; a2 ; a3 ; a4 ; b1 ; b2 ; b3 ; b4 ; c1 ; c2 ; c3 ; c4 ; d1 ; d2 ; d3 ; d4 2 R tais que
1 0
0 0

= a1

1 1
1 1

+ a2

1
1

1
1

+ a3

1 1
1 1

+ a4

1
1

1
1

0 1
0 0

= b1

1 1
1 1

+ b2

1
1

1
1

+ b3

1 1
1 1

+ b4

1
1

1
1

0 0
1 0

= c1

1 1
1 1

+ c2

1
1

1
1

+ c3

1 1
1 1

+ c4

1
1

1
1

+ d2

1
1

1
1

+ d3

1 1
1 1

+ d4

1
1

1
1

0 0
0 1
Atendendo a
2
1 1
6 1
1
6
4 1
1
1
1

1 1
1 1

= d1

1
1
1
1

1
1
1
1

j
j
j
j

1
0
0
0

0
1
0
0

0
0
1
0

3
0
0 7
7
!
0 5 L1 +L2 !L2
+L3 !L3
1 LL1 +L
!L
1

277

1
6 0
6
4 0
0

1
0
2
2

1
2
0
2

1
2
2
0

j
j
j
j

1
1
1
1

0
1
0
0

0
0
1
0

3
0
0 7
7 !
0 5 L2 $L4
1

1
6 0
! 6
L2 $L4 4 0
0

1
2
2
0

1
2
0
2

3
0 0
0 1 7
7
1 0 5
0 0
2
1
6 0
6
!
L2 +L3 !L3 4 0
0

j
j
j
j

1
0
2
2

Logo, tem-se

1
1
1
1

0
0
0
1

L2 +L3 !L3

1
2
0
0

1
2
2
0

1
0
2
4

1
6 0
6
4 0
0
j
j
j
j

1
1
0
1

1
2
0
0

1
2
2
2

0
0
0
1

0
0
1
1

1
0
2
2

j
j
j
j
3

1
1
0
1

0
0
0
1

0
0
1
0

0
1 7
7.
1 5
1

1 1
0 1

= a1

1 0
0 0

+ a2

1 2
0 0

+ a3

1 2
2 0

+ a4

1 0
2 4

0 0
0 1

= b1

1 0
0 0

+ b2

1 2
0 0

+ b3

1 2
2 0

+ b4

1 0
2 4

0 0
1 1

= c1

1 0
0 0

+ c2

1 2
0 0

+ c3

1 2
2 0

+ c4

1 0
2 4

0
1

1
1

= d1

1 0
0 0

+ d2

1 2
0 0

Isto , tem-se os seguintes sistemas:


8
1 = a1 + a2 + a3 + a4
>
>
<
1 = 2a2 + 2a3
0 = 2a3 + 2a4
>
>
:
1 = 4a4
8
0 = c1 + c2 + c3 + c4
>
>
<
0 = 2c2 + 2c3
1 = 2c3 + 2c4
>
>
:
1 = 4c4
que so equivalentes a

8
a1 =
>
>
>
>
>
>
>
>
< a2 =

>
>
c3 =
>
>
>
>
>
>
:
c4 =

+ d4

1 0
2 4

8
0 = b1 + b2 + b3 + b4
>
>
<
0 = 2b2 + 2b3
0 = 2b3 + 2b4
>
>
:
1 = 4b4
8
0 = d1 + d2 + d3 + d4
>
>
<
1 = 2d2 + 2d3
1 = 2d3 + 2d4
>
>
:
1 = 4d4
8
b1 =
>
>
>
>
>
>
>
>
< b2 =

1
4

1
4

>
>
a3 = 14
>
>
>
>
>
>
:
a4 = 14
8
c1 = 14
>
>
>
>
>
>
>
>
< c2 = 14

1 2
2 0

+ d3

1
4

1
4

>
>
b3 = 14
>
>
>
>
>
>
:
b4 = 14
8
d1 = 41
>
>
>
>
>
>
>
>
< d2 = 14

1
4

1
4

278

>
>
d3 =
>
>
>
>
>
>
:
d4 =

1
4

1
.
4

3
0
1 7
7
!
1 5 L3 +L4 !L4
0

Logo, a matriz SB1 !B2 de mudana da base B1 para a base B2 dada por:
2 1
3
1
1
1
SB1 !B2

6
6
6
6
6
6
6
6
4

Isto ,
1 2
3 4

1
4

1
4

1
4

1
4

1
4

1
4

1 1
1 1

=2

1
4

1
4

1
4

1
4

1
4

1
4

1
4

1
4

1 2
3 4

Assim, as coordenadas do vector


2

6
6
6
6
=6
6
6
6
4

1
4

1
4

1
4

7
7
7
7
7.
7
7
7
5

1
4

1
4

1
4

1
4

1
4

1
4

1
4

1
4

3
2

1
4

em relao base B2 so dadas por

1
4

1
4

1
1

1
1

72
7
7
76
76
74
7
7
5
+

3 6 7
6 3 7
1
6 7
2 7
7
2 7 6
6 7.
=
7
3 5 6
6 1 7
7
6
4
4 5
1
2

1 1
1 1

1
2

1
1

1
1

51. Seja B = fv1 ; v2 g uma base ordenada de P1 . Sejam (1; 1) e (2; 2) respectivamente
as coordenadas de dois polinmios 1 + t e 1 t em relao base B: Determine B.
Tem-se
1 + t = v1 v2
1+t
1
1
v1
,
=
,
1 t = 2v1 + 2v2
1 t
2 2
v2

Logo B =

3
4

+ 14 t;

v1
v2

1
4

3
t
4

1
2

1
2

1+t
1 t

3
4

+ 14 t

1
4

3
t
4

52. Sejam B1 = fv1 ; v2 g e B2 = fw1 ; w2 g duas bases ordenadas de P1 . Suponha que


(1; 1) e (2; 2) so respectivamente as coordenadas de um polinmio p (t) em relao s
bases B1 e B2 : Suponha ainda que (1; 1) e (2; 2) so respectivamente as coordenadas de
um polinmio q (t) em relao s bases B1 e B2 : Determine a matriz SB1 !B2 de mudana da
base B1 para a base B2 .
Seja
a b
SB1 !B2 =
.
c d
Tem-se
2
2

a b
c d

1
1

279

2
2

a b
c d

1
1

Logo

3 2
a
6 b 7 6
7 6
,6
4 c 5=4
d

8
2
3 2
2
=
a
b
2
>
>
<
6 2 7 6
2=c d
7 6
,6
4 2 5=4
2
=
a
+
b
>
>
:
2
2=c+d
3 12
3 2
1
1 0 0
2
6
7 6
0 0 1
1 7
7 6 2 7=6
1 1 0 0 5 4 2 5 4
0 0 1 1
2

280

1
0
1
0

1
0
1
0
3

2
0 7
7
0 5
2

0
1
0
1

32
0
a
6 b
1 7
76
0 54 c
1
d

e assim

7
7,
5

SB1 !B2 =

2
0

0
2

Resoluo da 2a Ficha de exerccios facultativos


1. Seja V um espao linear real e 0 o seu vector nulo.
(i) Suponhamos que u + v = u + w. Queremos ver que v = w. Ora,
v = 0 + v = (( u) + u) + v = ( u) + (u + v)

u+v=u+w

= ( u) + (u + w) = (( u) + u) + w = 0 + w = w.
Logo, v = w:
(ii) Queremos ver que 0 = 0 para todo o escalar
0+0= 0=

2 R. Ora,

(0 + 0) = 0 + 0 =) 0 = 0.
por (i)

(iii) Queremos ver que 0u = 0 para todo o vector u 2 V. Ora,


0u + 0 = 0u = (0 + 0) u = 0u + 0u =) 0 = 0u.
por (i)

(iv) Queremos ver que

( u) = u para todo o u 2 V. Ora,

u + ( u) = 0 =)

( u) = u.

(v) Queremos ver que o vector nulo 0 2 V nico. Ora, seja w 2 V tal que u + w = u,
para todo o u 2 V. Ento,
u + w = u = u + 0 =) w = 0.
por (i)

(vi) Queremos ver que o simtrico u de um qualquer vector u de V nico. Ora,


seja w 2 V tal que u + w = 0. Ento,
u + w = 0 = u + ( u) =) w =
por (i)

(vii) Queremos ver que (

1)u =

u.

u para todo o u 2 V. Ora,

u + ( 1) u = 1u + ( 1) u = (1 + ( 1)) u = 0u = 0.
Logo, como o simtrico nico, (

1)u =

u.

(viii) Queremos ver que: se u = 0, ento = 0 ou u = 0: Suponhamos que u = 0.


Se 6= 0, ento
1
1
1
u = 1u =
u = ( u) = 0 = 0.
por (iv)

Como

6= 0 =) u = 0, ento u 6= 0 =)
u = 0 =)

= 0. Logo,
=0_u=0

(ix) Queremos ver que: se u 6= 0 e u = u, ento


u = u. Ora, como u 6= 0 e (
) u = 0, ento
, = .
281

= . Suponhamos que u 6= 0 e
= 0, atendendo a (viii). Isto

2. O conjunto de todos os polinmios reais de grau igual a n:


+ an tn 2 Pn : a0 ; a1 ; :::; an 2 R e an 6= 0g ,

U = fa0 + a1 t +

com as operaes usuais, no um espao linear. Por exemplo: o polinmio nulo


p(t) = 0 2
= U.
3. (i) ? 6= P2

P3 e:
P2 = L

1; t; t2

Logo, P2 subespao de P3 .
(ii) ? 6= Pn

Pn+1 e:
Pn = L (f1; t; :::; tn g) .

Logo, Pn subespao de Pn+1 .


(iii) ? 6= Pn

P e:
Pn = L (f1; t; :::; tn g) .

Logo, Pn subespao de P .
4. (i) Seja
U = A 2 Mn
Sejam A1 ; A2 2 U e

n (R)

: A = AT :

2 R. Tem-se
A1 + A2 = AT1 + AT2 = (A1 + A2 )T 2 U

e, com A 2 U ,

A = AT = ( A)T 2 U .

Logo, U subespao de Mn

n (R).

(ii) Seja

U = fA 2 Mn

n (R)

: A invertvelg :

Por exemplo: a matriz nula no pertence a U . Logo, U no subespao de Mn


(iii) Seja
U = f(aij ) 2 Mn

n (R)

: aij = 0 se i 6= j, com i; j = 1; :::; ng :

Sejam
(bij ); (cij ) 2 U

2 R:

Tem-se
(bij ) + (cij ) = (bij + cij ) 2 U ,
pois bij + cij = 0 se i 6= j, com i; j = 1; :::; n. E, com (aij ) 2 U ,
(aij ) = ( aij ) 2 U ,
pois aij = 0 se i 6= j, com i; j = 1; :::; n. Logo, U subespao de Mn
(iv) Seja

U = fA 2 Mn

n (R)

282

: A singularg :

n (R).

n (R).

Por exemplo, para n = 2:


1 0
0 0

0 0
0 1

1 0
0 0

2 U , mas

Logo, U no subespao de Mn

0 0
0 1

1 0
0 1

2
= U.

n (R).

(v) Seja

U = f(aij ) 2 Mn

n (R)

: aij = 0 se i > j, com i; j = 1; :::; ng :

Sejam
(bij ); (cij ) 2 U

2 R:

Tem-se
(bij ) + (cij ) = (bij + cij ) 2 U ,
pois bij + cij = 0 se i > j, com i; j = 1; :::; n. E, com (aij ) 2 U ,
(aij ) = ( aij ) 2 U ,
pois aij = 0 se i > j, com i; j = 1; :::; n. Logo, U subespao de Mn

n (R).

5. Seja V o espao linear de todas as funes reais de varivel real.


(i) Seja
U = ff : Dom f

R ! R tais que 9k > 0 : jf (x)j

k; 8x 2 Dom f g

o conjunto de todas as funes limitadas. Sejam f1 ; f2 2 U e

2 R. Tem-se

f1 + f2 2 U ,
pois
j(f1 + f2 ) (x)j = jf1 (x) + f2 (x)j

jf1 (x)j + jf2 (x)j

k1 + k2 ,
f1 ;f2 2U

para todo o x 2 Dom f1 \ Dom f2 . E, com f 2 U ,


f 2 U,
pois
j( f ) (x)j = j j jf (x)j

f 2U

j j k,

para todo o x 2 Dom f . Logo, U subespao de V .


(ii) Seja

U = ff : Dom f

R ! R tais que f (x) = f ( x); 8x 2 Dom f g

o conjunto de todas as funes pares. Sejam f1 ; f2 2 U e

2 R. Tem-se

f1 + f2 2 U ,
pois
(f1 + f2 ) (x) = f1 (x) + f2 (x)

f1 ;f2 2U

283

f1 ( x) + f2 ( x) = (f1 + f2 ) ( x),

para todo o x 2 Dom f1 \ Dom f2 . E, com f 2 U ,


f 2 U,
pois
( f ) (x) = f (x) =

f ( x) = ( f ) ( x),

f 2U

para todo o x 2 Dom f . Logo, U subespao de V .

(iii) O conjunto de todas as funes racionais, isto , as que so quocientes de funes


polinomiais, um subespao de V
(iv) Seja
U = ff : Domf
Se f fr crescente ento
subespao de V .

R ! R tais que f crescenteg:

f decrescente, isto , f 2 U =)

f 2
= U . Logo, U no

(v) Seja
U = ff : Dom f
Sejam f1 ; f2 2 U e

R ! R tais que f (0) = f (1); 8x 2 Dom f g

2 R. Tem-se
f1 + f2 2 U ,

pois
(f1 + f2 ) (0) = f1 (0) + f2 (0)

f1 ;f2 2U

f1 (1) + f2 (1) = (f1 + f2 ) (1),

para todo o x 2 Dom f1 \ Dom f2 . E, com f 2 U ,


f 2 U,
pois
( f ) (0) = f (0) =

f 2U

f (1) = ( f ) (1),

para todo o x 2 Dom f . Logo, U subespao de V .


(vi) Seja

U = ff : Domf

R ! R tais que f (0) = 1 + f (1)g:

Sejam f1 ; f2 2 U . Tem-se
(f1 + f2 ) (0) = f1 (0) + f2 (0)

f1 ;f2 2U

2 + f1 (1) + f2 (1) = 2 + (f1 + f2 ) (1),

isto , f1 + f2 2
= U . Logo, U no subespao de V .
6. Dem. Seja fv1 ; v2 ; v3 g uma base de um espao linear V . Observe-se que
fv1 + v2 ; v2 + v3 ; v1 + v3 g

L (fv1 ; v2 ; v3 g) ,

pelo que
L(fv1 + v2 ; v2 + v3 ; v1 + v3 g)
284

L (fv1 ; v2 ; v3 g) .

Mas, como

8
v1 = 12 (v1 + v2 )
>
>
>
>
<
v2 = 12 (v1 + v2 )
>
>
>
>
:
v3 = 12 (v1 + v3 )

tem-se

L (fv1 ; v2 ; v3 g)

1
(v
2 2

+ v3 ) + 21 (v1 + v3 )

1
(v
2 1

+ v3 ) + 21 (v2 + v3 )

1
(v
2 1

+ v2 ) + 21 (v2 + v3 )

L(fv1 + v2 ; v2 + v3 ; v1 + v3 g).

Logo,
L(fv1 + v2 ; v2 + v3 ; v1 + v3 g) = L (fv1 ; v2 ; v3 g) = V .
Vejamos agora que o conjunto fv1 + v2 ; v2 + v3 ; v1 + v3 g linearmente independente:
Sejam

1;

2;

2 R tais que
(

1 (v1

3 )v1

+ v2 ) +

+(

2 (v2

2 )v2

+ v3 ) +

+(

3 (v1

3 )v3

+ v3 ) = 0. Isto ,

= 0.

Como fv1 ; v2 ; v3 g uma base de V , em particular linearmente independente. Logo,


8
< 1+ 3=0
1+ 2 = 0
:
2+ 3 = 0

o que equivalente ao sistema homogneo:


2
3
2

A4

2
3

3
0
5 = 4 0 5:
0

1 0 1
com A = 4 1 1 0 5. Como det A = 2 6= 0, ento A invertvel e tem-se 1 = 2 =
0 1 1
=
0.
Logo,
fv1 + v2 ; v2 + v3 ; v1 + v3 g uma base de V pois trata-se de um conjunto
3
de vectores linearmente independente que gera V .
7. Seja A uma matriz (real) invertvel do tipo n n. Suponhamos que fv1 ; v2 ; : : : ; vn g
uma base de Rn . Queremos provar que fAv1 ; Av2 ; : : : ; Avn g tambm uma base de
Rn .
Dem. Vejamos primeiro que o conjunto fAv1 ; Av2 ; : : : ; Avn g linearmente independente. Sejam 1 ; 2 ; : : : ; n 2 R tais que
1 (Av1 )

Queremos ver que


1 (Av1 )

2 (Av2 )

2 (Av2 )

= ::: =
+

n (Avn )

= 0.

= 0. Observe-se que

n (Avn )

= A( 1 v1 ) + A( 2 v2 ) +
= A( 1 v1 + 2 v2 +
+

+ A(
n vn ).

n vn )

Logo,
1 (Av1 )

2 (Av2 )

n (Avn )

= 0 () A( 1 v1 +

285

2 v2

n vn )

= 0.

Como A invertvel, tem-se


A 1 A( 1 v1 + 2 v2 +
I( 1 v1 + 2 v2 +
1 v1 + 2 v2 +

+ n vn ) = A 1 0 ,
+ n vn ) = 0 ,
+ n vn = 0.

Como fv1 ; v2 ; : : : ; vn g uma base de Rn , ento


1

= ::: =

= 0:

Logo, fAv1 ; Av2 ; : : : ; Avn g um subconjunto de Rn formado por n vectores linearmente


independentes. Como a dimenso de Rn n, ento
fAv1 ; Av2 ; : : : ; Avn g
uma base de Rn .
8. Sejam V um espao linear e S = fv1 ; v2 ; : : : ; vn g.

Dem. ()) Suponhamos que S uma base de V . Queremos provar que todo o vector
de V se escreve de maneira nica como combinao linear dos elementos de S. Assim,
seja v um vector qualquer de V . Como S uma base de V , ento em particular gera
V . Pelo que, existem 1 ; 2 ; : : : ; n 2 R tais que
v=

1 v1

Suponhamos que tambm existiam


v=

2 v2

1;

1 v1

2; : : : ;

2 v2

+
n

n vn .

2 R tais que

n vn .

Logo,
(

1 )v1

+(

2 )v2

+(

n )vn

= 0.

Como fv1 ; v2 ; : : : ; vn g um conjunto linearmente independente (por ser base), ento


temos
1 = 1; 2 = 2; : : : ; n = n.
Logo, conclui-se que todo o vector de V se escreve de maneira nica como combinao
linear dos elementos de S.
(() Suponhamos agora que todo o vector de V se escreve de maneira nica como
combinao linear dos elementos de S. Queremos provar que S = fv1 ; v2 ; : : : ; vn g
uma base de V . Como todo o vector de V se escreve como combinao linear dos
elementos de S, ento S gera V . Falta ver que S linearmente independente. Assim,
sejam 1 ; 2 ; : : : ; n 2 R tais que
1 v1

2 v2

n vn

= 0.

Como
0 = 0v1 + 0v2 +

+ 0vn ,

e uma vez que por hiptese todo o vector de V se escreve de maneira nica como
combinao linear dos elementos de S, conclui-se que
1

= ::: =

= 0.

Logo, S = fv1 ; v2 ; : : : ; vn g uma base de V . Fica assim provada a equivalncia referida


na questo.
286

9. Seja fv1 ; v2 g uma base de um espao linear U . Considere os vectores


e

w1 = av1 + bv2

w2 = cv1 + dv2 ;

com a; b; c; d 2 R. Queremos provar que fw1 ; w2 g tambm uma base de U se e s se


ad 6= bc.
Dem. (() Suponhamos que ad =
6 bc. Vejamos que fw1 ; w2 g uma base de U .
Vamos comear por vericar que o conjunto fw1 ; w2 g linearmente independente:
Sejam 1 ; 2 2 R tais que
1 w1 + 2 w2 = 0.
Queremos ver que

1 w1

= 0. Observe-se que
2 w2

= 1 (av1 + bv2 ) + 2 (cv1 + dv2 )


= ( 1 a + 2 c)v1 + ( 1 b + 2 d)v2 .

Logo,
1 w1

2 w2

= 0 , ( 1a +

2 c)v1

+ ( 1b +

2 d)v2

= 0.

Como o conjunto fv1 ; v2 g uma base de U , em particular linearmente independente.


Logo,
1 a + 2 c = 1 b + 2 d = 0 2 R.
Isto ,
a c
b d

0
0

Ou seja,
A = 0,
0
a c
1
e0=
. Como ad 6= bc e det A = ad
, =
0
b d
2
det A 6= 0, isto , A invertvel e como tal:
onde A =

A 1A = A 10 , I = 0 ,

bc, ento

= 0.

Logo, 1 = 2 = 0 e deste modo o conjunto fw1 ; w2 g linearmente independente.


Como dim U = 2 e como w1 ; w2 so dois vectores de U , linearmente independentes,
ento conclui-se que fw1 ; w2 g uma base de U (no sendo necessrio vericar se o
conjunto fw1 ; w2 g gera U ).
()) Reciprocamente, se fw1 ; w2 g uma base de U , em particular linearmente independente, e como tal tem-se
( 1 w1 +

2 w2

= 0) ) (

= 0) .

Isto , a equao
A = 0,
a c
0
1
, =
e0=
, tem como soluo nica
b d
0
2
equivalente a ter-se det A 6= 0, isto , ad 6= bc.

onde A =

287

= 0. O que

Demonstrao alternativa. Como o conjunto fv1 ; v2 g uma base do espao linear


U ento dim U = 2. Logo, se o conjunto fw1 ; w2 g fr linearmente independente ento
ser uma base do espao linear U . Assim, bastar provar que o conjunto fw1 ; w2 g
a b
linearmente independente se e s se a matriz
fr invertvel. Seja 0 o vector
c d
nulo do espao linear U . Sejam 1 ; 2 2 R tais que
1 w1

Queremos ver que


que

1 w1

2 w2

= 0:
a b
c d

= 0 se e s se a matriz

2 w2

fr invertvel. Observe-se

= 1 (av1 + bv2 ) + 2 (cv1 + dv2 )


= ( 1 a + 2 c)v1 + ( 1 b + 2 d)v2 .

Logo,
1 w1

2 w2

= 0 , ( 1a +

2 c)v1

+ ( 1b +

2 d)v2

= 0.

Como o conjunto fv1 ; v2 g uma base do espao linear U , em particular linearmente


independente. Logo,
1 a + 2 c = 1 b + 2 d = 0 2 R.
Isto ,
a c
b d

0
0

0
a c
1
e 00 =
. Como a equao
,
=
0
b d
2
A = 00 apenas admite a soluo trivial = 00 se e s se a matriz A fr invertvel e
a b
como a matriz A invertvel se e s se a matriz AT =
fr invertvel, tem-se
c d
ento o resultado pretendido.
Ou seja, A = 00 , onde A =

10. Sejam A uma matriz m

n e B uma matriz n

dim C (AB) = dim C (B)

p. Mostre que

dim (N (A) \ C (B)) .

Sugesto: Considere (no caso em que N (A) \ C (B) 6= f0g) uma base fx1 ; : : : ; xs g
para N (A) \ C (B) e suponha (no caso em que AB 6= 0) que fx1 ; : : : ; xs ; y1 ; : : : ; yt g
uma base para C (B). Mostre que fAy1 ; : : : ; Ayt g uma base para C (AB).
Dem. Se N (A) \ C (B) = f0g, ento dim (N (A) \ C (B)) = 0 e dim C (AB) =
dim C (B) :
Suponhamos ento que N (A) \ C (B) 6= f0g. Seja fx1 ; : : : ; xs g uma base para N (A) \
C (B) e suponhamos que AB 6= 0 (no caso em que AB = 0 tem-se dim C (AB) = 0 e
dim C (B) = dim (N (A) \ C (B))

uma vez que C (B)

N (A)):

Seja fx1 ; : : : ; xs ; y1 ; : : : ; yt g uma base para C (B). Nesse caso dim C (AB) = s + t.
Vejamos que fAy1 ; : : : ; Ayt g uma base para C (AB).
288

Seja b 2 C (AB). Tem-se ABz = b para algum z. Mas, como Bz 2 C (B), ento
existem escalares 1 ; : : : ; s ; 1 ; : : : ; t tais que
Bz =

s
X

i xi

i=1

t
X

j yj .

j=1

Logo,
s
X

b = ABz = A

i xi

i=1

t
X

j yj

i=1

s
X

i Axi

i=1

t
X

j Ayj

j=1

fx1 ;:::;xs g N (A)

t
X

j Ayj ,

j=1

isto , fAy1 ; : : : ; Ayt g gera C (AB).

Vejamos que fAy1 ; : : : ; Ayt g linearmente independente. Suponhamos que existiam


escalares 1 ; : : : ; t tais que
t
X
0=
j Ayj .
j=1

Tem-se

0=

t
X

j Ayj

t
X

=A

j=1

e ento

t
X
j=1

j yj

j yj

j=1

2 N (A) \ C (B). E assim, existem escalares


t
X

j yj =

j=1

Como

t
X

j yj

j=1

s
X

1; : : : ;

tais que

i xi :

i=1

s
X

i xi

i=1

t
X

j yj

j=1

s
X

i xi

=0

i=1

e atendendo a que fx1 ; : : : ; xs ; y1 ; : : : ; yt g uma base para C (B), tem-se


1

= ::: =

= ::: =

=0

e assim o conjunto fAy1 ; : : : ; Ayt g linearmente independente.

Logo, o conjunto fAy1 ; : : : ; Ayt g uma base para C (AB) e assim


dim C (B) = s + t = dim (N (A) \ C (B)) + dim C (AB) ,
, dim C (AB) = dim C (B)

dim (N (A) \ C (B)) .

11. Considere os seguintes r vectores de Rn :


x1 = (x11 ; x12 ; : : : ; x1n ); x2 = (x21 ; x22 ; : : : ; x2n ); : : : ; xr = (xr1 ; xr2 ; : : : ; xrn ):
Mostre que se jxjj j >

r
P

i=1(i6=j)

jxij j para todo o j = 1; : : : ; r ento o conjunto


x1 ; x2 ; : : : ; xr
289

linearmente independente.
Sugesto: Considere
v = (v1 ; : : : ; vn ) =
com

1;

2; : : : ;

1
1x

2 R e mostre que se existir

2
2x
j

r
rx ;

6= 0 (com j 2 f1; : : : ; rg) tal que

j j j > j i j;
para todo o i = 1; : : : ; r; ento vj 6= 0.

Dem. Seja

v = (v1 ; : : : ; vn ) =
com

1;

2; : : : ;

1
1x

2
2x

2 R. Suponhamos que existe

r
rx ;

6= 0 (com j 2 f1; : : : ; rg) tal que

j j j > j i j;
para todo o i = 1; : : : ; r. Queremos mostrar que vj 6= 0.

Suponhamos ento (com vista a uma contradio) que vj = 0. Nesse caso, teramos
r
X

i xij = 0 ,

r
X

r
X

r
X

j xjj =

|i=1 {z }

i=1
i6=j

= vj

Como

j j j jxjj j = j j xjj j =
e

i xij

i=1
i6=j

i=1
i6=j

i xij .

j i xij j =

r
X
i=1
i6=j

j i j jxij j

j ij j j j
i=1;:::;r

B
j jj @

r
X
i=1
i6=j

C
jxij jA

6= 0 (com j 2 f1; : : : ; rg) ento teramos


0
1
r
BX
C
jxjj j
jxij jA
@
i=1
i6=j

o que contradiz a hiptese de se ter

jxjj j >

r
X

i=1(i6=j)

jxij j

para todo o j = 1; : : : ; r. Logo mostrmos que a existir j 6= 0 (com j 2 f1; : : : ; rg)


tal que j j j > j i j; para todo o i = 1; : : : ; r; ento vj 6= 0, o que equivale a dizer que o
conjunto
x1 ; x2 ; : : : ; xr
linearmente independente.

290

12. Seja y 2 C (A + B). Ento existe x tal que


y = (A + B) x = Ax + Bx 2 C (A) + C (B) :
Logo
C (A + B)
13. Seja A 2 Mm

n (R).

C (A) + C (B) :

Vejamos que
N AT \ C (A) = f0g :

Seja y 2 N AT \ C (A). Ento existe x tal que


AT y = 0

y = Ax:

Logo
y T = x T AT
e
0 = xT AT y = xT AT y = y T y:
Isto
T

0=y y=

n
X

yi2

i=1

ou seja

y = (y1 ; :::; yn ) = (0; :::; 0) = 0.


Logo, sem perda de generalidade,
N (A) \ L (A) = f0g :
14. Como B = C + B

C e C (B) = C (C + B

car B = dim C (B)


Pelo que car B

car C

C (C) + C (B

C)

dim C (C) + dim C (B

C) ento

C) = car C + car (B

C) :

C). De um modo anlogo, como

car (B

C = B + [ (B

C)]

e
ento

C (C) = C (B + [ (B

C)])

C (B) + C (B

car C

car B

car (B

C) :

jcar B

car Cj

car (B

C) :

C)

Logo
15. Sejam ; ; ; 2 R tais que

v + Av + A2 v + A3 v = 0:

Multiplicando a igualdade anterior por A3 e atendendo a que A4 = 0 e assim A5 =


A6 = A7 = 0, ento A3 v = 0 e deste modo = 0 uma vez que A3 v 6= 0 (v 2
= N (A3 )).
Analogamente: multiplicando a igualdade Av+ A2 v+ A3 v = 0 por A2 tem-se = 0,
multiplicando a igualdade A2 v + A3 v = 0 por A tem-se = 0 e nalmente de A3 v =
0 obtm-se = 0. Logo, o conjunto fv; Av; A2 v; A3 vg linearmente independente.

291

Resoluo da 4a Ficha de exerccios para as aulas de problemas

1. Sejam a; b 2 R. A aplicao Ta;b : R ! R denida por Ta;b (x) = ax + b linear se e


s se b = 0 e a 2 R.
2. (i) Seja T : R2 ! R2 com T (x; y) = (x + 2y; 3x
1
3

M (T ; Bc2 ; Bc2 ) =

y). T linear e tem-se

2
1

uma vez que T (1; 0) = (1; 3) e T (0; 1) = (2; 1). Tem-se


N (T ) =
=

(x; y) 2 R2 : T (x; y) = (0; 0) = (x; y) 2 R2 : (x + 2y; 3x


(x; y) 2 R2 : x = 2y e 3x = y = f(0; 0)g .

y) = (0; 0) =

Logo T injectiva e dim N (T ) = 0. Uma vez que


dim

R2
|{z}

espao de partida

= dim N (T ) + dim I(T ),

ento dim I(T ) = 2. Vejamos como encontrar uma base para I(T ). Tem-se
I(T ) = f(x + 2y; 3x

y) : x; y 2 Rg = fx(1; 3) + y(2; 1) : x; y 2 Rg = L (f(1; 3); (2; 1)g) .

Como o conjunto f(1; 3); (2; 1)g linearmente independente e como gera I(T ) ento f(1; 3); (2; 1)g
uma base de I(T ).
Por outro lado, como I(T ) subespao de R2 e dim I(T ) = dim R2 ento I(T ) = R2 , isto
, T sobrejectiva. Sendo T sobrejectiva e tendo-se dim (espao de partida) = dim (espao de
chegada) ento T tambm injectiva, como se constatou no facto de se ter N (T ) = f(0; 0)g.
Como T injectiva e sobrejectiva, ento T bijectiva.
Observao: T injectiva se e s se N (T ) = f0g, onde 0 o vector nulo do espao de
partida.
Resoluo alternativa para encontrar uma base para I(T ). Sendo
1
3

M (T ; Bc2 ; Bc2 ) =

2
1

a matriz que representa a transformao linear T em relao base cannica Bc2 no espao
de partida e no espao de chegada, tem-se
T (x; y) = M (T ; Bc2 ; Bc2 )

x
y

Logo,
N (T ) = N M (T ; Bc2 ; Bc2 ) = N

1
3
292

2
1

=N

1
0

2
7

= f(0; 0)g

e
1
3

I(T ) = C M (T ; Bc2 ; Bc2 ) = C

2
1

= L (f(1; 3); (2; 1)g) .

O conjunto f(1; 3); (2; 1)g uma base de I(T ).


(ii) Seja T : R2 ! R2 com T (x; y) = (1
(0; 0).

y; 2x). T no linear pois T (0; 0) = (1; 0) 6=

(iii) Seja T : R3 ! R3 com T (x; y; z) = (x; 2x;


2
1
3
3
4
2
M (T ; Bc ; Bc ) =
1

x). T linear e tem-se


3
0 0
0 0 5,
0 0

uma vez que T (1; 0; 0) = (1; 2; 1); T (0; 1; 0) = (0; 0; 0) e T (0; 0; 1) = (0; 0; 0). Tem-se
N (T ) = (x; y; z) 2 R3 : T (x; y; z) = (0; 0; 0) = (x; y; z) 2 R3 : (x; 2x; x) = (0; 0; 0) =
= (0; y; z) 2 R3 : y; z 2 R = y(0; 1; 0) + z(0; 0; 1) 2 R3 : y; z 2 R =
= L (f(0; 1; 0); (0; 0; 1)g) .
Como o conjunto f(0; 1; 0); (0; 0; 1)g linearmente independente e como gera N (T ) ento
f(0; 1; 0); (0; 0; 1)g
uma base de N (T ). Logo, dim N (T ) = 2. Uma vez que
dim

R3
|{z}

espao de partida

= dim N (T ) + dim I(T ),

ento dim I(T ) = 1. Vejamos como encontrar uma base para I(T ). Tem-se
I(T ) = f(x; 2x; x) : x 2 Rg = fx(1; 2; 1) : x 2 Rg = L (f(1; 2; 1)g) .
Como o conjunto f(1; 2; 1)g linearmente independente e como gera I(T ) ento f(1; 2; 1)g
uma base de I(T ).
Por outro lado, como I(T ) 6= R3 ento T no sobrejectiva. Como N (T ) 6= f(0; 0; 0)g
ento T no injectiva.
Resoluo alternativa para encontrar bases para N (T ) e I(T ). Sendo
2
3
1 0 0
M (T ; Bc3 ; Bc3 ) = 4 2 0 0 5 ,
1 0 0

a matriz que representa a transformao linear T em relao base cannica Bc3 no espao
de partida e no espao de chegada, tem-se
2 3
x
T (x; y; z) = M (T ; Bc3 ; Bc3 ) 4 y 5 .
z
293

Logo,
02

31
02
31
1 0 0
1 0 0
N (T ) = N M (T ; Bc3 ; Bc3 ) = N @4 2 0 0 5A = N @4 0 0 0 5A = L (f(0; 1; 0); (0; 0; 1)g)
1 0 0
0 0 0
e

02

31
1 0 0
I(T ) = C M (T ; Bc3 ; Bc3 ) = C @4 2 0 0 5A = L (f(1; 2; 1)g) .
1 0 0

O conjunto f(0; 1; 0); (0; 0; 1)g uma base de N (T ) e o conjunto f(1; 2; 1)g uma base de
I(T ).
(iv) Seja T : R3 ! R2 com T (x; y; z) = (0; 0). T linear e tem-se
M (T ; Bc3 ; Bc2 ) =

0 0 0
0 0 0

uma vez que T (1; 0; 0) = T (0; 1; 0) = T (0; 0; 1) = (0; 0). Tem-se


N (T ) = (x; y; z) 2 R3 : T (x; y; z) = (0; 0) = (x; y; z) 2 R3 : x; y; z 2 R = R3 .
Uma base para N (T ) poder ser a base cannica Bc3 . Logo, dim N (T ) = 3. Uma vez que
dim

R3
|{z}

espao de partida

ento dim I(T ) = 0. De facto

= dim N (T ) + dim I(T ),

I(T ) = f(0; 0)g .

Por outro lado, como I(T ) 6= R2 ento T no sobrejectiva. Como N (T ) 6= f(0; 0; 0)g
ento T no injectiva.
Resoluo alternativa para encontrar uma base para N (T ). Sendo
M (T ; Bc3 ; Bc2 ) =

0 0 0
0 0 0

a matriz que representa a transformao linear T em relao s bases cannicas Bc3 e Bc2 nos
espaos de partida e de chegada respectivamente, tem-se
2 3
x
3
2 4
T (x; y; z) = M (T ; Bc ; Bc ) y 5 .
z
Logo,

N (T ) = N M (T ; Bc3 ; Bc2 ) = N

0 0 0
0 0 0

= R3 = L (f(1; 0; 0); (0; 1; 0); (0; 0; 1)g)

e
I(T ) = C M (T ; Bc3 ; Bc2 ) = C
294

0 0 0
0 0 0

= f(0; 0)g .

Uma base para N (T ) poder ser a base cannica Bc3 .


(v) Seja T : R2 ! R com T (x; y) =

3x. T linear e tem-se

M (T ; Bc2 ; Bc ) =
uma vez que T (1; 0) =
Tem-se
N (T ) =
=

3 0 ,

3 e T (0; 1) = 0. Note que Bc = f1g a base cannica de R.

(x; y) 2 R2 : T (x; y) = 0 = (x; y) 2 R2 : 3x = 0 =


(0; y) 2 R2 : y 2 R = y(0; 1) 2 R2 : y 2 R = L (f(0; 1)g) .

Como o conjunto f(0; 1)g linearmente independente e como gera N (T ) ento f(0; 1)g
uma base de N (T ). Logo, dim N (T ) = 1. Uma vez que
dim

R2
|{z}

espao de partida

= dim N (T ) + dim I(T ),

ento dim I(T ) = 1. Vejamos como encontrar uma base para I(T ). Tem-se
I(T ) = f 3x : x 2 Rg = L (f1g) .
Como o conjunto f1g linearmente independente e como gera I(T ) ento f1g uma base
de I(T ), a base cannica de R.
Por outro lado, como I(T ) subespao de R e dim I(T ) = dim R ento I(T ) = R, isto
, T sobrejectiva. Como N (T ) 6= f(0; 0)g ento T no injectiva.
Resoluo alternativa para encontrar bases para N (T ) e I(T ). Sendo
M (T ; Bc2 ; Bc ) =

3 0 ,

a matriz que representa a transformao linear T em relao s bases cannicas Bc2 no espao
de partida e Bc no espao de chegada, tem-se
T (x; y) = M (T ; Bc2 ; Bc )

x
y

Logo,
N (T ) = N M (T ; Bc2 ; Bc ) = N

3 0

= L (f(0; 1)g)

e
I(T ) = C M (T ; Bc2 ; Bc ) = C

3 0

= L (f 3g) = L (f1g) .

O conjunto f(0; 1)g uma base de N (T ) e o conjunto f1g uma base de I(T ).
(vi) T : R3 ! R3 com T (x; y; z) = (0; 1; 2). T no linear pois T (0; 0; 0) = (0; 1; 2) 6=
(0; 0; 0).
(vii) T : R ! R3 com T (x) = (2x; 0; x). T linear e tem-se
2
3
2
M (T ; Bc ; Bc3 ) = 4 0 5 ,
1
295

uma vez que T (1) = (2; 0; 1). Tem-se


N (T ) = fx 2 R : T (x) = (0; 0; 0)g = fx 2 R : (2x; 0; x) = (0; 0; 0)g = f0g .
Logo, dim N (T ) = 0. Uma vez que
dim

R
|{z}

espao de partida

= dim N (T ) + dim I(T ),

ento dim I(T ) = 1. Vejamos como encontrar uma base para I(T ). Tem-se
I(T ) = f(2x; 0; x) : x 2 Rg = fx(2; 0; 1) : x 2 Rg = L (f(2; 0; 1)g) .
Como o conjunto f(2; 0; 1)g linearmente independente e como gera I(T ) ento f(2; 0; 1)g
uma base de I(T ).
Por outro lado, como I(T ) 6= R3 ento T no sobrejectiva. Como N (T ) = f0g ento T
injectiva.
Resoluo alternativa para encontrar uma base para I(T ). Sendo
2
3
2
M (T ; Bc ; Bc3 ) = 4 0 5 ,
1

a matriz que representa a transformao linear T em relao s bases cannicas Bc no espao


de partida e Bc3 no espao de chegada, tem-se
T (x) = M (T ; Bc3 ; Bc3 ) [x] .
Logo,
02

31
02 31
2
2
3
@
4
5
A
@
4
0
0 5A = L (f0g) = f0g
N (T ) = N M (T ; Bc ; Bc ) = N
=N
1
0
02

31
2
I(T ) = C M (T ; Bc ; Bc3 ) = C @4 0 5A = L (f(2; 0; 1)g) .
1

O conjunto f(2; 0; 1)g uma base de I(T ).


(viii) T : R3 ! R2 com T (x; y; z) = (x2

y; 2y). T no linear, pois por exemplo:

T ((1; 0; 0) + (1; 0; 0)) = T (2; 0; 0) = (4; 0) 6= (2; 0) = T (1; 0; 0) + T (1; 0; 0).


(ix) Seja T : R4 ! R2 com T (x; y; z; w) = (x
M (T ; Bc4 ; Bc2 ) =

1
0

296

y; 3w). T linear e tem-se


1 0 0
0 0 3

uma vez que T (1; 0; 0; 0) = (1; 0); T (0; 1; 0; 0) = ( 1; 0); T (0; 0; 1; 0) = (0; 0) e T (0; 0; 0; 1) =
(0; 3). Tem-se
N (T ) =
=
=

(x; y; z; w) 2 R4 : T (x; y; z; w) = (0; 0) = (x; y; z; w) 2 R4 : (x y; 3w) = (0; 0) =


(x; y; z; w) 2 R4 : x = y e w = 0 = (y; y; z; 0) 2 R4 : y; z 2 R =
y(1; 1; 0; 0) + z(0; 0; 1; 0) 2 R4 : y; z 2 R = L (f(1; 1; 0; 0); (0; 0; 1; 0)g) .

Como o conjunto f(1; 1; 0; 0); (0; 0; 1; 0)g linearmente independente e como gera N (T ) ento
f(1; 1; 0; 0); (0; 0; 1; 0)g uma base de N (T ). Logo, dim N (T ) = 2. Uma vez que
dim

R4
|{z}

espao de partida

= dim N (T ) + dim I(T ),

ento dim I(T ) = 2. Vejamos como encontrar uma base para I(T ). Tem-se
I(T ) = f(x y; 3w) : x; y; w 2 Rg = fx(1; 0) + y( 1; 0) + w(0; 3) : x; y; w 2 Rg =
= L (f(1; 0); ( 1; 0); (0; 3)g) .
Como o conjunto f(1; 0); (0; 3)g linearmente independente e como gera I(T ) ento f(1; 0); (0; 3)g
uma base de I(T ).
Por outro lado, como I(T ) subespao de R2 e dim I(T ) = dim R2 ento I(T ) = R2 ,
isto , T sobrejectiva. Como N (T ) 6= f(0; 0; 0; 0)g ento T no injectiva.
Resoluo alternativa para encontrar bases para N (T ) e I(T ). Sendo
M (T ; Bc4 ; Bc2 ) =

1
0

1 0 0
0 0 3

a matriz que representa a transformao linear T em relao s bases cannicas Bc4 no espao
de partida e Bc2 no espao de chegada, tem-se
2
3
x
6 y 7
7
T (x; y; z; w) = M (T ; Bc4 ; Bc2 ) 6
4 z 5.
w
Logo,

N (T ) = N M (T ; Bc4 ; Bc2 ) = N

1
0

1 0 0
0 0 3

= L (f(1; 1; 0; 0); (0; 0; 1; 0)g)

e
I(T ) = C M (T ; Bc2 ; Bc2 ) = C

1
0

1 0 0
0 0 3

= L (f(1; 0); (0; 3)g) .

O conjunto f(1; 1; 0; 0); (0; 0; 1; 0)g uma base de N (T ) e o conjunto f(1; 0); (0; 3)g uma
base de I(T ).
(x) Seja T : R3 ! R4 com T (x; y; z) = ( z; y
2
0
6
0
M (T ; Bc3 ; Bc4 ) = 6
4 0
0
297

2z; 2y; y + z). T linear e tem-se


3
0
1
1
2 7
7,
2 0 5
1 1

uma vez que T (1; 0; 0) = (0; 0; 0); T (0; 1; 0) = (0; 1; 2; 1) e T (0; 0; 1) = ( 1; 2; 0; 1).
Tem-se
N (T ) =
=
=

(x; y; z) 2 R3 : T (x; y; z) = (0; 0; 0; 0) =


(x; y; z) 2 R3 : ( z; y 2z; 2y; y + z) = (0; 0; 0; 0) =
(x; 0; 0) 2 R3 : x 2 R = L (f(1; 0; 0)g) .

Como o conjunto f(1; 0; 0)g linearmente independente e como gera N (T ) ento f(1; 0; 0)g
uma base de N (T ). Logo, dim N (T ) = 1. Uma vez que
dim

R3
|{z}

espao de partida

= dim N (T ) + dim I(T ),

ento dim I(T ) = 2. Vejamos como encontrar uma base para I(T ). Tem-se
I(T ) = f( z; y

2z; 2y; y + z) : y; z 2 Rg = L (f(0; 1; 2; 1); ( 1; 2; 0; 1)g) .

Como o conjunto f(0; 1; 2; 1); ( 1; 2; 0; 1)g linearmente independente e como gera I(T )
ento f(0; 1; 2; 1); ( 1; 2; 0; 1)g uma base de I(T ).
Por outro lado, como I(T ) 6= R4 ento T no sobrejectiva. Como N (T ) 6= f(0; 0; 0)g
ento T no injectiva.
Resoluo alternativa para encontrar bases para N (T ) e I(T ). Sendo
2
3
0 0
1
6 0 1
2 7
7
M (T ; Bc3 ; Bc4 ) = 6
4 0 2 0 5,
0 1 1

a matriz que representa a transformao linear T em relao base cannica Bc3 no espao
de partida e no espao de chegada, tem-se
2 3
x
T (x; y; z) = M (T ; Bc3 ; Bc4 ) 4 y 5 .
z
Logo,

02

0
B
6
6 0
N (T ) = N M (T ; Bc3 ; Bc4 ) = N B
@4 0
0
02
02
31
0 0
1
B6 0 1 0 7C
B6
6
7C
B6
= NB
@4 0 2 0 5A = N @4
0 1 0
022

0
B
6
6
66 0
I(T ) = C M (T ; Bc3 ; Bc4 ) = C B
@44 0
0

0
1
2
1

0
1
2
1
0
0
0
0

31
1
C
2 7
7C =
0 5A
1
31
0
1
C
1 0 7
7C = L (f(1; 0; 0)g)
5
0 0 A
0 0

331
1
7C
2 7
77C = L (f(0; 1; 2; 1); ( 1; 2; 0; 1)g) .
5
0 5A
1
298

O conjunto f(1; 0; 0)g uma base de N (T ) e o conjunto f(0; 1; 2; 1); ( 1; 2; 0; 1)g uma
base de I(T ).
(xi) Seja T : R ! R2 com T (x) = (0; 0). T linear e tem-se
M (T ; Bc ; Bc2 ) =

0
0

uma vez que T (1) = (0; 0). Tem-se


N (T ) = fx 2 R : T (x) = (0; 0)g = fx : x 2 Rg = R.
Uma base para N (T ) poder ser a base cannica Bc = f1g. Logo, dim N (T ) = 1. Uma vez
que
dim
R
= dim N (T ) + dim I(T ),
|{z}
espao de partida

ento dim I(T ) = 0. De facto

I(T ) = f(0; 0)g .

Por outro lado, como I(T ) 6= R2 ento T no sobrejectiva. Como N (T ) 6= f0g ento T
no injectiva.
Resoluo alternativa para encontrar uma base para N (T ). Sendo
M (T ; Bc ; Bc2 ) =

0
0

a matriz que representa a transformao linear T em relao s bases cannicas Bc e Bc2 nos
espaos de partida e de chegada respectivamente, tem-se
T (x) = M (T ; Bc ; Bc2 ) x

Logo,
N (T ) = N M (T ; Bc ; Bc2 ) = N

0
0

= R = L (f1g)

I(T ) = C M (T ; Bc ; Bc2 ) = C

0
0

= f(0; 0)g .

Uma base para N (T ) poder ser a base cannica Bc = f1g.


(xii) Seja T : R3 ! R3 com T (x; y; z) = (x + 2y; 3z; x z). T linear e tem-se
2
3
1 2 0
M (T ; Bc3 ; Bc3 ) = 4 0 0 3 5 ,
1 0
1

uma vez que T (1; 0; 0) = (1; 0; 1); T (0; 1; 0) = (2; 0; 0) e T (0; 0; 1) = (0; 3; 1). Tem-se
N (T ) = (x; y; z) 2 R3 : T (x; y; z) = (0; 0; 0) =
= (x; y; z) 2 R3 : (x + 2y; 3z; x z) = (0; 0; 0) =
= f(0; 0; 0)g .
299

Logo, dim N (T ) = 0 e T injectiva. Uma vez que


dim

R3
|{z}

espao de partida

= dim N (T ) + dim I(T ),

ento dim I(T ) = 3. Vejamos como encontrar uma base para I(T ). Tem-se
I(T ) = f(x + 2y; 3z; x z) : x; y; z 2 Rg =
= fx(1; 0; 1) + y(2; 0; 0) + z(0; 3; 1) : x; y; z 2 Rg
= L (f(1; 0; 1); (2; 0; 0); (0; 3; 1)g) .
Como o conjunto f(1; 0; 1); (2; 0; 0); (0; 3; 1)g linearmente independente e como gera I(T )
ento f(1; 0; 1); (2; 0; 0); (0; 3; 1)g uma base de I(T ).
Por outro lado, como I(T ) subespao de R3 e dim I(T ) = dim R3 ento I(T ) = R3 ,
isto , T sobrejectiva.
Como T injectiva e sobrejectiva, ento T bijectiva.
Resoluo alternativa para encontrar uma base para I(T ). Sendo
2
3
1 2 0
M (T ; Bc3 ; Bc3 ) = 4 0 0 3 5 ,
1 0
1

a matriz que representa a transformao linear T em relao base cannica Bc3 no espao
de partida e no espao de chegada, tem-se
2 3
x
T (x; y; z) = M (T ; Bc3 ; Bc3 ) 4 y 5 .
z
Logo,

02

1 2
3
3
@
4
0 0
N (T ) = N M (T ; Bc ; Bc ) = N
1 0
02
31
02
1 2 0
1
@
4
5
A
@
4
1 0
1
0
= N
=N
0 0 3
0
02

1 2
3
3
@
4
0 0
I(T ) = C M (T ; Bc ; Bc ) = C
1 0

31
0
3 5A =
1
31
2
0
2
1 5A = f(0; 0; 0)g
0
3

31
0
3 5A = L (f(1; 0; 1); (2; 0; 0); (0; 3; 1)g) .
1

O conjunto f(1; 0; 1); (2; 0; 0); (0; 3; 1)g uma base de I(T ).
(xiii) Seja T : R3 ! R3 com T (x; y; z) = (x; y; z).
2
1 0
M (T ; Bc3 ; Bc3 ) = 4 0 1
0 0
300

T linear e tem-se
3
0
0 5,
1

uma vez que T (1; 0; 0) = (1; 0; 0); T (0; 1; 0) = (0; 1; 0) e T (0; 0; 1) = (0; 0; 1). Tem-se
N (T ) = (x; y; z) 2 R3 : T (x; y; z) = (0; 0; 0) = f(0; 0; 0)g .
Logo, dim N (T ) = 0 e T injectiva. Uma vez que
R3
|{z}

dim

espao de partida

= dim N (T ) + dim I(T ),

ento dim I(T ) = 3. Vejamos como encontrar uma base para I(T ). Tem-se
I(T ) = f(x; y; z) : x; y; z 2 Rg = R3 ,
isto , T sobrejectiva. Como o conjunto f(1; 0; 0); (0; 1; 0); (0; 0; 1)g linearmente independente e como gera I(T ) ento f(1; 0; 0); (0; 1; 0); (0; 0; 1)g uma base de I(T ).
Como T injectiva e sobrejectiva, ento T bijectiva.
(xiv) Seja T : R2 ! R2 com T (x; y) = (x cos
y sen ; x sen + y cos ),
linear e
cos
sen
cos
sen
M (T ; Bc3 ; Bc3 ) =
!
,
sen
cos
sen
cos

2 R. T

uma vez que T (1; 0) = (cos ; sen ) e T (0; 1) = ( sen ; cos ).


Atendendo ao exo 4 (viii) da cha 2, tem-se, para todo o 2 R,
M (T ; Bc3 ; Bc3 )

cos
sen

sen
cos

Logo
N (T ) = (x; y) 2 R2 : T (x; y) = (0; 0) = f(0; 0)g

e dim N (T ) = 0, isto , T injectiva.


Sendo T injectiva e tendo-se dim (espao de partida) = dim (espao de chegada) ento
T tambm sobrejectiva.
Como T injectiva e sobrejectiva, ento T bijectiva.
Como o conjunto f(1; 0); (0; 1)g linearmente independente e como gera I(T ) ento
f(1; 0); (0; 1)g uma base de I(T ).
(xv) Seja T : P2 ! P2 com
T (p (t)) = 2p (1

t)

tp0 (t) :

T linear uma vez que, para todos os p (t) ; p1 (t) ; p2 (t) 2 P2 , para todo o
T (p1 (t) + p2 (t)) =
=
=
=

2 R,

T ((p1 + p2 ) (t)) = 2 (p1 + p2 ) (1 t) t (p1 + p2 )0 (t) =


2p1 (1 t) + 2p2 (1 t) tp01 (t) tp02 (t) =
2p1 (1 t) tp01 (t) + 2p2 (1 t) tp02 (t) =
T (p1 (t)) + T (p2 (t)) ,

T ( p (t)) = T (( p) (t)) = 2 ( p) (1 t) t ( p)0 (t) =


= 2p (1 t) t p0 (t) = (2p (1 t) tp0 (t)) = T (p (t)).
301

Sendo B = f1; t; t2 g a base cannica de P2 , tem-se


2
3
2 2
2
3
4 5,
M (T ; B; B) = 4 0
0 0
0

uma vez que T (1) = 2

T (t2 ) = 2 (1
Uma base para N (T ):
Como

0 = 2; T (t) = 2 (1
t)2

t2t = 2

02

2
N (M (T ; B; B)) = N @4 0
0

ento

2
3
0

t)

4t + 2t2

3t e

1=2

2t2 = 2

4t:

31
2
4 5A = L (f(1; 4; 3)g) ,
0

N (T ) = a0 + a1 t + a2 t2 2 P2 : (a0 ; a1 ; a2 ) 2 L (f(1; 4; 3)g) = L

4t + 3t2

Como f1 4t + 3t2 g uma base de N (T ), dim N (T ) = 1. Logo, T no injectiva, uma


vez que dim N (T ) 6= 0.
Resoluo alternativa para encontrar uma base para N (T ):
N (T ) =

=
=
=
=

a0 + a1 t + a2 t2 2 P2 : T a0 + a1 t + a2 t2 = 0 =

a0 + a1 t + a2 t2 2 P2 : 2 a0 + a1 (1 t) + a2 (1 t)2
t (a1 + 2a2 t) = 0 =
2
a0 + a1 t + a2 t 2 P2 : 2a0 + 2a1 2a1 t + 2a2 4a2 t + 2a2 t2 a1 t 2a2 t2 = 0 =
a0 + a1 t + a2 t2 2 P2 : 2a0 + 2a1 + 2a2 + ( 3a1 4a2 ) t = 0 =
1
4
a2 e a0 = a2 =
a0 + a1 t + a2 t2 2 P2 : a1 =
3
3
1 4
4
1
a2
a2 t + a2 t2 2 P2 : a2 2 R = L
t + t2
= L 1 4t + 3t2 .
3
3
3 3

Como f1 4t + 3t2 g uma base de N (T ), dim N (T ) = 1.


Uma base para I(T ):
Como f1; t; t2 g gera P2 , tem-se
I (T ) = L

T (1) ; T (t) ; T t2

= L (f2; 2

3t; 2

4tg) = L (f2; 2

3tg) :

Uma vez que o conjunto f2; 2 3tg linearmente independente e gera I (T ), ento f2; 2
uma base de I (T ), tendo-se dim I (T ) = 2.
Como dim P2 = 3, tem-se I (T ) 6= P2 , pelo que T no sobrejectiva.
Resoluo alternativa para encontrar uma base para I(T ):
Sendo p (t) = a0 + a1 t + a2 t2 , com a0 ; a1 ; a2 2 R, tem-se
T (p (t)) = 2 a0 + a1 (1 t) + a2 (1 t)2
t (a1 + 2a2 t) =
= 2a0 + 2a1 2a1 t + 2a2 4a2 t + 2a2 t2 a1 t 2a2 t2 =
= a0 2 + a1 (2 3t) + a2 (2 4t) .
302

3tg

Logo, I(T ) = L (f2; 2 3t; 2 4tg) = L (f2; 2 3tg). Uma vez que o conjunto f2; 2
linearmente independente e gera I (T ), ento f2; 2 3tg uma base de I (T ).

3tg

(xvi) Seja T : P2 ! P2 com


T (p (t)) = p (0)

p ( 1) + (p ( 1) + p (1)) t + (p ( 1)

p (1)

2p (0)) t2 :

T linear uma vez que, para todos os p (t) ; p1 (t) ; p2 (t) 2 P2 , para todo o

2 R,

T (p1 (t) + p2 (t)) = T ((p1 + p2 ) (t)) =


= (p1 + p2 ) (0) (p1 + p2 ) ( 1) + ((p1 + p2 ) ( 1) + (p1 + p2 ) (1)) t +
+ ((p1 + p2 ) ( 1) (p1 + p2 ) (1) 2 (p1 + p2 ) (0)) t2
= p1 (0) p1 ( 1) + (p1 ( 1) + p1 (1)) t + (p1 ( 1) p1 (1) 2p1 (0)) t2 +
+p2 (0) p2 ( 1) + (p2 ( 1) + p2 (1)) t + (p2 ( 1) p2 (1) 2p2 (0)) t2
= T (p1 (t)) + T (p2 (t)) ,
T ( p (t)) = T (( p) (t)) =
= T (p (t)).

p (0)

p ( 1) + (p ( 1) + p (1)) t + (p ( 1)

Sendo B = f1; t; t2 g a base cannica de P2 , tem-se


2
0
1
4
2
0
M (T ; B; B) =
2
2

uma vez que T (1) = 1

T (t) = 0

1 + (1 + 1) t + (1

p (1)

2p (0)) t2 =

3
1
2 5,
0

2) t2 = 2t

( 1) + (( 1) + 1) t + (( 1)

2t2 ;

0) t2 = 1

2t2

e
T (t2 ) = 0

1 + (1 + 1) t + (1

Uma base para N (T ):


Como
02

31
02
0
1
1
0
@
4
5
A
@
4
2
0
2
2
N (M (T ; B; B)) = N
=N
2
2 0
0
= L (f( 1; 1; 1)g) ,

0) t2 =

1
0
2

1 + 2t:

31
02
1
2 0
5
A
@
4
2
0 1
=N
2
0 0

31
2
1 5A =
0

ento
N (T ) = a0 + a1 t + a2 t2 2 P2 : (a0 ; a1 ; a2 ) 2 L (f( 1; 1; 1)g) = L

1 + t + t2

Como f 1 + t + t2 g uma base de N (T ), dim N (T ) = 1. Logo, T no injectiva, uma vez


que dim N (T ) 6= 0.
303

Resoluo alternativa para encontrar uma base para N (T ):


N (T ) =
=

a0 + a1 t + a2 t2 2 P2 : T a0 + a1 t + a2 t2 = 0 =

a0 + a1 t + a2 t2 2 P2 : p (0) p ( 1) + (p ( 1) + p (1)) t+
+ (p ( 1) p (1) 2p (0)) t2 = 0

a0 + a1 t + a2 t2 2 P2 : p (0)
p ( 1)

p ( 1) = 0 e p ( 1) + p (1) = 0 e
p (1) 2p (0) = 0
8
9
< a0 + a1 t + a2 t2 2 P2 : a0 (a0 a1 + a2 ) = 0 e =
(a0 a1 + a2 ) + (a0 + a1 + a2 ) = 0 e
=
=
:
;
(a0 a1 + a2 ) (a0 + a1 + a2 ) 2a0 = 0

= a0 + a1 t + a2 t2 2 P2 : a1 = a2 e a0 = a2 =
=
a2 + a2 t + a2 t2 2 P2 : a2 2 R = a2 1 + t + t2 2 P2 : a2 2 R =
= L
1 + t + t2 .

Como f 1 + t + t2 g uma base de N (T ), dim N (T ) = 1.


Uma base para I(T ):
Como f1; t; t2 g gera P2 , tem-se
T (1) ; T (t) ; T t2

I (T ) = L

Uma vez que o conjunto f1

=L

2t2 ; 1

2t

2t2 ; 1 + 2t

=L

2t2 ; 1 + 2t

2t2 ; 1 + 2tg linearmente independente e gera I (T ), ento


1

2t2 ; 1 + 2t

uma base de I (T ), tendo-se dim I (T ) = 2.


Como dim P2 = 3, tem-se I (T ) 6= P2 , pelo que T no sobrejectiva.
Resoluo alternativa para encontrar uma base para I(T ):
Sendo p (t) = a0 + a1 t + a2 t2 , com a0 ; a1 ; a2 2 R, tem-se
T (p (t)) = p (0)
= a0

p ( 1) + (p ( 1) + p (1)) t + (p ( 1)

2p (0)) t2 =

a0 + a1 ( 1) + a2 ( 1)2 + a0 + a1 ( 1) + a2 ( 1)2 + a0 + a1 + a2 t+
+ a0 + a1 ( 1) + a2 ( 1)2

= a1

p (1)

a2 + (2a0 + 2a2 ) t + ( 2a0

Logo, I(T ) = L (f2t

2t2 ; 1

2a1 ) t2 = a0 2t

2t2 + a1 1

2t2 ; 1 + 2tg) = L (f2t


2t

2a0 t2 =

(a0 + a1 + a2 )

2t2 ; 1

2t2 ; 1

2t2 + a2 ( 1 + 2t) .

2t2 g). Como o conjunto

2t2

linearmente independente e gera I (T ), ento f2t

2t2 ; 1

2t2 g uma base de I (T ).

p (1) p (0)
.
p (0) p ( 1)
T linear uma vez que, para todos os p (t) ; p1 (t) ; p2 (t) 2 P2 , para todo o

(xvii) Seja T : P2 ! M2

(R) com T (p (t)) =

T (p1 (t) + p2 (t)) = T ((p1 + p2 ) (t)) =

304

(p1 + p2 ) (1) (p1 + p2 ) (0)


(p1 + p2 ) (0) (p1 + p2 ) ( 1)

2 R,

p1 (1) + p2 (1)
p1 (0) + p2 (0)
p1 (0) + p2 (0) p1 ( 1) + p2 ( 1)

p1 (1) p1 (0)
p1 (0) p1 ( 1)

p2 (1) p2 (0)
p2 (0) p2 ( 1)

= T (p1 (t)) + T (p2 (t)) ,


T ( p (t)) = T (( p) (t)) =

( p) (1) ( p) (0)
( p) (0) ( p) ( 1)

p (1) p (0)
p (0) p ( 1)

p (1)
p (0)

p (0)
p ( 1)

= T (p (t)).

Sendo B1 = f1; t; t2 g a base cannica de P2 e


B2 =
a base cannica de M2

0 1
0 0

0 0
1 0

1
0
0
1

3
1
0 7
7,
0 5
1

1
6 1
M (T ; B1 ; B2 ) = 6
4 1
1
T (1) =

Clculo de N (T ):
Como

0 0
0 1

(R) tem-se

uma vez que

ento

1 0
0 0

1 1
1 1

T (t) =

1
0

0
1

T (t2 ) =

31
02
1 1 1
1
B6 1 0 0 7C
B6 0
6
7C
B6
N (M (T ; B1 ; B2 )) = N B
@4 1 0 0 5A = N @4 0
1
1 1
0
02
31
1 1
1
B6 0
C
2 0 7
6
7C = f(0; 0; 0)g ;
=NB
@4 0 0
1 5A
0 0
0
02

1 0
0 1

31
1
C
1 7
7C =
5
1 A
0

1
1
1
2

N (T ) = a0 + a1 t + a2 t2 2 P2 : (a0 ; a1 ; a2 ) = (0; 0; 0) = f0g .

Logo, T injectiva uma vez que dim N (T ) = 0.


Resoluo alternativa para calcular N (T ):
N (T ) =

p (t) = a0 + a1 t + a2 t2 2 P2 : T (p (t)) =

0 0
0 0

p (1) p (0)
p (0) p ( 1)

=
0 0
0 0

p (t) = a0 + a1 t + a2 t2 2 P2 :

a0 + a1 t + a2 t2 2 P2 :

a0 + a1 t + a2 t2 2 P2 : a0 = 0 e a1 = a2 = 0 = f0g .

a0 + a1 + a2
a0
a0

305

a0
a1 + a2

=
0 0
0 0

Uma base para I(T ):


Como f1; t; t2 g gera P2 , tem-se
I (T ) = L

T (1) ; T (t) ; T t2
1 1
1 1

Uma vez que o conjunto


gera I (T ), ento

1 1
1 1

=L
1
0

1 1
1 1

0
1
1
0

1 0
0 1

;
0
1

1
0

0
1

1 0
0 1

linearmente independente e

1 0
0 1

uma base de I (T ), tendo-se dim I (T ) = 3.


Como dim M2 2 (R) = 4, tem-se I (T ) 6= M2 2 (R), pelo que T no sobrejectiva.
Resoluo alternativa para encontrar uma base para I(T ):
Sendo p (t) = a0 + a1 t + a2 t2 , com a0 ; a1 ; a2 2 R, tem-se
T (p (t)) =

p (1) p (0)
p (0) p ( 1)

a0 a0
a0 a0

Logo, I(T ) = L

1 1
1 1

1
0

a1
0

1 1
1 1

= a0

1 1
1 1

;
;

a0
a1 + a2

0
a1

1
0

+ a1
0
1

a0 + a1 + a2
a0
a0
+

0
1

+ a2

1 0
0 1
1
0

0
1

a2 0
0 a2
1 0
0 1

. Como o conjunto
;

1 0
0 1

linearmente independente e gera I (T ), ento uma base de I (T ).


3. Considere a transformao linear T : R3 ! R3 que em relao base cannica
(ordenada) Bc3 = f(1; 0; 0); (0; 1; 0); (0; 0; 1)g de R3 representada pela matriz
2
3
1
2 1
1 0 5.
M (T ; Bc3 ; Bc3 ) = 4 1
2
1 0
Tem-se

3 2
x
1
3
3 4
5
4
T (x; y; z) = M (T ; Bc ; Bc ) y = 1
z
2

Tem-se

32 3
2 1
x
5
4
1 0
y 5 = (x + 2y + z; x + y; 2x
1 0
z

N (T ) = (x; y; z) 2 R3 : T (x; y; z) = (0; 0; 0) =


= (x; y; z) 2 R3 : (x + 2y + z; x + y; 2x y) = (0; 0; 0) =
= f(0; 0; 0)g .
306

y).

Logo, dim N (T ) = 0 e T injectiva. Uma vez que


dim

R3
|{z}

espao de partida

= dim N (T ) + dim I(T ),

ento dim I(T ) = 3. Vejamos como encontrar uma base para I(T ). Tem-se
I(T ) = f(x + 2y + z; x + y; 2x

y) : x; y; z 2 Rg = L (f(1; 1; 2); (2; 1; 1); (1; 0; 0)g) .

Como o conjunto f(1; 1; 2); (2; 1; 1); (1; 0; 0)g linearmente independente e como gera I(T )
ento f(1; 1; 2); (2; 1; 1); (1; 0; 0)g uma base de I(T ).
Por outro lado, como I(T ) subespao de R3 e dim I(T ) = dim R3 ento I(T ) = R3 ,
isto , T sobrejectiva.
Como T injectiva e sobrejectiva, ento T bijectiva.
Resoluo alternativa para encontrar uma base para I(T ). Sendo
2
3
1
2 1
1 0 5,
M (T ; Bc3 ; Bc3 ) = 4 1
2
1 0

a matriz que representa a transformao linear T em relao base cannica Bc3 no espao
de partida e no espao de chegada, tem-se
2 3
x
3
3 4
T (x; y; z) = M (T ; Bc ; Bc ) y 5 .
z
Logo,

02

31
1 2 1
N (T ) = N M (T ; Bc3 ; Bc3 ) = N @4 1 1 0 5A =
2
1 0
02
31
02
31
1 2
1
1 2
1
1
1 5A = N @4 0
1
1 5A = f(0; 0; 0)g
= N @4 0
0
5
2
0 0
3

02

1
I(T ) = C M (T ; Bc3 ; Bc3 ) = C @4 1
2

31
2 1
1 0 5A = L (f(1; 1; 2); (2; 1; 1); (1; 0; 0)g) .
1 0

O conjunto f(1; 1; 2); (2; 1; 1); (1; 0; 0)g uma base de I(T ).

4. Considere a base ordenada B = fv1 ; v2 g de R2 , em que v1 = (1; 1) e v2 = (1; 0) e seja


T : R2 ! R2 a transformao linear tal que
T (v1 ) = (1; 2), T (v2 ) = ( 3; 1).
(i) Tem-se T (2; 1) = T ((1; 1) + (1; 0)) |{z}
= T (1; 1) + T (1; 0) = (1; 2) + ( 3; 1) =
T linear

( 2; 1).

307

(ii) Seja (x; y) 2 R2 . Tem-se


(x; y) = y(1; 1) + (x

y)(1; 0).

Logo,
T (x; y) = T (y(1; 1) + (x

y)(1; 0)) |{z}


= yT (1; 1) + (x

y)T (1; 0) =

T linear

= y(1; 2) + (x

y)( 3; 1) = ( 3x + 4y; x

3y).

(iii) Tem-se
3
1

M (T ; Bc2 ; Bc2 ) =

4
3

uma vez que, pela alnea (ii), T (1; 0) = ( 3; 1) e T (0; 1) = (4; 3).
Observao: Poderamos ter calculado T (1; 0) e T (0; 1) sem recorrer alinea (ii), uma
vez que
(1; 0) = 0(1; 1) + (1; 0) e (0; 1) = (1; 1) (1; 0).
Logo, sendo T linear, tem-se (usando s o enunciado)
T (1; 0) = ( 3; 1) e T (0; 1) = T (1; 1)

T (1; 0) = (1; 2)

( 3; 1) = (4; 3).

(iv) Tem-se
0
1

SBc2 !B =

1
1

uma vez que


(1; 0) = 0(1; 1) + (1; 0) e (0; 1) = (1; 1)

(1; 0).

Tem-se
1 1
1 0

SB!Bc2 =
uma vez que

(1; 1) = (1; 0) + (0; 1) e (1; 0) = (1; 0) + 0(0; 1).


As coordenadas do vector (2; 1) na base B so dadas por:
SBc2 !B

2
1

0
1

1
1

2
1

1
1

Observao 1: Na verdade poderamos ter determinado as coordenadas do vector (2; 1) na


base B usando a denio de coordenadas de um vector numa base:
(2; 1) = (1; 1) + (1; 0).
Logo, as coordenadas do vector (2; 1) na base B so precisamente 1 e 1.
Observao 2: Tem-se
SB!Bc2 = SBc2 !B

e SBc2 !B = SB!Bc2
308

(v) Determinemos a matriz M (T ; B; B) usando s a denio de matriz que representa


uma transformao linear em relao a uma base ordenada B no espao de partida e no
espao de chegada. Tem-se
2 1
M (T ; B; B) =
,
3
4
uma vez que
2(1; 1) + 3(1; 0) e T (1; 0) = ( 3; 1) = (1; 1)

T (1; 1) = (1; 2) =

4(1; 0).

Determinemos agora as coordenadas do vector T (2; 1) na base B sem usar as alneas anteriores. Tem-se
T (2; 1) = T ((1; 1) + (1; 0)) |{z}
= T (1; 1) + T (1; 0) =
T linear

= (1; 2) + ( 3; 1) = ( 2; 1) =

(1; 1)

(1; 0).

Logo, as coordenadas do vector T (2; 1) na base B so 1 e 1.


Resoluo alternativa: Determinemos a matriz M (T ; B; B) e as coordenadas do vector
T (2; 1) na base B usando as alneas anteriores. Tem-se
M (T ;Bc2 ;Bc2 )

(R2 ; Bc2 )

(R2 ; Bc2 )

SBc2 !B # I
(R2 ; B)

I # SBc2 !B
(R2 ; B)

M (T ;B;B)

Logo,
M (T ; B; B) = SBc2 !B M (T ; Bc2 ; Bc2 ) SBc2 !B
=

0
1

1
1

3
1

4
3

1 1
1 0

1
4

= SBc2 !B M (T ; Bc2 ; Bc2 )SB!Bc2 =


3
7

1 1
1 0

2
3

1
4

Alm disso tem-se


coordenadas de (2; 1)
na base Bc2

coordenadas de T (2; 1)
na base Bc2

M (T ;Bc2 ;Bc2 )

SBc2 !B # I

I # SBc2 !B

coordenadas de (2; 1)
na base B

coordenadas de T (2; 1)
na base B.

M (T ;B;B)

Logo, sendo 2 e 1 as coordenadas do vector (2; 1) na base Bc2 ento as coordenadas do vector
T (2; 1) na base B so dadas por
M (T ; B; B)SBc2 !B

2
1

2
3

1
4

0
1

1
1

2
1

1
4

3
7

2
1

1
1

(vi) Determinemos a matriz M (T ; Bc2 ; B) usando s a denio de matriz que representa


uma transformao linear em relao s bases ordenadas no espao de partida e no espao
de chegada. Tem-se
1
3
M (T ; Bc2 ; B) =
,
4 7
309

uma vez que


T (1; 0) = ( 3; 1) = (1; 1)

4(1; 0)

e
T (0; 1) = T ((1; 1) (1; 0)) = T (1; 1) T (1; 0) =
= (1; 2) ( 3; 1) = (4; 3) = 3(1; 1) + 7(1; 0).
Resoluo alternativa: Tendo em conta o diagrama
(R2 ; Bc2 )

SBc2 !B # I
(R2 ; B)

M (T ;Bc2 ;Bc2 )

M (T ;B;B)

(R2 ; Bc2 )

I # SBc2 !B
(R2 ; B)

tem-se
2
3

M (T ; Bc2 ; B) = M (T ; B; B)SBc2 !B =

1
4

0
1

1
1

1
4

3
7

(vii) Determinemos a matriz M (T ; B; Bc2 ) usando s a denio de matriz que representa


uma transformao linear em relao s bases ordenadas no espao de partida e no espao
de chegada. Tem-se
1
3
M (T ; B; Bc2 ) =
,
2 1
uma vez que
T (1; 1) = (1; 2) = (1; 0) 2(0; 1)
e
T (1; 0) = ( 3; 1) =

3(1; 0) + (0; 1).

Resoluo alternativa: Tendo em conta o diagrama


(R2 ; B)

SB!Bc2 # I
(R2 ; Bc2 )

M (T ;B;B)

T
T

M (T ;Bc2 ;Bc2 )

(R2 ; B)

I # SB!Bc2
(R2 ; Bc2 )

tem-se
M (T ; B; Bc2 ) = M (T ; Bc2 ; Bc2 )SB!Bc2 =

3
1

4
3

1 1
1 0

1
2

3
1

5. Considere as transformaes lineares T1 e T2 cujas matrizes que as representam em


relao s bases cannicas (ordenadas) de R2 e R3 so dadas respectivamente por
2
3
0 1
2 0 1
M (T1 ; Bc3 ; Bc2 ) =
e M (T2 ; Bc2 ; Bc3 ) = 4 0 1 5 .
1 1 0
1 1
310

Tem-se T1 : R3 ! R2 com

3
x
T1 (x; y; z) = M (T1 ; Bc3 ; Bc2 ) 4 y 5 =
z

Tem-se T2 : R2 ! R3 com

T2 (x; y) = M (T2 ; Bc2 ; Bc3 )

x
y

Logo, tem-se T1 T2 : R2 ! R2 linear com

2 0 1
1 1 0

3
0 1
=4 0 1 5
1 1

3
x
4 y 5 = (2x + z; x + y).
z
x
y

= (y; y; x + y).

x
=
(T1 T2 ) (x; y) = M (T1 ; Bc3 ; Bc2 )M (T2 ; Bc2 ; Bc3 )
y
2
3
0 1
2 0 1 4
x
1 3
0 1 5
=
=
1 1 0
y
0 2
1 1

e T2 T1 : R3 ! R3 linear com
(T2

x
y

= (x + 3y; 2y)

3 2
3
x
0 1
2 0 1
T1 ) (x; y; z) = M (T2 ; Bc2 ; Bc3 )M (T1 ; Bc3 ; Bc2 ) 4 y 5 = 4 0 1 5
1 1 0
z
1 1
2
32 3
1 1 0
x
= 4 1 1 0 5 4 y 5 = (x + y; x + y; 3x + y + z).
3 1 1
z

3
x
4 y 5=
z

Resoluo alternativa: Tendo-se T1 : R3 ! R2 com T1 (x; y; z) = (2x + z; x + y) e


T2 : R2 ! R3 com T2 (x; y) = (y; y; x + y), ento T1 T2 : R2 ! R2 linear com
(T1 T2 ) (x; y) = T1 (T2 (x; y)) = T1 (y; y; x + y) = (x + 3y; 2y)
e T2 T1 : R3 ! R3 linear com
(T2 T1 ) (x; y; z) = T2 (T1 (x; y; z)) = T2 (2x + z; x + y) = (x + y; x + y; 3x + y + z).

6. Considere a transformao linear T : R3 ! R3 denida por


T (x; y; z) = (2y; y

x; x).

Considere a base ordenada B = fv1 ; v2 ; v3 g de R3 com


v1 = (1; 0; 1), v2 = (1; 2; 0), v3 = ( 1; 1; 1).
Tem-se

2
M (T ; B; B) = 4 1
1
311

4
3
5

3
5
3 5,
4

uma vez que


T (1; 0; 1) = (0; 1; 1) = 2(1; 0; 1)
T (1; 2; 0) = (4; 1; 1) =
T ( 1; 1; 1) = (2; 2; 1) =

(1; 2; 0) + ( 1; 1; 1),
5( 1; 1; 1) e

4(1; 0; 1) + 3(1; 2; 0)
5(1; 0; 1) + 3(1; 2; 0)

4( 1; 1; 1):

7. Seja
Bc2

1 0
0 0

0 1
0 0

0 0
1 0

0 0
0 1

a base cannica (ordenada) de M2 2 (R). Considere a transformao linear


S : M2 2 (R) ! M2 2 (R) denida por S(A) = AT .
Tem-se

1
6
0
M (S; Bc2 2 ; Bc2 2 ) = 6
4 0
0

uma vez que


S

1 0
0 0
0 0
1 0

1 0
0 0

0
0
1
0

0 1
0 0

0 1
0 0

3
0
0 7
7,
0 5
1

0
1
0
0

0 0
0 1

0 0
1 0
0 0
0 1

8. Considere a transformao linear T : R3 ! R3 e a base cannica (ordenada)


Bc3 = fv1 ; v2 ; v3 g de R3 , com v1 = (1; 0; 0), v2 = (0; 1; 0), v3 = (0; 0; 1).
Suponha que se tem
T (v3 ) = 3v1 + v2

2v3 ,

T (v2 + v3 ) = v1

T (v1 + v2 + v3 ) = v2 + v3 .

Logo,
T (0; 0; 1) = T (v3 ) = (3; 1; 2),
T (0; 1; 0) = T (v2 ) = T (v2 + v3 )

T (v3 ) =

2v1

v2 + 2v3 = ( 2; 1; 2)

e
T (1; 0; 0) = T (v1 ) = T (v1 + v2 + v3 )

T (v2 + v3 ) =

v1 + v2 + v3 = ( 1; 1; 1).

Assim:
(i)
T (2v1

v2 + 3v3 ) = 2T (v1 )
312

T (v2 ) + 3T (v3 ) =

= 2( 1; 1; 1)

( 2; 1; 2) + 3(3; 1; 2) = (9; 6; 6);

(ii)

1
3
3
4
1
M (T ; Bc ; Bc ) =
1

2
1
2

3
3
1 5.
2

(iii) Seja B1 = Bc3 a base cannica ordenada de R3 . Determinemos uma base ordenada
B2 = fw1 ; w2 ; w3 g de R3 de modo a que a matriz M (T ; B1 ; B2 ) que represente T em relao
a essas bases B1 e B2 seja a matriz identidade:
2
3
1 0 0
4 0 1 0 5.
0 0 1
Tem-se T (1; 0; 0) = w1 ; T (0; 1; 0) = w2 e T (0; 0; 1) = w3 . Logo,

B2 = f( 1; 1; 1); ( 2; 1; 2); (3; 1; 2)g .


9. Considere a transformao linear T : R2 ! R3 que em relao s bases ordenadas
B1 = fu1 ; u2 g de R2 e B2 = fv1 ; v2 ; v3 g de R3 com
u1 = (1; 1), u2 = (2; 1), v1 = (1; 0; 1), v2 = (1; 1; 2), v3 = (0; 1; 1),
representada pela matriz

3
1 2
M (T ; B1 ; B2 ) = 4 1 1 5 .
3 0
0

Considere ainda as bases ordenadas B1 = u1 ; u2 de R2 e B2 = v1 ; v2 ; v3 de R3 com


0

u1 = (1; 0), u2 = (1; 1), v1 = (1; 0; 0), v2 = (1; 1; 0), v3 = (1; 1; 1).
(i) Tem-se
( 1; 2) = (1; 1)

(2; 1).

Logo, as coordenadas do vector ( 1; 2) na base B1 so 1 e 1. Deste modo, as coordenadas


do vector T ( 1; 2) na base B2 so dadas por
2
3
2
3
1 2
1
1
1
M (T ; B1 ; B2 )
=4 1 1 5
= 4 2 5.
1
1
3 0
3
(ii) Tem-se
( 1; 2) =

3(1; 0) + 2(1; 1).

Logo, as coordenadas do vector ( 1; 2) na base B10 so


313

3 e 2.

Resoluo alternativa: Tem-se


0
1

SB1 !B10 =
0

uma vez que u1 = 0u1 + u2 e u2 = 3u1


do vector ( 1; 2) na base B1 so 1 e 1,
so dadas por
1
=
SB1 !B10
1

3
1

u2 . Tendo em conta (por (i)) que as coordenadas


ento as coordenadas do vector ( 1; 2) na base B10
0
1

3
1

1
1

3
2

(iii) Uma vez que (por (i)) as coordenadas do vector T ( 1; 2) na base B2 so


3, ento
T ( 1; 2) = (1; 0; 1) 2(1; 1; 2) + 3(0; 1; 1) = ( 3; 1; 8).

1;

2 e

Por outro lado, tem-se


( 3; 1; 8) =

4(1; 0; 0) + 9(1; 1; 0)

8(1; 1; 1).

Logo, as coordenadas do vector T ( 1; 2) na base B20 so 4; 9 e 8.


Resoluo alternativa: Determinemos a matriz de mudana de base SB2 !B20 . Tem-se
SB2 !B20
0

1
=4 1
1
0

0
1
2
0

3
1
2 5,
1

uma vez que v1 = v1 v2 + v3 ; v2 = 0v1 v2 + 2v3 e v3 = v1 + 2v2 v3 . Tendo em


conta que (por (i)) as coordenadas do vector T ( 1; 2) na base B2 so 1; 2 e 3, ento as
coordenadas do vector T ( 1; 2) na base B20 so dadas por
2
3 2
32
3 2
3
1
1
0
1
1
4
1 2 54 2 5 = 4 9 5.
SB2 !B20 4 2 5 = 4 1
3
1
2
1
3
8
(iv) Determinemos uma base para N (T ). Seja u 2 R2 e sejam (
de u em relao base
B1 = f(1; 1); (2; 1)g :

1;

2)

as coordenadas

Tem-se
u 2 N (T ) , (

1;

2)

2 N (M (T ; B1 ; B2 ))

e como
02

31
02
1 2
1
N (M (T ; B1 ; B2 )) = N @4 1 1 5A = N @4 0
3 0
0

31
02
31
2
1 2
3 5A = N @4 0 3 5A = f(0; 0)g ,
6
0 0

N (T ) = f0(1; 1) + 0(2; 1)g = f(0; 0)g .

Assim, dim N (T ) = 0 e T injectiva.


314

(v) Determinemos uma base para I (T ). Como f(1; 1); (2; 1)g gera R2 , tem-se
I(T ) = L (fT (1; 1); T (2; 1)g) =
= L (f1(1; 0; 1) + ( 1) (1; 1; 2) + 3(0; 1; 1); 2(1; 0; 1) + 1(1; 1; 2) + 0(0; 1; 1)g) =
= L (f(0; 2; 4); (3; 1; 4)g) .
Uma vez que o conjunto f(0; 2; 4); (3; 1; 4)g linearmente independente e gera I (T ), ento
f(0; 2; 4); (3; 1; 4)g
uma base de I (T ), tendo-se dim I (T ) = 2.
Como dim R3 = 3, tem-se I (T ) 6= R3 , pelo que T no sobrejectiva.
(vi) Determinemos a expresso geral de T , isto , T (x; y), para todo o (x; y) 2 R2 .
Considerando as bases cannicas de R2 e de R3 respectivamente:
Bc2 = f(1; 0); (0; 1)g ;

Bc3 = f(1; 0; 0); (0; 1; 0); (0; 0; 1)g ,

tem-se
2

1 1
4
= 0 1
1 2

M (T ; Bc2 ; Bc3 ) = SB2 !Bc3 M (T ; B1 ; B2 ) SB1 !Bc2


32
3
2
3
0 3
0
1 2
1
1
1 2
1 54 1 1 5
= 4 2 1 5 31
1
1
3
1
3 0
4 4

Logo, para todo o (x; y) 2 R2 ,


T (x; y) = M (T ; Bc2 ; Bc3 )

x
y

1
4
= 1
0

3
1
1 5
4

x
y

2
3

1
3

1
4
= 1
0

3
x y
= 4 x + y 5 = (x
4y

3
1
1 5.
4

y; x + y; 4y) .

Resoluo alternativa alnea (v) para encontrar uma base para I(T ):
Tem-se
I(T ) = T (x; y) : (x; y) 2 R2 = (x y; x + y; 4y) : (x; y) 2 R2 =
= (x; x; 0) + ( y; y; 4y) : (x; y) 2 R2 =
= x (1; 1; 0) + y ( 1; 1; 4) : (x; y) 2 R2 =
= L (f(1; 1; 0) ; ( 1; 1; 4)g)
Como o conjunto f(1; 1; 0) ; ( 1; 1; 4)g linearmente independente e gera I (T ), ento
f(1; 1; 0) ; ( 1; 1; 4)g
uma base de I (T ).
Note que:
L (f(1; 1; 0) ; ( 1; 1; 4)g) = L (f(0; 2; 4); (3; 1; 4)g) .

315

(vii) Tem-se
(R2 ; B1 )

SB1 !B10 # I
(R2 ; B10 )

M (T ;B1 ;B2 )

M (T ;B10 ;B20 )

(R3 ; B2 )

I # SB2 !B20
(R3 ; B20 )

Logo,
1

M (T ; B10 ; B20 ) = SB2 !B20 M (T ; B1 ; B2 ) SB1 !B10


= SB2 !B20 M (T ; B1 ; B2 )SB10 !B1 =
2
32
3
2
3
2
3
1
0
1
1 2
2 2
0
2
1=3 1
1=3 1
1 2 54 1 1 5
3 5
=4 1
=4 6
= 4 1 6 5.
1=3 0
1=3 0
0
4
1
2
1
3 0
4 4
10. Considere a transformao linear T : R3 ! R2 denida por
T (x; y; z) = (x + y; x + y

z).

(i) Tem-se
M (T ; Bc3 ; Bc2 ) =

1 1
1 1

0
1

uma vez que T (1; 0; 0) = (1; 1); T (0; 1; 0) = (1; 1) e T (0; 0; 1) = (0; 1).
(ii) Tem-se
N (T ) =
=
=

(x; y; z) 2 R3 : T (x; y; z) = (0; 0) =


(x; y; z) 2 R3 : (x + y; x + y z) = (0; 0) =
(x; x; 0) 2 R3 : x 2 R = L (f(1; 1; 0)g) .

Logo, o conjunto f(1; 1; 0)g uma base de N (T ) e dim N (T ) = 1. T no injectiva, uma


vez que N (T ) 6= f(0; 0)g.
(iii) Tem-se
I(T ) = f(x + y; x + y

z) : x; y; z 2 Rg = C

1 1
1 1

0
1

= L (f(1; 1); (0; 1)g) .

Como o conjunto f(1; 1); (0; 1)g linearmente independente e como gera I(T ) ento f(1; 1); (0; 1)g
uma base de I(T ) e tem-se dim I(T ) = 2.
Por outro lado, como I(T ) subespao de R2 e dim I(T ) = dim R2 ento I(T ) = R2 ,
isto , T sobrejectiva.
(iv) O vector (1; 0; 0) uma soluo particular da equao linear
T (x; y; z) = (1; 1).
Logo, a soluo geral da equao linear T (x; y; z) = (1; 1) dada por:
f(1; 0; 0)g + N (T ) = (1 + t; t; 0) 2 R3 : t 2 R .
316

(v) No existe nenhum vector (a; b) 2 R2 para o qual a equao linear T (x; y; z) = (a; b)
seja impossvel, uma vez que T sobrejectiva.
(vi) No existe nenhum vector (a; b) 2 R2 para o qual a equao linear T (x; y; z) = (a; b)
seja possvel e determinada, uma vez que T no injectiva.
11. Considere a transformao linear T : R3 ! R3 cuja matriz M (T ; Bc3 ; Bc3 ) que a
representa em relao base cannica (ordenada) Bc3 de R3 dada por
2
3
1 2 2
M (T ; Bc3 ; Bc3 ) = 4 2 1 4 5 .
0 0 2
(i) Seja (x; y; z) 2 R3 . Tem-se
2 3 2
32 3
x
1 2 2
x
T (x; y; z) = M (T ; Bc3 ; Bc3 ) 4 y 5 = 4 2 1 4 5 4 y 5 = (x + 2y + 2z; 2x + y + 4z; 2z).
z
0 0 2
z
(ii) Tem-se
02

31
02
1 2 2
1
N (T ) = N @4 2 1 4 5A = N @4 0
0 0 2
0

Logo, T injectiva e dim N (T ) = 0.

31
2 2
3 0 5A = f(0; 0; 0)g .
0 2

(iii) Tem-se
I(T ) = f(x + 2y + 2z; 2x + y + 4z; 2z) : x; y; z 2 Rg =
= fx(1; 2; 0) + y(2; 1; 0) + z(2; 4; 2) : x; y; z 2 Rg
= L (f(1; 2; 0); (2; 1; 0); (2; 4; 2)g) .
Como o conjunto f(1; 2; 0); (2; 1; 0); (2; 4; 2)g linearmente independente e como gera I(T )
ento f(1; 2; 0); (2; 1; 0); (2; 4; 2)g uma base de I(T ) e tem-se dim I(T ) = 3. Por outro lado,
como I(T ) subespao de R3 e dim I(T ) = dim R3 ento I(T ) = R3 , isto , T sobrejectiva.
(iv) Como T (1; 1; 0) = T (1; 0; 0) + T (0; 1; 0) = (2; 1; 0) + (1; 2; 0) = (3; 3; 0), ento o
vector (1; 1; 0) uma soluo particular da equao linear
T (x; y; z) = (3; 3; 0).
Logo, a soluo geral da equao linear T (x; y; z) = (3; 3; 0) dada por:
f(1; 1; 0)g + N (T ) = f(1; 1; 0)g .
(v) No existe nenhum vector (a; b; c) 2 R3 para o qual a equao linear T (x; y; z) =
(a; b; c) seja impossvel, uma vez que T sobrejectiva.
317

(vi) No existe nenhum vector (a; b; c) 2 R3 para o qual a equao linear T (x; y; z) =
(a; b; c) seja possvel e indeterminada, uma vez que T injectiva.
12. Considere a transformao linear T : R3 ! R3 cuja matriz M (T ; B; B) que a
representa em relao base (ordenada) B = fv1 ; v2 ; v3 g de R3 com
v1 = (1; 1; 1), v2 = (1; 1; 0), v3 = (1; 0; 0),
dada por

3
1 2 2
M (T ; B; B) = 4 2 4 4 5 .
0 0 2

(i) Seja A = M (T ; B; B). Seja u 2 R3 e sejam (


base B. Tem-se
u 2 N (T ) , ( 1 ; 2 ;

1;

3)

2;

3)

as coordenadas de u em relao

2 N (A)

e como
02

31
02
31
1 2 2
1 2 0
N (A) = N @4 2 4 4 5A = N @4 0 0 0 5A = f( 2y; y; 0) : y 2 Rg = L (f( 2; 1; 0)g) ,
0 0 2
0 0 2
N (T ) = L (f( 2) (1; 1; 1) + 1(1; 1; 0) + 0(1; 0; 0)g) = L (f(1; 1; 2)g) .

O conjunto f(1; 1; 2)g uma base de N (T ) pois gera N (T ) e linearmente independente.


Assim, dim N (T ) = 1. T no injectiva, uma vez que N (T ) 6= f(0; 0; 0)g.
Como
dim
R3
= dim N (T ) + dim I(T ),
|{z}
espao de partida

ento dim I(T ) = 2 e assim I(T ) 6= R3 (pois


Expresso geral de T :
2
32
1 1 1
1
T (x; y; z) = 4 1 1 0 5 4 2
1 0 0
0
= (8x

2y

dim R3 = 3), isto , T no sobrejectiva.


32
3
2 2
1 1 1
4 4 54 1 1 0 5
0 2
1 0 0

3z; 6x

3z; 2x

z).

3
x
4 y 5=
z

Clculo alternativo de N (T ): Tem-se


N (T ) = (x; y; z) 2 R3 : T (x; y; z) = (0; 0; 0) =
= (x; y; z) 2 R3 : (8x 2y 3z; 6x 3z; 2x
= (x; y; z) 2 R3 : z = 2x e x = y
= (x; x; 2x) 2 R3 : x 2 R =
= L (f(1; 1; 2)g) .

z) = (0; 0; 0) =

(ii) Quanto ao contradomnio:


I(T ) = L (fT (1; 1; 1); T (1; 1; 0); T (1; 0; 0)g) =
318

= L(f1(1; 1; 1) + 2(1; 1; 0) + 0(1; 0; 0); 2(1; 1; 1)+


+4(1; 1; 0) + 0(1; 0; 0); 2(1; 1; 1) + 4(1; 1; 0) + 2(1; 0; 0)g) =
= L (f(3; 3; 1); (6; 6; 2); (8; 6; 2)g) = L (f(6; 6; 2); (8; 6; 2)g) = L (f(8; 6; 2); ( 2; 0; 0)g) .
Como o conjunto f(8; 6; 2); ( 2; 0; 0)g linearmente independente e como gera I(T ) ento
f(8; 6; 2); ( 2; 0; 0)g
uma base de I(T ) e tem-se dim I(T ) = 2.
Clculo alternativo de I(T ): Tem-se
I(T ) = f(8x 2y 3z; 6x 3z; 2x z) : x; y; z 2 Rg =
= L (f(8; 6; 2); ( 2; 0; 0); ( 3; 3; 1)g) =
= L (f(8; 6; 2); ( 2; 0; 0)g) = C M (T ; Bc3 ; Bc3 ) .
(iii) fcil ver que (2; 4; 0) 2
= I(T ). Logo, a equao linear T (x; y; z) = (2; 4; 0) no tem
solues.
(iv) Tem-se T (1; 1; 1) = 1(1; 1; 1) + 2(1; 1; 0) + 0(1; 0; 0) = (3; 3; 1) e assim
T

1
;
3

1
;
3

1
3

1; 1;

1
3

Logo, a soluo geral de


T (x; y; z) =

1; 1;

1
3

dada por:
(x; y; z) 2 R3 : T (x; y; z) =
=

1
;
3

1; 1;
1
;
3

1
3

1
;
3

1
;
3

1
3

+ N (T ) =

1
) + s (1; 1; 2) : s 2 R .
3

(v) Por exemplo o vector (1; 0; 0) ou qualquer vector (a; b; c) 2 I(T ), uma vez que sendo
T no injectiva, sempre que a equao linear fr possvel, ela ser indeterminada.
(vi) Tem-se
T (v1 ) = (1; 1; 1) + 2(1; 1; 0) + 0(1; 0; 0) = (3; 3; 1);
T (v2 ) = 2(1; 1; 1) + 4(1; 1; 0) + 0(1; 0; 0) = (6; 6; 2)
e
T (v3 ) = 2(1; 1; 1) + 4(1; 1; 0) + 2(1; 0; 0) = (8; 6; 2).
Logo,
T (1; 0; 0) = T (v3 ) = (8; 6; 2);
T (0; 1; 0) = T (v2 )

T (v3 ) = ( 2; 0; 0)

319

e
T (0; 0; 1) = T (v1 )

T (v2 ) = ( 3; 3; 1).

Assim,

8
3
3
4
M (T ; Bc ; Bc ) = 6
2

e deste modo, para (x; y; z) 2 R3 ,

3
3
3 5
1

2
0
0

3 2
x
8
3
3 4
5
4
T (x; y; z) = M (T ; Bc ; Bc ) y = 6
z
2
= (8x

2y

3z; 6x

2
0
0

3z; 2x

z).

32 3
x
3
5
4
3
y 5=
1
z

13. Considere a transformao linear T : R3 ! R3 denida por


T (x; y; z) = (x + y + z; x + 2y

4z; z).

(i) Tendo em conta que T (1; 0; 0) = (1; 1; 0); T (0; 1; 0) = (1; 2; 0) e T (0; 0; 1) = (1; 4; 1),
tem-se
2
3
1 1 1
4 5
M (T ; Bc3 ; Bc3 ) = 4 1 2
0 0 1
que representa T em relao base cannica (ordenada) Bc3 de R3 .
(ii) A matriz M (T ; Bc3 ; Bc3 ) invertvel pois
2
3
2
1 1 1
1 1
4 5!4 0 1
M (T ; Bc3 ; Bc3 ) = 4 1 2
0 0 1
0 0

Logo, T injectiva e como tal invertvel, tendo-se


1

M (T ; Bc3 ; Bc3 )
Determinemos (M (T ; Bc3 ; Bc3 ))
2
1
3
3
4
1
M (T ; Bc ; Bc ) j I =
0
2
1
! 4 0
0

Logo,

= M (T

3
1
5 5.
1

; Bc3 ; Bc3 ).

.
3
2
1 j 1 0 0
1 1
5
4
4 j 0 1 0 ! 0 1
1 j 0 0 1
0 0
3
2
1 0 j 1 0
1
1 0
1 0 j
1 1 5 5!4 0 1
0 1 j 0 0 1
0 0

1
2
0

M (T ; Bc3 ; Bc3 )

2
=4 1
0
320

1
1
0

3
6
5 5
1

1 j
5 j
1 j

0 j
0 j
1 j

3
1 0 0
1 1 0 5!
0 0 1
3
2
1
6
1 1
5 5.
0
0
1

e como tal, para (x; y; z) 2 R3 ,


T

(x; y; z) = M (T ; Bc3 ; Bc3 )


= (2x

Observao: T

T =T
T

3 2
x
2
1
4 y 5=4 1
z
0

1
1
0

6z; x + y + 5z; z).

32 3
6
x
5 54 y 5 =
1
z

= I. Isto , para qualquer (x; y; z) 2 R3 ;

T (x; y; z) = T

(x; y; z) = (x; y; z),

como se pode ver:


T

T (x; y; z) = T 1 (T (x; y; z)) = T 1 (x + y + z; x + 2y 4z; z) =


= (2x + 2y + 2z x 2y + 4z 6z; x y z + x + 2y
= (x; y; z);
T

4z + 5z; z) =

(x; y; z) = T T 1 (x; y; z) = T (2x y 6z; x + y + 5z; z) =


= (2x y 6z x + y + 5z + z; 2x y 6z 2x + 2y + 10z
= (x; y; z).

4z; z) =

Demonstrao alternativa da injectividade de T : Tem-se


N (T ) = (x; y; z) 2 R3 : T (x; y; z) = (0; 0; 0) =
= (x; y; z) 2 R3 : (x + y + z; x + 2y 4z; z) = (0; 0; 0) =
= f(0; 0; 0)g .
Logo, T injectiva.
(iii) Sendo T injectiva, como os espaos de partida e de chegada tm a mesma dimenso,
ento T sobrejectiva. Logo, T linear e bijectiva, isto , T um isomorsmo.
(iv) Tem-se
T (x; y; z) = (1; 1; 2) , (x; y; z) = T

(1; 1; 2) = ( 11; 10; 2).

Logo, a soluo geral da equao linear T (x; y; z) = (1; 1; 2) : f( 11; 10; 2)g.
14. Seja
Bc2

1 0
0 0

0 1
0 0

0 0
1 0

0 0
0 1

a base cannica (ordenada) de M2 2 (R). Considere a transformao


T : M2 2 (R) ! M2 2 (R) denida por T (X) = AX

321

XA, com A =

0 1
1 0

(i) Sejam X; X1 ; X2 2 M2 2 (R) e

2 R. Tem-se

T (X1 + X2 ) = A(X1 + X2 ) (X1 + X2 )A = AX1 + AX2 X1 A


= AX1 X1 A + AX2 X2 A = T (X1 ) + T (X2 )

X2 A =

e
T ( X) = A( X)
(ii) Seja

a b
c d

a b
c d

( X)A =

(AX

XA) = T (X).

2 M2 2 (R). Tem-se
0 1
1 0

a b
c d

a b
c d

0 1
1 0

b+c
d a

d a
b c

Logo, a expresso geral de T dada por:


a b
c d

(iii) Tem-se

0
6
1
M (T ; Bc2 2 ; Bc2 2 ) = 6
4 1
0

uma vez que


1 0
0 0

b+c
d a

0 1
1 0

1 0
0 0

d a
b c

1
0
0
1

1 0
0 0

1
0
0
1

3
0
1 7
7,
1 5
0

0 1
1 0

0
1

1
0

0 1
0 0

0 1
1 0

0 1
0 0

0 1
0 0

0 1
1 0

1
0

0
1

0 0
1 0

0 1
1 0

0 0
1 0

0 0
1 0

0 1
1 0

1
0

0
1

0 0
0 1

0 1
1 0

0 0
0 1

0 0
0 1

0 1
1 0

0 1
1 0

(iv) Tem-se
N (T ) =
=

a b
b a

X=

a b
c d

2 M2 2 (R) : T (X) =

2 M2 2 (R) : a; b 2 R

Logo, dim N (T ) = 2. Como N (T ) 6=

0 0
0 0

322

=L

1 0
0 1

0 0
0 0
;

=
0 1
1 0

ento T no injectiva.

(v) Atendendo a que dim N (T ) = 2 e dim M2 2 (R) = 4, ento dim I(T ) = 2. T no


sobrejectiva uma vez que I(T ) 6= M2 2 (R). Determinemos uma base para I(T ). Tem-se
I(T ) =

b+c
a+d

1
0

= L
1
0

0
1

d a
b c

0
1

= L

Como o conjunto

a b
c d

T (X) : X =

1
0
0
1

0
1

0 1
1 0

0 1
1 0

2 M2 2 (R) : a; b; c; d 2 R
1
0

0 1
1 0

2 M2 2 (R)

gera I(T ) e linearmente independente, ento

uma base de I(T ).


15. Considere as transformaes lineares T1 ; T2 : R2 ! R2 denidas respectivamente por
y) e T2 (x; y) = (2x + y; x

T1 (x; y) = (x + y; x

2y).

(i) Tem-se
M (T1 ; Bc2 ; Bc2 ) =

1
1

1
1

M (T2 ; Bc2 ; Bc2 ) =

2
1

1
2

uma vez que T1 (1; 0) = (1; 1); T1 (0; 1) = (1; 1); T2 (1; 0) = (2; 1) e T2 (0; 1) = (1; 2).
(ii) A matriz M (T2 T1 ; Bc2 ; Bc2 ) que representa T2
denada) Bc2 de R2 , dada por

T1 em relao base cannica (or-

M (T2 T1 ; Bc2 ; Bc2 ) = M (T2 ; Bc2 ; Bc2 )M (T1 ; Bc2 ; Bc2 ) =


2 1
1 1
3 1
=
=
1
2
1
1
1 3
(iii) Tem-se, para qualquer (x; y) 2 R2 ,
(T2 T1 )(x; y) = M (T2 T1 ; Bc2 ; Bc2 )
3 1
1 3

x
y

x
y

= (3x + y; x + 3y).

(iv) Tem-se, para qualquer (x; y) 2 R2 ,


T1 (x; y) = M (T1 ; Bc2 ; Bc2 )
=

1
1

1
1
323

x
y

x
y

= (x + y; x

y)

e
x
y

T2 (x; y) = M (T2 ; Bc2 ; Bc2 )


2
1

1
2

x
y

= (2x + y; x

2y).

(T2 T1 )(x; y) = T2 (T1 (x; y)) = T2 (x + y; x

y) =

Logo,
= (2x + 2y + x

y; x + y

2x + 2y) = (3x + y; x + 3y):

(v) Tem-se
N (T1 ) = (x; y) 2 R2 : T (x; y) = (0; 0) = (x; y) 2 R2 : (x + y; x

y) = (0; 0) = f(0; 0)g

e
N (T2 ) = (x; y) 2 R2 : T (x; y) = (0; 0) = (x; y) 2 R2 : (2x + y; x

2y) = (0; 0) = f(0; 0)g .

Logo, T1 e T2 so injectivas e como tal so invertveis.


(vi) Tem-se ento
M (T1 ; Bc2 ; Bc2 )

= M (T1 1 ; Bc2 ; Bc2 ) e

Determinemos (M (T1 ; Bc2 ; Bc2 ))

= M (T2 1 ; Bc2 ; Bc2 )

e (M (T2 ; Bc2 ; Bc2 )) .

1
1

1
0

M (T1 ; Bc2 ; Bc2 ) j I

M (T2 ; Bc2 ; Bc2 ) j I

M (T2 ; Bc2 ; Bc2 )

2
1

2
0

1=2
1=2

1 j 1 0
1 j 0 1

1 j
2 j

1
0

0 j 1=2 1=2
2 j
1 1

1 j 1 0
2 j 0 1
0
j
5=2 j

2
0

4=5 2=5
1=2 1

1 0
1 1

1 0 j 1=2
0 1 j 1=2

1
j
5=2 j
!

1
0
1=2 1

1 0 j 2=5
0 1 j 1=5

!
1=2
1=2

!
1=5
2=5

Logo,
M (T1 ; Bc2 ; Bc2 )

1=2
1=2

M (T2 ; Bc2 ; Bc2 )

2=5
1=5

1=5
2=5

e como tal, para (x; y) 2 R2 ,


T1 1 (x; y) = M (T1 ; Bc2 ; Bc2 )

x
y

1=2
1=2

1=2
1=2

x
y

1 1
1
x + y; x
2
2 2

1
y ,
2

T2 1 (x; y) = M (T2 ; Bc2 ; Bc2 )

x
y

2=5
1=5

1=5
2=5

x
y

2
1 1
x + y; x
5
5 5

2
y ,
5

324

e nalmente
T1

T2

T2 1 (x; y) =
2
1 1
= T1 1
x + y; x
5
5 5
3
1
1
=
x
y; x +
10
10 10

(x; y) = T1

2
y =
5
3
y .
10

(vii) Tem-se
M ((T2 T1 ) 1 ; Bc2 ; Bc2 ) = M (T1
= M (T1 ; Bc2 ; Bc2 )

T2 1 ; Bc2 ; Bc2 ) = M (T1 1 ; Bc2 ; Bc2 )M (T2 1 ; Bc2 ; Bc2 ) =


1

M (T2 ; Bc2 ; Bc2 )

1=2
1=2

1=2
1=2

2=5
1=5

1=5
2=5

3=10
1=10

1=10
3=10

De facto,
3=10
1=10

M ((T2 T1 ) 1 ; Bc2 ; Bc2 ) =

1=10
3=10

3 1
1 3

= M (T2 T1 ; Bc2 ; Bc2 )

(viii) Tendo em conta (vii) tem-se


(T2 T1 ) 1 (x; y) =

3=10
1=10

1=10
3=10

x
y

1
1
3
y; x + y .
10 10
10

3
x
10

Logo, como seria de esperar,


(T2 T1 ) 1 (x; y) = T1

T2

(x; y).

1 0
invertvel, pois det A = 1 6= 0,
2 1
T injectiva. Logo, se a equao linear T (x; y) = (1; 2) tiver soluo, ela nica. Como
1
1
1
0
C (A) = I (T ) e uma vez que
2 C (A) pois:
=1
+0
, ento (1; 0) a
2
2
2
1
soluo nica da equao linear T (x; y) = (1; 2).
Resoluo alternativa da equao linear T (x; y) = (1; 2):
Como A invertvel, T invertvel e
16. Seja A = M (T ; Bc2 ; Bc2 ). Como A =

T (x; y) = (1; 2) , (x; y) = T

17. Tem-se M (T1 ; Bc2 ; Bc1 ) =

(1; 2) = A

1
2

1 0
2 1

1
2

1
0

1 0 , pois T1 (1; 0) = 1 e T1 (0; 1) = 0. Logo

M T2 T1 ; Bc2 ; Bc2 = M T2 ; Bc1 ; Bc2 M T1 ; Bc2 ; Bc1 =


325

1
0

1 0

1 0
0 0

e assim N (T2

1 0
= L (f(0; 1)g). Pelo que
0 0
T1 ), uma vez que f(0; 1)g linearmente independente e gera

T1 ) = N (M (T2 T1 ; Bc2 ; Bc2 )) = N

f(0; 1)g base de N (T2


N (T2 T1 ).

1 0
1
, tem-se T (1; 0; 1) = 1(1; 1) (0; 1) = (1; 0),
1 0
1
T (0; 1; 1) = 0(1; 1) + 0(0; 1) = (0; 0) e T (1; 0; 1) = 1(1; 1) (0; 1) = (1; 0). Por outro lado,
como B1 = f(1; 0; 1); (0; 1; 1); (0; 0; 1)g gera o "espao de partida" R3 , tem-se
18. Como M (T ; B1 ; B2 ) =

I (T ) = L (fT (1; 0; 1); T (0; 1; 1); T (0; 0; 1)g) = L (f(1; 0)g) .


Pelo que f(1; 0)g base de I (T ), pois (1; 0) linearmente independente e gera I (T ).
1 0 1
Tem-se dim I (T ) = car (M (T ; B1 ; B2 )) = car
= 1. Como I (T ) 6= R2 ,
0 0 0
pois dim I (T ) = 1 6= 2 = dim R2 , ento T no sobrejectiva.
19. Considere a transformao linear T1 : R3 ! R2 denida por T1 (x; y; z) = (2x + y; y +
2z). Considere ainda a transformao linear T2 : R2 ! R3 cuja representao matricial em
2
3
3
relao base (ordenada)
2 B = 3f(2; 1); (1; 2)g de R e base cannica Bc de R dada pela
2 1
matriz M (T2 ; B; Bc3 ) = 4 1 1 5.
1 2
(i)
(x; y; z) 2 R3 : T1 (x; y; z) = (0; 0) = (x; y; z) 2 R3 : (2x + y; y + 2z) = (0; 0) =
o
n
y
yo n y
3
=
; y;
: y 2 R = L (f(1; 2; 1)g) .
= (x; y; z) 2 R : x = z =
2
2
2

N (T1 ) =

O conjunto f(1; 2; 1)g gera N (T1 ) e linearmente independente, logo uma base de N (T1 ).
Tem-se
dim N (T1 ) = 1 e dim N (T1 ) + dim I(T1 ) = dim R3 ,

e assim dim I(T1 ) = 2. Logo, como I(T1 ) um subespao de R2 e dim I(T1 ) = dim R2 = 2,
ento I(T1 ) = R2 e assim, T1 sobrejectiva.
(ii) Como B = f(2; 1); (1; 2)g gera o "espao de partida" R2 , tem-se
I (T2 ) = L (fT2 (2; 1); T2 (1; 2)g) = L (f(2; 1; 1) ; (1; 1; 2)g) .

Como o conjunto f(2; 1; 1) ; (1; 1; 2)g gera I (T2 ) e linearmente independente, ento uma
base de I (T2 ).
Tem-se
dim I (T2 ) = 2 e dim N (T2 ) + dim I (T2 ) = dim R2 ,
e assim dim N (T2 ) = 0. Logo, T2 injectiva.
(iii) Tem-se
2
3
1
2 1
4 2 1 1 5
1 1 2

2L1 +L2 !L2


L1 +L3 !L3

1
4 0
0

2
5
3
326

3
1
1 5
1

3
L +L3 !L3
5 2

1
4 0
0

2
5
0

3
1
1 5
8
5

logo o conjunto f(1; 2; 1) ; (2; 1; 1) ; (1; 1; 2)g gera N (T1 ) + I(T2 ) e linearmente independente, ento uma base de N (T1 ) + I(T2 ).
Logo, como N (T1 ) + I(T2 ) um subespao de R3 e dim (N (T1 ) + I(T2 )) = dim R3 = 3,
ento N (T1 ) + I(T2 ) = R3 .
Tem-se
dim (N (T1 ) \ I(T2 )) = dim N (T1 ) + dim I(T2 )
2
(iv) Como (1; 0) = (2; 1)
3
T2 (1; 0) = T2
=

2
(2; 1)
3

2
(2; 1; 1)
3

dim (N (T1 ) + I(T2 )) = 1 + 2

1
(1; 2) e (0; 1) =
3

1
2
(2; 1) + (1; 2), tem-se
3
3

2
1
T2 (2; 1)
T2 (1; 2) =
T linear 3
3
1
4 2 2
1 1 2
(1; 1; 2) =
; ;
+
;
;
=
3
3 3 3
3 3 3
1
(1; 2)
3

3 = 0.

1
1; ; 0
3

e
2
1
(2; 1) + (1; 2)
=
T linear
3
3
1
2
2
(2; 1; 1) + (1; 1; 2) =
;
3
3
3

T2 (0; 1) = T2
=

1
T2 (2; 1) +
3
1 1
;
+
3 3

2
T2 (1; 2) =
3
2 2 4
; ;
=
3 3 3

1
0; ; 1
3

Logo, a matriz M (T2 ; Bc2 ; Bc3 ) que representa T2 em relao s bases cannicas Bc2 e Bc3 de R2
e R3 respectivamente, dada por
2
3
1
0
M (T2 ; Bc2 ; Bc3 ) = 4 1=3 1=3 5 .
0
1
(v) A matriz M (T1 ; Bc3 ; Bc2 ) que representa T1 em relao s bases cannicas Bc3 e Bc2 de
R3 e R2 respectivamente, dada por
M (T1 ; Bc3 ; Bc2 ) =

2 1 0
0 1 2

uma vez que


T1 (1; 0; 0) = (2; 0);

T1 (0; 1; 0) = (1; 1) e T1 (0; 0; 1) = (0; 2).

Logo, a matriz que representa T1 T2 em relao base cannica Bc2 de R2 dada por
2
3
1
0
2 1 0 4
7=3 1=3
1=3 1=3 5 =
M (T1 T2 ; Bc2 ; Bc2 ) = M (T1 ; Bc3 ; Bc2 )M (T2 ; Bc2 ; Bc3 ) =
0 1 2
1=3 7=3
0
1
Logo, tem-se

(T1 T2 ) (x; y) =

7=3 1=3
1=3 7=3
327

x
y

Assim, como a matriz

7=3 1=3
1=3 7=3

invertvel, a soluo geral da equao (T1 T2 ) (x; y) =

8=3
, dada
8=3
x
y

7=3 1=3
1=3 7=3

8=3
8=3

7=16
1=16

1=16
7=16

8=3
8=3

1
1

20. Considere a transformao linear T1 : R2 ! R3 denida por T1 (x; y) = (2x + y; 0; x +


2y). Considere ainda a transformao linear T2 : R3 ! R2 cuja representao matricial em
relao base (ordenada) B = f(1; 1; 1); (1; 1; 0); (1; 0; 0)g de R3 e base cannica Bc2 de R2
dada pela matriz:
1
1 1
.
M (T2 ; B; Bc2 ) =
1 1
1
(i) T2 (0; 1; 0) = T2 (1; 1; 0) T2 (1; 0; 0) = ( 1; 1) (1; 1) = ( 2; 2).
T2 (0; 0; 1) = T2 (1; 1; 1) T2 (1; 1; 0) = (1; 1) ( 1; 1) = (2; 2).
(ii) Tem-se
I (T1 ) = T1 (x; y) : (x; y) 2 R2 = (2x + y; 0; x + 2y) : (x; y) 2 R2 =
= fx(2; 0; 1) + y(1; 0; 2) : x; y 2 Rg = L (f(2; 0; 1); (1; 0; 2)g) .
Como o conjunto f(2; 0; 1); (1; 0; 2)g gera I (T1 ) e linearmente independente, ento uma
base de I (T1 ).
Como dim I(T1 ) = 2 < 3 = dim R3 ento I(T1 ) 6= R3 e assim, T1 no sobrejectiva.
(iii)
N M (T2 ; B; Bc2 ) = N
= f(y

1
1

1
1

1
1

1
0

=N

1 1
0 0

z; y; z) : y; z 2 Rg = L (f(1; 1; 0); ( 1; 0; 1)g) :

Como os vectores (1; 1; 0) e ( 1; 0; 1) so as coordenadas na base B de vectores que geram o


ncleo de T2 , tem-se
1(1; 1; 1) + 1(1; 1; 0) + 0(1; 0; 0) = (2; 2; 1)
e
1(1; 1; 1) + 0(1; 1; 0) + 1(1; 0; 0) = (0; 1; 1)
Como o conjunto f(2; 2; 1); (0; 1; 1)g gera N (T2 ) e linearmente independente, ento
uma base de N (T2 ). Como N (T2 ) 6= f0g ento T2 no injectiva.
(iv) Pela denio de M (T2 ; B; Bc2 ) tem-se T2 (1; 0; 0) = (1; 1). Atendendo alnea
a), tem-se T2 (0; 1; 0) = ( 2; 2) e T2 (0; 0; 1) = (2; 2). Logo, a matriz M (T2 ; Bc3 ; Bc2 ) que
representa T2 em relao s bases cannicas Bc3 e Bc2 de R3 e R2 respectivamente, dada por
M (T2 ; Bc3 ; Bc2 ) =
328

1
1

2
2

2
2

Por outro lado, como T1 (1; 0) = (2; 0; 1) e T1 (0; 1) = (1; 0; 2). Logo, a matriz M (T1 ; Bc2 ; Bc3 )
que representa T1 em relao s bases cannicas Bc2 e Bc3 de R2 e R3 respectivamente, dada
por
2
3
2 1
M (T1 ; Bc2 ; Bc3 ) = 4 0 0 5 .
1 2
Logo, a matriz que representa T2 T1 em relao base cannica Bc2 de R2 dada por
2
3
2 1
1
2 2 4
4
5
0 0 5=
M (T2 T1 ; Bc2 ; Bc2 ) = M (T2 ; Bc3 ; Bc2 )M (T1 ; Bc2 ; Bc3 ) =
1 2
2
4
5
1 2

Logo, tem-se

(T2 T1 ) (x; y) =

4
4

5
5

x
y

(T2 T1 ) (x; y) = ( 1; 1) ,

4
4

5
5

x
y

e assim,
1
1

A soluo geral de (T2 T1 ) (x; y) = ( 1; 1) dada por:


4
4

Soluo particular de
1
;0
4

Como o vector
N

5
5

x
y

1
1

uma soluo particular de


4
4

5
5

4 5
0 0

=N

+ Soluo geral de
4
4

5
5

x
y

4
4

5
5

1
1

x
y

5
;1
4

=L

ento, a soluo geral de (T2 T1 ) (x; y) = ( 1; 1) dada por:


1
;0 + N
4

4
4

5
5

1
;0 + s
4

5
;1
4

:s2R .

21. Considere a transformao linear T : R3 ! P3 denida por


T (1; 1; 1) = 2 + 2t2 ;

T (1; 1; 1) =

t3

e T ( 1; 1; 1) = 2 + t + 2t2 + t3 .

(i) Determinemos a expresso geral de T , isto , determinemos T (x; y; z) para qualquer


(x; y; z) 2 R3 .
Seja (x; y; z) 2 R3 . Como f(1; 1; 1); (1; 1; 1); ( 1; 1; 1)g gera R3 , existem escalares
; ; 2 R tais que
(x; y; z) = (1; 1; 1) + (1; 1; 1) + ( 1; 1; 1).
Atendendo
2
1
4 1
1

a
1
1
1

3
2
1 j x
1
1 j y 5!4 0
1 j z
0

1
2
2

3
2
1 j
x
1
2 j y x 5!4 0
0 j z+x
0
329

1
2
0

3
1 j
x
2 j y x 5,
2 j y+z

0
0

tem-se
8
<

+
=x
2 +2 =y
:
2 =y+z

Logo

x ,

8
>
>
>
>
<

1
(x + y)
2
1
= (x + z)
2
1
= (y + z) .
2
=

>
>
>
>
:

1
1
1
(x + y) (1; 1; 1) + (x + z) (1; 1; 1) + (y + z) ( 1; 1; 1),
2
2
2
e assim, como T linear,
(x; y; z) =

T (x; y; z) =
=

1
1
1
(x + y) T (1; 1; 1) + (x + z) T (1; 1; 1) + (y + z) T ( 1; 1; 1) =
2
2
2

1
1
1
(x + y) 2 + 2t2 + (x + z) t t3 + (y + z) 2 + t + 2t2 + t3 =
2
2
2
1
1
= x + 2y + z + (y x) t + (x + 2y + z) t2 + (y x) t3 .
2
2

(ii) Tem-se
N (T ) = (x; y; z) 2 R3 : T (x; y; z) = 0 =
=

(x; y; z) 2 R3 : x + 2y + z +
=

1
(y
2

x) t + (x + 2y + z) t2 +

(x; y; z) 2 R3 : x + 2y + z = 0 e

= (x; y; z) 2 R3 : x = y

e z=

1
(y
2

1
(y
2

x) t3 = 0 + 0t + 0t2 + 0t3

x) = 0

3y = y(1; 1; 3) 2 R3 : y 2 R = L (f(1; 1; 3)g)

Logo, o conjunto f(1; 1; 3)g uma base de N (T ) e dim N (T ) = 1. T no injectiva, uma


vez que N (T ) 6= f(0; 0; 0)g.
(iii) Determine, se possvel, uma base para o contradomnio I(T ). Determine a dimenso
de I(T ). Diga se T sobrejectiva.
Como f(1; 1; 1); (1; 1; 1); ( 1; 1; 1)g gera R3 ; tem-se
2 + 2t2 ; t

I (T ) = L (fT (1; 1; 1); T (1; 1; 1); T ( 1; 1; 1)g) = L


Como:

2
6 0
6
4 2
0

0
1
0
1

3
2
2
2
6
7
1 7
0
!6
4 0
2 5
0
1

0
1
0
0

3
2
1 7
7
0 5
0

t3 ; 2 + t + 2t2 + t3

ento o conjunto f2 + 2t2 ; t t3 g linearmente independente e gera I(T ); sendo assim


uma base de I(T ).
Logo, tem-se dim I(T ) = 2.
Por outro lado, como I(T ) subespao de P3 e dim P3 = 4 ento I(T ) 6= P3 , isto , T
no sobrejectiva.

330

(iv) Atendendo a ter-se


T (1; 1; 1) = 2 + 2t2 ;

T (1; 1; 1) =

t3

e T ( 1; 1; 1) = 2 + t + 2t2 + t3 .

1
2 + 2t2 = T ( 1; 1; 1)
2 | {z }

1 + t + t2 + t3 = 2| + t +{z2t2 + t}3
= T ( 1;1;1)

=T

1
T (1; 1; 1) =
T linear
2

= T (1;1; 1)

1
(1; 1; 1)
2

( 1; 1; 1)

=T

3 1 3
; ;
2 2 2

3 1 3
; ;
2 2 2

uma soluo particular da equao linear T (x; y; z) = 1 + t + t2 + t3 .


Como, a soluo geral de T (x; y; z) = 1 + t + t2 + t3 dada por:
Soluo particular de T (x; y; z) = 1 + t + t2 + t3 + (Soluo geral de T (x; y; z) = 0)

e como a soluo geral de T (x; y; z) = 0 dada por


N (T ) = (x; y; z) 2 R3 : T (x; y; z) = 0 = L (f(1; 1; 3)g)
ento, a soluo geral de T (x; y; z) = 1 + t + t2 + t3 dada por:
3 1 3
; ;
2 2 2

3 1 3
; ;
2 2 2

+ L (f(1; 1; 3)g) =

+ s(1; 1; 3) : s 2 R .

2 R. Considere a transformao linear T : R3 ! P2 denida por

22. Seja

T (x; y; z) = z

y + (y

x) t + xt2 .

(i) Tem-se
N (T ) = (x; y; z) 2 R3 : T (x; y; z) = 0 =

= (x; y; z) 2 R3 : z

x) t + xt2 = 0 + 0t + 0t2 + 0t3 =

y + (y

= (x; y; z) 2 R3 : z = y

e (y = x ou

= (0; y; z) 2 R3 : z = y

=
Logo, se

8
< f(0; 0; 0)g
:

se

e (y = 0 ou = 0) =
8
6= 0
< f(0; 0; 0)g se
=
:
=0
L (f(0; 1; 1)g) se
=0

6= 0

fy(0; 1; 1) 2 R3 : y 2 Rg

= 0) e x = 0 =

se

= 0 ento f(0; 1; 1)g uma base de N (T0 ) e assim T0 no injectiva.


8
6 0
=
< 0 se
dim N (T ) =
:
1 se
= 0:

Logo, como N (T ) = f(0; 0; 0)g, para todo o


2 Rn f0g.

2 Rn f0g, ento T injectiva, para todo o

(ii) Seja (x; y; z) 2 R3 , tem-se


T (x; y; z) = z

y + (y

x) t + xt2 = z + x
331

t + t2 + y ( 1 + t)

Logo,
t + t2 + y ( 1 + t) : x; y; z 2 R =
I(T ) = T (x; y; z) : (x; y; z) 2 R3 = z + x
8
t + t2 ; 1 + tg) se
6= 0
< L (f1;
2
= L 1;
t+t ; 1+ t =
:
L (f1; t2 g) se
=0

Se
6= 0 ento o conjunto f1;
t + t2 ; 1 + tg linearmente independente e gera
I (T ), sendo assim uma base de I (T ).
Se = 0 ento o conjunto f1; t2 g linearmente independente e gera I (T0 ), sendo assim
uma base de I (T0 ).
Logo
8
6= 0
< 3 se
dim I(T ) =
:
2 se
= 0:

Como I (T ) um subespao de P2 e neste caso (


I (T ) = P2 , isto , T sobrejectiva se 6= 0.
Se = 0, como I (T0 ) 6= P3 , T0 no sobrejectiva.
Note que: para todo o 2 R,
dim

R3
|{z}

espao de partida

(iii) Considere

6= 0) dim I (T ) = dim P2 , ento

= dim N (T ) + dim I(T ),

= 0 e resolva a equao linear T0 (x; y; z) = 1 + t2 .Atendendo a ter-se


T0 (1; 0; 1) = 1 + t2

ento (1; 0; 1) uma soluo particular da equao linear T0 (x; y; z) = 1 + t2 .


Como, a soluo geral de T0 (x; y; z) = 1 + t2 dada por:
Soluo particular de T0 (x; y; z) = 1 + t2 + (Soluo geral de T0 (x; y; z) = 0)
e como a soluo geral de T0 (x; y; z) = 0 dada por
N (T0 ) = (x; y; z) 2 R3 : T0 (x; y; z) = 0 = L (f(0; 1; 1)g)
ento, a soluo geral de T0 (x; y; z) = 1 + t2 dada por:
(1; 0; 1) + L (f(0; 1; 1)g) = f(1; 0; 1) + s(0; 1; 1) : s 2 Rg .

23. Considere o espao linear P2 dos polinmios reais de varivel real e de grau menor
ou igual a 2. Considere a transformao linear T : P2 ! P2 denida por
T (p (t)) = p0 (t)
onde p0 (t) a derivada de primeira ordem de p (t).
332

2p (t) ,

(i) Seja p (t) 2 P2 . p (t) = a0 + a1 t + a2 t2 ; com a0 ; a1 ; a2 2 R. Tem-se


T a0 + a1 t + a2 t2 = a0 + a1 t + a2 t2
= a1 + 2a2 t

2a0

2a2 t2 =

2a1 t

2 a0 + a1 t + a2 t2 =

2a0 + a1 + (2a2

2a1 ) t

2a2 t2 .

Logo, a expresso geral de T : P2 ! P2 dada por:


T a0 + a1 t + a2 t2 =

2a0 + a1 + (2a2

2a1 ) t

2a2 t2 .

(ii) Seja B = f1; t; t2 g a base cannica (ordenada) de P2 . Determinemos a matriz


M (T ; B; B) que representa T em relao base B.
Como
T (1) = 0 2 = 2,
T (t) = 1 2t;
T t2 = 2t 2t2
tem-se

2
M (T ; B; B) = 4 0
0

1
2
0

3
0
2 5
2

(iii) Como a transformao linear T : P2 ! P2 invertvel, pois M (T ; B; B) invertvel


ento T linear e bijectiva, isto , T um isomorsmo. Sendo T um isomorsmo, T 1
tambm um isomorsmo.
Seja p (t) 2 P2 . p (t) = a0 + a1 t + a2 t2 ; com a0 ; a1 ; a2 2 R.
Tem-se
1
p (t)
2

1 0
p (t)
4
=

1 00
p (t) =
8

1
a0 + a1 t + a2 t2
2

1
a1
4

1
a1
2

1
a0
2

1
a2 +
4

1
(a1 + 2a2 t)
4

1
a2 t
2

a2 2
t
2

1
2a2 =
8

(*)

3 2
3 12
3
a0
2 1
0
a0
2 2 5 4 a1 5 =
(M (T ; B; B)) 1 4 a1 5 = 4 0
a2
0
0
2
a2
3
2
3
2 1
3
2
1
1
1
1
1
a
a
a
a0
2
4
4
2 0
4 1
4 2
1
1 54
1
1
5
a
a
a1 5 = 4
=4 0
2
2
2 1
2 2
1
1
0
0
a2
a
2
2 2

Logo
T

1
a0
2

(p (t)) =

1
a1
4

1
a2 +
4

1
a1
2

1
a2 t
2

a2 2
t
2

(**)

Atendendo a (*) e a (**) conclui-se que a expresso geral do isomorsmo T


T

(p (t)) =

1
p (t)
2

para todo o p (t) 2 P2 .

333

1 0
p (t)
4

1 00
p (t)
8

dada por

(iv) Tem-se
p0 (t)

3t)2 , T (p (t)) = (2

2p (t) = (2
=

(ii)

Logo, p (t) =

1
(2
2
5
4

1
(2 (2
4

3t)2

+ 32 t

9 2
t
2

3t)2

T um isomorsmo

p (t) = T

1
(2 ( 3) ( 3)) =
8

3t) ( 3))

3t)2 =

(2

5 3
+ t
4 2

(ii)

9 2
t.
2

a nica soluo da equao diferencial linear


p0 (t)

3t)2 .

2p (t) = (2

24. Considere o espao linear P2 dos polinmios reais de varivel real e de grau menor
ou igual a 2. Considere a transformao linear T : P2 ! P2 denida por
T (p (t)) = t2 p00 (t)

2p (t) ,

onde p00 (t) a derivada de segunda ordem de p (t).


(i) Seja p (t) 2 P2 . p (t) = a0 + a1 t + a2 t2 ; com a0 ; a1 ; a2 2 R. Tem-se
T a0 + a1 t + a2 t2 = t2 a0 + a1 t + a2 t2
= t2 2a2

2a0

00

2a2 t2 =

2a1 t

2 a0 + a1 t + a2 t2 =
2a0

2a1 t.

Logo, a expresso geral de T : P2 ! P2 dada por:


T a0 + a1 t + a2 t2 =

2a0

2a1 t.

(ii) Seja B = f1; t; t2 g a base cannica (ordenada) de P2 . Determinemos a matriz


M (T ; B; B) que representa T em relao base B.
Como
T (1) = 0 2 = 2,
T (t) = 2t;
T t2 = 2t2 2t2 = 0
tem-se

2
4
0
M (T ; B; B) =
0
(iii) Uma base para N (T ):
Como

ento

02

2
N (M (T ; B; B)) = N @4 0
0

3
0 0
2 0 5
0 0

31
0 0
2 0 5A = L (f(0; 0; 1)g) ,
0 0

N (T ) = a0 + a1 t + a2 t2 2 P2 : (a0 ; a1 ; a2 ) 2 L (f(0; 0; 1)g) = L

t2

Como ft2 g uma base de N (T ), dim N (T ) = 1. Logo, T no injectiva, uma vez que
dim N (T ) 6= 0.
334

Resoluo alternativa para encontrar uma base para N (T ):

N (T ) =
=
=
=

a0 + a1 t + a2 t2
a0 + a1 t + a2 t2
a0 + a1 t + a2 t2
a0 + a1 t + a2 t2

2 P2
2 P2
2 P2
2 P2

: T a0 + a1 t + a2 t2 = 0 =
: t2 2a2 2 a0 + a1 t + a2 t2 = 0 =
: 2a0 2a1 t = 0 =
: a0 = 0 e a1 = 0 = L t2 .

Como ft2 g uma base de N (T ), dim N (T ) = 1.


Uma base para I(T ):
Como f1; t; t2 g gera P2 , tem-se
I (T ) = L

T (1) ; T (t) ; T t2

= L (f 2; 2t; 0g) = L (f 2; 2tg) :

Uma vez que o conjunto f 2; 2tg linearmente independente e gera I (T ), ento f 2; 2tg
uma base de I (T ), tendo-se dim I (T ) = 2.
Como dim P2 = 3, tem-se I (T ) 6= P2 , pelo que T no sobrejectiva.
(iv) (a) Resolva, em P2 ; a equao diferencial linear t2 p00 (t) 2p (t) = 2
Como
2
3
02
31
2
2 0 0
4 1 5 2 C (M (T ; B; B)) = C @4 0
2 0 5A ;
0
0
0 0
uma vez que
32
2
3 2
3
2 0 0
1
2
4 1 5=4 0
2 0 5 4 21 5 ,
0
0 0
0
0
ento

t.

1
1 + t uma soluo particular da equao diferencial linear
2
t2 p00 (t)

Como a soluo geral de t2 p00 (t)


Soluo particular de t2 p00 (t)
e como a soluo geral de t2 p00 (t)

2p (t) = 2

2p (t) = 2

2p (t) = 2

1
1+ t+L
2

t dada por:

t + Soluo geral de t2 p00 (t)

2p (t) = 0 dada por


N (T ) = L

ento a soluo geral de t2 p00 (t)

t:

2p (t) = 2
t2

t2

t dada por:
1
1 + t + at2 : a 2 R .
2

(b) Resolva, em P2 ; a equao diferencial linear 2tp0 (t)


335

2p (0) = 2

t.

2p (t) = 0

Seja T1 (p (t)) = 2tp0 (t)


Logo

2p (0), em que p (t) = a0 + a1 t + a2 t2 ; com a0 ; a1 ; a2 2 R.

T1 (p (t)) = 2tp0 (t)


Como

2p (0) = 2t (a1 + 2a2 t)

2a0 =

2a0 + 2a1 t + 4a2 t2

uma vez que T1 (1) =


(ordenada) de P2
Logo

3
2 0 0
M (T1 ; B; B) = 4 0 2 0 5 ;
0 0 4

2; T1 (t) = 2t; T1 (t2 ) = 4t2 , onde B = f1; t; t2 g a base cannica


2

3 2
a0
2tp0 (t) 2p (0) = 2 t , T1 (p (t)) = 2 t , M (T1 ; B; B) 4 a1 5 = 4
a2
2
3
2
3
a0
2
4 a1 5 = (M (T1 ; B; B)) 1 4 1 5
,
M (T1 ;B;B) invertvel
a2
0
32
3 2
3 2 1
2
3 2
3 12
2
0 0
2
a0
2 0 0
2
, 4 a1 5 = 4 0 2 0 5 4 1 5 = 4 0 21 0 5 4 1 5 = 4
0 0 41
0
0
a2
0 0 4

3
2
1 5 ,
0

1
1
2

Isto , a soluo geral de

2tp0 (t)

2p (0) = 2

:
1

5.

1
t .
2

Vericao:
T1

1
t
2

= 2t

1
t
2

1
0
2

= 2t

1
2

+2=2

Nota importante: Como


dim N (T1 ) = dim N (M (T1 ; B; B)) = 0
ento T1 injectiva e tendo-se
dim

R3
|{z}

espao de partida

= dim N (T1 ) + dim I(T1 ) = dim I(T1 ),

ento I(T1 ) = R3 , isto , T1 sobrejectiva e uma base para I(T1 ) por exemplo
B = 1; t; t2
a base cannica (ordenada) de P2 .
Clculo alternativo de uma base de I(T1 ):
336

t.

Seja p (t) = a0 + a1 t + a2 t2 ; com a0 ; a1 ; a2 2 R. Como


T1 (p (t)) = T1 a0 + a1 t + a2 t2 = 2tp0 (t)

2a0 + 2a1 t + 4a2 t2

2p (0) =

ento
2; 2t; 4t2

I(T1 ) = fT1 (p (t)) : p (t) 2 P2 g = L

Como f1; t; t2 g gera P2 , tem-se

T (1) ; T (t) ; T t2

I (T1 ) = L

2; 2t; 4t2

=L

e sendo o conjunto f 2; 2t; 4t2 g linearmente independente ento


2; 2t; 4t2
uma base de I (T1 ), tendo-se
dim

R3
|{z}

espao de partida

= dim N (T1 ) + dim I(T1 ) = dim N (T1 ) + 3 , dim N (T1 ) = 0,

isto , T1 injectiva.
25. Seja U o subespao das matrizes simtricas de M2
U = A 2 M2

(R), isto ,

(R) : A = AT .

Considere a transformao linear T : U ! U denida por


T (A) = AB + BA
com B =

(i) Seja

0 1
.
1 0
a b
b c
a b
b c

2 U , com a; b; c 2 R. Tem-se
=

a b
b c

0 1
1 0

0 1
1 0

a b
b c

2b a + c
a + c 2b

Logo, a expresso geral de T : U ! U dada por:


T

a b
b c

2b a + c
a + c 2b

(ii) Determinemos uma base para U e a matriz que representa T em relao a essa base.
Seja A 2 U . Tem-se
A=

a b
b c

=a

1 0
0 0

+b

337

0 1
1 0

+c

0 0
0 1

com a; b; c 2 R. Como o conjunto


1 0
0 0

B=

0 1
1 0

0 0
0 1

gera U e linearmente independente, ento B uma base de U . Por outro lado, como
T

1 0
0 0

1 0
0 0

=
= 0

0 1
1 0

0 1
1 0

=
= 2

0 0
0 1

1 0
0 0

1 0
0 0
0 0
0 1

=
= 0

1 0
0 0

0 1
1 0
+1
0 1
1 0
+0
0 1
1 0
+1

0 1
1 0

0 1
1 0

+0

0 1
1 0

0 1
1 0

+2

0 1
1 0

0 1
1 0

+0

ento a matriz que representa T em relao base


2
0
4
M (T ; B; B) = 1
0

1 0
0 0

0 1
1 0

2 0
0 2

0 1
1 0

0 0
0 1
0 1
1 0
0 0
0 1
0 0
0 1
0 0
0 1

B dada por:
3
2 0
0 1 5.
2 0

(iii) Uma base para N (T ):


Como
02
31
02
31
0 2 0
1 0 1
N (M (T ; B; B)) = N @4 1 0 1 5A = N @4 0 2 0 5A = L (f(1; 0; 1)g) ,
0 2 0
0 0 0

ento

N (T ) =

A=

a b
b c

2 U : (a; b; c) 2 L (f(1; 0; 1)g)

=L

1
0

0
1

1 0
uma base de N (T ), dim N (T ) = 1. Logo, T no injectiva, uma
0
1
vez que dim N (T ) 6= 0.
Como

Resoluo alternativa para encontrar uma base para N (T ):

338

A=

a b
b c

2 U : T (A) =

A=

a b
b c

2U :A

A=

a b
b c

2U :

A=

a b
b c

2 U : 2b = 0 e a + c = 0

A=

c 0
0 c

0
1

uma base de N (T ), dim N (T ) = 1.

N (T ) =

1
0

Como

0 0
0 0

0 1
1 0

=
0 1
1 0

2b a + c
a + c 2b

:c2R

0 0
0 0

1 0
0 1

=L

0 0
0 0

A=

1
0

=L

0
1

Uma base para I(T ):


1 0
0 0

Como

0 1
1 0

I (T ) = L

0 0
0 1

gera U , tem-se

1 0
0 0

;T

= L

0 1
1 0

2 0
0 2

= L

0 1
1 0

2 0
0 2

0 1
1 0

2 0
0 2

Uma vez que o conjunto

0 1
1 0
;

0 0
0 1

;T

0 1
1 0

linearmente independente e gera I (T ),

ento
0 1
1 0

2 0
0 2

uma base de I (T ), tendo-se dim I (T ) = 2.


Como dim U = 3, tem-se I (T ) 6= U , pelo que T no sobrejectiva.
(iv) Resolva, em U; a equao linear T (A) = B.
Como
0 1
1 0
ento

1
2

1
2

0
1
2

0 1
1 0

0 1
1 0

1
2

0
1
2

=T

1
2

0
1
2

uma soluo particular da equao linear T (A) = B.


0 12
Como a soluo geral de T (A) = B dada por:
(Soluo particular de T (A) = B) + (Soluo geral de T (A) = 0)
339

e como a soluo geral de T (A) = 0 dada por


1
0

N (T ) = L

0
1

ento a soluo geral de T (A) = B dada por:


1
2

0
1
2

1
0

+L

0
1

1
2

26. Considere a transformao linear T : M2


a representa em relao s bases ordenadas
1 1
1 0

B1 =
de M2

1 1
0 1

(R) e B2 = f1 + t; t + t2 ; t2 + t3 ; t3 g de
2
1
6 0
M (T ; B1 ; B2 ) = 6
4 0
0
a b
c d

(i) Seja

2 M2

+a
0

0
1
2

:a2R .

(R) ! P3 cuja matriz M (T ; B1 ; B2 ) que


0 1
1 1

1 0
1 1

P3 dada por
3
1 1 1
1 1 1 7
7 . (*)
0 1 1 5
0 0 1

(R), com a; b; c; d 2 R. De (*), tem-se

1 1
1 0

= 1+t

1 1
0 1

= 1 + t + t + t2 = 1 + 2t + t2

0 1
1 1

= 1 + t + t + t2 + t2 + t3 = 1 + 2t + 2t2 + t3

1 0
1 1

= 1 + t + t + t2 + t2 + t3 + t3 = 1 + 2t + 2t2 + 2t3

como
1 0
0 0
0 1
0 0
0 0
1 0
0 0
0 1

1
3
1
=
3
1
=
3

1 1
1 0

1 1
1 0
1 1
1 0
2
3

1 1
1 0

1
3
1
+
3
2
3

1 1
0 1

1 1
0 1
1 1
0 1
1
3

1 1
0 1

340

2
3
1
+
3
1
+
3
+

0 1
1 1
0 1
1 1
0 1
1 1
1
3

0 1
1 1

1
3
2
3
1
+
3

1 0
1 1

1 0
1 1
1 0
1 1
1
3

1 0
1 1

ento
a b
c d

T linear

1 0
0 0

aT

0 1
0 0

+bT

0 0
1 0

+cT

+dT

0 0
0 1

1
2
1
1
(1 + t) +
1 + 2t + t2
1 + 2t + 2t2 + t3 +
1 + 2t + 2t2 + 2t3 +
3
3
3
3
1
1
1
2
+b
(1 + t) +
1 + 2t + t2 +
1 + 2t + 2t2 + t3
1 + 2t + 2t2 + 2t3 +
3
3
3
3
1
2
1
1
+c
(1 + t)
1 + 2t + t2 +
1 + 2t + 2t2 + t3 +
1 + 2t + 2t2 + 2t3 +
3
3
3
3
2
1
1
1
+d
(1 + t) +
1 + 2t + t2 +
1 + 2t + 2t2 + t3 +
1 + 2t + 2t2 + 2t3
3
3
3
3

= a

=a

1 1
+ t
3 3

1 2
t +b
3

1 1
+ t
3 3

1 2
t
3

2
1 1
+ t + t2 + t3 +d
3 3
3

t3 +c

1
1
1
1
1
1
4
1
a + b + c + d t+
= a+ b+ c+ d+
3
3
3
3
3
3
3
3
Logo, a expresso geral de T : M2

a b
b c

1
a
3

5
1 4
+ t + t2 + t3
3 3
3

1
2
5
b + c + d t2 +( b + c + d) t3
3
3
3

(R) ! P3 dada por:

1 1 1 1
1
1
1
4
= a+ b+ c+ d+ a + b + c + d t+
3 3 3 3
3
3
3
3

1
a
3

1
2
5
b + c + d t2 +( b + c + d) t3 .
3
3
3

(ii) Como a transformao linear T : M2 2 (R) ! P3 invertvel, pois M (T ; B1 ; B2 )


invertvel ento T linear e bijectiva, isto , T um isomorsmo. Sendo T um isomorsmo,
T 1 tambm um isomorsmo.
Determinemos a expresso geral do isomorsmo T 1 , isto , determinemos
T

a0 + a1 t + a2 t2 + a3 t3 .

Primeiro determinemos M (T ; B2c 2 ; B3c ), onde


B2c

1 0
0 0

0 1
0 0

0 0
1 0

0 0
0 1

e
B3c = 1; t; t2 ; t3
so respectivamente as bases cannicas de M2 2 (R) e de P3 .
A matriz de mudana da base B1 para a base B2c 2 dada por:
2
3
1 1 0 1
6 1 1 1 0 7
7
SB1 !B2c 2 = 6
4 1 0 1 1 5.
0 1 1 1
341

A matriz de mudana da base B2 para a base B3c dada por:


2
3
1 0 0 0
6 1 1 0 0 7
7
SB2 !B3c = 6
4 0 1 1 0 5.
0 0 1 1
Logo, a matriz que representa T em relao s bases B2c

e B3c dada por:

M (T ; B2c 2 ; B3c ) = SB2 !B3c M (T ; B1 ; B2 ) SB1 !B2c


2

1
6 1
=6
4 0
0
2

1
6 1
=6
4 0
0

1
2
1
0

1
2
2
1

0
1
1
0

0
0
1
1

1
2
2
2

32
0
1 1
7
6
0 76 0 1
0 54 0 0
1
0 0
2 1
1

36
6
6
76
76
56
6
6
4

1
3

1
3

2
3

1
3

2
3

1
3

1
3

32
1
1 1
7
6
1 76 1 1
1 54 1 0
1
0 1
3
2
2

1
1
1
0

2
3

1
3

1
3

1
3

1
3

1
3

7 6
7 6
7 6
7 6
7=6
7 6
7 6
7 6
5 4

0
1
1
1
1
3
1
3

3
1
0 7
7
1 5
1

1
3

1
3

1
3

1
3

1
3

4
3

2
3

5
3

1
3

1
3

1 1 1

7
7
7
7
7.
7
7
7
5

Note que a expresso geral de T obtida na alnea (i) pode ser obtida atravs da matriz
M (T ; B2c 2 ; B3c ) anterior:
as coordenadas de T

3 6
6
a
6
6
7
6
c
c 6 b 7
M (T ; B2 2 ; B3 ) 4 5 = 6
6
c
6
6
d
4
2

Logo

a b
b c

a b
b c

na base B3c so dadas por


1
3

1
3

1
3

1
3

1
3

1
3

1
3

4
3

2
3

5
3

1
3

1
3

1 1 1

72
7
7
76
76
74
7
7
5

3 2 1
3
a
a + 13 b + 13 c + 13 d
3
1
1
4
6 1
7
b 7
7 = 6 23 a + 13 b + 13 c + 35 d 7 .
c 5 4 3c 3b 3a + 3d 5
d
c b+d

1 1 1 1
1
1
1
4
= a+ b+ c+ d+ a + b + c + d t+
3 3 3 3
3
3
3
3

1
a
3

1
2
5
b + c + d t2 +( b + c + d) t3
3
3
3

Seja p (t) 2 P3 , isto , p (t) = a0 + a1 t + a2 t2 + a3 t3 , com a0 ; a1 ; a2 ; a3 2 R.

342

Atendendo a que as coordenadas de T


so dadas por:
2
3 2
a0
1 2
6
7
6
1
1
6 a1 7 = 6 2
M (T ; B2c 2 ; B3c )
4 a2 5 4 3
2
a3
1 1

(a0 + a1 t + a2 t2 + a3 t3 ) em relao base B2c


32
1
a0
7
6
1 7 6 a1
0 5 4 a2
0
a3

2
1
1
0

tem-se

T
+ (2a0

a0 + a1 t + a2 t2 + a3 t3 = (2a1

a1 + a2

0 1
0 0

a3 )

+ (3a0

Ou seja, a expresso geral do isomorsmo T


1

3
2a1 a0 2a2 + a3
7 6 2a0 a1 + a2 a3 7
7=6
7,
5 4 3a0 2a1 + a2
5
a1 a0

0 0
1 0

+ (a1

a1 + a2
a1 a0

: P3 ! M2

a3

+
0 0
0 1

a0 )

(R) dada por:

2a1 a0 2a2 + a3 2a0


3a0 2a1 + a2

a0 + a1 t + a2 t2 + a3 t3 =

1 0
0 0

2a2 + a3 )

2a1 + a2 )

2a1 a0 2a2 + a3 2a0


3a0 2a1 + a2

a0

a1 + a2
a1 a0

a3

Tem-se de facto:
T

T = IM2

2 (R)

= IP3 .

(iii) Atendendo alnea anterior, a soluo geral da equao linear


T

a b
c d

= 1 + 2t + 3t2 + 4t3

dada por:
a b
c d

=T

1 + 2t + 3t2 + 4t3 =

1
3

6+4 2
4+3

2+3
2 1

1
2

1
1

27. Seja U o espao linear das funes reais de varivel real duas vezes diferencivel.
Considere a transformao linear T : U ! U denida por
T (f ) = f 00

2f 0 + f .

Considere o subespao S = ff 2 U : f 00 2f 0 + f = 0g de U .
(i) Mostre que o conjunto fet ; tet g uma base de S. Sugesto: Mostre que se f 2 S,
ento f (t) e t um polinmio de grau menor ou igual a 1.
Seja f 2 S. Como
f (t) e

t 00

= f 0 (t) e t f (t) e t = f 00 (t) e t


= (f 00 (t) 2f 0 (t) + f (t)) e t = 0
f 2S

343

f 0 (t) e

f 0 (t) e

+ f (t) e

ento existe c 2 R tal que para todo o t 2 R


t 0

f (t) e

= c.

Assim, existe d 2 R tal que para todo o t 2 R


t

f (t) e

= ct + d 2 P1 = L (f1; tg) .

Logo
et ; tet

f (t) 2 L

Tem-se assim:
et ; tet

S=L

onde o conjunto fet ; tet g linearmente independente uma vez que o conjunto f1; tg linearmente independente.
Logo o conjunto fet ; tet g uma base de S.
(ii) Mostre que dados a; b 2 R, existe uma nica funo f 2 S tal que f (0) = a e
f 0 (0) = b.
Sejam a; b 2 R. Sejam f; g 2 S tais que
f (0) = g (0) = a e f 0 (0) = g 0 (0) = b:
Como S = L (fet ; tet g), existem

1;
t
1e

f (t) =

2;

1;

t
1 te

2 R tais que
t
2e

e g (t) =

t
2 te .

Como f (0) = g (0) = a tem-se


a = f (0) =

e a = g (0) =

2.

Logo
=

2:

Por outro lado, como f 0 (0) = g 0 (0) = b,


b = f 0 (0) =

t
1e

t 0
1 te t=0

t
1e

t
1e

t
1 te t=0

b = g 0 (0) =

t
2e

t 0
2 te t=0

t
2e

t
2e

t
2 te t=0

2.

t
1 te

t
2e

e
Assim,

e uma vez que

2,

ento

Deste modo, para todo o t 2 R


f (t) =

t
1e

isto ,
f = g.
344

t
2 te

= g (t) ;

Pelo que dados a; b 2 R, existe uma nica funo f 2 S tal que f (0) = a e f 0 (0) = b.
(iii) Determine a nica soluo f da equao diferencial linear T (f ) = 1 que verica
f (0) = 1 e f 0 (0) = 0.
A funo identicamente igual a 1 : f = 1 (f (t) = 1;para todo o t 2 R) uma soluo
particular de
ff 2 U : T (f ) = 1 e f (0) = 1 e f 0 (0) = 0g :
Atendendo alnea anterior, existe uma nica funo f 2 S tal que f (0) = 0 e f 0 (0) = 0.
Como
f (t) = et + tet
e
e 0 = f 0 (0) =

0 = f (0) =
ento

f (t) = 0,
para todo o t 2 R, a soluo geral de
ff 2 U : T (f ) = 0 e f (0) = 0 e f 0 (0) = 0g
Como a soluo geral de
ff 2 U : T (f ) = 1 e f (0) = 1 e f 0 (0) = 0g :
dada por:
(Soluo particular de ff 2 U : T (f ) = 1 e f (0) = 1 e f 0 (0) = 0g) +
+ (Soluo geral de ff 2 U : T (f ) = 0 e f (0) = 0 e f 0 (0) = 0g) ,
ento a soluo geral de
ff 2 U : T (f ) = 1

e f (0) = 1 e f 0 (0) = 0g

dada por:
f (t) = 1,
para todo o t 2 R.
28) (i)
0
1

M (T ; B; B) =

1
0

(ii) Como det M (T ; B; B) 6= 0 ento T invertvel e


T (u) = (2; 3; 3; 2) , u = T
Como
M (T

; B; B) =

0
1

1
0
345

(2; 3; 3; 2).

0 1
1 0

ento atendendo a que as coordenadas de (2; 3; 3; 2) em B so 2 e


2v1 3v2 , tem-se que
0 1
2
3
=
1 0
3
2

3 pois (2; 3; 3; 2) =

so as coordenadas de u na base B. Logo


T (u) = (2; 3; 3; 2) , u =

3v1

2v2 = ( 3; 2; 2; 3);

ou seja u = ( 3; 2; 2; 3) a nica soluo da equao linear T (u) = (2; 3; 3; 2):


(iii) Como
R(1; 0; 0; 0) = R (v1 ) + R (w2 ) = v2 = (0; 1; 1; 0);
R(0; 1; 0; 0) = R (v2 ) + R (w1 )
R(0; 0; 1; 0) = R (w1 )

R (w2 ) = v1 = (1; 0; 0; 1);


R (w2 ) = (0; 0; 0; 0)

e
R(0; 0; 0; 1) = R (w2 ) = (0; 0; 0; 0)
ento, sendo Bc a base cannica de R4 ,

pelo que

0
6 1
M (R; Bc ; Bc ) = 6
4 1
0

1
0
0
1

3
0
0 7
7
0 5
0

0
0
0
0

3
2
6 3 7
7
R(u) = (2; 3; 3; 2) , M (R; Bc ; Bc )u = 6
4 3 5,
2

, u 2 f(a; b; c; d) 2 R4 :

b = 2; a =

3; c; d 2 Rg = f( 3; 2; c; d) : c; d 2 Rg:

Isto , a soluo geral de R


(u) = (2; 3; 3; 2)
:
f( 3; 2; c; d) : c; d 2 Rg:
29) a)
T2 (1) = 1
T2 (t) = 2 + 8t

t = 0 (1 + t) + 1 (1
2t2 = 5 (1 + t)

logo

0
M (T2 ; B; B1 ) = 4 1
0
346

t) + 0t2 ;

3 (1
3
5
3 5:
2

t)

2t2 ;

b)
1
0

N (M (T1 ; B1 ; B2 )) = N

2 0
1 1

= f( 2y; y; y) : y 2 Rg = L (f( 2; 1; 1)g) :

Logo
N (T1 ) = L

t) + 1t2

( 2) (1 + t) + 1 (1

=L

3t + t2

Base para
N (T1 ) :

3t + t2 :

T1 sobrejectiva:
dim I(T1 ) = dim P2

dim N (T1 ) = 2 = dim P1 :

c)
T1 (t) =

1
[T1 (1 + t)
2

T1 (1

t)] =

1
1
[1 (1 + t) + 0 (1 + 2t) 2 (1 + t) + 1 (1 + 2t)] = t , T1 (2t) = t
2
2
(uma vez que T1 linear), logo a soluo geral da equao T1 (p (t)) = t :
=

f2tg + N (T1 ) = 2t + c

3t + t2 : c 2 R :

d) f1; tg uma base de P1 . Como


(T1 T2 ) (1) = T1 (T2 (1)) = T1 (1

t) = 2 (1 + t)

1 (1 + 2t) = 1

e
(T1 T2 ) (t) = T1 (T2 (t)) = T1 2 + 8t
= 5T1 (1 + t)
= 5 [1 (1 + t) + 0 (1 + 2t)]

3T1 (1

3 [2 (1 + t)

t)

(1 + 2t)]

ento T1 T2 = I.

347

2t2 =

2T1 t2 =
2 [0 (1 + t) + 1 (1 + 2t)] = t;

Resoluo da 5a Ficha de exerccios para as aulas de problemas


1. Seja

3
9 8 7
A = 4 6 5 4 5.
3 2 1

Como
det (A

9 8 7
0I) = det 4 6 5 4
3 2 1
2
1 2
3
= det 4 0
0
6

3
2
3
3 2 1
1 2 3
5 = det 4 6 5 4 5 = det 4 4 5 6 5 =
9 8 7
7 8 9
3
2
3
3
1 2
3
6 5 = det 4 0
3
6 5=0
12
0 0
0
{z
}
|

ento 0 valor prprio de A e atendendo a (*) (1; 2; 1) 2 N (A) = L f(1; 2; 1)g, logo
tem-se
2
3 2 3
2
3
1
0
1
A4 2 5 = 4 0 5 = 04 2 5
1
0
1
isto , (1; 2; 1) um vector prprio de A associado ao valor prprio 0.

2. Tem-se
2
32
3
2
3
2
32
3
2
3
5 5 5
0
0
5 5 5
1
1
4 5 5 5 54 1 5 = 04 1 5
4 5 5 5 54 1 5 = 04 1 5.
e
5 5 5
1
1
5 5 5
0
0
2
3
5 5 5
Logo, 0 um valor prprio de 4 5 5 5 5 e (0; 1; 1) e (1; 1; 0) so dois vectores prprios
5 5 5
(associados ao valor prprio 0) linearmente independentes.

3. Determinemos os valores prprios de uma matriz A 2


cujo determinante seja igual a 6.
a b
Seja A =
2 M2 2 (R). Tem-se
c d
tr A = 5 , a + d = 5
Sejam

2 cujo trao seja igual a 5 e

det A = 6 , ad

dois valores prprios de A. Como


tr A =

e
348

det A =

1 2

bc = 6.

ento
1

=5

1 2

=6

Logo
[

=5

(5

2)

= 6] , ( = 3 ou

= 2),

isto , os valores prprios de A so 3 e 2.

4. Determinemos uma matriz A real simtrica (AT = A) 2 2 cujos valores prprios


sejam 2 e 2 e tal que (2; 1) seja um vector prprio associado ao valor prprio 2.
a b
Seja A =
2 M2 2 (R) tal que A = AT . Logo b = c. Alm disso, sendo 2 e 2
c d
dois valores prprios de A tem-se
0 = det (A + 2I) = det

a+2
b
b
d+2

b2 + 2a + 2d + ad + 4

b2

e
0 = det (A

2I) = det

2
b

b
d

2a

2d + ad + 4

sendo (2; 1) um vector prprio associado ao valor prprio 2 tem-se


a b
b d
Logo

e assim

2
1

=2

2
1

, (2a + b = 4 e 2b + d = 2).

8
a = 56
>
>
b + 2a + 2d + ad + 4 = 0
>
>
<
b2 2a 2d + ad + 4 = 0
b = 85
,
2a
+
b
=
4
>
>
>
>
:
>
>
2b + d = 2
:
d=
8
>
>
<

A=

a b
b d

=4

6
5

8
5

8
5

6
5

6
5

5.

5. Considere a transformao linear T : R3 ! R3 que admite os vectores prprios


v1 = (1; 2; 1);

v2 = ( 1; 0; 1);

v3 = (0; 1; 0);

associados respectivamente aos valores prprios 1; 2 e 3.


Determinemos a expresso geral de T .
Seja (x; y; z) 2 R3 . Existem ; ; 2 R tais que
(x; y; z) = (1; 2; 1) + ( 1; 0; 1) + (0; 1; 0).

349

Logo
2

1
4 2
1

e assim

3
2
1 0 j x
1
5
4
0 1 j y ! 0
1 0 j z
0

x+y

z,

3
2
1 0 j
x
1
5
4
2 1 j y 2x ! 0
2 0 j z x
0

= 12 ( x + z),

3
0 j
x
1 j y 2x 5
1 j z y+x

1
2
0

= 21 (x + z). Pelo que

1
1
(x + z) T (1; 2; 1) + ( x + z) T ( 1; 0; 1) + ( x + y z) T (0; 1; 0) =
2
2
1
1
= (x + z) (1; 2; 1) + ( x + z) 2( 1; 0; 1) + ( x + y z) 3(0; 1; 0) =
2
2
3
1
3
1
=
x
z; 3y 2x 2z; z
x
2
2
2
2
ou seja, a expresso geral de T dada por:
T (x; y; z) =

T (x; y; z) =

3
x
2

1
z; 3y
2

2x

3
2z; z
2

1
x .
2

6. Considere a transformao linear T : R3 ! R3 denida por


T (x; y; z) = (0; y + 3z; 3y + z).
(i) T (v1 ) = (0; 4; 4). Como no existe 2 R tal que T (v1 ) = v1 , ento v1 no vector
prprio de T .
T (v2 ) = (0; 2; 2) = ( 2)(0; 1; 1) = ( 2)v2 . Logo, v2 um vector prprio de T
associado ao valor prprio 2.
T (v3 ) = (0; 0; 0) = 0(1; 0; 0) = 0v3 . Logo, v3 um vector prprio de T associado ao valor
prprio 0.
T (v4 ) = (0; 10; 6). Como no existe 2 R tal que T (v4 ) = v4 , ento v4 no vector
prprio de T .
T (v5 ) = (0; 12; 12) = 4(0; 3; 3) = 4v5 . Logo, v5 um vector prprio de T associado ao
valor prprio 4.
(ii) Determinemos os valores prprios de T . Seja A = M (T ; Bc3 ; Bc3 ). Tem-se
2
3
0 0 0
A = 4 0 1 3 5,
0 3 1

uma vez que T (1; 0; 0) = (0; 0; 0), T (0; 1; 0) = (0; 1; 3) e T (0; 0; 1) = (0; 3; 1) constituem
respectivamente a 1a , 2a e 3a colunas de A.
O polinmio caracterstico dado por
det(A

I) =
=

0
0 1
0
3
((1
)

0
3
1
3) ((1
350

(1
) + 3) =

)2

9 =

( 2

) (4

).

Os valores prprios de T so os valores prprios de A, isto , so os valores de


quais det(A
I) = 0. Logo, os valores prprios de T so
= 0,

2 e

para os

= 4.

(iii) Como 0 valor prprio de T ento T no invertvel. Como T tem 3 valores


prprios distintos, os vectores prprios correspondentes a cada um deles iro ser linearmente
independentes e como tal ir existir uma base de R3 formada s com vectores prprios de T ,
ou seja, T diagonalizvel.
(iv) O subespao prprio E
E

dado por

= N (T
02

1 I)

=
31

base cannica

N (A
02

1 I)

0 0 0
0 0
= N @4 0 1 3 5A = N @4 0 1
0 3 1
0 0
= f(x; y; z) : y = z = 0g =
= f(x; 0; 0) : x 2 Rg = L (f(1; 0; 0)g) .

O conjunto f(1; 0; 0)g uma base de E 1 .


Os vectores prprios de T associados ao valor prprio

= N (A)
31
0
3 5A =
8

= 0 so

u = (s; 0; 0), com s 2 Rn f0g .


O subespao prprio E
E

dado por

= N (T
02

=
=
=

= N (A + 2I)
31
2 0 0
2 0 0
N @4 0 3 3 5A = N @4 0 3 3 5A =
0 3 3
0 0 0
f(x; y; z) : x = 0 e y + z = 0g =
f(x; y; z) : x = 0 e y + z = 0g =
f(0; z; z) : z 2 Rg = L (f(0; 1; 1)g) .
2 I)

=
31

base cannica

N (A
02

O conjunto f(0; 1; 1)g uma base de E 2 .


Os vectores prprios de T associados ao valor prprio

2 I)

2 so

u = (0; s; s), com s 2 Rn f0g .


O subespao prprio E
E

dado por

= N (T
02
=

=
=
=

N (A
4I)
3 I) = N (A
31
02
31
4 0
0
4 0 0
3 3 5A = N @4 0
3 3 5A =
N @4 0
0
3
3
0
0 0
f(x; y; z) : x = 0 e
y + z = 0g =
f(x; y; z) : x = 0 e y = zg =
f(0; z; z) : z 2 Rg = L (f(0; 1; 1)g) .
3 I)

base cannica

351

O conjunto f(0; 1; 1)g uma base de E 3 .


Os vectores prprios de T associados ao valor prprio

= 4 so

u = (0; s; s) , com s 2 Rn f0g .

7. Considere a transformao linear T : R2 ! R2 denida por


T (1; 2) = (5; 5) = T (2; 1).
(i) Como
(1; 1) =

(1; 2) + (2; 1)

Tem-se
T (v1 ) = T (1; 1) = T [ (1; 2) + (2; 1)]
=

T (1; 2) + T (2; 1) =

T linear

(5; 5) + (5; 5) = (0; 0) = 0(1; 1) = 0v1 :

Como

1
1
(1; 1) = (1; 2) + (2; 1)
3
3

Tem-se
T (v2 ) = T (1; 1) = T

1
1
(1; 2) + (2; 1)
3
3

T linear

1
1
T (1; 2) + T (2; 1) =
3
3

1
10
10
[(5; 5) + (5; 5)] = (1; 1) = v2 :
3
3
3
.
Logo, v2 um vector prprio de T associado ao valor prprio 10
3
=

(ii) Como 0 valor prprio de T ento T no invertvel. Como os vectores v1 = (1; 1)


e v2 = (1; 1) formam uma base de R2 pois so dois vectores linearmente independentes em
R2 e dim R2 = 2 e alm disso, v1 e v2 so vectores prprios de T , ento existe uma base de
R2 formada s com vectores prprios de T , ou seja, T diagonalizvel.
(iii) Seja Bvp = fv1 ; v2 g = f(1; 1); (1; 1)g. Tem-se
M (T ; Bvp ; Bvp ) =

0
0

0
10
3

uma vez que T (v1 ) = 0v1 = 0v1 +0v2 e T (v2 ) = 10


v = 0v1 + 10
v e deste modo as coordenadas
3 2
3 2
10
a
a
(0; 0) e (0; 3 ) constituem respectivamente a 1 e 2 colunas de M (T ; Bvp ; Bvp ).
Logo, Bvp uma base de R2 em relao qual T pode ser representada por uma matriz
diagonal, por ser uma base formada s com vectores prprios de T .
(iv) Seja A = M (T ; Bvp ; Bvp ), com Bvp = f(1; 1); (1; 1)g. O polinmio caracterstico
dado por
10
0
det(A
I) =
=
.
10
0
3
3
352

Os valores prprios de T so os valores prprios de A, isto , so os valores de


quais det(A
I) = 0. Logo, os valores prprios de T so
1

O subespao prprio E
E

=0 e

para os

10
.
3

dado por

= N (T
1 I) = f (1; 1) + (1; 1) : ( ; ) 2 N (A
= f (1; 1) + (1; 1) : ( ; ) 2 L (f(1; 0)g)g =
= f (1; 1) : 2 Rg = L (f(1; 1)g) .

O conjunto f(1; 1)g uma base de E 1 .


Os vectores prprios de T associados ao valor prprio

1 I)g

2 I)g

= 0 so

u = (s; s) , com s 2 Rn f0g .


O subespao prprio E
E

dado por

= N (T
2 I) = f (1; 1) + (1; 1) : ( ; ) 2 N (A
= f (1; 1) + (1; 1) : ( ; ) 2 L (f(0; 1)g)g =
= f (1; 1) : 2 Rg = L (f(1; 1)g) .

O conjunto f(1; 1)g uma base de E 2 .


Os vectores prprios de T associados ao valor prprio

10
3

so

u = (s; s) , com s 2 Rn f0g .

8. Considere a transformao linear T : R3


R representada pela matriz:
2
0 1
A=4 0 1
0 1
3

! R3 que em relao base cannica de


3
0
0 5.
0

(i) Sejam v1 = (1; 0; 0), v2 = (1; 1; 1), v3 = (0; 0; 1). Atendendo matriz, tem-se
T (v1 ) = T (1; 0; 0) = 0(1; 0; 0) + 0(0; 1; 0) + 0(0; 0; 1) =
= (0; 0; 0) = 0(1; 0; 0) = 0v1 ;
T (v2 ) = T (1; 1; 1) = 1(1; 0; 0) + 1(0; 1; 0) + 1(0; 0; 1) =
(1; 1; 1) = 1(1; 1; 1) = 1v2 ;
T (v3 ) = T (0; 0; 1) = 0(1; 0; 0) + 0(0; 1; 0) + 0(0; 0; 1) =
(0; 0; 0) = 0(0; 0; 1) = 0v3 :

Logo, v1 um vector prprio de T associado ao valor prprio 0; v2 um vector prprio de T


associado ao valor prprio 1; v3 um vector prprio de T associado ao valor prprio 0.
353

(ii) Como 0 valor prprio de T ento T no invertvel. Como os vectores v1 =


(1; 0; 0); v2 = (1; 1; 1) e v3 = (0; 0; 1) formam uma base de R3 pois so trs vectores linearmente independentes em R3 e dim R3 = 3 e alm disso, v1 ; v2 e v3 so vectores prprios
de T , ento existe uma base de R3 formada s com vectores prprios de T , ou seja, T
diagonalizvel.
(iii) Seja A = M (T ; Bc3 ; Bc3 ). Tem-se

3
0 1 0
A = 4 0 1 0 5,
0 1 0

uma vez que T (1; 0; 0) = (0; 0; 0), T (0; 1; 0) = (1; 1; 1) e T (0; 0; 1) = (0; 0; 0) constituem
respectivamente a 1a , 2a e 3a colunas de A.
Determinemos os valores prprios de T . Os valores prprios de T so os valores prprios
de A, isto , so os valores de para os quais det(A
I) = 0.
O polinmio caracterstico dado por
1
det(A

0
0

I) =

0
0

(1

).

Logo, os valores prprios de T so


1

O subespao prprio E
E

= N (T
02

=0 e

= 1.

dado por

= N (A
1 I) =
02
31
31
0 1 0
1
0
1
0 5A = N @4 0 1 0 5A =
= N @4 0 1
1
0 1 0
0
1
1
02
31
0 1 0
@
4
0 0 0 5A = (x; y; z) 2 R3 : y = 0 =
= N
0 0 0
= f(x; 0; z) : x; z 2 Rg = L (f(1; 0; 0); (0; 0; 1)g) .
1 I)

O conjunto f(1; 0; 0); (0; 0; 1)g uma base de E 1 .


Os vectores prprios de T associados ao valor prprio

= 0 so

u = (s; 0; t), com s; t 2 Rn f0g .


O subespao prprio E
E

= N (T
02

dado por

= N (A
2 I) =
31
02
1
0
1 1
2
@
4
5
A
@
4
0 1
0
0 0
= N
=N
2
0
1
0 1
2
= f(x; y; z) : x + y = 0 e y z = 0g =
= f(x; x; x) : x 2 Rg = L (f(1; 1; 1)g) .
2 I)

354

31
0
0 5A =
1

O conjunto f(1; 1; 1)g uma base de E 2 .


Os vectores prprios de T associados ao valor prprio

= 1 so

u = (s; s; s), com s 2 Rn f0g .


(iv) possvel ter ento uma base de R3 constituda s por vectores prprios de T :
Bvp = f(1; 0; 0); (1; 1; 1); (0; 0; 1)g ,
uma vez que
dim E

+ dim E

Note ainda que

= 3.

M (T ; Bvp ; Bvp ) = 4 0
0

com

M (T ; Bvp ; Bvp ) = 4 0
0
SBc3 !Bvp

= SBvp !B3c

0
2

3
0
0 5
1

3
0
0 5 = SBc3 !Bvp A SBc3 !Bvp

3
1 1 0
=4 0 1 0 5
0 1 1

e A = M (T ; Bc3 ; Bc3 ).

Isto , a matriz A diagonalizvel e a matriz M (T ; Bvp ; Bvp ) diagonal tendo-se


A

(R3 ; Bc3 )

SBc3 !Bvp # I
(R3 ; Bvp )
Em resumo, existe P

(R3 ; Bc3 )

I # SBc3 !Bvp
(R3 ; Bvp )

M (T ;Bvp ;Bvp )

= SBvp !B3c tal que


2

com D = M (T ; Bvp ; Bvp ) = 4 0


0

0
2

D = P AP
3
0
0 5.

9. Considere a transformao linear T : R2 ! R2 que em relao base ordenada


B1 = f(1; 2) ; (2; 1)g de R2 representada pela matriz:
A=

2 3
3 2

(i) Tem-se
det (A

0I) = det A =
355

5 6= 0.

Logo, como 0 no valor prprio de T ento T invertvel.


Os valores prprios de T so os valores prprios de A, isto , so os valores de
quais det(A
I) = 0.
O polinmio caracterstico dado por
det(A

I) =

3
3

)2

= (2

=( 1

9 = [(2

) (5

3] [(2

para os

) + 3] =

Logo, os valores prprios de T so


1

1 e

= 5.

Como T tem 2 valores prprios distintos, os vectores prprios correspondentes a cada um


deles iro ser linearmente independentes e como tal ir existir uma base de R2 formada s
com vectores prprios de T , ou seja, T diagonalizvel.
(ii) O subespao prprio E
E

= N (T

dado por

= f (1; 2) + (2; 1) : ( ; ) 2 N (A
3 3
(1; 2) + (2; 1) : ( ; ) 2 N
=
3 3

1 I)

( 1) I)g =

3 3
=
0 0
= f (1; 2) + (2; 1) : ( ; ) 2 L (f( 1; 1)g)g =
= f ( 1; 1) : 2 Rg = L (f( 1; 1)g) .
(1; 2) + (2; 1) : ( ; ) 2 N

O conjunto f( 1; 1)g uma base de E 1 .


Os vectores prprios de T associados ao valor prprio

1 so

u = ( s; s), com s 2 Rn f0g .


O subespao prprio E
E

= N (T
=

dado por

= f (1; 2) + (2; 1) : ( ; ) 2 N (A 5I)g =


3 3
(1; 2) + (2; 1) : ( ; ) 2 N
=
3
3
1 I)

3 3
0 0
= f (1; 2) + (2; 1) : ( ; ) 2 L (f(1; 1)g)g =
= f (1; 1) : 2 Rg = L (f(1; 1)g) .
=

(1; 2) + (2; 1) : ( ; ) 2 N

O conjunto f(1; 1)g uma base de E 2 .


Os vectores prprios de T associados ao valor prprio

u = (s; s), com s 2 Rn f0g .


356

= 5 so

(iii) possvel ter uma base de R2 constituda s por vectores prprios de T :


Bvp = f( 1; 1); (1; 1)g ,
uma vez que
dim E

+ dim E

= 2 = dim R2 .

Logo,
M (T ; Bvp ; Bvp ) =

1 0
0 5

uma vez que


T ( 1; 1) =

1(

1; 1) =

1(

1; 1) + 0(1; 1)

e
T (1; 1) =

2 (1; 1)

= 0( 1; 1) +

2 (1; 1).

Deste modo, ( 1 ; 0) e (0; 2 ) constituem respectivamente a 1a e 2a colunas de M (T ; Bvp ; Bvp ).


Alm disso, sendo B1 = f(1; 2) ; (2; 1)g, tem-se
1

M (T ; Bvp ; Bvp ) = SB1 !Bvp A SB1 !Bvp


com
SB1 !Bvp

= SBvp !B1 =

1
1

1
3
1
3

e A = M (T ; B1 ; B1 )

uma vez que


( 1; 1) = (1; 2)

(2; 1)

e (1; 1) =

1
1
(1; 2) + (2; 1) .
3
3

Logo, a matriz A diagonalizvel e tem-se


D = P AP
com
P

1
1

= SBvp !B1 =

1
3
1
3

e D = M (T ; Bvp ; Bvp ) =

1 0
0 5

Observao:
P

(R2 ; B1 )

"I
(R2 ; Bvp )

(R2 ; B1 )

I#P
(R ; Bvp )
2

10. Seja V um espao linear de dimenso nita. Seja T : V ! V uma transformao linear tal que T 2 = T . Uma tranformao linear nas condies anteriores chama-se projeco.
(i) Mostre que os valores prprios de T so 0 e 1:
Dem. Seja um valor prprio de T . Logo existe v 6= 0 tal que
T (v) = v.
Por outro lado, como
v = T (v) = T 2 (v) = (T

T ) (v) = T (T (v)) = T ( v)
357

T linear

T (v) =

v=

tem-se
v=

v,

) v = 0 , ( = 0 ou

(1

v6=0

= 1) .

Logo, os valores prprios de T so 0 e 1.


(ii) Tem-se
T 2 = T , (T

I) T = 0

logo, para todo o u 2 V


(T

I) (T (u)) = 0 (u) = 0 ,T (u) 2 N (T

I)

pelo que
I (T )
Seja agora u 2 N (T
modo

I). Logo (T

N (T

I) .

I) (u) = 0, isto , T (u) = u, ou seja u 2 I (T ). Deste


N (T

I)

I (T )

e assim
I (T ) = N (T

I) .

Por outro lado, sendo n = dim V , atendendo a que


n = dim

V
|{z}

espao de partida

= dim N (T

= dim N (T ) + dim I (T ) =

0I) + dim N (T

1I) = mg (0) + mg (1)

isto ,
n = mg (0) + mg (1)
ento T diagonalizvel, uma vez que existir assim uma base de V formada s com vectores
prprios de T .

11. Considere a transformao linear T : R3 ! R3 denida por


T (x; y; z) = (x; y; x

y) .

(i) Determinemos os valores prprios e os subespaos prprios de T .


Seja Bc3 = f(1; 0; 0); (0; 1; 0); (0; 0; 1)g a base cannica de R3 . Seja A = M (T ; Bc3 ; Bc3 ).
Tem-se
2
3
1
0 0
1 0 5,
A=4 0
1
1 0

uma vez que T (1; 0; 0) = (1; 0; 1), T (0; 1; 0) = (0; 1; 1) e T (0; 0; 1) = (0; 0; 0) constituem
respectivamente a 1a , 2a e 3a colunas de A.
Determinemos os valores prprios de T . Os valores prprios de T so os valores prprios
de A, isto , so os valores de para os quais det(A
I) = 0.

358

O polinmio caracterstico dado por


1
det(A

0
0
1

I) =

0
0

(1

)2 .

Logo, os valores prprios de T so


1

O subespao prprio E
E

=0 e

= 1.

dado por

= N (T
02

= N (A 0I) =
31
02
31
1
0 0
1 0 0
1 0 5A = N @4 0 1 0 5A =
= N @4 0
1
1 0
0 0 0
= f(0; 0; z) : z 2 Rg = L (f(0; 0; 1)g) .
1 I)

O conjunto f(0; 0; 1)g uma base de E 1 .


Os vectores prprios de T associados ao valor prprio

= 0 so

u = (0; 0; s), com s 2 Rn f0g .


O subespao prprio E
E

= N (T

dado por
2 I)

= N (A

02

0
I) = N @4 0
1

0
0
1

31
0
0 5A =
1

= (x; y; z) 2 R3 : x + y + z = 0 =
= (x; y; z) 2 R3 : x = y z =
= f( y z; y; z) : y; z 2 Rg = L (f( 1; 1; 0); ( 1; 0; 1)g) .
O conjunto f( 1; 1; 0); ( 1; 0; 1)g uma base de E 2 .
Os vectores prprios de T associados ao valor prprio
u=( s

= 1 so

t; s; t), com s; t 2 Rn f0g .

(ii) Tem-se T 2 = T , razo pela qual a transformao linear T uma projeco. Como
f( 1; 1; 0); ( 1; 0; 1); (0; 0; 1)g uma base de R3 formada s por vectores prprios de T , cujos
valores prprios associados so respectivamente 1 e 0, tendo-se
T ( 1; 1; 0) = 1( 1; 1; 0) = ( 1; 1; 0)
T ( 1; 0; 1) = 1( 1; 0; 1) = ( 1; 0; 1)
T (0; 0; 1) = 0(0; 0; 1) = (0; 0; 0).
Assim, T projecta os elementos de R3 sobre um plano, paralelamente a um vector, sendo o
plano dado por:
L (f( 1; 1; 0); ( 1; 0; 1)g)
359

isto , por:
x+y+z =0
e o vector dado por:
(0; 0; 1).
12. Considere a transformao linear T : R3 ! R3 que representa geometricamente a
projeco sobre o plano x + y + z = 0, paralelamente ao vector (0; 0; 1).
(i) O plano
(x; y; z) 2 R3 : x + y + z = 0 = L (f( 1; 1; 0); ( 1; 0; 1)g)
tal que
T ( 1; 1; 0) = ( 1; 1; 0) e T ( 1; 0; 1) = ( 1; 0; 1)
e o vector (0; 0; 1) tal que
T (0; 0; 1) = (0; 0; 0)
Ou seja, os vectores que denem o plano so vectores (de I (T )) (linearmente independentes)
prprios de T associados ao valor prprio 1 e o vector (0; 0; 1) um vector (de N (T )) prprio
de T associado ao valor prprio 0:
(ii) Seja (x; y; z) 2 R3 . Como
f( 1; 1; 0); ( 1; 0; 1); (0; 0; 1)g
uma base de R3 , as coordenadas de (x; y; z) em relao base ordenada anterior iro ser
; ; tais que
(x; y; z) = ( 1; 1; 0) + ( 1; 0; 1) + (0; 0; 1).
Atendendo
2
1
4 1
0
e assim

2
3
1
1 0 j x
4
5
0
0 0 j y !
1 1 j z
0

= x + y + z,

y;

3
2
1 0 j
x
1
5
4
1 0 j x+y !
0
1 1 j
z
0
= y. Pelo que

T (x; y; z) = yT ( 1; 1; 0) + ( x
= y( 1; 1; 0) + ( x

3
1 0 j
x
1 0 j
x+y 5
0 1 j x+y+z

y) T ( 1; 0; 1) + (x + y + z) T (0; 0; 1) =

y) ( 1; 0; 1) + (x + y + z) (0; 0; 0) =
= (x; y; x

y),

isto , a expresso geral de T dada por:


T (x; y; z) = (x; y; x

y).

13. Considere a transformao linear T : R2 ! R2 que em relao base cannica de


R2 representada pela matriz:
2 1
A=
.
0 2
360

(i) O polinmio caracterstico dado por


det(A

I) =

1
0

= (2

)2 =

4 + 4.

Os valores prprios de T so os valores prprios de A, isto , so os valores de


quais det(A
I) = 0. Logo, o valor prprio de T

para os

= 2.
O subespao prprio E dado por
E

= N (T

I) = N (A

2I) = N

0 1
0 0

(x; y) 2 R2 : y = 0 = f(x; 0) : x 2 Rg = L (f(1; 0)g) .

O conjunto f(1; 0)g uma base de E .


Os vectores prprios de T associados ao valor prprio

= 2 so

u = (s; 0), com s 2 Rn f0g .


(ii) No existe nenhuma base de R2 constituda s por vectores prprios de T uma vez
que dim E = 1 < 2 = dim R2 . Logo, T no diagonalizvel.
14. Considere a transformao linear T : R3 ! R3 denida por
T (x; y; z) = (3x; 2y + z; 2z).
Seja A = M (T ; Bc3 ; Bc3 ). Tem-se

3
3 0 0
A = 4 0 2 1 5,
0 0 2

uma vez que T (1; 0; 0) = (3; 0; 0), T (0; 1; 0) = (0; 2; 0) e T (0; 0; 1) = (0; 1; 2) constituem
respectivamente a 1a , 2a e 3a colunas de A.
(i) O polinmio caracterstico dado por
3
det(A

I) =

0
0
0

0
1

2
0

= (3

) (2

)2 =

+7

16 + 12.

Os valores prprios de T so os valores prprios de A, isto , so os valores de


quais det(A
I) = 0. Logo, os valores prprios de T so
1

=3 e

361

= 2.

para os

O subespao prprio E
E

dado por

= N (T
02

= N (A
I) =
31 1 02
0 0
0
0 0
@
4
5
A
@
4
0
1 1
0
1
= N
=N
0 0
1
0 0
= f(x; y; z) : y = z = 0g =
= f(x; 0; 0) : x 2 Rg = L (f(1; 0; 0)g) .
1 I)

O conjunto f(1; 0; 0)g uma base de E 1 .


Os vectores prprios de T associados ao valor prprio

31
0
0 5A =
1

= 3 so

u = (s; 0; 0), com s 2 Rn f0g .


O subespao prprio E
E

dado por

= N (T
02

= N (A
2 I) =
31
0
1 5A = f(x; y; z) : x = z = 0g =
0
2 Rg = L (f(0; 1; 0)g) .

2 I)

1 0
@
4
0 0
= N
0 0
= f(0; y; 0) : y

O conjunto f(0; 1; 0)g uma base de E 2 .


Os vectores prprios de T associados ao valor prprio

= 2 so

u = (0; s; 0), com s 2 Rn f0g .


(ii) No existe nenhuma base de R3 constituda s por vectores prprios de T uma vez
que
dim E 1 + dim E 2 = 2 < 3 = dim R3 :
Logo, a matriz A no diagonalizvel, isto , no existe nenhuma base de R3 em relao
qual T possa ser representada por uma matriz diagonal.
15. Considere a transformao linear T : R3 ! R3 denida por
T (x; y; z) = (y + z; 2y + z; y + 2z).
Seja A = M (T ; Bc3 ; Bc3 ). Tem-se

3
0 1 1
A = 4 0 2 1 5,
0 1 2

uma vez que T (1; 0; 0) = (0; 0; 0), T (0; 1; 0) = (1; 2; 1) e T (0; 0; 1) = (1; 1; 2) constituem
respectivamente a 1a , 2a e 3a colunas de A.

362

(i) O polinmio caracterstico dado por


det(A

1
1
0 2
1
=
(2
)2 + =
(2
)2
0
1
2
[((2
) 1) ((2
) + 1)] =
(1
) (3
)
3
2
+4
3 .

I) =
=
=

1 =

(ii) Os valores prprios de T so os valores prprios de A, isto , so os valores de


os quais det(A
I) = 0. Logo, os valores prprios de T so
1

O subespao prprio E
E

= 0,

=1 e

= 3.

dado por

= N (T
02

= N (A
1 I) =
31
0
2
31
0 1 1
0 1 1
= N @4 0 2 1 5A = N @4 0 1 0 5A =
0 1 2
0 0 1
= f(x; y; z) : y = z = 0g =
= f(x; 0; 0) : x 2 Rg = L (f(1; 0; 0)g) .

1 I)

O conjunto f(1; 0; 0)g uma base de E 1 .


Os vectores prprios de T associados ao valor prprio

= 0 so

u = (s; 0; 0), com s 2 Rn f0g .


O subespao prprio E
E

dado por

= N (T
02
=

=
=
=

= N (A
2 I) =
31
02
31
1 1 1
1 1 1
N @4 0 1 1 5A = N @4 0 1 1 5A =
0 1 1
0 0 0
f(x; y; z) : x + y + z = 0 e y + z = 0g =
f(x; y; z) : x = 0 e y + z = 0g =
f(0; z; z) : z 2 Rg = L (f(0; 1; 1)g) .
2 I)

O conjunto f(0; 1; 1)g uma base de E 2 .


Os vectores prprios de T associados ao valor prprio

= 1 so

u = (0; s; s), com s 2 Rn f0g .

363

para

O subespao prprio E
E

dado por

= N (T
02

=
=
=

= N (A
I) =
31 3
02
31
3 1
1
3 1 1
1 1 5A = N @4 0
1 1 5A =
N @4 0
0
1
1
0
0 0
f(x; y; z) : 3x + y + z = 0 e
y + z = 0g =
2
(x; y; z) : x = z e y = z =
3
2
z; z; z : z 2 R = L (f(2; 3; 3)g) .
3
3 I)

O conjunto f(2; 3; 3)g uma base de E 3 .


Os vectores prprios de T associados ao valor prprio

= 3 so

u = (2s; 3s; 3s) , com s 2 Rn f0g .


(iii) possvel ter uma base de R3 constituda s por vectores prprios de T :
Bvp = f(1; 0; 0); (0; 1; 1); (2; 3; 3)g ,
uma vez que
dim E

+ dim E

+ dim E

= 3 = dim R3 .

Logo, a matriz que representa T na base Bvp dada por


2
3 2
0 0 0
1
M (T ; Bvp ; Bvp ) = 4 0 1 0 5 = 4 0
0 0 3
0

uma vez que

0
2

3
0
0 5,
3

T (1; 0; 0) = (0; 0; 0) = 0(1; 0; 0) + 0(0; 1; 1) + 0 (2; 3; 3) ,


T (0; 1; 1) = (0; 1; 1) = 0(1; 0; 0) + 1(0; 1; 1) + 0 (2; 3; 3)
e
T (2; 3; 3) = (6; 9; 9) = 0(1; 0; 0) + 0(0; 1; 1) + 3 (2; 3; 3) .
Deste modo, ( 1 ; 0; 0), (0;
de M (T ; Bvp ; Bvp ).

2 ; 0)

e (0; 0;

3)

constituem respectivamente a 1a , 2a e 3a colunas

(iv) Seja A a matriz que representa T na base cannica de R3 , isto , A = M (T ; Bc3 ; Bc3 ).
Tem-se, por (iii),
2
3 2
3
0 0
0 0 0
1
0 5 = 4 0 1 0 5.
M (T ; Bvp ; Bvp ) = 4 0
2
0 0
0 0 3
3
364

Logo, atendendo ao diagrama


A

(R3 ; Bc3 )

(R3 ; Bc3 )

I # SBc3 !Bvp
(R3 ; Bvp )

"I
SBc3 !Bvp
3
(R ; Bvp )

M (T ;Bvp ;Bvp )

tem-se
1

D = P AP
com

= SBc3 !Bvp

= SBvp !B3c

3 2
3
0
0 0 0
0 5 = 4 0 1 0 5,
0 0 3
3

D = M (T ; Bvp ; Bvp ) = 4 0
0

com

3
0 2
1 3 5
1 3

1
4
= 0
0

e A = M (T ; Bc3 ; Bc3 ).

Isto , a matriz A diagonalizvel e a matriz M (T ; Bvp ; Bvp ) diagonal.


(v) Atendendo a que
D = P AP

tem-se
A=P

DP .

Logo,
An

1
1 n
4
= P D P = 0
0
2
1 n
0
3
3
1
4
0 2 + 21 3n
=
1
0
+ 12 3n
2
2

32
32 n
1
0 2
0
0 0
n
5
4
5
4
0
1 3
0 1
0
n
0
1 3
0 0 3
3
1 n
3
3
1
+ 12 3n 5
2
1
+ 21 3n
2

3 2
x
T n (x; y; z) = An 4 y 5 = 4
z

1 n
3 y + 31 3n z
3
1
1 n
1
+ 23 y +
+
2
2
1
1 n
1
+ 23 y + 2 +
2

para todo o (x; y; z) 2 R3 .

3
0 2
1 3 5
1 3

1 n
3
2
1 n
3
2

z 5,
z

16. Considere a transformao linear T : R3 ! R3 que em relao base B =


f(0; 1; 0) ; (1; 0; 1) ; (1; 0; 1)g (ordenada) de R3 representada pela matriz:
2
3
7 4 2
1 5.
A=4 1 7
1 2 10
365

Logo, a matriz que representa T em relao base cannica Bc de R3 dada por:


2
3
7 4 2
1 5 (SB!Bc ) 1 =
B = M (T ; Bc ; Bc ) = SB!Bc 4 1 7
1 2 10
2
32
3 2
32
3
0 1 1
7 4 2
9 0
0
0 1 0
1 5
1 5 4 21 0
1 5.
= 4 1 0 0 54 1 7
=4 3 7
2
1
1
0
1 1
1 2 10
0 2
3
2 8
2
Note que deste modo, para todo o (x; y; z) 2 R3 tem-se
2 3
x
4
T (x; y; z) = B y 5 = (9x; 3x + 7y z; 3x
z

2y + 8z) .

(i) O polinmio caracterstico dado por


9

det(A

I) = det(B
= (9
=

)
(

3
3

I) =
2

9)2 (

0
1

7
2

= (9

) [(7

) (8

15 + 54 = (9

)(

9) (

6) =

6) .

(ii) Os valores prprios de T so os valores prprios de B, isto , so os valores de


os quais det(B
I) = 0. Logo, os valores prprios de T so
1

O subespao prprio E
E

=9 e

= 6.

dado por

= N (T
02

0
@
4
3
= N
3

= N (B
I) =
31 1 02
0
0
0
5
A
@
4
2
1
0
=N
2
1
3

1 I)

0
0
2

31
0
0 5A =
1

= (x; y; z) 2 R3 : 3x 2y z = 0 =
= f(x; y; 3x 2y) : x; y 2 Rg = L (f(1; 0; 3); (0; 1; 2)g) .
O conjunto f(1; 0; 3); (0; 1; 2)g uma base de E 1 .
Os vectores prprios de T associados ao valor prprio
u = (s; t; 3s
O subespao prprio E
E

= 9 so

2t), com s; t 2 Rn f0g .

dado por

= N (T
02

2] =

= N (B
I) =
31 2 02
3 0
0
3 0
@
4
5
A
@
4
3 1
1
0 1
= N
=N
3
2 2
0 0
= f(x; y; z) : 3x = 0 e y z = 0g =
= f(0; z; z) : z 2 Rg = L (f(0; 1; 1)g) .
2 I)

366

31
0
1 5A =
0

para

O conjunto f(0; 1; 1)g uma base de E 2 .


Os vectores prprios de T associados ao valor prprio

= 6 so

u = (0; s; s), com s 2 Rn f0g .


(iii) possvel ter uma base de R3 constituda s por vectores prprios de T :
Bvp = f(1; 0; 3); (0; 1; 2); (0; 1; 1)g ,
uma vez que
dim E

+ dim E

= 3 = dim R3 .

Logo, a matriz que representa T na base Bvp dada por


2
3 2
9 0 0
1
M (T ; Bvp ; Bvp ) = 4 0 9 0 5 = 4 0
0 0 6
0

0
1

uma vez que

3
0
0 5,
2

T (1; 0; 3) = (9; 0; 27) = 9(1; 0; 3) + 0(0; 1; 2) + 0(0; 1; 1),


T (0; 1; 2) = (0; 9; 18) = 0(1; 0; 3) + 9(0; 1; 2) + 0(0; 1; 1)
e
T (0; 1; 1) = (0; 6; 6) = 0(1; 0; 3) + 0(0; 1; 2) + 6(0; 1; 1).
Deste modo, ( 1 ; 0; 0), (0; 2 ; 0) e (0; 0;
de M (T ; Bvp ; Bvp ).
Logo, atendendo ao diagrama

3)

constituem respectivamente a 1a , 2a e 3a colunas

(R3 ; Bc3 )

(R3 ; Bc3 )

I # SBc3 !Bvp
(R3 ; Bvp )

SBc3 !Bvp
"I
(R3 ; Bvp )

M (T ;Bvp ;Bvp )

tem-se
1

D = P BP
com

D = M (T ; Bvp ; Bvp ) = 4 0
0

com
P

= SBc3 !Bvp

= SBvp !B3c

,
3 2
3
0
9 0 0
0 5 = 4 0 9 0 5,
0 0 6
2

0
1

1
4
= 0
3

3
0 0
1 1 5
2 1

e B = M (T ; Bc3 ; Bc3 ).

Isto , a matriz B diagonalizvel e a matriz M (T ; Bvp ; Bvp ) diagonal.


(iv) Atendendo a que
D = P BP
367

tem-se
1

B=P

DP .

Logo,
Bn = P
2

= 4

= 4
2

= 4

e
2

32 n
32
3
0 0
9
0 0
1 0 0
1 n
1 1 5 4 0 9n 0 5 4 0 1 1 5
D P
2 1
0 0 6n
3
2 1
3
2
3
1 0
9n 0 0
1
0
0
n
5
4
0 1
0 9
0
1 1=3
1=3 5 =
n
3
2
0 0 6
1 2=3 1=3
3
2
3
9n
0
0
1
0
0
0
9n
6n 5 4 1 1=3
1=3 5 =
n
n
n
9 3 9 ( 2) 6
1 2=3 1=3
3
9n
0
0
1 n
9 + 13 6n 5
9n 6n 13 9n + 32 6n
3
2 n
9n 6n
9 + 32 6n 32 9n + 13 6n
3
1
4
= 0
3
32
0
1 54
1

3
1 1
0 0 5
1 1

0
4
= 1
0

Por outro lado,

An = (SB!Bc )
9n

4 9n 6n
9n 6n
2 2 n 1 n
6 + 39
3
1 n
1 n
4
6
= 39
3
1 n
1 n
6
9
3
3

B n SB!Bc =

32
0
0
1 n
1 n 54
1
9 + 36
3
2 n
1 n
9
+
6
0
3
3
3
2 n
2 n
9
6
3
3
1 n
1 n 5
6
9
.
3
3
4 n
1 n
9
6
3
3

0
1 n
9 + 32 6n
3
2 n
9 + 32 6n
3

3 2
x
T n (x; y; z) = B n 4 y 5 = 4 (9n
z
(9n

4 n
9
3
2 n
6
3
2 n
9
3

4 n
6
3
1 n
9
3
2 n
6
3

9n x
6n ) x + 31 9n + 32 6n y +
2 n
6n ) x +
9 + 32 6n y +
3

para todo o (x; y; z) 2 R3 .

17. Sabendo que os vectores (1; 1; 1); (1; 0;


2
1
4
A= a
d

existem

1;

2 R tais que

(1; 1; 1) 2 N (A
isto ,

1 I),

2
4

a
d

1
c

3
1 1
0 0 5=
1 1

+ 31 6n z 5 ,
+ 31 6n z

1) e (1; 1; 0) so vectores prprios da matriz


3
1 1
b c 5,
e f

(1; 0; 1) 2 N (A
1

1 n
9
3
2 n
9
3

368

2 I)

32

e (1; 1; 0) 2 N (A

3 2 3
1
0
54 1 5 = 4 0 5,
1
0

3 I),

a
d
1

a
d

1
c

1
c

32

3 2 3
1
0
54 0 5 = 4 0 5
1
0

32

3 2 3
1
0
54 1 5 = 4 0 5.
0
0

Logo, tem-se respectivamente


8
8
3
=
0
>
>
1 = 3
1
>
>
>
>
>
>
<
<
a+b+c=3
a+b+c
1 = 0 ,
>
>
>
>
>
>
>
>
:
:
d + e + f = 3,
d+e+f
=
0
1
8
8
=
0
>
>
2
2 = 0
>
>
>
>
>
>
<
<
a c=0
a=c
,
>
>
>
>
>
>
>
>
:
:
d f+ 2=0
d=f
e
8
8
=
0
>
>
3
>
> 3=0
>
>
>
>
<
<
a b+ 3 =0 ,
a=b
>
>
>
>
>
>
>
>
:
:
d e=0
d = e.
Assim,

8
>
>
>
>
>
>
>
>
<
>
>
>
>
>
>
>
>
:

=3

=0

=0

a = b = c = d = e = f = 1.

18. Considere a transformao linear T : M2 2 (R) ! M2 2 (R) denida por


T (A) = A + AT .
(i) Seja
Bc2

1 0
0 0

0 1
0 0

a base cannica (ordenada) de M2 2 (R).

369

0 0
1 0

0 0
0 1

A matriz M (T ; Bc2 2 ; Bc2 2 ) que representa T em relao base cannica (ordenada) Bc2
dada por
2
3
2 0 0 0
6 0 1 1 0 7
7
M (T ; Bc2 2 ; Bc2 2 ) = 6
4 0 1 1 0 5,
0 0 0 2
uma vez que
1 0
0 0

2 0
0 0

1 0
0 0

=2

+0

0 1
0 0

+0

0 0
1 0

+0

0 0
0 1

0 1
0 0

0 1
1 0

=0

1 0
0 0

0 1
0 0

0 0
1 0

+0

0 0
0 1

0 0
1 0

0 1
1 0

=0

1 0
0 0

0 1
0 0

0 0
1 0

+0

0 0
0 1

+0

0 1
0 0

+0

e
T

0 0
0 1

0 0
0 2

1 0
0 0

=0

0 0
1 0

+2

0 0
0 1

(ii) Seja A = M (T ; Bc2 2 ; Bc2 2 ). O polinmio caracterstico dado por


2
det(A

I) =

0
0
0
0

0
1

1
1
0

= (2
) [((1
=
(2
)3 .

0
0
0

= (2

)2 (1

)2

1 =

2
1) ((1

) + 1)] =

Os valores prprios de T so os valores prprios de A, isto , so os valores de


quais det(A
I) = 0. Logo, os valores prprios de T so
1

=0 e

370

= 2.

para os

O subespao prprio E
E

= N (T

1 I)

dado por
a b
c d

2 M2 2 (R) : (T

a b
c d

2 M2 2 (R) : T

a b
c d

2 M2 2 (R) :

2a b + c
c + b 2d

1a

1b

1c

1d

a b
c d

2 M2 2 (R) :

2a b + c
c + b 2d

0 0
0 0

a b
c d

2 M2 2 (R) :

2a b + c
c + b 2d

a b
c d

2 M2 2 (R) : 2a = 0 e b + c = 0 e 2d = 0

0
c

c
0
0
1

= L

a b
c d

a b
c d

1 I)

2 M2 2 (R) : c 2 R
1
0

a b
c d

1I

0 0
0 0

0 0
0 0

=
0 0
0 0

0 0
0 0

=
0 0
0 0

=
=

0
1
uma base de E 1 .
1 0
Os vectores prprios de T associados ao valor prprio

O conjunto

U=
O subespao prprio E
E

= N (T

2 I)

0
s

s
0

dado por
a b
c d

2 M2 2 (R) : (T

a b
c d

2 M2 2 (R) : T

a b
c d

2 M2 2 (R) :

2a b + c
c + b 2d

a b
c d

2 M2 2 (R) :

2a b + c
c + b 2d

a b
c d

2 M2 2 (R) :

a b
c d

2 M2 2 (R) : b = c

a c
c d

2 M2 2 (R) : a; c; d 2 R

1 0
0 0

= 0 so

, com s 2 Rn f0g .

= L

0 1
1 0

a b
c d

0
c+b

0 0
0 1

=
.

371

a b
c d

2I

b+c
0

a b
c d

2 I)

2a

2b

2c

2d

2a 2b
2c 2d
=

0 0
0 0

0 0
0 0

0 0
0 0

=
=

0 0
0 0

0 0
0 0
=

=
=

1 0
0 1
0 0
;
;
uma base de E 2 .
0 0
1 0
0 1
Os vectores prprios de T associados ao valor prprio 2 = 2 so

O conjunto

r s
s t

U=

, com r; s; t 2 Rn f0g .

(iii) possvel ter uma base de M2 2 (R) constituda s por vectores prprios de T :
1 0
0 0

Bvp =

0 1
1 0

0
1

1
0

0 0
0 1

uma vez que


dim E

+ dim E

= 4 = dim M2 2 (R).

Logo, a matriz que representa T na base Bvp


2
2 0 0
6 0 2 0
M (T ; Bvp ; Bvp ) = 6
4 0 0 0
0 0 0

uma vez que

dada por
3 2
0
2
7
6
0 7 6 0
=
0 5 4 0
2
0

0
2

0
0

0
0

3
0
0 7
7,
0 5
2

1 0
0 0

2 0
0 0

=2

1 0
0 0

+0

0 1
1 0

+0

0
1

1
0

+0

0 0
0 1

0 1
1 0

0 2
2 0

=0

1 0
0 0

+2

0 1
1 0

+0

0
1

1
0

+0

0 0
0 1

0
1

1
0

0 0
0 0

1 0
0 0

=0

0 1
1 0

+0

+0

0
1

1
0

+0

0 0
0 1

e
T

0 0
0 1

0 0
0 2

=0

1 0
0 0

Deste modo, ( 2 ; 0; 0; 0), (0; 2 ; 0; 0), (0; 0;


1a , 2a , 3a e 4a colunas de M (T ; Bvp ; Bvp ).
Logo, atendendo ao diagrama
(M2 2 (R); Bc2 2 )
1

SBc2

!Bvp

"I

(M2 2 (R); Bvp )

0 1
1 0

+0
1 ; 0)

e (0; 0; 0;

1
0

tem-se
1

!Bvp

(M2 2 (R); Bvp )

M (T ;Bvp ;Bvp )

+2

0 0
0 1

constituem respectivamente a

I # SBc2

372

2)

0
1

(M2 2 (R); Bc2 2 )

D = P AP

+0

com

com

6 0
D = M (T ; Bvp ; Bvp ) = 6
4 0
0
1

0
0

0
0

= SBc2

= SBvp !B2c

!Bvp

3 2
0
2 0 0
7
6
0 7 6 0 2 0
=
0 5 4 0 0 0
0 0 0
2

3
0
0 7
7,
0 5
2

e A = M (T ; Bc2 2 ; Bc2 2 ).

Isto , a matriz A diagonalizvel e a matriz M (T ; Bvp ; Bvp ) diagonal.


2 1
. Tem-se
2 5

19. (i) Seja A1 =

det(A1

I) =

1
2

= (2

) (5

)+2=

7 + 12 = (3

Os valores prprios de A1 so
1

O subespao prprio E
E

=3 e

= 4.

dado por

= N (A
= N

1 I)

=
1

1 1
2 2

=N

1 1
= (x; y) 2 R2 :
0 0
= f(y; y) : y 2 Rg = L (f(1; 1)g) .

= N

O conjunto f(1; 1)g uma base de E 1 .


Os vectores prprios de A1 associados ao valor prprio

x+y =0 =

= 3 so

u = (s; s), com s 2 Rn f0g .


O subespao prprio E
E

dado por

= N (A2
= N

2 I)

=
1

2 1
2 1

=N

2 1
= (x; y) 2 R2 :
0 0
= f(x; 2x) : x 2 Rg = L (f(1; 2)g) .

= N

O conjunto f(1; 2)g uma base de E 2 .


Os vectores prprios de A1 associados ao valor prprio

2x + y = 0 =

u = (s; 2s), com s 2 Rn f0g .


373

= 4 so

) (4

).

possvel ter uma base de R2 constituda s por vectores prprios de A1 :


Bvp = f(1; 1); (1; 2)g ,
uma vez que
dim E

+ dim E

= 2.

Logo, a matriz A1 diagonalizvel e tem-se


D1 = P1 A1 P1 1 ,
com
P1

1 1
1 2

= SBvp !B2c =

e
D1 =

3 0
0 4

3
2 1 1
(ii) Seja A2 = 4 0 3 1 5. Tem-se
0 1 3
2
det(A2

1
0
0

I) =
= (2

1
1

3
1

= (2

) (3

)2

1 =

) [(3

1] [(3

) + 1] = (2

)2 (4

).

Os valores prprios de A2 so
1

O subespao prprio E
E

= N (A2
02
= N @4
02

=2 e

= 4.

dado por

1 I)

=
1

0
0

3
1
31

1
1

31

02

31
0 1 1
5A = N @4 0 1 1 5A =
0 1 1

0 1 1
= N @4 0 0 0 5A = (x; y; z) 2 R3 : y + z = 0 =
0 0 0
= f(x; z; z) : x; z 2 Rg = L (f(1; 0; 0); (0; 1; 1)g) .
O conjunto f(1; 0; 0); (0; 1; 1)g uma base de E 1 .
Os vectores prprios de A2 associados ao valor prprio

= 2 so

u = (s; t; t), com s; t 2 Rn f0g .

374

O subespao prprio E
E

= N (A2
02

2 I)

= N @4

dado por

=
1

0
0

02

1
1

31

31

02

2
5A = N @4 0
0

2 1 1
@
4
0
1 1 5A = (x; y) 2 R2 :
= N
0
0 0
= f(z; z; z) : z 2 Rg = L (f(1; 1; 1)g) .

1
1
1

31
1
1 5A =
1

2x + y + z = 0 e

O conjunto f(1; 1; 1)g uma base de E 2 .


Os vectores prprios de A2 associados ao valor prprio

y+z =0 =

= 4 so

u = (s; s; s), com s 2 Rn f0g .


possvel ter uma base de R3 constituda s por vectores prprios de A2 :
Bvp = f(1; 0; 0); (0; 1; 1); (1; 1; 1)g ,
uma vez que
dim E

+ dim E

= 3.

Logo, a matriz A2 diagonalizvel e tem-se


D 2 = P 2 A2 P 2 1 ,
com
1

P2
e

= SBvp !B3c

D2 = 4 0
0

1
4
= 0
0

3
0 1
1 1 5
1 1

3 2
3
0
2 0 0
0 5 = 4 0 2 0 5.
0 0 4
2

3
1 1 0
(iii) Seja A3 = 4 1 1 0 5. Tem-se
0 0 0
1
det(A3

I) =
= (

1
1
0

0
0

=(

) (1

)2

1 =

(2

).

) [(1

1] [(1

Os valores prprios de A3 so
1

=0 e
375

= 2.

) + 1] =

O subespao prprio E
E

= N (A3
02

dado por
1 I)

= N @4

1
0

02

=
1

0
0

0
31

31

02

31
1 1 0
5A = N @4 1 1 0 5A =
0 0 0

1 1 0
@
4
0 0 0 5A = (x; y; z) 2 R3 : x + y = 0 =
= N
0 0 0
= f( y; y; z) : y; z 2 Rg = L (f( 1; 1; 0); (0; 0; 1)g) .
O conjunto f( 1; 1; 0); (0; 0; 1)g uma base de E 1 .
Os vectores prprios de A2 associados ao valor prprio

= 0 so

u = ( s; s; t), com s; t 2 Rn f0g .


O subespao prprio E
E

= N (A2
02

= N @4

02

dado por

2 I)

=
1

1
0

0
0

31

02

31

1
5A = N @4 1
0

1 1 0
= N @4 0 0 0 5A = (x; y) 2 R2 :
0 0
2
= f(y; y; 0) : y 2 Rg = L (f(1; 1; 0)g) .

1
1
0

31
0
0 5A =
2

x+y =0 e

O conjunto f(1; 1; 0)g uma base de E 2 .


Os vectores prprios de A3 associados ao valor prprio

2z = 0 =

= 2 so

u = (s; s; 0), com s 2 Rn f0g .


possvel ter uma base de R3 constituda s por vectores prprios de A3 :
Bvp = f( 1; 1; 0); (0; 0; 1); (1; 1; 0)g ,
uma vez que
dim E

+ dim E

= 3.

Logo, a matriz A3 diagonalizvel e tem-se


D 3 = P 3 A3 P 3 1 ,
com
P3

= SBvp !B3c

3
1 0 1
=4 1 0 1 5
0 1 0

e D3 = 4 0
0
376

0
1

3 2
3
0
0 0 0
0 5 = 4 0 0 0 5.
0 0 2
2

20. Considere a transformao linear T :


4
R representada pela matriz
2
0 0
6 a 0
6
4 0 b
0 0

R4 ! R4 que em relao base cannica de


0
0
0
c

3
0
0 7
7,
0 5
0

com a; b; c 2 R.
Determinemos os valores prprios de T . Tem-se
0
a
0
0

0
0

b
0

=(

)4 =

O valor prprio de T = 0.
O subespao prprio E dado por
02
0
0
B6 a
0
6
E = NB
@4 0
b
0
0
c
=

0
0
0

31
0
C
0 7
7C = N
0 5A

02

0
B6 a
B6
@4 0
0

0
0
b
0

0
0
0
c

(x; y; z; w) 2 R4 : ax = 0 e by = 0 e cz = 0 .

31
0
C
0 7
7C =
0 5A
0

Assim, para que exista uma base de R4 constituda s por vectores prprios de T necessrio
que se tenha
a = b = c = 0.
Caso contrrio, teramos
dim E < 4.

21. (i) Como Au1 = ( + 1)u1 e Au2 = (


1)u2 ento u1 ; u2 so vectores prprios de
A, associados respectivamente aos valores prprios + 1 e
1.
(ii)
p( ) = det(A
Para ( + 1 6= 3 e
todos distintos.

I) = (3
1 6= 3) ,

) (

)2

1 = (3

)(

1)(

62 f2; 4g os valores prprios de A: 3;

+ 1):
+1 e

1 so

(iii) Para 62 f2; 4g, fu1 g; fu2 g e f(3


; 2 6 + 8; 1)g so bases de E +1 , E 1 e
E3 , respectivamente.
Para = 2, E3 = E +1 . Alm disso, fu2 g uma base de E 1 e fu1 g uma base de E3 .
Para = 4, E3 = E 1 . Alm disso, fu2 g uma base de E3 e fu1 g uma base de E +1 .
(iv) Para 62 f2; 4g a matriz A diagonalizvel, pois os seus valores prprios so todos
distintos. Se = 2 ou = 4 ento 3 valor prprio de A e mg (3) = 1 < 2 = ma (3), pelo
que A no diagonalizvel. Logo A diagonalizvel , 62 f2; 4g.
377

Resoluo da 3a Ficha de exerccios facultativos


1. Seja T : Rn ! Rn uma transformao linear invertvel. Seja u um vector prprio de
T associado a um valor prprio de T . Verique que u tambm um vector prprio de T 1
e determine o valor prprio de T 1 que lhe est associado.
Dem. Tem-se
T (u) = u,
com u 6= 0. Como T invertvel e T

linear,
1

u=T
Por outro lado, tem-se

( u) = T

(u).

6= 0 uma vez que u 6= 0 e T invertvel. Logo,


1

T
1

Isto , u um vector prprio de T

(u) =

u.
1

associado ao valor prprio

de T

2. Seja V um espao linear. Seja T : V ! V uma transformao linear. Seja u um


vector prprio de T associado a um valor prprio de T . Verique que u tambm um
vector prprio de T 2 associado ao valor prprio 2 de T 2 .
Dem. Tem-se
T (u) = u,
com u 6= 0. Logo, como T linear,
T 2 (u) = (T

T ) (u) = T (T (u)) = T ( u) = T (u) =

isto , u um vector prprio de T 2 associado ao valor prprio


3. Seja A uma matriz do tipo n n. Mostre que se
um valor prprio de Ak , onde k um inteiro positivo.

u=

u,

de T 2 .

um valor prprio de A ento

Dem. Sendo k um inteiro positivo, tem-se


Ak
Logo, se
positivo.

I = (A

I)(Ak

um valor prprio de A ento

+ Ak
k

+A

k 2

k 1

I).

um valor prprio de Ak , onde k um inteiro

4. Uma matriz A do tipo n n diz-se nilpotente se Al = 0 para algum inteiro positivo


l. Mostre que se A nilpotente ento o nico valor prprio de A 0.
Dem. Suponhamos que Al = 0 para algum inteiro positivo l. Seja um valor prprio
de A. Pelo exo anterior, l um valor prprio de Al . Como Al = 0, ento:
0 = det(Al

I) = det(
378

I) = ( 1)n l .

Logo

= 0 e como tal, 0 o nico valor prprio de A.


n. Verique que A e AT tm os mesmos valores prprios.

5. Seja A uma matriz n


Dem. Tem-se
det(A

I)T = det(AT

I) = det (A

I).

Isto , as matrizes A e AT tm os mesmos valores prprios.


6. Seja A uma matriz n n cuja soma das suas colunas constante e igual a r. Mostre
que r um valor prprio de A:
Dem. Tem-se

6
6
=6
4

a11 a12
6 7 6 a21 a22
6 7 6
A 6 7 = 6 ..
..
...
4 5 4 .
.
an1 an2
1
2
3
3 2
a1n
a12
a11
6
6
7
7
a21 7 6 a22 7
6 a2n
+ 6 ..
.. 7 + 6 .. 7 +
4 .
. 5 4 . 5

an1

1
1
..
.

ann

an2

a1n
a2n
..
.

32
76
76
76
54

ann
3 2
7 6
7 6
7=6
5 4

r
r
..
.
r

1
1
..
.
1
3

7
7
7=
5

6
7
6
7
7 = r6
4
5

1
1
..
.
1

7
7
7.
5

Logo r um valor prprio de A, associado ao vector prprio (1; 1; : : : ; 1).

7. Seja A 2 Mn n (R). Seja P uma matriz diagonalizante para A. Determine uma


matriz diagonalizante para AT em termos de P .
Dem. Tem-se
D = P AP
e
D = DT = P AP
Logo, a matriz (P

1 T

= P

1 T

AT P T .

1 T

) uma matriz diagonalizante para AT .

8. Seja Q uma matriz n n real ortogonal, isto , tal que Q 1 = QT . Mostre que se n
fr mpar ento Q tem o valor prprio 1 ou tem o valor prprio 1.
Dem. Atendendo a que QQT = I tem-se
(det Q)2 = det Q det Q = det Q det QT = det QQT = det I = 1 , (det Q = 1 ou det Q =
Logo:
Se det Q = 1
det (Q

I) = det Q I

QT
379

= det Q det I

QT =

1).

= ( 1) det Q det Q

det Q det (Q

n mpar

, 2 det (Q

I)

I) = 0 , det (Q

det (Q

I) ,

I) = 0

isto , 1 valor prprio de Q;


Se det Q = 1
det (Q + I) = det Q I + QT = det Q det I + QT =
h
i
T
T
= det Q det Q + I = det (Q + I) = det (Q + I) ,

, 2 det (Q + I) = 0 , det (Q + I) = 0 , det (Q


isto ,

( 1) I) = 0

1 valor prprio de Q.

9. Determine uma matriz A real 2


Dem. Seja A =

a b
c d

2 M2

2 tal que det A < 0. Mostre que A diagonalizvel.


(R). Sejam
1 2

dois valores prprios de A. Como

= det A < 0

ento 1 e 2 so dois valores prprios distintos de A, pelo que os vectores prprios correspondentes so linearmente independentes, constituindo assim uma base de R2 , razo pela
qual A diagonalizvel.
10. Seja A uma matriz n n e seja um valor prprio de A com multiplicidade algbrica
igual a n. Mostre que se A fr diagonalizvel ento A uma matriz diagonal.
Dem. Seja um valor prprio de A com multiplicidade algbrica igual a n. Como A
do tipo n n, ento o nico valor prprio de A. Assim, A fr diagonalizvel se e s se
dim N (A

I) = mg ( ) = ma ( ) = n

o que equivalente a ter-se


A
isto ,

I = 0 (matriz nula)
2

6
6
A= I=6
4

0
..
.

0
...
..
.

ou seja, A uma matriz diagonal.

...
..
0

3
0
.. 7
. 7
7
0 5

11. Seja V um espao linear e seja T : V ! V uma transformao linear tal que todos
os vectores no nulos de V so vectores prprios. Mostre que T tem um nico valor prprio.

380

Dem. Suponhamos, com vista a uma contradio, que 1 e 2 eram dois valores prprios
distintos de T . Sejam v1 e v2 vectores prprios de T associados respectivamente aos valores
prprios 1 e 2 . Logo, o conjunto fv1 ; v2 g linearmente independente. Por outro lado
T (v1 + v2 ) = T (v1 ) + T (v2 ) =

1 v1

2 v2

e como cada vector no nulo de V um vector prprio de T , ento v1 + v2 um vector


prprio de T e assim, existe um escalar 3 tal que
T (v1 + v2 ) =

(v1 + v2 ) =

3 v1

3 v2 .

Deste modo, tem-se


1 v1

2 v2

3 v1

3 v2

ou seja
(

3 ) v1

+(

2 ) v2

= 0.

Como o conjunto fv1 ; v2 g linearmente independente, ento ter-se-ia


1

eram dois valores prprios distintos de

isto ,
1

contrariando o facto de se ter assumido que


T.
Logo, T tem um nico valor prprio.

=
1

12. Sejam A e B duas matrizes do tipo n


valores prprios.
Dem. Sejam A; B 2 Mn
det (AB

n (R).

n. Mostre que AB e BA tm os mesmos

Atendendo a que

0I) = det (AB) = det (BA) = det (BA

0I) ;

0 valor prprio de AB se e s se 0 valor prprio de BA.


Seja um valor prprio de AB, com 6= 0. Ento existe u 6= 0 tal que ABu = u. Seja
w = Bu: Como u 6= 0 e B invertvel ento w 6= 0. Logo,
(BA) w = (BA) Bu = B (AB) u = B u =

(Bu) = w.

Isto , valor prprio de BA com w como vector prprio associado.


Seja um valor prprio de BA, com 6= 0. Ento existe u 6= 0 tal que BAu = u. Seja
w = Au: Como u 6= 0 e A invertvel ento w 6= 0. Logo,
(AB) w = (AB) Au = A (BA) u = A u =
Isto ,

(Au) = w.

valor prprio de AB com w como vector prprio associado.

13. Sejam A e B duas matrizes tais que AB = BA. Mostre que A e B tm um vector
prprio em comum.
381

Sugesto: Sendo um valor prprio de A, considere C a matriz cujas colunas formam


uma base ordenada S de N (A
I) e verique que (A
I) BC = 0. Finalmente considere
a matriz P cujas colunas so respectivamente as coordenadas das colunas de BC em relao
base S e sendo v um vector prprio de P mostre que Cv um vector prprio comum a A
e B.
Dem. Suponhamos que as matrizes quadradas A e B so do tipo n n. Seja um valor
prprio de A. Tem-se N (A
I) 6= f0g. Seja r = dim N (A
I). Seja C a matriz n r
cujas colunas formam uma base ordenada S de N (A
I). Temse
(A

I) BC = ABC

BC

AB=BA

BAC

BC = B (A

I) C = B0 = 0

Seja P = (pij ) a matriz r r cujas colunas so respectivamente as coordenadas das


colunas de BC em relao base S. Tem-se, para k = 1; :::; r
r
r
X
X
pik [C] i =
[C] i pik .
[BC] k =
|{z}
| {z }
i=1
i=1
coluna i de C

coluna k de BC

Logo, tem-se

BC = CP .

Seja v um vector prprio de P associado a um valor prprio . Tem-se v 6= 0 e Cv 6= 0


pois C tem caracterstica mxima (= no de colunas). Alm disso,
(Cv) ,

B (Cv) = (BC) v = (CP ) v = C (P v) = C ( vI) =


isto , Cv um vector prprio de B associado ao valor prprio .
Por outro lado, tem-se
(Cv) ,

A (Cv) = (AC) v = ( IC) v =

isto , Cv um vector prprio de A associado ao valor prprio .


Logo, Cv um vector prprio comum a A e B.
14. Seja A uma matriz n

n e sejam
(A

1;

escalares, com

1 I) (A

2 I)

= 0:

6=

2,

tais que

Atendendo a que
det (A

1 I) det (A

2 I)

= 0 , (det (A

1 I)

= 0 ou

det (A

2 I)

= 0)

ento 1 valor prprio de A ou 2 valor prprio de A. Suponhamos sem perda de


generalidade (uma vez que (A
1 I) (A
2 I) = (A
2 I) (A
1 I)) que 1 um valor
prprio de A. Atendendo a que
C (A

ento
n

nul (A

2 I)

= car (A

2 I)

2 I)

isto ,
n
Logo, atendendo a que nul (A

= dim C (A

nul (A
1 I)

nul (A

N (A

1 I)

2 I)

6= f0g
dim N (A
2 I) .

+ nul (A

+ nul (A
1 I)

1 I)

2 I)

+ nul (A

ou seja, A diagonalizvel.
382

n, tem-se
2 I)

=n

1 I)

= nul (A

1 I)

Resoluo da 6a Ficha de exerccios para as aulas de problemas


1. (i) Consideremos a aplicao h; i : R2

R2 ! R, denida por

h(x1 ; x2 ); (y1 ; y2 )i = x21 y12 + x22 y22 ,


com (x1 ; x2 ); (y1 ; y2 ) 2 R2 .
Por exemplo
h(1; 1); (1; 0) + (1; 0)i = h(1; 1); (2; 0)i = 4 6= 2 = h(1; 1); (1; 0)i + h(1; 1); (1; 0)i .
Logo, esta aplicao h; i no um produto interno, uma vez que a condio de linearidade
no vericada.
(ii) Consideremos a aplicao h; i : R2

R2 ! R, denida por

h(x1 ; x2 ); (y1 ; y2 )i = x1 y1

x1 y2 + 3x2 y2 ,

x2 y1

com (x1 ; x2 ); (y1 ; y2 ) 2 R2 .


Tem-se
h(x1 ; x2 ); (y1 ; y2 )i =

1
1

x1 x2

1
3

y1
y2

p
p
1
1
simtrica e os seus valores prprios ( 2 + 2 e 2
2) so todos
1 3
positivos, logo, a aplicao h; i dene um produto interno em R2 .
0
0
Resoluo alternativa: Para todos os (x1 ; x2 ); (x1 ; x2 ); (y1 ; y2 ) 2 R2 e 2 R tem-se:
e como

h(x1 ; x2 ); (y1 ; y2 )i =
=
=
=

x1 y1 x2 y1 x1 y2 + 3x2 y2 =
y1 x1 y1 x2 y2 x1 + 3y2 x2 =
y1 x1 y2 x1 y1 x2 + 3y2 x2 =
h(y1 ; y2 ); (x1 ; x2 )i .
D
E D
E
0
0
0
0
(x1 ; x2 ) + (x1 ; x2 ); (y1 ; y2 ) = (x1 + x1 ; x2 + x2 ); (y1 ; y2 ) =
0

= (x1 + x1 )y1
0
= x1 y1 + x1 y1
= x1 y1 x 2 y1

(x2 + x2 )y1 (x1 + x1 )y2 + 3(x2 + x2 )y2 =


0
0
0
x2 y1 x2 y1 x1 y2 x1 y2 + 3x2 y2 + 3x2 y2 =
0
0
0
0
x1 y2 + 3x2 y2 + x1 y1 x2 y1 x1 y2 + 3x2 y2 =
D 0 0
E
= h(x1 ; x2 ); (y1 ; y2 )i + (x1 ; x2 ); (y1 ; y2 ) .
h (x1 ; x2 ); (y1 ; y2 )i = h x1 ; x2 ); (y1 ; y2 )i =
= x 1 y1
x 2 y1
x 1 y2 + 3 x 2 y2 =
= (x1 y1 x2 y1 x1 y2 + 3x2 y2 ) =
=
h(x1 ; x2 ); (y1 ; y2 )i .
383

h(x1 ; x2 ); (x1 ; x2 )i = x21

p
x2 )2 + ( 2x2 )2

2x1 x2 + 3x22 = (x1

e
h(x1 ; x2 ); (x1 ; x2 )i = 0 , (x1 x2 = 0 e
, (x1 = x2 e x2 = 0)
, (x1 = 0 e x2 = 0):

2x2 = 0) ,

Logo:
h(x1 ; x2 ); (x1 ; x2 )i > 0,
para todo o (x1 ; x2 ) 6= (0; 0).
Assim, a aplicao h; i : R2

R2 ! R, denida por

h(x1 ; x2 ); (y1 ; y2 )i = x1 y1

x2 y1

x1 y2 + 3x2 y2

um produto interno.
(iii) Consideremos a aplicao h; i : R2

R2 ! R, denida por

h(x1 ; x2 ); (y1 ; y2 )i =

2x1 y1 + 3x2 y2 ,

com (x1 ; x2 ); (y1 ; y2 ) 2 R2 .


Tem-se
h(x1 ; x2 ); (y1 ; y2 )i =

2 0
0 3

x1 x2

y1
y2

2 0
no so todos positivos ( 2 e 3), logo, a aplicao
0 3
h; i no dene um produto interno em R2 , uma vez que a condio de positividade no
satisfeita.
Como os valores prprios de

Resoluo alternativa: Vejamos que a condio de positividade no satisfeita.


r
3
jx2 j .
h(x1 ; x2 ); (x1 ; x2 )i = 0 , 2x21 + 3x22 = 0 , x1 =
2
Logo, por exemplo tem-se:
* r
! r
!+
3
3
=0 e
;1 ;
;1
2
2

!
3
; 1 6= (0; 0).
2

Assim, a condio:
h(x1 ; x2 ); (x1 ; x2 )i > 0, 8(x1 ; x2 ) 6= (0; 0)

no satisfeita. Logo, a aplicao h; i : R2

R2 ! R, denida por

h(x1 ; x2 ); (y1 ; y2 )i =
no um produto interno.
384

2x1 y1 + 3x2 y2

2. (i) Consideremos a aplicao h; i : R3

R3 ! R, denida por

h(x1 ; x2 ; x3 ); (y1 ; y2 ; y3 )i = x1 y1 + x2 y2 + x3 y3 ,
com (x1 ; x2 ; x3 ); (y1 ; y2 ; y3 ) 2 R3 .
Tem-se
h(x1 ; x2 ; x3 ); (y1 ; y2 ; y3 )i =
2

1
4
e como 0
0
aplicao h; i

x1 x2 x3

32
3
1 0 0
y1
4 0 1 0 5 4 y2 5 .
0 0 1
y3

3
0 0
1 0 5 simtrica e os seus valores prprios (1) so todos positivos, logo, a
0 1
dene um produto interno em R3 .
0

Resoluo alternativa: Para todos os (x1 ; x2 ; x3 ); (x1 ; x2 ; x3 )(y1 ; y2 ; y3 ) 2 R3 e


tem-se:
h(x1 ; x2 ; x3 ); (y1 ; y2 ; y3 )i = x1 y1 + x2 y2 + x3 y3 =
= y1 x1 + y2 x2 + y3 x3 =
= h(y1 ; y2 ; y3 ); (x1 ; x2 ; x3 )i .
E
D
0
0
0
(x1 ; x2 ; x3 ) + (x1 ; x2 ; x3 ); (y1 ; y2 ; y3 ) =
=

E
0
0
0
(x1 + x1 ; x2 + x2 ; x3 + x3 ); (y1 ; y2 ; y3 ) =
0

= (x1 + x1 )y1 + (x2 + x2 )y2 + (x3 + x3 )y3 =


0
0
0
= x1 y1 + x1 y1 + x2 y2 + x2 y2 + x3 y3 + x3 y3 =
0
0
0
= x1 y1 + x 2 y2 + x3 y3 + x1 y1 + x2 y2 + x3 y3 =
E
D 0 0 0
= h(x1 ; x2 ; x3 ); (y1 ; y2 ; y3 )i + (x1 ; x2 ; x3 ); (y1 ; y2 ; y3 ) .
h (x1 ; x2 ; x3 ); (y1 ; y2 ; y3 )i = h x1 ; x2 ; x3 ); (y1 ; y2 ; y3 )i =
= x 1 y1 + x 2 y2 + x 3 y3 =
= (x1 y1 + x2 y2 + x3 y3 ) =
=
h(x1 ; x2 ; x3 ); (y1 ; y2 ; y3 )i .
h(x1 ; x2 ; x3 ); (x1 ; x2 ; x3 )i = x21 + x22 + x23

e
h(x1 ; x2 ; x3 ); (x1 ; x2 ; x3 )i = 0 , (x1 = 0 e x2 = 0 e x3 = 0):
Logo:
h(x1 ; x2 ; x3 ); (x1 ; x2 ; x3 )i > 0, 8(x1 ; x2 ; x3 ) 6= (0; 0; 0).
385

2R

Assim, a aplicao h; i : R3

R3 ! R, denida por

h(x1 ; x2 ; x3 ); (y1 ; y2 ; y3 )i = x1 y1 + x2 y2 + x3 y3
um produto interno, o chamado produto interno usual de R3 .
(ii) Consideremos a aplicao h; i : R3

R3 ! R, denida por

h(x1 ; x2 ; x3 ); (y1 ; y2 ; y3 )i = x1 y2

x2 y1 .

Tem-se
x1 x2 x3

h(x1 ; x2 ; x3 ); (y1 ; y2 ; y3 )i =
2

32
3
0 1 0
y1
4 1 0 0 5 4 y2 5 .
0 0 0
y3

3
0 1 0
e como 4 1 0 0 5 no simtrica, logo, a aplicao h; i no dene um produto interno
0 0 0
3
em R .
Resoluo alternativa: Por exemplo
h(1; 1; 1); (1; 0; 0)i =

1 6= 1 = h(1; 0; 0); (1; 1; 1)i .

Logo, esta aplicao h; i no um produto interno, uma vez que a condio de simetria no
vericada.
(iii) Consideremos a aplicao h; i : R3

R3 ! R, denida por

h(x1 ; x2 ; x3 ); (y1 ; y2 ; y3 )i = 2x1 y1 + x1 y3 + x3 y1 + 2x2 y2 + x3 y3 ,


com (x1 ; x2 ; x3 ); (y1 ; y2 ; y3 ) 2 R3 .
Tem-se
x1 x2 x3

h(x1 ; x2 ; x3 ); (y1 ; y2 ; y3 )i =
2

32
3
2 0 1
y1
4 0 2 0 5 4 y2 5 .
1 0 1
y3

3
2 0 1
e como 4 0 2 0 5 simtrica e os seus valores prprios
1 0 1
2
3
2
0
1
2
2
0 5 = (2
det 4 0
) det
1
1
0
1
= (2

) [(2

= (2

) (1

3
2

1] = (2
p !
5
2

386

1
1

3 +1 =
p !
3
5
+
2
2

( 32 +
R3 .

5 3
;
2 2

5
;2
2

) so todos positivos, logo, a aplicao h; i dene um produto interno em


0

Resoluo alternativa: Para todos os (x1 ; x2 ; x3 ); (x1 ; x2 ; x3 )(y1 ; y2 ; y3 ) 2 R3 e


tem-se:

2R

h(x1 ; x2 ; x3 ); (y1 ; y2 ; y3 )i = 2x1 y1 + x1 y3 + x3 y1 + 2x2 y2 + x3 y3 =


= 2y1 x1 + y3 x1 + y1 x3 + 2y2 x2 + y3 x3 =
= h(y1 ; y2 ; y3 ); (x1 ; x2 ; x3 )i .
D
E
E D
0
0
0
0
0
0
(x1 ; x2 ; x3 ) + (x1 ; x2 ; x3 ); (y1 ; y2 ; y3 ) = (x1 + x1 ; x2 + x2 ; x3 + x3 ); (y1 ; y2 ; y3 ) =
0

= 2(x1 + x1 )y1 + (x1 + x1 )y3 + (x3 + x3 )y1 + 2(x2 + x2 )y2 + (x3 + x3 )y3 =
0
0
0
0
0
= 2x1 y1 + 2x1 y1 + x1 y3 + x1 y3 + x3 y1 + x3 y1 + 2x2 y2 + 2x2 y2 + x3 y3 + x3 y3 =
0
0
0
0
0
= 2x1 y1 + x1 y3 + x3 y1 + 2x2 y2 + x3 y3 + 2x1 y1 + x1 y3 + x3 y1 + 2x2 y2 + x3 y3 =
E
D 0 0 0
= h(x1 ; x2 ; x3 ); (y1 ; y2 ; y3 )i + (x1 ; x2 ; x3 ); (y1 ; y2 ; y3 ) .
h (x1 ; x2 ; x3 ); (y1 ; y2 ; y3 )i = h x1 ; x2 ; x3 ); (y1 ; y2 ; y3 )i =
= 2 x 1 y1 + x 1 y3 + x 3 y1 + 2 x 2 y2 + x 3 y3 =
= (2x1 y1 + x1 y3 + x3 y1 + 2x2 y2 + x3 y3 ) =
=
h(x1 ; x2 ; x3 ); (y1 ; y2 ; y3 )i .
h(x1 ; x2 ; x3 ); (x1 ; x2 ; x3 )i = 2x21 + 2x1 x3 + 2x22 + x23 =
2
p
2x2
= x21 + (x1 + x3 )2 +

e
h(x1 ; x2 ; x3 ); (x1 ; x2 ; x3 )i = 0 , (x1 = 0 e x1 + x3 = 0 e
, (x1 = 0 e x2 = 0 e x3 = 0):

2x2 = 0) ,

Logo:
h(x1 ; x2 ; x3 ); (x1 ; x2 ; x3 )i > 0, 8(x1 ; x2 ; x3 ) 6= (0; 0; 0).

Assim, a aplicao h; i : R3

R3 ! R, denida por

h(x1 ; x2 ; x3 ); (y1 ; y2 ; y3 )i = 2x1 y1 + x1 y3 + x3 y1 + 2x2 y2 + x3 y3


um produto interno.

por

3. Sejam (x1 ; x2 ); (y1 ; y2 ) 2 R2 . Consideremos a aplicao h; i : R2


h(x1 ; x2 ); (y1 ; y2 )i =

x1 x2

3 2
2 3

y1
y2
387

R2 ! R, denida

= 3x1 y1 + 2x2 y1 + 2x1 y2 + 3x2 y2 .

3 2
simtrica e tem os seus valores prprios (1 e 5) todos
2 3
positivos, ento esta aplicao dene em R2 um produto interno. Alm disso, verica-se
h(1; 0); (0; 1)i = 2, uma vez que

Atendendo a que a matriz

3 2
2 3

h(1; 0); (1; 0)i h(1; 0); (0; 1)i


h(0; 1); (1; 0)i h(0; 1); (0; 1)i

4. Considere os vectores u =

p1 ;
5

p1
5

ev=

p2 ; p3
30
30

. Considere o produto interno

denido em R2 por
h(x1 ; x2 ); (y1 ; y2 )i = 3x1 y1 + 2x2 y2 .
Tem-se
1
p ;
5

hu; vi =

1
p
5

3
2
p ;p
30 30

1 2
= 3p p + 2
5 30

1
p
5

3
p =0
30

e
hu; ui = 3

1
p
5

1
p
5

+2

=1

hv; vi = 3

2
p
30

+2

3
p
30

= 1.

Logo, o conjunto fu; vg ortonormado relativamente ao produto interno anterior.


No entanto, relativamente ao produto interno usual h; i0 denido em R2 :
h(x1 ; x2 ); (y1 ; y2 )i0 = x1 y1 + x2 y2 ,
tem-se

2
13
1
, hu; ui0 =
e hv; vi0 = .
5
30
150
Logo, o conjunto fu; vg no ortonormado relativamente ao produto interno usual denido
em R2 .
hu; vi0 =

5. Considere em R4 o produto interno usual.


Seja U = L (f(1; 0; 0; 0); (1; 0; 0; 1)g). Logo, o subespao de R4 ortogonal a U dado por:
U? =

(x; y; z; w) 2 R4 : h(x; y; z; w); (1; 0; 0; 0)i = 0


h(x; y; z; w); (1; 0; 0; 1)i = 0
=N

1 0 0 0
1 0 0 1

=N

1 0 0 0
0 0 0 1

= (x; y; z; w) 2 R4 : x = 0 e w = 0 = (0; y; z; 0) 2 R4 : y; z 2 R =
= L (f(0; 1; 0; 0); (0; 0; 1; 0)g) .
Como o conjunto f(0; 1; 0; 0); (0; 0; 1; 0)g independente e gera U ? ento uma base de
U ? e tem-se
R4 = U U ? =
388

= L (f(1; 0; 0; 0); (1; 0; 0; 1)g)

L (f(0; 1; 0; 0); (0; 0; 1; 0)g) .

6. Considere em R3 o produto interno denido por:


h(x1 ; x2 ; x3 ); (y1 ; y2 ; y3 )i = x1 y1 + x1 y2 + x2 y1 + 2x2 y2 + x3 y3 ,
isto , por
h(x1 ; x2 ; x3 ); (y1 ; y2 ; y3 )i =

x1 x2 x3

(i) Seja u = (x1 ; x2 ; x3 ) 2 R3 . Tem-se

32
3
1 1 0
y1
4 1 2 0 5 4 y2 5 .
0 0 1
y3

q
p
kuk = h(x1 ; x2 ; x3 ); (x1 ; x2 ; x3 )i = x21 + 2x1 x2 + 2x22 + x23 .

(ii) Considere os vectores u1 = (1; 0; 0), u2 = ( 1; 1; 0) e u3 = (0; 0; 1). Tem-se


arccos

hu1 ; u2 i
0
= arccos
= ,
ku1 k ku2 k
1:1
2

arccos

hu2 ; u3 i
0
= arccos
=
ku2 k ku3 k
1:1
2

arccos

hu1 ; u3 i
0
= arccos
=
ku1 k ku3 k
1:1
2

(iii) Atendendo a que

hu1 ; u2 i = hu2 ; u3 i = hu1 ; u3 i = 0 e

ku1 k = ku2 k = ku3 k = 1

ento o conjunto fu1 ; u2 ; u3 g uma base ortonormada de R3 .


Seja u = (x1 ; x2 ; x3 ) 2 R3 . Tem-se
u = hu; u1 i u1 + hu; u2 i u2 + hu; u3 i u3 =
= (x1 + x2 ) u1 + x2 u2 + x3 u3 .
Logo, as coordenadas de um vector u = (x1 ; x2 ; x3 ) 2 R3 em relao base ortonormada
fu1 ; u2 ; u3 g so dadas por:
x1 + x2 , x2 e x3 .

7. Considere R4 com o produto interno usual. Seja


U = L (f(1; 0; 1; 0); ( 1; 2; 0; 1); (2; 0; 2; 1)g) .

389

Determinemos
2
1
6 0
6
4 1
0

a dimenso de U e uma base ortonormada


3
2
3
2
1 2
1
1 2
1
1
7
6
7
6
2 0 7
0 2 0 7
0 2
!6
!6
5
4
5
4
0 0
0 2
0
1 4
1 1
0 1 1
0 0

para U . Tem-se
3
2
2
1
1
7
6
0 7
0 2
!6
5
4
4
0 0
1
0 0

3
2
0 7
7.
4 5
0

Logo, o conjunto fv1 ; v2 ; v3 g, com v1 = (1; 0; 1; 0); v2 = ( 1; 2; 0; 1) e v3 = (2; 0; 2; 1), uma


base de U e como tal dim U = 3.
Sejam
u1 = v 1 , u2 = v 2

proju1 v2

e u3 = v 3

proju1 v3

proju2 v3 .

Logo, o conjunto fu1 ; u2 ; u3 g, com u1 = (1; 0; 1; 0),


u2 = ( 1; 2; 0; 1)

1
; 2;
2

1
(1; 0; 1; 0) =
2

1
;1
2

e
0
1
1
1
(1; 0; 1; 0)
; 2;
;1
2
11=2
2
2
21 4 21 13
1
( 1; 4; 1; 2) =
; ; ;
= (2; 0; 2; 1) +
11
11 11 11 11

u3 = (2; 0; 2; 1)

uma base ortogonal de U . Uma base ortonormada para U :

( p

2
; 0;
2

!
2
;0 ;
2

u1
u2
u3
;
;
=
ku1 k ku2 k ku3 k
)
p
p
p
p !
22 2 22
22 22
21
4
21
13
;
;
;
; p
;p
;p
;p
:
22
11
22 11
1067 1067 1067 1067

8. (i) O conjunto f(0; 1; 1); (0; 0; 1)g gera U e linearmente independente logo uma
base de U . Atendendo ao mtodo de ortogonalizao de Gram-Schmidt, uma base ortogonal
para U : fu1 ; u2 g em que u1 = (0; 1; 1) e
u2 = (0; 0; 1)

Proj (0; 0; 1) = (0; 0; 1)


(0;1;1)

= (0; 0; 1)
Assim uma base ortogonal para U :
Tem-se
V = (x; y; z) 2 R3 : y

1
(0; 1; 1) =
2
(0; 1; 1); 0;

h(0; 0; 1); (0; 1; 1)i


(0; 1; 1) =
k(0; 1; 1)k2
0;
1 1
;
2 2

1 1
;
2 2

z = 0 = f(x; y; y) : x; y 2 Rg = L (f(1; 0; 0); (0; 1; 1)g) :

Atendendo a que h(1; 0; 0); (0; 1; 1)i = 0, uma base ortonormada para V :
390

(1; 0; 0)
(0; 1; 1)
;
k(1; 0; 0)k k(0; 1; 1)k

p !)
2 2
(1; 0; 0); 0;
;
:
2 2
p

(ii) Como
U? =

(0; 1; 1); 0;

1 1
;
2 2

= L (f(1; 0; 0)g) ;

uma base ortonormada para R3 que inclui dois vectores geradores de U :


(
p p !
p p !)
2 2
2 2
(1; 0; 0); 0;
;
; 0;
;
:
2 2
2 2
Como
V ? = (x; y; z) 2 R3 : y

z=0

= (x; y; z) 2 R3 : h(x; y; z); (0; 1; 1)i = 0

= (L (f(0; 1; 1)g))?

= L (f(0; 1; 1)g) ;

e atendendo alnea anterior, uma base ortonormada para R3 que inclui dois vectores geradores de V :
(
p p !
p
p !)
2 2
2
2
(1; 0; 0); 0;
;
; 0;
;
:
2 2
2
2

(iii) O elemento de U mais prximo de (1; 1; 1) :


PU (1; 1; 1) = (1; 1; 1)
= (1; 1; 1)

PU ? (1; 1; 1) =

h(1; 1; 1); (1; 0; 0)i (1; 0; 0) = (0; 1; 1):

A distncia entre (1; 1; 1) e V ? :

d (1; 1; 1); V ? = kPV (1; 1; 1)k

1
4
9. Seja A = 0
2
L (A) respectivamente

(1;1;1)2V

k(1; 1; 1)k =

3
0 2
0 0 5 e considere o produto interno usual. Sejam N (A), C (A) e
0 1
o ncleo, espao das colunas e espao das linhas de A.

(i) O conjunto f(1; 0; 2); (2; 0; 1)g uma base para C (A) pois gera C (A) e linearmente
independente.
391

O conjunto f(1; 0; 2); (2; 0; 1); (0; 1; 0)g uma base para R3 . Como (2; 0; 1) e (0; 1; 0) so
ortogonais, basta aplicar Gram-Schmidt a (1; 0; 2):
(1; 0; 2)
= (1; 0; 2)

P(2;0;1) (1; 0; 2)

h(1; 0; 2); (2; 0; 1)i


(2; 0; 1)
k(2; 0; 1)k2

4
(2; 0; 1) =
5

= (1; 0; 2)
Logo, o conjunto
(
(0; 1; 0)
(2; 0; 1)
;
;
k(2; 0; 1)k k(0; 1; 0)k

3
; 0;
5
3
; 0;
5

6
5
6
5

P(0;1;0) (1; 0; 2) =
h(1; 0; 2); (0; 1; 0)i
(0; 1; 0) =
k(0; 1; 0)k2
3
6
; 0;
5
5

p
p !
2 5
5
; 0;
; (0; 1; 0);
5
5

uma base ortonormada para R3 que inclui dois vectores de C (A):


p

5
; 0; 2 5 5
5

p !)
2 5
5
; 0;
5
5

p
p
2 5
5
;
0;
5
5

(ii) O elemento de L (A) mais prximo de (1; 1; 1) :


PL(A) (1; 1; 1) = (1; 1; 1)
= (1; 1; 1)
= (1; 1; 1)

PN (A) (1; 1; 1)

N (A)=L(f(0;1;0)g)

h(1; 1; 1); (0; 1; 0)i


(0; 1; 0) =
k(0; 1; 0)k2
(0; 1; 0) = (1; 0; 1).

A distncia entre (1; 1; 1) e N (A) :


d ((1; 1; 1); N (A)) = P(N (A))? (1; 1; 1) = PL(A) (1; 1; 1) = k(1; 0; 1)k =

1
4
10. Seja A = 0
1
L (A) respectivamente

2:

3
0 1
2 0 5 e considere o produto interno usual. Sejam N (A), C (A) e
0 1
o ncleo, espao das colunas e espao das linhas de A.

(i) Tem-se (N (A))? = L (A). O conjunto f(1; 0; 1); (0; 2; 0)g uma base para N (A) pois
gera N (A) e linearmente independente. Como h(1; 0; 1); (0; 2; 0)i = 0, os vectores (1; 0; 1)
e (0; 2; 0) so ortogonais. Logo, o conjunto
( p
)
p !
(1; 0; 1)
(0; 2; 0)
2
2
;
=
; 0;
; (0; 1; 0)
k(1; 0; 1)k k(0; 2; 0)k
2
2
uma base ortonormada para (N (A))? .
392

(ii) O conjunto f(1; 0; 1); (0; 2; 0)g uma base para C (A) pois gera C (A) e linearmente
independente.
O conjunto f(1; 0; 1); (0; 2; 0); (0; 0; 1)g uma base para R3 . Como (1; 0; 1) e (0; 2; 0) so
ortogonais, basta aplicar Gram-Schmidt a (0; 0; 1):
(0; 0; 1)
= (0; 0; 1)

P(1;0;1) (0; 0; 1)

P(0;2;0) (0; 0; 1) =

h(0; 0; 1); (1; 0; 1)i


(1; 0; 1)
k(1; 0; 1)k2
= (0; 0; 1)

Logo, o conjunto
(
(1; 0; 1)
(0; 2; 0)
;
;
k(1; 0; 1)k k(0; 2; 0)k

1
; 0;
2
1
; 0;
2

1
2
1
2

1
(1; 0; 1) =
2
)

h(0; 0; 1); (0; 2; 0)i


(0; 2; 0) =
k(0; 2; 0)k2
1
1
; 0;
2
2

( p

p !
2
2
; 0;
; (0; 1; 0);
2
2

p !)
2
2
; 0;
2
2

p
p

p
2
2
;
0;
2
2

uma base ortonormada para R3 que inclui dois vectores de C (A):

e (0; 1; 0).

(iii) O elemento de L (A) mais prximo de (1; 2; 3) :


PL(A) (1; 2; 3) = (1; 2; 3)
= (1; 2; 3)
= (1; 2; 3)

PN (A) (1; 2; 3)

N (A)=L(f( 1;0;1)g)

h(1; 2; 3); ( 1; 0; 1)i


( 1; 0; 1) =
k( 1; 0; 1)k2
( 1; 0; 1) = (2; 2; 2).

A distncia entre (1; 2; 3) e L (A)? :

p
d (1; 2; 3); (L (A))? = PL(A) (1; 2; 3) = k(2; 2; 2)k = 2 3:

11. Seja U = L (f(1; 1; 1; 0); (0; 1; 1; 1)g). Seja (x; y; z; w) 2 U . Ento existem x; y 2 R
tais que
(x; y; z; w) = x(1; 1; 1; 0) + y(0; 1; 1; 1):
Deste modo, o seguinte sistema (nas variveis x e y) tem que ser possvel e determinado:
8
>
> x=x
<
x+y =y
x+y =z
>
>
:
y=w
Considerando ento a matriz aumentada deste sistema, tem-se:
2
3
22
33
22
1 0 j x
1 0 j
x
6 1 1 j y 7
66 0 1 j y x 77
66
7
77
6
66
66
!
4 1 1 j z 5 L1 +L!
4
4
5
5
0 1 j z x
L2 +L3 !L3 44
2 !L2
L1 +L3 !L3
L2 +L4 !L4
0 1 j w
0 1 j
w
393

1
0
0
0

0
1
0
0

33
j
x
7
j
y x 7
77 .
5
5
j
z y
j x y+w

Logo, para que o sistema anterior seja possvel e determinado, preciso que se tenha z y = 0
e x y + w = 0.
Assim, U = f(x; y; z; w) 2 R4 : x y + w = 0 e z y = 0g, isto ,
U = N (A), com A =

1
0

1 0 1
1 1 0

12. Seja B = f(1; 0); (1; 1)g uma base de R2 . Vamos denir um produto interno em R2
em relao ao qual a base B ortonormada.
Seja Bc2 = f(1; 0); (0; 1)g a base cannica de R2 . A matriz de mudana de base de Bc2
para B dada por
SBc2 !B = SB!Bc2
Sejam u; v 2 R2 . Tem-se

1
0

u = (x1 ; x2 )

1
1

1
0

1
1

e v = (y1 ; y2 ) ,

onde x1 ; x2 e y1 ; y2 so as coordenadas na base Bc2 de u e v respectivamente. Seja S = SBc2 !B .


Logo, tem-se a aplicao h; i : R2 R2 denida por
hu; vi = (Su)T G (Sv) , com G =

hv1 ; v1 i hv1 ; v2 i
hv2 ; v1 i hv2 ; v2 i

1 0
0 1

ou seja,
T

1 1
x1
1 0
h(x1 ; x2 ) ; (y1 ; y2 )i =
0
1
x2
0 1
= x1 y1 + x1 y2 + x2 y1 + 2x2 y2 .

1
0

1
1

y1
y2

Como
h(x1 ; x2 ) ; (y1 ; y2 )i = x1 y1 + x1 y2 + x2 y1 + 2x2 y2 =

x1 x2

1 1
1 2

p
1 1
simtrica, sendo os seus valores prprios ( 21 5+ 32 e
1 2
ento a expresso
h(x1 ; x2 ) ; (y1 ; y2 )i = x1 y1 + x1 y2 + x2 y1 + 2x2 y2
e a matriz

3
2

y1
y2
1
2

5) positivos,

dene um produto interno em R2 . Alm disso, fcil vericar que para este produto interno
a base B = f(1; 0); (1; 1)g ortonormada:
h(1; 0) ; (1; 1)i = 0 e h(1; 0) ; (1; 0)i = h(1; 1) ; (1; 1)i = 1.

13. Considere a aplicao h; i : R3

R3 ! R denida por

h(x1 ; x2 ; x3 ); (y1 ; y2 ; y3 )i = x1 y1
394

x1 y2

x2 y1 + 4x2 y2 + x3 y3 .

(i) Tem-se
2

x1 x2 x3

h(x1 ; x2 ; x3 ); (y1 ; y2 ; y3 )i =

32
3
1 0
y1
4 0 5 4 y2 5 .
0 1
y3

1
4 1
0

3
1
1 0
p
Como 4 1 4 0 5 simtrica e os seus valores prprios ( 5+2 13 e
0
0 1
positivos, logo, a aplicao h; i dene um produto interno em R3 .

p
2

13

) so todos

R3 . Uma base ortonormada para V :

(ii) Seja V = L (f(3; 4; 0)g)

(3; 4; 0)
k(3; 4; 0)k

(3; 4; 0)
7

3 4
; ;0
7 7

O ponto de V mais prximo de (0; 1; 0)


PV (0; 1; 0) =

(0; 1; 0);

3 4
; ;0
7 7

3 4
; ;0
7 7

13
7

3 4
; ;0
7 7

39 52
; ;0 .
49 49

Nota. Em alternativa, como dim V = 1,


PV (0; 1; 0) = proj(3;4;0) (0; 1; 0) =

h(0; 1; 0); (3; 4; 0)i


13
(3; 4; 0) =
(3; 4; 0) =
2
49
k(3; 4; 0)k

39 52
; ;0 .
49 49

(iii) Tem-se
V ? = (x; y; z) 2 R3 : h(x; y; z); (3; 4; 0)i = 0 =
= (x; y; z) 2 R3 : 3x 4x 3y + 16y = 0 =
= (x; y; z) 2 R3 : x + 13y = 0 =
= (13y; y; z) 2 R3 : y; z 2 R =
= L (f(13; 1; 0); (0; 0; 1)g) .
Como o conjunto fv1 ; v2 g, com v1 = (13; 1; 0) e v2 = (0; 0; 1), independente e gera V ? ento
uma base de V ? . Sejam
u1 = v 1

e u2 = v 2

proju1 v2 .

Logo, o conjunto fu1 ; u2 g, com u1 = (13; 1; 0) e u2 = (0; 0; 1)


base ortogonal de V ? .

0(13; 1; 0) = (0; 0; 1), uma

(iv) Seja
B=

3 4
; ;0 ;
7 7

13
1
p
;p
; 0 ; (0; 0; 1) :
147 147
395

Como

13
1
p
;p
; 0 ; (0; 0; 1)
147 147

uma base ortonormada para V ? , ento B uma base ortonormada de R3 . Atendendo a


que
PV

3 4
; ;0
7 7

3 4
; ;0 ;
7 7

3 4
; ;0
7 7

3 4
; ;0
7 7

3 4
3 4
3 4
; ;0
; ;0 =
; ;0 =
7 7
7 7
7 7
1
3 4
13
= 1
; ;0 + 0 p
;p
; 0 + 0(0; 0; 1),
7 7
147 147
=

PV

1
13
p
;p
;0
147 147

=
= 0
= 0

13
1
3 4
3 4
p
;p
;0 ;
; ;0
; ;0 =
7 7
7 7
147 147
3 4
; ; 0 = (0; 0; 0) =
7 7
3 4
13
1
; ;0 + 0 p
;p
; 0 + 0(0; 0; 1)
7 7
147 147

e
PV (0; 0; 1) =

(0; 0; 1);

= 0
= 0

3 4
; ;0
7 7

3 4
; ;0
7 7

3 4
; ; 0 = (0; 0; 0) =
7 7
13
3 4
1
; ;0 + 0 p
;p
; 0 + 0(0; 0; 1),
7 7
147 147

a matriz que representa PV em relao base


2
1 0
4 0 0
0 0

B dada por:
3
0
0 5.
0

14. Consideremos em R3 o produto interno usual. Seja U = L


Tem-se
U? = N

0 1
4=5 0

0
3=5

=N

0 1
4 0

0
3

(0; 1; 0);

= L (f(3; 0; 4)g) .

Logo,
PU ? (1; 2; 3) =

h(1; 2; 3); (3; 0; 4)i


(3; 0; 4) =
k(3; 0; 4)k2
396

4
; 0;
5

9
12
; 0;
5
5

3
5

e assim
PU (1; 2; 3) = (1; 2; 3)

9
12
; 0;
5
5

PU ? (1; 2; 3) = (1; 2; 3)

4
3
; 2;
5
5

Deste modo,
(1; 2; 3) =
com

4
; 2; 35
5

2U e

9
; 0; 12
5
5

4
3
; 2;
5
5

9
12
; 0;
5
5

2 U ?.

15. Considere R4 com o produto interno usual.


(i) Seja U = L (f(1; 0; 0; 0); (1; 1; 0; 1)g). Logo,
U ? = (x; y; z; w) 2 R4 : h(x; y; z; w); (1; 0; 0; 0)i = 0 e h(x; y; z; w); (1; 1; 0; 1)i = 0 .
Tem-se ento:

8
< x=0
:

Logo,

x+y+w =0

8
< x=0
:

y=

w.

U ? = (0; w; z; w) 2 R4 : z; w 2 R = L (f(0; 1; 0; 1); (0; 0; 1; 0)g) .


Como
h(0; 1; 0; 1); (0; 0; 1; 0)i = 0

ento o conjunto f(0; 1; 0; 1); (0; 0; 1; 0)g uma base ortogonal de U ? .


(ii) Seja U = L (f(1; 0; 1; 1)g). Logo,
U ? = (x; y; z; w) 2 R4 : h(x; y; z; w); (1; 0; 1; 1)i = 0 .
Tem-se ento:
x+z+w =0,x=

w.

Logo,
U? = ( z

w; y; z; w) 2 R4 : y; z; w 2 R = L (f(0; 1; 0; 0); ( 1; 0; 1; 0); ( 1; 0; 0; 1)g) ,

pois
( z

w; y; z; w) = y(0; 1; 0; 0) + z( 1; 0; 1; 0) + w( 1; 0; 0; 1).

Como o conjunto f(0; 1; 0; 0); ( 1; 0; 1; 0); ( 1; 0; 0; 1)g independente (basta colocar esses
trs vectores como linhas ou como colunas de uma matriz e aplicar de seguida o mtodo de
eliminao de Gauss obtendo-se uma matriz em escada de linhas) e gera U ? ento uma
base de U ? .
Como (0; 1; 0; 0) e ( 1; 0; 1; 0) so ortogonais, basta aplicar Gram-Schmidt a ( 1; 0; 0; 1):
( 1; 0; 0; 1)

P(0;1;0;0) ( 1; 0; 0; 1)

397

P(

1;0;1;0) (

1; 0; 0; 1) =

= ( 1; 0; 0; 1)

h( 1; 0; 0; 1); (0; 1; 0; 0)i


(0; 1; 0; 0)
k(0; 1; 0; 0)k2
= ( 1; 0; 0; 1)

h( 1; 0; 0; 1); ( 1; 0; 1; 0)i
( 1; 0; 1; 0) =
k( 1; 0; 1; 0)k2

1
( 1; 0; 1; 0) =
2

1
; 0;
2

1
;1 .
2

Logo, o conjunto
(0; 1; 0; 0); ( 1; 0; 1; 0);

1
; 0;
2

1
;1
2

uma base ortogonal de U ? .


(iii) Seja U = f(x; y; z; w) 2 R4 : x + 2y + z + 2w = 0g. Logo, atendendo a que o produto interno o usual (de R4 ), Tem-se:
U = (x; y; z; w) 2 R4 : h(x; y; z; w); (1; 2; 1; 2)i = 0 = (L (f(1; 2; 1; 2)g))? .
Assim,
U ? = (L (f(1; 2; 1; 2)g))?? = L (f(1; 2; 1; 2)g) .
Logo, o conjunto f(1; 2; 1; 2)g uma base ortogonal de U ? .
(iv) Seja U = f(x; y; z; w) 2 R4 : x z = 0 e 2x
a que o produto interno o usual (de R4 ), Tem-se:
U =

y + 2z

w = 0g. Logo, atendendo

(x; y; z; w) 2 R4 : h(x; y; z; w); (1; 0; 1; 0)i = 0 e h(x; y; z; w); (2; 1; 2; 1)i = 0

= (L (f(1; 0; 1; 0); (2; 1; 2; 1)g))? .


Assim,

U ? = (L (f(1; 0; 1; 0); (2; 1; 2; 1)g))?? = L (f(1; 0; 1; 0); (2; 1; 2; 1)g) .


Como
h(1; 0; 1; 0); (2; 1; 2; 1)i = 0

ento o conjunto f(1; 0; 1; 0); (2; 1; 2; 1)g uma base ortogonal de U ? .


16. Considere R3 com o produto interno usual. Considere tambm o seguinte subespao
de R3 :
U = L (f(1; 1; 1); (1; 0; 0)g) .
(i) Aplicando o mtodo de ortogonalizao de Gram-Schmidt, sejam
v1 = (1; 1; 1) e v2 = (1; 0; 0)

398

proj(1;1;1) (1; 0; 0).

Tem-se ento:
v2 = (1; 0; 0)

proj(1;1;1) (1; 0; 0)
h(1; 0; 0); (1; 1; 1)i
= (1; 0; 0)
(1; 1; 1)
k(1; 1; 1)k2
1
= (1; 0; 0)
(1; 1; 1)
3
2 1 1
=
;
;
.
3 3 3

Logo, o conjunto
(1; 1; 1);

2
;
3

1
;
3

1
3

1
3

uma base ortogonal de U , ento

uma base ortogonal de U .


(ii) Como o conjunto (1; 1; 1);
k(1; 1; 1)k =
ento o conjunto
(
(1; 1; 1)
;
k(1; 1; 1)k

2
;
3
2
;
3

1
;
3
1
;
3

1
3
1
3

2
;
3

1
;
3

2
;
3

3 e

1
;
3

1
3

( p

p p !
3 3 3
;
;
;
3 3 3

6
,
3

6
;
3

6
;
6

p !)
6
6

uma base ortonormada de U .


Por outro lado, tem-se:
U ? = (x; y; z) 2 R3 : h(x; y; z); (1; 1; 1)i = 0 e h(x; y; z); (1; 0; 0)i = 0 =
1 1 1
1 0 0

=N
Logo,

8
< y+z =0
:

Assim,

x=0

0 1 1
1 0 0

=N

8
< y=
:

x = 0.

U ? = (0; z; z) 2 R3 : z 2 R = L (f(0; 1; 1)g) .


Como
k(0; 1; 1)k =
ento o conjunto
0;

1 1
p ;p
2 2

uma base ortonormada de U ? .


399

0;

2,
p !)
2 2
;
2 2

Deste modo, uma vez que se tem


R3 = U

U ?,

ento
(3; 2; 1) = PU (3; 2; 1) + PU ? (3; 2; 1) =
p p p !+ p p p !
3 3 3
3 3 3
= (3; 2; 1);
;
;
;
;
+
3 3 3
3 3 3
*
p
p
p !+ p
p
p !
6
6
6
6
6
6
+ (3; 2; 1);
;
;
;
;
+
3
6
6
3
6
6
*
p p !
p p !+
2 2
2 2
;
0;
;
=
+ (3; 2; 1); 0;
2 2
2 2
*

3 3
1 1
3; ;
+ 0; ;
.
2 2
2 2
| {z }
|
{z
}
2U

Isto ,

(3; 2; 1) =

2U ?

3 3
1 1
3; ;
+ 0; ;
.
2 2
2 2
| {z }
|
{z
}
2U

2U ?

(iii) A distncia entre o ponto (1; 0; 1) e o plano f(1; 1; 0)g + U dada por:
d((1; 0; 1); f(1; 1; 0)g+U ) = kPU ? ((1; 0; 1)

(1; 1; 0))k = kPU ? (0; 1; 1)k

(0; 1;1)2U ?

k(0; 1; 1)k =

(iv) A distncia entre o ponto (x; y; z) e o subespao U dada por:


d((x; y; z); U ) = kPU ? ((x; y; z)
*

(0; 0; 0))k = kPU ? (x; y; z)k


p p !
p p !+
2 2
2 2
0;
=
(x; y; z); 0;
;
;
2 2
2 2
p
2
= j y + zj
.
2

17. Considere R4 com o produto interno usual. Considere tambm o seguinte subespao
de R4 :
U = (x; y; z; w) 2 R4 : x y + z = 0 e y z + w = 0 .
(i) Tem-se ento
U = (y

z; y; z; z

y) 2 R4 : y; z 2 R = L(f(1; 1; 0; 1); ( 1; 0; 1; 1)g).


400

2.

Aplicando o mtodo de ortogonalizao de Gram-Schmidt, sejam


v1 = (1; 1; 0; 1) e v2 = ( 1; 0; 1; 1)

proj(1;1;0;

1) (

1; 0; 1; 1).

Tem-se ento:
v2 = ( 1; 0; 1; 1)

proj(1;1;0; 1) ( 1; 0; 1; 1)
h( 1; 0; 1; 1); (1; 1; 0; 1)i
= ( 1; 0; 1; 1)
(1; 1; 0; 1)
k(1; 1; 0; 1)k2
2
= ( 1; 0; 1; 1) + (1; 1; 0; 1)
3
1 2
1
=
; ; 1;
.
3 3
3

Logo, o conjunto
1 2
1
; ; 1;
3 3
3

(1; 1; 0; 1);
uma base ortogonal de U . Como
k(1; 1; 0; 1)k =

1
1 2
; ; 1;
3 3
3

3 e

15
,
3

ento o conjunto
( p

p
3 3
;
; 0;
3 3

p !
3
;
3

p
p
p !)
15 2 15 3 15 15
;
;
;
15
15
15
15

uma base ortonormada de U .


(ii) Como
U = (x; y; z; w) 2 R4 : x

y+z =0 e y

z+w =0

e atendendo ao produto interno usual de R4 , Tem-se:


U =

(x; y; z; w) 2 R4 : h(x; y; z; w); (1; 1; 1; 0)i = 0 e h(x; y; z; w); (0; 1; 1; 1)i = 0

= (L (f(1; 1; 1; 0) ; (0; 1; 1; 1)g))? .


Logo,

U ? = (L (f(1; 1; 1; 0) ; (0; 1; 1; 1)g))?? = L (f(1; 1; 1; 0) ; (0; 1; 1; 1)g) .


Aplicando o mtodo de ortogonalizao de Gram-Schmidt, sejam
v1 = (1; 1; 1; 0)

e v2 = (0; 1; 1; 1)

401

proj(1;

1;1;0)

(0; 1; 1; 1) .

Tem-se ento:
v2 = (0; 1; 1; 1)

proj(1; 1;1;0) (0; 1; 1; 1)


h(0; 1; 1; 1) ; (1; 1; 1; 0)i
= (0; 1; 1; 1)
(1; 1; 1; 0)
k(1; 1; 1; 0)k2
2
= (0; 1; 1; 1) + (1; 1; 1; 0)
3
2 1 1
=
; ;
;1 .
3 3 3

Logo, o conjunto
(1; 1; 1; 0) ;

2 1
; ;
3 3

1
;1
3

uma base ortogonal de U ? . Como


k(1; 1; 1; 0)k =

2 1
; ;
3 3

3 e

1
;1
3

15
,
3

ento o conjunto
( p

!
p
3 3
;
;0 ;
3 3

3
;
3

p
p
2 15 15
;
;
15
15

p !)
15 15
;
15
5

uma base ortonormada de U ? .


(iii) A projeco ortogonal PU de R4 sobre U denida por:
PU

R 4 ! R4
*

p
p !+ p p
3 3
3
3 3
(x; y; z; w) !
(x; y; z; w);
;
; 0;
;
; 0;
3 3
3
3 3
*
p
p
p !+
p
15 2 15 3 15 15
;
;
;
+ (x; y; z; w);
15
15
15
15
uma vez que o conjunto
( p

p !
3
;
3

3 3
;
; 0;
3 3

p !
3
+
3
p
p
p
p !
15 2 15 3 15 15
;
;
;
,
15
15
15
15

p
p
p !)
15 2 15 3 15 15
;
;
;
15
15
15
15

uma base ortonormada de U . Logo, a projeco ortogonal de (0; 0; 1; 0) sobre U dada


por:
*
p p
p !+ p p
p !
3 3
3
3 3
3
PU (0; 0; 1; 0) =
(0; 0; 1; 0);
;
; 0;
;
; 0;
+
3 3
3
3 3
3
*
p
p
p
p !+
p
p
p
p !
15 2 15 3 15 15
15 2 15 3 15 15
+ (0; 0; 1; 0);
;
;
;
;
;
;
15
15
15
15
15
15
15
15
=

1 2 3 1
; ; ;
5 5 5 5

:
402

A projeco ortogonal PU ? de R4 sobre U ? denida por:


PU ?

R 4 ! R4
*

!+ p
!
p p
p p
3
3 3
3
3 3
(x; y; z; w) !
(x; y; z; w);
;
;
;0
;
;
;0 +
3
3 3
3
3 3
*
p
p
p
p !+
p
p
2 15 15
15 15
2 15 15
+ (x; y; z; w);
;
;
;
;
;
15
15
15
5
15
15
uma vez que o conjunto
( p

3
;
3

3 3
;
;0 ;
3 3

p
p
2 15 15
;
;
15
15

15 15
;
15
5

p !
15 15
;
,
15
5

!)

uma base ortonormada de U ? . Logo, a projeco ortogonal de (0; 0; 1; 0) sobre U ? dada


por:
*
!+ p
!
p
p p
p p
3
3 3
3
3 3
(0; 0; 1; 0);
PU ? (0; 0; 1; 0) =
;
;
;0
;
;
;0 +
3
3 3
3
3 3
*
p
p
p
p !+
p
p
p
p !
2 15 15
2 15 15
15 15
15 15
+ (0; 0; 1; 0);
;
;
;
;
;
;
15
15
15
5
15
15
15
5
1 1 1
2
1 1
;
; ;0 +
;
; ;
3 3 3
15 15 15
Nota muito importante: Uma vez que se tem
=

R4 = U

1
5

1
;
5

2 2
; ;
5 5

1
5

U ?,

ento para todo o (x; y; z; w) 2 R4 ,

(x; y; z; w) = PU (x; y; z; w) + PU ? (x; y; z; w).

Logo, uma vez calculado PU (0; 0; 1; 0) pela denio, como se fz atrs, obtendo-se PU (0; 0; 1; 0) =
1 2 3 1
; ; ; , ento no precisamos de efectuar o clculo de PU ? (0; 0; 1; 0) pela denio.
5 5 5 5
Basta efectuar:
PU ? (0; 0; 1; 0) = (0; 0; 1; 0)

PU (0; 0; 1; 0)
1 2 3 1
; ; ;
= (0; 0; 1; 0)
5 5 5 5
1 2 2 1
=
;
; ;
.
5 5 5 5

se:

(iv) Seja Bc4 = f(1; 0; 0; 0); (0; 1; 0; 0); (0; 0; 1; 0); (0; 0; 0; 1)g a base cannica de R4 . Tem-

PU (1; 0; 0; 0) =

(1; 0; 0; 0);

(1; 0; 0; 0);

1 1
; ; 0;
3 3

1
3

p
p !+ p p
p !
3 3
3
3 3
3
;
; 0;
;
; 0;
+
3 3
3
3 3
3
p
p
p
p !+
p
p
p
p !
15 2 15 3 15 15
15 2 15 3 15 15
;
;
;
;
;
;
15
15
15
15
15
15
15
15

1
;
15

2
;
15
403

3
;
15

1
15

2 1
; ;
5 5

1
;
5

2
5

PU (0; 1; 0; 0) =

p !+ p p
p !
3 3
3
3 3
3
;
; 0;
;
; 0;
+
3 3
3
3 3
3
p
p
p
p !+
p
p
p
p !
15 2 15 3 15 15
15 2 15 3 15 15
;
;
;
;
;
;
15
15
15
15
15
15
15
15

(0; 1; 0; 0);

(0; 1; 0; 0);
1 1
; ; 0;
3 3

PU (0; 0; 1; 0) =

1
3

(0; 0; 1; 0);

(0; 0; 1; 0);
1 2 3 1
; ; ;
5 5 5 5

PU (0; 0; 0; 1) =

.
p
p !+ p p
p !
3 3
3
3 3
3
;
; 0;
;
; 0;
+
3 3
3
3 3
3
p
p
p
p
p !+
p !
p
p
15 2 15 3 15 15
15 2 15 3 15 15
;
;
;
;
;
;
15
15
15
15
15
15
15
15

(0; 0; 0; 1);
1
;
3

2 4 6 2
1 3 2 1
; ; ;
=
; ; ;
.
15 15 15 15
5 5 5 5
p !+ p p
p !
p p
3 3
3
3 3
3
;
; 0;
;
; 0;
+
3 3
3
3 3
3
p
p
p
p !+
p
p
p
p !
15 2 15 3 15 15
15 2 15 3 15 15
;
;
;
;
;
;
15
15
15
15
15
15
15
15
+

(0; 0; 0; 1);

1
1
; 0;
3
3

1 2 3 1
; ; ;
15 15 15 15

2
;
5

1 1 2
; ;
5 5 5

Logo, a representao matricial de PU : R4 ! R4 em relao base cannica de R4 , dada


por:
2
3
2=5
1=5
1=5
2=5
6 1=5
3=5
2=5
1=5 7
7.
M (PU ; Bc4 ; Bc4 ) = 6
4 1=5
2=5
3=5
1=5 5
2=5
1=5
1=5
2=5
Tem-se:
*
!+ p
!
p p
p p
p
3
3 3
3
3 3
;
;
;0
;
;
;0 +
PU ? (1; 0; 0; 0) =
(1; 0; 0; 0);
3
3 3
3
3 3
*
p
p
p
p
p !+
p
p
p !
2 15 15
2 15 15
15 15
15 15
(1; 0; 0; 0);
;
;
;
;
;
;
15
15
15
5
15
15
15
5
1
;
3

PU ? (0; 1; 0; 0) =

1 1
; ;0 +
3 3

(0; 1; 0; 0);

(0; 1; 0; 0);

1 1
; ;
3 3

2 2
3
;
=
;
15 5
5
!+ p
p
p p
3
3 3
3
;
;
;0
;
3
3 3
3
p
p
p
p !+
2 15 15
15 15
;
;
;
15
15
15
5

1
;0 +
3

4 2
; ;
15 15

2 1
; ;
15 15
404

1 1
;
15 5

1 1 2
; ;
5 5 5

!
p
3 3
;
;0 +
3 3
p
p
p
p !
2 15 15
15 15
;
;
;
15
15
15
5

1 2
; ;
5 5

2 1
;
5 5

PU ? (0; 0; 1; 0) =

(0; 0; 1; 0);

(0; 0; 1; 0);
1
;
3

PU ? (0; 0; 0; 1) =

(0; 0; 0; 1);
(0; 0; 0; 1);
2 1
; ;
5 5

1 3
;
5 5

!+

1
;
5

2 2
; ;
5 5

3
3 3
3
3 3
;
;
;0
;
;
;0 +
3
3 3
3
3 3
p
p
p
p !+
p
p
p
p !
2 15 15
15 15
2 15 15
15 15
;
;
;
;
;
;
15
15
15
5
15
15
15
5

1 1
; ;0 +
3 3

2
;
15

1 1
; ;
15 15

1
5

1
5

!+ p
!
p p
p p
3
3 3
3
3 3
;
;
;0
;
;
;0 +
3
3 3
3
3 3
p
p
p
p !+
p
p
p !
p
15 15
2 15 15
15 15
2 15 15
;
;
;
;
;
;
15
15
15
5
15
15
15
5

Logo, a representao matricial de PU ? : R4 ! R4 em relao base cannica de R4 , dada


por:
2
3
3=5
1=5
1=5
2=5
6 1=5
2=5
2=5
1=5 7
7.
M (PU ? ; Bc4 ; Bc4 ) = 6
4 1=5
2=5
2=5
1=5 5
2=5
1=5
1=5
3=5
(v) Escolhendo um ponto de U , por exemplo (0; 0; 0; 0), a distncia entre (0; 0; 1; 0) e U
dada por:
d((0; 0; 1; 0); U ) = kPU ? ((0; 0; 1; 0)
*

(0; 0; 1; 0);

+ (0; 0; 1; 0);
1
;
3

(0; 0; 0; 0))k = kPU ? (0; 0; 1; 0)k =

!+ p
!
p p
p p
3
3 3
3
3 3
;
;
;0
;
;
;0 +
3
3 3
3
3 3
p
p
p
p !+
p
p
2 15 15
15 15
2 15 15
;
;
;
;
;
15
15
15
5
15
15

1 1
; ;0 +
3 3

2
;
15

1 1
; ;
15 15

1
5

1
;
5

2 2
; ;
5 5

1
5

p !
15 15
;
15
5
p
10
=
.
5

(vi) A distncia entre (x; y; z; w) e U dada por:


d((x; y; z; w); U ) = kPU ? ((x; y; z; w)
=

(x; y; z; w);

+ (x; y; z; w);

(0; 0; 0; 0))k = kPU ? (x; y; z; w)k =

!+ p
!
p p
p p
3
3 3
3
3 3
;
;
;0
;
;
;0 +
3
3 3
3
3 3
p
p
p
p !+
p
p
2 15 15
15 15
2 15 15
;
;
;
;
;
15
15
15
5
15
15

405

p !
15 15
;
=
15
5

!
p ! p
p p
3
3
3
3
3 3
x
y
+z
;
;
;0 +
=
3
3
3
3
3 3
p
p
p
p !
p
p
2 15
15
15
15
2 15 15
+ x
+y
z
+w
;
;
15
15
15
5
15
15
p

2
3
1
1 1
w+ x
y + z; w
5
5
5
5 5
q
1
=
(2w + 3x y + z)2 + (w
5

p !
15 15
;
=
15
5

1
2
x+ y
5
5

2 1
z; x
5 5

1
w
5

2
2 3
2
1
y + z; w + x + y
5
5 5
5
5

1
z
5

x + 2y

2z)2 + (x

2y + 2z)2 + (3w + 2x + y

z)2 .

18. Em P2 :
hp(t); q(t)i = p( 1)q( 1) + p(0)q(0) + p(1)q(1).
Considere tambm o seguinte subespao de P2 :
U = fp(t) 2 P2 : p(0) = 0g .
(i) Em P2 , para p(t) = a0 + a1 t + a2 t2

e q(t) = b0 + b1 t + b2 t2 tem-se

hp(t); q(t)i = p( 1)q( 1) + p(0)q(0) + p(1)q(1) =


= (a0

a1 + a2 ) (b0

b1 + b2 ) + a0 b0 + (a0 + a1 + a2 ) (b0 + b1 + b2 ) =

= 3a0 b0 + 2a0 b2 + 2a1 b1 + 2a2 b0 + 2a2 b2 =


2
32
3
3 0 2
b0
= a0 a1 a1 4 0 2 0 5 4 b1 5 .
2 0 2
b1

Assim, relativamente base cannica ordenada f1; t; t2 g de P2 :


3
2
32
3 0 2
b0
hp(t); q(t)i = a0 a1 a1 4 0 2 0 5 4 b1 5
2 0 2
b1
onde

3 2
3
3 0 2
h1; 1i h1; ti h1; t2 i
4 0 2 0 5 = 4 ht; 1i ht; ti ht; t2 i 5
2 0 2
ht2 ; 1i ht2 ; ti ht2 ; t2 i

3
3 0 2
p
Como 4 0 2 0 5 simtrica e os seus valores prprios ( 21 17 + 52 ;
2 0 2
positivos, logo, a aplicao h; i dene um produto interno em P2 .
(ii) Tem-se:
U = a1 t + a2 t2 : a1 ; a2 2 R = L(ft; t2 g).
406

5
2

1
2

17 e 2) so todos

Aplicando o mtodo de ortogonalizao de Gram-Schmidt, sejam


p1 (t) = t e p2 (t) = t2

ht2 ; ti
t.
ktk2

Logo,
p2 (t) = t2

( 1)2 ( 1) + 02 0 + 12 1
t = t2 .
( 1):( 1) + 0:0 + 1:1

Logo, o conjunto ft; t2 g uma base ortogonal de U . Assim, o conjunto


(p p )
t2
t t2
2
2 2
t
; 2
= p ;p
t;
t
=
ktk kt k
2
2
2 2
uma base ortonormada de U .
(iii) Tem-se:
U ? = p(t) 2 P2 : hp(t); ti = 0 e
Logo,

Logo,

8
< (a0
:

(a0

p(t); t2 = 0 .

a1 + a2 )( 1)2 + a0 0 + a0 + a1 + a2 = 0
a1 + a2 )( 1) + a0 0 + a0 + a1 + a2 = 0
U? =

8
< a0 =
:

a2

a1 = 0.

a2 + a2 t2 : a2 2 R = L(f 1 + t2 g).

Como k 1 + t2 k = 1 ento f 1 + t2 g uma base ortonormada de U ? .


Observao. Note que P2 = U U ? , tendo-se, neste caso, dim U = 2 e dim U ? = 1.
(iv) A projeco ortogonal PU de P2 sobre U denida por:
P 2 ! P2
*
*
p +p
p +p
2
2
2 2
2 2
p(t) !
p(t);
t
t + p(t);
t
t,
2
2
2
2
PU

uma vez que o conjunto

p )
2
2 2
t;
t
2
2

(p

uma base ortonormada de U . Logo, a projeco ortogonal de 1 + t sobre U dada por:


*
*
p +p
p +p
2
2
2 2
2 2
PU (1 + t) = 1 + t;
t
t + 1 + t;
t
t = t + t2
2
2
2
2
A projeco ortogonal PU ? de R3 sobre U ? denida por:
PU ? : P2 ! P 2
p(t) ! p(t); 1 + t2 ( 1 + t2 ),
407

uma vez que o conjunto


f 1 + t2 g

uma base ortonormada de U ? . Logo, a projeco ortogonal de 1 + t sobre U ? dada por:


PU ? (1 + t) = 1 + t; 1 + t2 ( 1 + t2 ) = 1

t2

Nota muito importante: Uma vez que se tem


U ?,

P2 = U
ento para todo o p(t) 2 P2 ,

p(t) = PU (p(t)) + PU ? (p(t)).


Logo, uma vez calculado PU ? (1+t) pela denio, como se fz atrs, obtendo-se PU ? (1+t) =
1 t2 , ento no precisamos de efectuar o clculo de PU (1 + t) pela denio. Basta efectuar:
PU (1 + t) = 1 + t

PU ? (1 + t) = t + t2 .

(v) Seja B = f1; t; t2 g a base cannica de P2 . Atendendo alnea (iii), tem-se


* p +p
* p +p
2
2
2 2
2 2
PU (1) = 1;
t
t + 1;
t
t = t2
2
2
2
2
* p +p
* p +p
2
2
2 2
2 2
PU (t) = t;
t
t + t;
t
t =t
2
2
2
2
* p +p
* p +p
2
2
2 2
2 2
PU (t2 ) = t2 ;
t
t + t2 ;
t
t = t2
2
2
2
2
PU ? (1) = 1; 1 + t2

1 + t2 = 1

PU ? (t) = t; 1 + t2

1 + t2 = 0

PU ? (t2 ) = t2 ; 1 + t2
e assim

t2

1 + t2 = 0

Note que

3
0 0 0
M (PU ; B; B) = 4 0 1 0 5
1 0 1
2
3
1 0 0
M (PU ? ; B; B) = 4 0 0 0 5 .
1 0 0
I = PU + PU ? .

(vi) Escolhendo um ponto de U , por exemplo t, a distncia entre 1 + t e U dada por:


d(1 + t; U ) = kPU ? (1 + t

t)k = kPU ? (1)k =


408

1; 1 + t2 ( 1 + t2 ) = 1.

(vii) Escolhendo um ponto de U , por exemplo o polinmio nulo 0, a distncia entre


a0 + a1 t + a2 t2 e U , com a0 ; a1 ; a2 2 R, dada por:
d(a0 + a1 t + a2 t2 ; U ) = PU ? (a0 + a1 t + a2 t2 ) =
=

a0 + a1 t + a2 t2 ; 1 + t2 ( 1 + t2 ) = ja0 j 1

t2 = ja0 j .

19. Considere no espao linear M2 2 (R) o produto interno denido da seguinte forma:
hA; Bi = tr(AB T ).
Considere tambm o subespao U de M2 2 (R) constitudo por todas as matrizes simtricas
reais do tipo 2 2:
a b
U=
2 M2 2 (R) : b = c .
c d
(i) Sejam x; y 2 R e A; A0 ; B 2 M2 2 (R). Tem-se
hxA + yA0 ; Bi = tr((xA + yA0 ) B T ) = tr(xAB T + yA0 B T )

tr linear

= xtr(AB T ) + ytr(A0 B T ) = x hA; Bi + y hA0 ; Bi

hA; Bi = tr(AB T ) = tr( AT

a b
c d
a b
c d

B T ) = tr( BAT ) = tr(BAT ) = hB; Ai


!
T
a b
a b
= a2 + b2 + c2 + d2 0
c d
c d

hA; Ai = tr(AAT ) = tr
para todo o

2 M2 2 (R) e
a b
c d

a b
c d

=0,a=b=c=d=0,

0 0
0 0

Logo, a aplicao h; i dene um produto interno em M2 2 (R).


(ii) Tem-se:
a b
b d

U=

1 0
0 0

=L

: a; b; d 2 R
0 1
1 0

0 0
0 1

0 1
1 0

+d

pois
a b
b d

1 0
0 0

=a

1 0
0 1
0 0
;
;
0 0
1 0
0 1
linearmente independente pois se tivermos:
O conjunto

1 0
0 0

0 1
1 0

+b

0 0
0 1

uma base de U , uma vez que gera U , e

+
409

0 0
0 1

0 0
0 0

ento
1

0 0
0 0

1 0
0 1
0 0
;
;
linearmente
0 0
1 0
0 1
independente. Vamos aplicar agora a este conjunto o mtodo de ortogonalizao de GramSchmidt. Sejam
Logo,

e como tal, o conjunto

A1 =

1 0
0 0

A3 =

0 0
0 1

0 1
1 0

A2 =

0 0
0 1

projA1

0 1
1 0

projA1

0 0
0 1

projA2

Logo,
0 1
1 0

A2 =

0 1
1 0

0 1
1 0

0 1
1 0

0 1
1 0

0 1
1 0

projA1
0 1
1 0

tr

tr

; A1 A1

kA1 k2
0 1
AT1
1 0
kA1 k2
0 0
A1
1 0

kA1 k2
0A1
0 1
2 =
1 0
kA1 k

=
A1
=

=
.

e
A3 =

0 0
0 1

0 0
0 1

0 0
0 1

0 0
0 1

0 0
0 1

0 0
0 1

projA1
0 0
0 1

tr

tr

projA2

; A1 A1

kA1 k2
0 0
AT1
0 1
kA1 k2
0 0
A1
0 0

kA1 k2
0A1
0A2
=
2
kA1 k
kA2 k2
410

A1

0 0
0 1

0 0
0 1

; A2 A2

kA2 k2
0 0
AT2
0 1

tr

tr

0 0
0 1

0 0
1 0

kA2 k2

kA2 k2
.

A2
=

=
A2
=

1 0
0 0

Logo, o conjunto

0 1
1 0

0 0
0 1

uma base ortogonal de U . Como:

q
p
hA1 ; A1 i = tr (A1 AT1 ) =
kA1 k =
q
p
kA2 k =
hA2 ; A2 i = tr (A2 AT2 ) =
kA3 k =

hA3 ; A3 i =

tr (A3 AT3 ) =

s
s
s

tr

1 0
0 0

= 1,

tr

1 0
0 1

tr

0 0
0 1

= 1,

2,

ento o conjunto
1 0
0 0

1
;p
2

0 1
1 0

0 0
0 1

1 0
0 0

"

2
2

0
p

2
2

0 0
0 1

uma base ortonormada de U .


(iii) Tem-se
a b
2 M2 2 (R) :
c d
a b
0 1
;
=0 e
c d
1 0

U? =

Logo,

Ou seja,
0 b
b 0

U? =

a
c
a
c

b
d
b
d

1
0
0
0

;
;

8
a=0
>
>
>
>
<
b+c=0
>
>
>
>
:
d = 0.
:b2R

=L

0 1
1 0

0
0
0
1

=0 e
=0 .

0 1
1 0

Como
0 1
1 0

v
u
u
= ttr
=
=

s
s

0 1
1 0

0 1
1 0

0 1
1 0

0 1
1 0

tr

1 0
0 1

tr
411

0
1
=

1
0
2,

ento o conjunto

("

#)

2
2

0p

2
2

uma base ortonormada de U ? .


1 0
0 1
;
0 0
0 0
Atendendo alnea (iii), tem-se
(iv) Seja B =

PU

PU ?

1 0
0 0
0 1
0 0

1 0
0 0

0 0
0 1

0 0
0 1

0 1
0 0

0 0
0 1

0 0
0 1

0 0
1 0

0 0
0 1

0 0
0 1

0 0
0 1
*
1
=
0
*
0
=
0

0 0
0 1
"

;
0
0
1
0

"

0p

2
2

0p

2
2

412

#+ "

1 0
0 0
#

1 0
0 0

2
2

1 0
0 0
#
1
2

1
2

2
2

0
1 0
0 0
#
1
2

1
2

2
2

2
2

2
2

2
2

0 0
=
0 1
p #+ "
p

0 0
0 0
1 0
=
;
0 1
0 1
0 0
*
"
p #+ "
2
0 0
0
0
p
+
; p2 2
2
0 1
0
2
2
+

PU ?

a base cannica de M2 2 (R).

0 0
0 0
1 0
=
;
1 0
1 0
0 0
*
"
#+
"
p
2
0 0
0
0
p
+
; p2 2
2
1 0
0
2
2
+

PU

0 0
0 1

0 1
0 1
1 0
=
;
0 0
0 0
0 0
*
"
p #+ "
2
0 1
0
0
p
+
; p2 2
2
0 0
0
2
2
+

PU

0 0
1 0

1 0
1 0
1 0
=
;
0 0
0 0
0 0
*
"
p #+ "
2
0
1 0
0
p
+
; p2 2
2
0 0
0
2
2
+

PU

0
1 0
0 0
#

0 0
0 1
0p

2
2

2
2

0p

2
2

0
p

2
2

0 0
0 0
0
1
2

1
2

PU ?
PU ?

0 0
1 0
0 0
0 1

0 0
1 0

0 0
0 1

"
;

2
2

0p

2
2

"

0
2
2

2
2

0
6 0
M (PU ? ; B; B) = 6
4 0
0

Note que

0
1
2

1
2

1
2

2
2

2
2

0p

2
2

0
p

2
2

1
2

0
1
2

0 0
0 0

3
0
0 7
7
0 5
1

1 0 0
6 0 1 1
2
2
M (PU ; B; B) = 6
4 0 1 1
2
2
0 0 0

0p

#+ "

0p

e assim

#+ "

1
2

3
0
0 7
7.
0 5
0

I = PU + PU ? .
1 1
0 1

(v) A projeco ortogonal da matriz


PU ?

1 1
0 1

= proj2
6
4

2
2

3
2
2 7
5

0p

sobre U ? dada por:


1 1
0 1

p #
p #+ "
2
2
0
1 1
0p
2
2
p
=
=
;
2
2
0 1
0
0
2
2
0
1"
"
p #T
p #
2
2
1
1
0
0
2
2
A
p
p
= tr @
=
2
2
0 1
0
0
2
2
"
p #! "
p #
2
2
1 1
0
0
2
2
p
p
= tr
=
2
2
0 1
0
0
2
2
" p
p #! "
p #
2
2
2
0
2
2
p2
p
= tr
=
2
2
0
0
2
2
p #
p "
2
2
0p
0 12
2
=
=
.
1
2
0
2
0
2

"

Como se tem:

ento para todo

a b
c d

M2 2 (R) = U

U ?,

2 M2 2 (R),
a b
c d

= PU

a b
c d
413

+ PU ?

a b
c d

Logo,
PU

1 1
0 1

1 1
0 1

1 1
0 1
1
2

1
2

(vii) A distncia entre

1 1
0 1

1 1
0 1
;U

1 1
0 1

(vi) A matriz simtrica mais prxima da matriz

PU

1
2

1
2

1 1
0 1

PU ?

1 1
0 1

1
2

1
2

a matriz

e U dada por:

=
=
=
=

PU ?
s

tr
tr

1 1
0 1
1
2

0
1
2

1
2

414

1
2

1
2

0
1
4

1
4

1
2
1
2

1
2

1
2

2
.
2

1
2

0
=

0
0

(viii) A distncia entre

a b
c d

;U

a b
c d

e U dada por:
a b
c d

PU ?

proj2

=
=
=
=

*
tr
tr

6
4

3
2
2 7
5

0p

2
2

"

a b
c d

a b
c d

"

a b
c d
p
1
2b
2p
1
2d
2

"

415

a b
c d

0p

2
2

0p

2
2

0
p

2
2

1
2a
2p
1
2c
2

2
2

0
p

2
2

0
p

2
2

=
#+ "

#+

#!
=

0p

2
2

2
2

=
=
p

2
jb
2

cj .

Resoluo da 4a Ficha de exerccios facultativos


1. Seja V um espao euclidiano real. As alneas (i), (ii), (iii) e (iv) so consequncia da
denio de produto interno.
Sejam u; v; w 2 V;

2 R.

(v) Atendendo condio de linearidade do produto interno:


hu + w; v + wi = hu; vi + hu; wi + hw; vi + hw; wi =
= hu; vi + hu; wi + hw; vi + kwk2 .
(vi) Atendendo condio de linearidade do produto interno:
hu; 0i = hu; 0vi = 0 hu; vi = 0 e

h0; ui = h0v; ui = 0 hv; ui = 0.

(vii) Se hu; vi = 0 ento


ku + vk2 =
=
=
=
=
isto , ku + vk = ku
Se ku + vk = ku
equao

hu + v; u + vi = kuk2 + 2 hu; vi + kvk2 =


kuk2 + kvk2 =
kuk2 2 hu; vi + kvk2 =
hu v; u vi =
ku vk2 ,

vk.

vk ento ku + vk2 = ku

vk2 e esta ltima equao equivalente

kuk2 + 2 hu; vi + kvk2 = kuk2

2 hu; vi + kvk2 ,

isto , hu; vi = 0.

(viii) Atendendo a que


ku + vk2 = hu + v; u + vi = kuk2 + 2 hu; vi + kvk2 ,
ento tem-se ku + vk2 = kuk2 + kvk2 se e s se hu; vi = 0.
(ix) Seja c 2 R. Se hu; vi = 0 ento

ku + cvk2 = hu + cv; u + cvi = kuk2 + 2c hu; vi + c2 kvk2 =


= kuk2 + c2 kvk2 kuk2 ,
para todo o real c, isto , ku + cvk
Se ku + cvk

kuk para todo o real c.

kuk para todo o real c, ento

kvk2 c2 + 2 hu; vi c

0,

para todo o real c, se e s se hu; vi = 0 (frmula resolvente).


416

(x) Se hu + v; u

vi = 0 ento
0 = hu + v; u

vi = kuk2

kvk2 .

Logo, kuk = kvk.

Se kuk = kvk ento


Logo, hu + v; u

0 = kuk2

kvk2 = hu + v; u

vi .

vi = 0.

(xi)

ku

vk2 + ku + vk2 = hu v; u vi + hu + v; u + vi =
= kuk2 2 hu; vi + kvk2 + kuk2 + 2 hu; vi + kvk2 =
= 2 kuk2 + 2 kvk2 .

2. Seja V um espao euclidiano real.


(i) Seja u 2 V . Se hu; vi = 0 para qualquer v 2 V ento, em particular para v = u,
tem-se
hu; ui = 0.
Logo, u = 0.
(ii) Sejam u; v 2 V . Se u = v ento
hu; wi = hv; wi ,
para qualquer w 2 V .

Se hu; wi = hv; wi para qualquer w 2 V , ento


hu

v; wi = 0,

para qualquer w 2 V . Logo, atendendo alnea anterior, tem-se u = v.


3. Seja V um espao euclidiano com dim V = n. Seja S = fu1 ; :::; un g uma base ortonormada de V . Seja T : V ! V uma transformao linear. A matriz A = (aij ) que
representa T em relao base S dada por
A = (aij ) = (hT (uj ); ui i) ,
uma vez que, para j = 1; :::; n,
T (uj ) = hT (uj ); u1 i u1 + ::: + hT (uj ); un i un .
4. Seja V um espao euclidiano de dimenso n. Seja fu1 ; :::; uk g um conjunto linearmente
independente de k vectores de V . Considere a transformao linear T : V ! V denida
por
k
X
T (v) =
hv; ui i ui ,
i=1

com v 2 V .

417

Mostre que T invertvel se e s se k = n.


Dem. Atendendo a que T invertvel se e s se N (T ) = f0g, bastar ver que
N (T ) = f0g se e s se k = n.
Se N (T ) = f0g ento teremos k = n, caso contrrio, isto , caso k < n ter-se-ia
(L (fu1 ; :::; uk g))? 6= f0g :
Assim, para v 2 (L (fu1 ; :::; uk g))? , com v 6= 0, teramos T (v) = 0, ou seja N (T ) 6=
f0g. O que no pode ser pois suposemos N (T ) = f0g. Logo, se N (T ) = f0g ento
tem-se k = n.
Suponhamos agora que se tem k = n. Nesse caso, o conjunto fu1 ; :::; un g uma base
de V . Queremos ver que se tem N (T ) = f0g. Seja v 2 V tal que T (v) = 0. Logo,
n
X
i=1

hv; ui i ui = 0.

Assim, atendendo a que o conjunto fu1 ; :::; un g linearmente independente, tem-se


hv; ui i = 0, para todo o i = 1; :::; n. Finalmente, como o conjunto fu1 ; :::; un g gera V ,
tem-se hv; ui = 0, para qualquer u 2 V . Logo v = 0 e assim N (T ) = f0g.
5. Seja V um espao euclidiano real. Seja T : V ! V uma transformao linear tal que
kT (w)k = kwk para qualquer w 2 V . Mostre que
hT (u); T (v)i = hu; vi ,
para quaisquer u; v 2 V .

Dem. Sejam u; v 2 V . Tem-se


hu; vi =

1
ku + vk2
2
=

kuk2

kvk2 =

1
kT (u) + T (v)k2
2

1
kT (u + v)k2
2

kT (u)k2

kT (u)k2

kT (v)k2 =

kT (v)k2 = hT (u); T (v)i .

6. Seja U uma matriz unitria. Isto :


UH = U
Seja

um valor prprio de U e v um vector prprio associado:


U v = v.

Logo
v H U H = (U v)H = ( v)H = v H
e assim
vH U H U v = vH
, v H Iv = v H

U v , v H U H U v = v H (U v) ,

v , v H v = v H v j j2 , 1

418

j j2 kvk2 = 0 , j j = 1.
v6=0

Resoluo da 7a Ficha de exerccios para as aulas de problemas


2

3
2 0 1
1. A = 4 0 1 0 5.
1 0 2

02

I) = det @4

det(A
Tem-se

0
0
1

31

1
0

1
0

5A =

dim N (A

3I) = mg (3)

dim N (A

1I) = mg (1)

3) (

ma (3) = 1

ma (1) = 2.

A simtrica
A simtrica

1)2 :

3
3
1
2
3 2 5. Como A simtrica ento ortogonalmente diagonalizvel,
2. A = 4 1
2 2
0
isto , existem uma matriz ortogonal P T (P T P = P P T = I; isto , P T = P 1 ) e uma matriz
diagonal D tais que
D = P AP T .
Como
3
det(A

1
1
2

I) =

os valores prprios de A so 2 e

2
2

2) ( + 4)2 ,

4 e tem-se

N (A

1 1
; ;1
2 2

2I) = L

N (A + 4I) = L (f(1; 1; 0) ; (2; 0; 1)g) .

Note-se que os vectores de N (A 2I) so ortogonais aos vectores de N (A + 4I). Para


determinar uma base ortonormada de R3 formada s com vectores prprios de A basta
aplicar o mtodo de Gram-Schmidt ao conjunto
1 1
; ; 1 ; (2; 0; 1) ; (1; 1; 0) .
2 2
Como, relativamente ao produto interno usual em R3 ,
1 1
; ; 1 ; (2; 0; 1)
2 2
basta substituir (1; 1; 0) pelo vector: (1; 1; 0)
(1; 1; 0)

(1; 1; 0) ;
1 1
; ;1 ;
2 2

1 1
; ;1
2 2
1 1
; ;1
2 2

proj(
1 1
; ;1
2 2

419

=0

1 1
; ;1
2 2

) (1; 1; 0)

proj(2;0;1) (1; 1; 0) =

h(1; 1; 0) ; (2; 0; 1)i


(2; 0; 1) =
h(2; 0; 1) ; (2; 0; 1)i

1 1
; ;1
2 2

= (1; 1; 0)

3
2

2
(2; 0; 1) =
5

1
; 1;
5

Base ortonormada de R3 formada s com vectores prprios de A:


(
1
1
1
1 1
; ;1 ;
(2; 0; 1) ; 1
1 1
2 2
k(2; 0; 1)k
; ;1
; 1; 25
2 2
5
=

p p !
6 6 6
;
;
;
6 6 3

p
p !
5
2 5
; 0;
;
5
5

Logo

6
P T = SBvp !Bc = 4

e
2

2
D=4 0
0

0
4
0

1
6p

6
0
2
0 5 = 4 5p 5
1
4
30
| 30

3
4
1
3. A =
0
det(A

1
2
1

1
6

6
p6
6
p6
6
3

p0
1
30
6
{z
=P

3
0
1 5. Como
3
3
1
1 2
I) =
0
1

0
1

p
30 30
;
;
30
6
p

p
2 5
5

1
6
3p
1
5
5 p
1
30
15

30
p30
30
6
p
30
15

5
5

32

3
1
2

54
}

2
5

2
5

1
; 1;
5
p

30
15

!)

3
7
5

1
3
2

1) (

32
2
6
2 54
0
|

6
p6
6
p6
6
3

3) (

p
2 5
5

5
5

{z

30
p30
30
6
p
30
15

=P T

4) ,

os valores prprios de A so 1; 3 e 4. Como A simtrica (A = AT ) e denida positiva uma vez


que os valores prprios de A so todos positivos, ento existe B simtrica e denidas positiva
tal que A = B 2 . De facto, como A ortogonalmente diagonalizvel (por ser simtrica) tem-se
2

A = P T DP = P T (D0 ) P = P T D0 P
com

3
1 0 0
D=4 0 3 0 5
0 0 4

P T D0 P = BB = B 2

Note-se que sendo os 3 valores prprios distintos, os correspondentes espaos prprios so


ortogonais entre si. Base ordenada ortonormada de R3 formada s com vectores prprios de
A:
1
1
1
(1; 2; 1) ;
( 1; 0; 1) ;
(1; 1; 1) =
k(1; 2; 1)k
k( 1; 0; 1)k
k(1; 1; 1)k
( p p p !
p
p ! p
p p !)
6 6 6
2
2
3
3 3
=
;
;
;
; 0;
;
;
;
.
6 3 6
2
2
3
3 3
420

3
7
5
}

Logo

6
P T = SBvp !Bc = 4

Assim pode ter-se


2
1 p0
0
4
D = 0
3
0 0

ou

6
B=4

6
p6
6
p3
6
6

2
2

3
p3
3
3

3
p3
3
3

ou

6
B=4

6
p6
6
p3
6
6

De facto:
2 1p
3 + 56
2
1
4
3p
1
5
3
6
2

2
2

2
2

4
3

1
3
1
3

32

3
3p

32
1 p0 0
74
6
0
3 0 54
5
0
0 2

3
3p

3
p3
3
3

2
2

2
2

32
0
6
3 0 54
0
2

1
74
5 0
0

0
p

6
p6
6
p3
6
6

3
3p

3
p3
3
3

2
2

6
p6
6
p3
6
6

3T

3
3p

2
2

3
p3
3
3

2
2

2
2

2 1p

7
5 =4

3
p3
3
3

3
0
3 0 5
0
2

1
0
4
ou D = 0
0

7
5.
2

3
3p

quadradas seguintes:
p
3 2 p6
2
0
0
2
p6
p
6
3 0 5 4 p36 p0
6
2
0 2

32

2
2

6
p6
6
p3
6
6

por exemplo
3
2
3
0
1 p0 0
0 5 ou D0 = 4 0
3 0 5
2
0
0 2

e nesse caso tem-se as 3 razes


32
2 p
p
p
6
2
3
1
2
3p
74
6 p66
3
B = 4 p3
0
5 0
p
p3
6
2
3
0
6

3
3p

0
p

5
6

3+

5
6

1
3p
1
3
2

3T

2 1p

7
5 =4

1
2

3T

5
6

7
5 =4 p
1
2

1
2
1
3

3+

1
2
1
3

5
6

1
3

3 + 12
1p
1
3
2

p 32 2 1 p
p 32 2 5 1 p
1
1
1
3
3
+
1
3
3
2
2
2
2
6
2
1
5 =4
5 =4
1
0
1
p
p
p
p 3 5
1
5
1
1
1
1
1
3
+
3
1
3
+
3+ 6
2
6
2
2
2
2
2
2
3
3
1 0
1 5.
=4 1 2
0
1 3

5
6

4
3

p 3
3
5
p
1
3 + 56
2

1
3

1
2

1
3

1
2

1
3

5
6

4
3

4
3

p 3
3
5
p 1 1
1
3+ 2
2

1
0
1

3
5
6

1
2
1
3

1
3

1
2

1
3

5
6

1
2

4.
Q (x1 ; x2 ; x3 ) = 3x21
=

=
3)

x1 x2 x3

2
6
4

6
p6
6
p3
6
6

x1 x2 x3
p

3
p3
3
3

2
2

2
2

3
3p

Q : R3 ! R
2

2x1 x2 + 2x22

2x2 x3 + 3x23 =
32
3
0
x1
1 5 4 x2 5 =
3
x3
2
p
p
3 p

3
1
1 2
0
1
32
1 0 0
74
6
5 0 3 0 54
0 0 4

421

6
p6
6
p3
6
6

2
2

2
2

3
3p

3
p3
3
3

3T 2
3
x1
7 4
x2 5 =
5
x3

3+

5
6

1
3p
1
3
2

3+

5
6

1
3p
1
3
2

5.

32

5 =

02

B6
= @4

3T 2
31T 2
x1
1 0 0
3 7 4 x 5C 4 0 3 0
5
A
2
p3
3
x3
0 0 4
3
2
1 0
4
0 3
= y1 y2 y3
0 0

6
p6
6
p3
6
6

2
2

3
3p

2
2

fazendo a mudana de varivel


2 p
6
4

6
p6
6
p3
6
6

3
p3
3
3

2
2

2
2

3
p3
3
3

6
p6
6
p3
6
6

2
2

3
3p

2
2

32
3
0
y1
0 5 4 y2 5
4
y3

3T 2
3
x1
7 4
x2 5 =
5
x3

3T 2
3 2
3
x1
y1
7 4
x2 5 = 4 y2 5 .
5
x3
y3

3
3p

32
56
4

Tem-se ento a forma quadrtica diagonal:

Q0 (y1 ; y2 ; y3 ) = y12 + 3y22 + 4y32 .

5. A =

2
1

1+i
.
3

(i)
2

AA =
AH A =

1
2
1

1+i
i
3
1+i
i
3

2
1
H

1+i
i
3

2
1

1+i
3

5 + 5i
5i
11

5 + 5i
11

5
5

5i

Logo A normal.
(ii)
A=

2
1

1+i
i
3

2
1

1+i
i
3

= AH

logo A hermitiana e em particular normal:


AAH = AA = AH A.
(iii) Como A hermitiana ento unitariamente diagonalizvel (embora o recproco
no seja verdadeiro), isto , existem uma matriz unitria U H (U H U = U U H = I; isto ,
U H = U 1 ) e uma matriz diagonal D tais que
D = U AU H .
Note-se que: A normal , A unitariamente diagonalizvel.
Como
det(A

I) =

2
1

1+i
i 3
422

=(

1) (

4) ,

os valores prprios de A so 1 e 4 e tem-se


N (A
N (A

1I) = L (f( 1

i; 1)g)

1 1
+ i; 1
2 2

4I) = L

Note-se que os vectores de N (A 1I) so ortogonais aos vectores de N (A 4I). Logo,


uma base ortonormada de C2 formada s com vectores prprios de A pode ser:
(
)
1
1
1 1
( 1 i; 1) ; 1 1
+ i; 1
=
k( 1 i; 1)k
2
2
+
i;
1
2
2
=

p !
3
3
i;
;
3
3

p p !)
6
6
6
+
i;
.
6
6
3

3
3

Logo
U H = SBvp !Bc =

"

3
3 p

3
i
3

3
3

p
6
6
+
i
6 p 6
6
3

e
D=

1 0
0 4

"
|

3
3 p

3
i
3

3
3

{z

p
6
6
+
i
6 p 6
6
3

=U

#H

2
1

1+i
i
3

"
|

3
3 p

3
3

6. Determine a soluo de mnimos quadrados de Au = b, com


2
3
2 3
1 2
3
4
5
4
A= 2 4
e b= 2 5
1
2
1

3
i
3

{z

=U H

p
6
6
+
i
6 p 6
6
3

calculando o correspondente vector erro de mnimos quadrados.


Tem-se car AT A = car A = 2 e como tal a soluo de mnimos quadrados nica e
dada por:
1 T
u = AT A
A b=
0
2
31 1
2 3
3
1 2
1 2 1 4
1 2 1 4 5
6=5
@
5
A
2 4
2 =
=
.
2 4
2
2 4
2
1=10
1
2
1
O vector erro de mnimos quadrados b Au dado por:
2 3 2
3
2 8 3
3
1 2
5
6=5
b Au = 4 2 5 4 2 4 5
= 4 45 5
1=10
0
1
1
2

sendo o erro de mnimos quadrados dado por:


kb

Auk =

8
;
5
423

4
;0
5

4p
5.
5

7. Resolva o seguinte sistema de equaes diferenciais determinando a respectiva soluo


geral.
8 0
2
3 2
3
u1 (0)
2
< u1 = 2u3
4 u2 (0) 5 = 4 1 5 .
u0 = u1 + 2u2 + u3
com
: 20
u3 = u1 + 3u3
u3 (0)
0
8 0
2 0
3 2
32
3
u1 (t)
0 0
2
u1 (t)
< u1 = 2u3
u0 = u1 + 2u2 + u3 , 4 u02 (t) 5 = 4 1 2 1 5 4 u2 (t) 5 : ( )
: 20
u3 = u1 + 3u3
u03 (t)
1 0 3
u3 (t)
2
3
0 0
2
1 e 2 so os valores prprios da matriz A = 4 1 2 1 5, sendo os espaos prprios
1 0 3
dados por:
N (A 1I) = L (f( 2; 1; 1)g)
N (A

2I) = L (f(0; 1; 0) ; ( 1; 0; 1)g) .

Como existe2uma base de3R3 formada s por vectores prprios:


0 0
2
4
ento a matriz 1 2 1 5 diagonalizvel. Assim, fazendo
1 0 3
2
3
2
0
1
2
1
1
4
5
4
1
1
P = 1 0
tem-se P =
0 1
1
1
e

3
2
2 0 0
0
,D=4 0 2 0 5=P4 1
0 0 1
1
2
3
2
0 0
2
, 4 1 2 1 5 = P 14
1 0 3

o sistema (*) equivalente a


2 0
3 0
2
u1 (t)
4 u02 (t) 5 = @P 1 4
u03 (t)
2 0
3 2
u1 (t)
, P 4 u02 (t) 5 = 4
u03 (t)

1
0
0

3
1
2 5
1

3
2
1 5P 1 ,
3
3
2 0 0
0 2 0 5P.
0 0 1
0
2
0

3 12
2 0 0
0 2 0 5PA4
0 0 1
30 2
2 0 0
0 2 0 5 @P 4
0 0 1

Assim, considerando a mudana de varivel


2
3
2
3
v1 (t)
u1 (t)
4 v2 (t) 5 = P 4 u2 (t) 5
v3 (t)
u3 (t)
424

f(0; 1; 0) ; ( 1; 0; 1) ; ( 2; 1; 1)g

3
u1 (t)
u2 (t) 5 ,
u3 (t)
31
u1 (t)
u2 (t) 5A
u3 (t)

tem-se

3 2
30 2
31
2 0 0
v10 (t)
v1 (t)
4 v20 (t) 5 = 4 0 2 0 5 @P 4 v2 (t) 5A ,
v30 (t)
0 0 1
v3 (t)
8 0
v1 (t)
>
>
=2
>
>
8 0
>
v (t)
>
< v10 (t)
< v1 (t) = 2v1 (t)
2
v20 (t) = 2v2 (t)
=1 ,
,
,
Se v 6=0; > v2 (t)
: 0
v3 (t) = v3 (t) v2 6=0 e1 v3 6=0 >
>
>
v30 (t)
>
>
=1
:
v3 (t)
8
8
< log jv1 (t)j = 2t + k1
< v1 (t) = c1 e2t
log jv2 (t)j = t + k2 ,
v2 (t) = c2 et
,
:
:
log jv3 (t)j = t + k2
v3 (t) = c3 et
2

com c1 ; c2 ; c3 2 R. Logo
3 2
2
3
2
0
u1 (t)
c1 e2t
4 u2 (t) 5 = P 1 4 c2 et 5 = 4 1
0
c3 et
u3 (t)
Como

tem-se

3
3 2
32
c2 et 2c3 et
2
c1 e2t
1 5 4 c2 et 5 = 4 c3 et + c1 e2t 5 .
c2 et + c3 et
c3 et
1

1
0
1

8
< u1 (0) = 2
u2 (0) = 1
:
u3 (0) = 0

3 2
c1
1 1
4 c2 5 = 4 1 0
c3
1 0

32
3 2
3
1
2
1
2 54 1 5 = 4 2 5
1
0
2

e ento a soluo geral do sistema de equaes


2
3 2
u1 (t)
4 u2 (t) 5 = 4
u3 (t)

diferenciais lineares dada por


3
2et + 4et
2et + e2t 5 .
2et 2et

425

Você também pode gostar